Sie sind auf Seite 1von 91

Labor Law 1

SECTION 14: TERMINATION OF


EMPLOYMENT
A. GENERAL CONCEPT

14.01 SECURITY OF TENURE


A. NATURE OF SECURITY OF TENURE
SONZA V ABS-CBN BROADCASTING CORP
[PAGE 42]
ALHAMBRA INDUSTRIES V. NLRC (RUPISAN)
238 SCRA 232
BELLOSILLO; November 18, 1994
NATURE
Special civil action in the Supreme Court. Certiorari
FACTS
- Alhambra employed Rupisan as salesman on 6-mo
probationary basis. Alhambra made surprise audit, alleged
violations were purportedly committed by him. He was placed
under 1-mo preventive suspension. He protested. He alleges
that charges against him had become academic when he was
given clearance of all accountabilities.
- A day before end of suspension, he was terminated. He sued
Alhambra.
- Labor Arbiter found that the termination was for just cause,
but there was a violation of due process (failure to furnish copy
of audit report).
- Both parties appealed to NLRC which affirmed Arbiters
findings.
ISSUE
WON NLRC committed grave abuse of discretion in sustaining
finding of Labor Arbiter that Rupisan was illegally dismissed but
directing his reinstatement so he could have explained
HELD
YES
- Employment is no longer just an ordinary human acctivity. For
most families the main source of their livelihood, employment
has now leveled off with property rights which no one may be
deprived of without due process of law.
- Termination of employment is not anymore a mere cessation
or severance of contractual relationship but an economic
phenomenon affecting members of the family. This explains
why under the broad principles of social justice the dismissal of
employees is adequately protected by the laws of the state.
- A termination without just cause entitles a worker to
reinstatement regardless of whether he was accorded due
process. On the other hand, termination of a worker for cause,
even without procedural due process, does not warrant
reinstatement, but the employer incurs liability for damages.
- Since the Labor Arbiter found a valid ground for dismissal, it
erred when it directed reinstatement.
- To order reinstatement and compel the parties to start the
procedure from step one would be circuitous because almost
invariably that same issue of validity of the ground of dismissal
would be brought back to the Labor Arbiter for adjudication. We
laid down in Wenphil Corporation v. NLRC3 that an otherwise
justly grounded termination without procedural due process
would only sanction payment of damages
- Standards of due process in judicial as well as administrative
proceedings have long been established. In its bare minimum
due process of law simply means giving notice and opportunity

A2010

- 186 -

Disini

to be heard before judgment is rendered.


- When the private respondent
filed a complaint against
petitioner, he was afforded the right to an investigation by the
labor arbiter.
- Although belatedly, private respondent was afforded due
process before the labor arbiter wherein the just cause of his
dismissal had been established. With such finding, it would be
arbitrary and unfair to order his reinstatement with backwages.
- It will be highly prejudicial to the interests of the employer to
impose on him the services of an employee who has been
shown to be guilty of the charges that warranted his dismissal
from employment. Indeed, it will demoralize the rank and file.
- However, the petitioner must nevertheless be held to account
for failure to extend to private respondent his right to an
investigation before causing his dismissal.

MANILA ELECTRIC COMPANY V NLRC (LOMABAO,


MASAYA)
186 SCRA 763
NARVASA; July 2, 1991
NATURE
CERTIORARI
FACTS
- Jose Masaya made an unauthorized electric service connection
which supplied electricity to the house of Antonio Sanchez (who
paid the former Php 200 for making the said connection.)
- Sanchez neither applied with Meralco for electric service nor
made the requisite deposit for it.
- This clandestine and illicit connection was eventually
discovered by Meralco who then charged him (through a letter)
with a violation of the Company Code on Employee Discipline,
and thereafter conducted a formal investigation of the matter.
- Those who gave testimony at that investigation were Jose
Masaya himself, and Renato Repuyan, Meralco field
investigator.
- Prior to being interrogated about the illegal connection and in
response to preliminary questions by the investigator, Masaya
stated for the record that he had received the letter accusing
him of misconduct, that he had a copy of the code of discipline
and understood the nature of the precise charge against him,
and that he did not need to be assisted by a lawyer or a
representative of his Union because he said that what he was
about to say was pawing katotohanan lamang.
- Repuyan testified on the fact of the undenied and indisputable
installation of the illegal electrical connection at the residence
of Antonio Sanchez (his description of the manner of its
accomplishment being substantially the same as Masaya's
own), and also, the disclosures made to him by Sanchez's house
helpers and the owner of the house
- After the investigation, and on the basis of the results thereof,
Meralco filed with the Ministry of Labor and Employment an
application for clearance to terminate Masaya's services,
serving copy on the latter.
- Meralco also placed Masaya under preventive suspension.
- A week later, Masaya filed a complaint for illegal dismissal
against Meralco.
- After the trial, LA Andres M. Lomabao rendered a decision in
Masaya's favor; saying that the record of the investigation
conducted by Meralco should not be accorded credence; that
Meralco's contention that Masaya had "surreptitiously effected
the direct connection of . . . electric service" was not credible,
because Masaya "was employed as a bill collector, not as a
lineman collector, hence, he does not know how to install
electrical connection;" and that the money received by Masaya
from Sanchez (P200 or P250) was not in consideration of any
clandestine connection but was accepted as "representation
expenses in following up Mr. Sanchez' application for
installation of electric facilities . . . with the Engineer's Office at
the City Hall of Manila.

Labor Law 1
- NLRC affirmed the Arbiter's decision; said that since Meralco
was charging Masaya of a criminal offense, it should prove
beyond reasonable doubt (pbrd) said crime which it was not
able to do as it was not shown that Masaya was given the
opportunity to be heard by counsel or at least, a representative
to confront his accuser; that based on the doctrine of PBRD,
there is no causal connection between Masaya' s duties to the
crime imputed to him, mere substantial evidence is insufficient
to hold Masaya guilty of installing electrical connection let alone
deprive him of his right to labor."
ISSUE
WON the LA & the NLRC committed GABD in failing to take into
consideration or excluding Masayas admissions in their
prononcement that Masaya was illegally dismissed
HELD
YES
Reasoning
NLRCS ERROR:
- Masaya was in truth asked if he wished to be assisted by a
lawyer or a representative of his Union, and his response was in
the negative because, in his own words, "ang sasabihin ko
naman dito ay pawang katotohanan lamang"
- In administrative or quasi-judicial proceedings, PBRD is not
required as basis for a judgment of the legality of an employer's
dismissal of an employee, nor even preponderance of evidence,
substantial evidence being sufficient.
- LC: the rules of evidence prevailing in courts of law or equity
shall not be controlling and it is the spirit and intention of this
Code that the Commission and its members and the Labor
Arbiters shall use every and all reasonable means to ascertain
the facts in each case speedily and objectively and without
regard to the technicalities of law or procedure, all in the
interest of due process. . . .
- SC: the ground for an employer's dismissal of an employee
need be established only by substantial evidence.
- It is absolutely of no consequence that the misconduct with
which an employee may be charged also constitutes a criminal
offense
-The proceedings being administrative, the quantum of proof is
governed by the substantial evidence rule and not, as the
respondent Commission seems to imagine, by the rule
governing judgments in criminal actions.
-The Court cannot close its eyes to the following facts of record,
to wit:
1) the reality of the illegal electrical connection;
2) the letter to Masaya accusing him of misconduct
3) Masaya's acknowledgment that, having a copy of the
company's code of discipline, he understood the nature of the
accusation against him, and his declining to be assisted by a
lawyer or a representative of his Union because, according to
him, "ang sasabihin ko naman dito ay pawang katotohanan
lamang;"
4) his voluntary admission that it was he who had made the
illegal electrical connection, describing the manner by which
he had made it, and that he had received P250.00 from the
occupant of the house, Antonio Sanchez; and
5) his plea to the company for forgiveness for having made
the illegal connection.
- on record: testimony regarding identification of Masaya by
Antonio Sanchez' servants and by Castaeda, the owner of the
house occupied by Sanchez.
- nothing in the record to demonstrate that Masaya's
admissions were made otherwise than voluntarily.
- Such an offense is obviously of so serious a character as to
merit the penalty of dismissal from employment, as stated in
the Meralco Code on Employee Discipline:
SECTION 7. Dishonesty.
xxx xxx xxx
3) Directly or indirectly tampering with electric meters or
metering installation of the Company or the installation of
any device, with the purpose of defrauding the Company.

A2010

- 187 -

Disini

-The Labor Code pronounces "fraud or willful breach by the


employee of the trust reposed in him by his employer or
duly authorized representative," or "serious misconduct"
on the part of the employee to be lawful ground to
terminate employment.
Ratio And this Court has held that the "dismissal of a
dishonest employee is as much in the interests of labor
as it is of management. The labor force in any company
is protected and the workers' security of tenure
strengthened when pilferage of equipment, goods and
products which endangers the viability of an employer
and, therefore, the workers' continued employment is
minimized or eliminated and consequently labormanagement relations based on mutual trust and
confidence are promoted."
(*IN short: Tenurial Security is not an absolute right for the law
provides that an employee may be dismissed for just cause. )
Disposition Petition for certiorari is GRANTED, the decisions of
the NLRC and LA are ANNULLED AND SET ASIDE, and the
petitioner's termination of the employment of private
respondent is AUTHORIZED and APPROVED

CITYTRUST BANKING CORPORATION V NLRC (RUIZ)


258 SCRA 621
MENDOZA; July 11, 1996
NATURE
Special civil action in the Supreme Court. Certiorari
FACTS
- Private respondent Ruiz was the internal auditor of petitioner
Citytrust Banking Corporation. She was designated manager of
the Quiapo branch of the bank, but she refused the
appointment on the ground that it was a demotion. As a
consequence, she was suspended and, upon clearance given by
the Department of Labor, she was terminated on November 8,
1974.
- Private respondent filed a complaint for illegal dismissal. She
was ordered reinstated as branch manager, the NLRC urging
her to accept the position, otherwise her refusal would be
considered a ground for her loss of employment. Private
respondent appealed to the Minister of Labor (now Secretary of
Labor and Employment) but again she lost. Both parties then
appealed to the Office of the President, which ordered
petitioner to reinstate private respondent to her former position
as internal auditor and to pay her backwages from the time her
compensation was withheld up to the time of her reinstatement.
- Petitioner moved for a reconsideration on the ground that the
position of internal auditor had been abolished (although the
position of resident inspector was created in its stead), and
therefore in lieu of reinstatement, it should only be made to pay
private respondent's separation pay.
The Office of the
President modified its decision and ordered petitioner to
reinstate private respondent to a substantially equivalent
position without loss of seniority rights and to grant her the
benefits and privileges to which she would be entitled had she
not been dismissed.
- Subsequently, petitioner reinstated private respondent as
manager of the Auditing Department.
Private respondent
accepted the appointment but questioned her reinstatement to
that position on the ground that it was not substantially
equivalent to the position of resident inspector (the position
created in place of internal auditor). She also questioned the
award of backwages as the report of the socio-economic analyst
allegedly did not include backwages from April 1974 to June
1974 when she was on leave with pay and vacation and sick
leave in 1974 and other fringe benefits to which she was
entitled before her termination.
- Labor Arbiter Apolinario N. Lumabao issued an order holding
that the position of manager of the Auditing Department was
not substantially equivalent to that of resident inspector.
possible as it appears (that) the position is already filled up (,) to
relocate complainant to a substantially equivalent position with

Labor Law 1
all the emoluments and privileges of a Resident Inspector.
Respondent is hereby further ordered to pay.
- The NLRC affirmed the Labor Arbiter's order with modification
by ordering the following to be added to the award: (a) Her
vacation and sick leave privilege during the period of her
separation in accordance with the disposition hereinbefore
stated in the body of this Resolution, and (b)the normal
increases which complainant would have received during the
period of her separation.
- In connection with the computation of the award in her favor,
private respondent sought the production of the bank's payrolls
for 1974-1981. Her motion was opposed by petitioner which
offered instead P74,344.00, the total amount of backwages as
computed by the socio-economic analyst of the Department of
Labor, plus P9,040.00 in transportation allowance and
P1,050.00 mid-year bonus for 1974.
- Private respondent refused the offer, hence the NLRC directed
the analyst to compute the award on the basis of the payrolls
from 1974 to 1981. Petitioner appealed to the NLRC en banc,
but its petition was dismissed, on the ground that the order
appealed from was interlocutory.
- Petitioner filed a petition for Certiorari and Prohibition with this
Court, assailing the dismissal of its appeal. The petition was at
first dismissed for lack of merit.
Petitioner's motion for
reconsideration was also dismissed. On July 21, 1986 this Court
modified its decision and petitioner was ordered to pay private
respondent "backwages limited to three years without
qualification or deduction at the salary rate of private
respondent at the time of dismissal."
- The Labor Arbiter issued an alias writ of execution after finding
that the amount corresponded to the amount found due private
respondent in the decision of the NLRC and the resolution of
this Court, consisting of salary differentials and other fringe
benefits which were not paid to her from the time that she was
reinstated on August 14, 1978 as manager of the Auditing
Department.
- Petitioner moved to quash the alias writ of execution. As its
motion was denied, it filed a petition for Injunction in the NLRC
en banc to stop the implementation of the alias writ of
execution and prayed for a recomputation of the monetary
award pursuant to this Court's resolution of July 21, 1986. Its
petition was, however, denied, as was its motion for
reconsideration, in the resolutions of the NLRC. Hence, this
petition.

A2010

Disini

PHILIPS SEMICONDUCTORS V FADRIQUELA


[PAGE 77]
QUIJANO V BARTOLABAC
480 SCRA 204
TINGA; January 27, 1999

ISSUE
WON private respondent is entitled to only three years of
backwages and no more
HELD
NO
- Private respondent is, in addition, entitled to reinstatement
without loss of seniority rights. Art. 280 of the Labor Code
provides:
ART. 280.
Security of Tenure. In cases of regular
employment, an employer shall not terminate the services of
an employee except for a just cause or when authorized by
this title. An employee who is unjustly dismissed from
work shall be entitled to reinstatement without loss of
seniority rights and to his backwages computed from the
time his compensation was withheld from him up to the time
of his reinstatement. (emphasis supplied)
- Backwages are for earnings which a worker has lost due to his
illegal dismissal. Private respondent was illegally dismissed
from November 8, 1974 to August 13, 1978. In its May 28,
1985 Report, the socio-economic analyst computed private
respondent's backwages for this period but he erroneously
considered as backwages private respondent's salary
differential from August 14, 1978 to October 31, 1984. On
August 14, 1978, private respondent had already been
reinstated, albeit to a lower paying position as manager of the
Auditing Department. Hence the award of backwages should be
up to August 13, 1978 only. What she was entitled to receive
after that date was the difference between the salary of internal

- 188 -

auditor (resident inspector) and that of manager of the Auditing


Department to which she was actually appointed. This position,
as already noted, was found to be not a substantially equivalent
position to that of internal auditor or resident inspector.
- The resolution of July 21, 1986 of this Court, which limited the
award of backwages, referred to the backwages for the period
November 8, 1974 to August 13, 1978 as component of the
relief granted by law to those who are illegally dismissed. The
Court at that time limited the award of backwages to three
years without qualification and deduction to avoid delays
incident to the determination of the earnings of the laid-off
employees during the pendency of the case and of deducting
them from the backwages later awarded.
- The second component of the relief granted under then Art.
280 of the Labor Code was reinstatement either to their former
position or if, this was not possible, to a substantially equivalent
position. Reinstatement contemplates a restoration to a position
from which one has been removed or separated so that the
employee concerned may resume the functions of the position
he already held. Private respondent was the internal auditor of
petitioner at the time of her dismissal. Since this position had
been replaced by the position of resident inspector, private
respondent should have been appointed resident inspector.
The position of manager of the Auditing Department to which
she was appointed was not a substantially equivalent position,
as found by the Labor Arbiter in his order of February 26, 1979
and later by the NLRC.
- The order to reinstate an employee to a former position or to a
substantially equivalent position is a positive mandate of the
law with which strict compliance is required.
This is an
affirmation that those deprived of a recognized and protected
interest should be made whole so that the employer will not
profit from his misdeeds.
- Since private respondent retired from the bank on March 1,
1991, reinstatement is now academic. She should therefore be
paid the difference in pay of a resident inspector and a
manager of the Auditing Department from August 14, 1978 up
to March 1, 1991.
Disposition Petition dismissed.

FACTS
- Quijano was employed by Mercury Drug Corporation as a
warehouseman --- a clerical/rank and file position. He was
dismissed, so he filed a complaint with the NLRC for illegal
dismissal. The case reached the SC. In 1998, the SC ruled for
his reinstatement to his old position or to a substantially similar
position. The SC denied the companys mfr, and came out with
a resolution in 1999 for Quijanos reinstatement.
- Whats this case all about, then? The respondents in this case
are the LA and the NLRC commissioner, respectively. Quijano
filed a case against then for violation of Canon 1 and Rule 1.01
of the Code of Professional Responsibility. WHY? They gave out
orders contrary to the resolution of the SC. The LA said to make
him self-service attendant because accdg to mercury there
were only 4 positions open. All 4 positions required college
graduates, but LA said he thinks Quijano could handle the selfservice attendant job. The NLRC commissioner said since there
are no available positions, he should just be given separation
pay.
ISSUE
WON Bartolabac & Quimpo erred
HELD
YES

Labor Law 1
- The decision of the SC was already final and executory. They
had no place to use discretion in executing a final and
executory order of the Supreme Court. SUPREME. If the final &
executory orders of the SC would be second-guessed by other
bodies, then cases would never reach finality. The
implementation of the final and executory decision is
mandatory. (The court was disappointed in the IBP
recommendation to dismiss the complaint against Bartolabac &
Quimpo.)
- The SC wont compel to instantly restore the position of
warehouseman if it had already been abolished. It ruled that
Quijano should be reinstated to original or substantially similar
position. They took notice of Mercury Drugs nationwide
operation. SC couldnt believe that they wouldnt have a
position for Quijano.
- Our Constitution mandates that no person shall be deprived of
life, liberty, and property without due process of law. It should
be borne in mind that employment is considered a property
right and cannot be taken away from the employee without
going through legal proceedings. In the instant case,
respondents wittingly or unwittingly dispossessed complainant
of his source of living by not implementing his reinstatement. In
the process, respondents also run afoul of the public policy
enshrined in the Constitution ensuring the protection of the
rights of workers and the promotion of their welfare.
Disposition Bartolabac & Quimpo suspended from the practice
of law for 3 months for violation of Canon 1 and Rule 1.01 of
CPR.

A2010

- 189 -

Disini

school. As a matter of fact, 6 out of the nine 9 students and


their
parents/guardians
retracted
and
withdrew
their
statements.
- NLRC reversed LAs decision, saying the dismissal was valid
and legal.
ISSUE
WON dismissal was valid and legal
HELD
NO
- In view of the foregoing, the conclusion of the NLRC is
unwarranted.
No due process The committee refused to revise the rules of
procedure. As a result, Lorlene wasnt afforded a chance defend
herself and to examine / cross-examine the accusers.
Failure to prove by substantial evidence The evidence of
Ateneo didnt measure up to the standard laid down in Ang
Tibay v CIR: "substantial evidence is more than mere scintilla. It
means such relevant evidence as a reasonable mind might
accept as adequate to support a conclusion."
Lorlenes evidence She was able to prove that shes a
competent and dedicated teacher of Ateneo for 17 years.
- Employment is not merely a contractual relationship; it has
assumed the nature of property right. It may spell the
difference whether or not a family will have food on their table,
roof over their heads and education for their children. It is for
this reason that the State has taken up measures to protect
employees from unjustified dismissals. It is also because of this
that the right to security of tenure is not only a statutory right
but, more so, a constitutional right.
Disposition NLRC decision reversed and set aside. LA decision
reinstated, affirmed and adopted.

B. IMPORTANCE OF EMPLOYMENT
EMPLOYMENT
GONZALES V NLRC (ATENEO DE DAVAO
UNIVERSITY)
313 SCRA 169
BELLOSILLO; August 26, 1999
FACTS
- Lorlene Gonzales was a Grade 6 teacher in Ateneo de Davao
University from 1974 to 1993, when she was terminated. In
1991, the Grade School Headmaster sent her a letter informing
her of 2 complaints from parents of her students for alleged use
of corporal punishment. She demanded to know who the
parents were because Ateneo wouldnt tell her. When she found
out that Ateneo was soliciting complaints from parents of her
students, she demanded an investigation.
Ateneo sent her a notice of investigation, schedule,
Committee composition, affidavits of the parents, and the rules
of procedure. She refused to take part in the investigation
unless the rules of procedure were revised. The committee,
under advise of counsel, did not revise the rules, since it had
been used for a different teacher in the past. The investigation
went on, without her participation. In 1993, she was asked to
tender her resignation, otherwise she would be considered
resigned.
- Lorlene filed for illegal dismissal with the LA. The LA found
that she was indeed illegally dismissed because although she
was afforded due process, Ateneo failed to establish substantial
evidence as to Lorlenes guilt. It was established that she is a
very good teacher, equipped with the appropriate educational
qualifications, trainings, seminars and work experiences. Such
fact was affirmed by her present and former students, their
parents, colleagues and the former headmaster of the grade

C. STATE REGULATION - RATIONALE


RATIONALE
LLOSA-TAN V SILAHIS INTERNATIONAL HOTEL
181 SCRA 738
PARAS; February 5, 1990
NATURE
Petition for certiorari seeking to set aside the decision and
resolutions of the NLRC
FACTS
- The complainant was a front office cashier of Silahis
International Hotel since November 2, 1976 until her questioned
dismissal on October 30, 1982.
- Since 1977, the Silahis International Hotel, had a standing
corporate policy (Corporate Policy No. 014), which orders all
cashiers of SMC and its affiliates to refuse the cashing of
personal checks of employees and officials, endorsement by
any executive of the Sulo Management Company, or Philippine

Labor Law 1
Village Hotel or Silahis International Hotel or Sulo Hotel
notwithstanding, because based on experience, a number of
these checks unfortunately bounce to the detriment of SMC and
its affiliates.
- On August 22, 1982, while petitioner was on duty, she was
approached by Mr. Gayondato, the general cashier of Puerto
Azul Beach Resorta sister company of Silahis International
Hotel and nephew of the Executive Vice President, to encash
two (2) US dollar checks with a combined value of US$1,200.00
or P10,389.60.
- Although petitioner politely explained the existence of Policy
No. 014 prohibiting such transactions, Gayondato persisted and
assured that the presentation of aforesaid checks to the front
office cashier was upon instructions of the Executive Vice
President.
- Petitioner, eventually encashed the aforesaid checks,
notwithstanding Corporate Policy No. 014.
- Thereafter, the said checks bounced.
- On October 1, 1982, respondent Vanessa Suatengco issued a
memorandum to the petitioner requiring her to explain in
writing why she should not be terminated for encashing the two
(2) personal checks without proper authorization.
- Despite petitioner's explanation, her services were terminated
effective October 30, 1982.
- Petitioner filed a complaint against respondents for illegal
dismissal.
- Labor Arbiter Virginia G. Son rendered a decision in favor of
petitioner.
- Hotel appealed the decision of the LA to the NLRC, and the
NLRC rendered a decision setting aside the decision of the
Labor Arbiter and dismissing the complaint for illegal dismissal
for lack of merit
- Petitioners 2 MFRs having been denied, recourse was made to
the SC
ISSUE
WON the acts of petitioner constitute gross negligence resulting
in a valid ground for the termination of her employment
HELD
NO
- Gross negligence has been defined as the want of any or slight
care or the utter disregard of consequences.
- Admittedly, the encashment of the checks in question is a
violation of Policy No. 014 of said hotel. But as found by the
Labor Arbiter, it was established that: (a) complainant was not
motivated by bad faith; (b) Policy No. 014 is not strictly or
consistently enforced but has been relaxed repeatedly to meet
business exigencies; and (c) complainant's encashment of the
checks in question was not only with the knowledge but with
clearance from her superiors who are more knowledgeable as to
the circumstances under which the enforcement of the same
may be relaxed.
- Moreover, it cannot be said that complainant was precipitate
or that she has acted in utter disregard of consequences. On
the contrary, she refused to encash subject checks despite the
request of Mr. Gayondato, the general cashier of Puerto Azul,
but was persuaded only upon the assurances of the latter that
such was the wish of the Executive Vice President and that said
encashment was necessary to meet certain disbursements in
Puerto Azul. In addition, she informed personally Mr. Samuel
Grulla, Assistant Manager of the Silahis International Hotel, of
said encashment, who also told her that such is "alright".
- Finally, against the background of her previous experience
when she refused to encash a similar check for Mr. Katte, the
Food and Beverage Manager of Silahis International Hotel, and
that she was reprimanded by the management of the Silahis
International Hotel for her refusal, as well as threatened with
suspension or dismissal from her job, coupled with the advice of
Mr. Nestor Famatigan, Jr., Silahis International Hotel
Comptroller, to use her discretion in handling similar requests in
the future, it is not at all surprising that she opted to take
subject course of action.

A2010

- 190 -

Disini

- It is well settled that dismissal based on loss of trust and


confidence arising from alleged misconduct of employee, is not
to be used as a shield to dismiss an employee arbitrarily.
Although the power to dismiss is a normal prerogative of the
employer, the same is not without limitations. The right of the
employer must not be exercised arbitrarily and without just
cause. Otherwise, the constitutional guarantee of security of
tenure of the workers would be rendered nugatory. While
dismissing or laying off of an employee is a management's
prerogative, it must nevertheless be done without abuse of
discretion. Furthermore, the right of employer to freely select or
discharge his employees is regulated by the State, because the
preservation of the lives of the citizens is a basic duty of the
State, more vital than the preservation of the corporate profit.
In addition, security of tenure is a right of paramount value
guaranteed by the Constitution and should not be denied on
mere speculation.
Protection for labor and social justice
provisions of the Constitution and the labor laws and rules and
regulations are interpreted in favor of the exercise of labor
rights.
Disposition The assailed decision of the NLRC is DISMISSED,
and SET ASIDE and private respondent Silahis International
Hotel is ordered to reinstate petitioner Anita Llosa-Tan to her
former position or similar position without loss of seniority rights
with full backwages beginning October 30, 1982 for a period of
three (3) years therefrom.

D. COVERAGE
CONTRACT EMPLOYEE
LABAJO V ALEJANDRO
165 SCRA 747
FELICIANO; September 26, 1988
NATURE
Petition for certiorari with preliminary injunction to review NLRC
resolution
FACTS
- The 6 private respondents had all been contracted by the
petitioners to work as classroom teachers at the San Andres HS,
a private learning institution situated in Maramag, Bukidnon.
They then filed a complaint before the Ministry of Labor and
Employment, alleging that they had each received a letter from
petitioner Fr. Labajo, Director of the San Andres High School
which contained: Please be informed that your service at the
San Andres High School will be terminated effective March 31,
1985.Thank you for all services you have rendered to the
school. Thus, their dismissal was without justifiable cause and
violated their rights to due process and security of tenure.
Petitioners Claims
> It was admitted that they had not paid in full the employment
benefits claimed by the teachers. It was alleged, however, that
private respondents, prior to their acceptance of teaching jobs
at the San Andres High School, "were already made aware that
the school could not give them everything due them under
existing laws" and, hence, were estopped from claiming such
benefits.
> At time of their dismissal, they were merely probationary
employees of the San Andres HS whose services were
terminated for just cause (upon expiration on 31 March 1985 of
their respective contracts and before any of them had achieved
regular or permanent status in their jobs.)
* Labor Arbiter ruled in favor of the teachers. It held that they
were not probationary employees, and that they could only be
dismissed for cause and only after having been accorded due
process.
* NLRC affirmed Labor Arbiters decision.

Labor Law 1
ISSUE
WON the respondents were illegally dismissed
HELD
NO
Ratio As probationary and contractual employees, private
respondents enjoyed security of tenure, but only to a limited
extent i.e., they remained secure in their employment during
the period of time their respective contracts of employment
remained in effect. As petitioners were not under obligation to
renew those contracts of employment, the separation of private
respondents in this case cannot be said to have been without
justifiable cause, much less illegal.
Reasoning
- Par 75 of the Manual of Regulations for Private Schools is
applicable in this case: Full-time teachers who have rendered
three years of satisfactory service shall be considered
permanent. This 3-year period is the maximum period or upper
limit of probationary employment allowed. Whether or not one
has indeed attained permanent status in one's employment,
before the passage of 3 years, is a matter of proof.
- NONE of them had been able to accumulate at least 3 years of
service with the San Andres HS at the time of their separation.
- Private respondent AMAR argued that the 12 years of teaching
experience he had accumulated prior to his acceptance of
employment at San Andres qualified him as a regular employee
thereof. This is not persuasive since it is the length of time Mr.
Amar has been teaching at San Andres that is material in
determining whether or not he in fact qualified as a regular
employee.
- Respondent ALEJANDRO asserted that her appointment as
"Night Principal" after having served a year thereat as a nonregular full-time teacher amounted to a promotion which
raised her status to that of a regular employee. This is also not
persuasive because mere appointment as "Night Principal" is
not, by itself and absent any additional evidence, sufficient
proof that her employment status had in fact been upgraded
from probationary to regular.
- The contracts of employment entered into by the San Andres
HS separately with each of the respondents stipulated, among
others: (a) that employment of the individual concerned took
effect at the beginning of the school year, or sometime in the
month of June; and (b) that payment of that individual's salary
would be made "every month for 10 months." We read these
stipulations together to mean that such contracts each had an
effective term of ten (10) months, i.e., from June until either
March or April of the following year. New contracts for another
period of ten months were negotiated between them at the
beginning of each school year. It does not appear from the
record or from the stipulations in those contracts, however, that
renewal was obligatory upon either party.
- Private respondents claimed that Fr. Labajos allegedly
"unusual antedated letter of termination" did not sufficiently
inform them of the reasons for their dismissal, nor did it satisfy
the due process requirements in termination cases. These
contentions ignore the fact that their employment was on a
contractual basis and for a stipulated period of time.
- The use of the word "terminated" was inept and unfortunate
but need not preclude recognition of the real nature of that
letter. Such letter was either a formal reminder that their
contracts were due to expire OR advance notice that such
contracts would no longer be renewed for the next school year
OR both. Assuming that prior notice of expiration of the
contractual term was necessary in this case, we consider that
Fr. Labajo's letter substantially complied with that requirement.
* Since the six (6) private respondents were not illegally
dismissed, the twin remedies of reinstatement and backwages
are not available to them. Dispositive NLRC Resolution is SET
ASIDE, except for the portion directing petitioners to pay
P52,173.67 in favor of private respondents.

PROBATIONARY EMPLOYEE

A2010

- 191 -

Disini

SKILLWORD MANAGEMENT AND MARKETING


CORPORATION V NLRC (MANUEL)
186 SCRA 465
MEDIALDEA; June 13, 1990
NATURE
Petition for certiorari
FACTS
- On June 24, 1983, Francisco Manuel was deployed to Saudi
Arabia to work as driver by petitioner Skillworld Management
and Marketing, a duly licensed recruitment agency operated by
petitioners-spouses Serafin and Alicia Ramos. Upon his arrival in
Jeddah, Manuel signed a 2-year employment contract with his
foreign employer, petitioner Shary Limousine for a monthly
basic salary of $300. 2 months later, Manuel was repatriated to
the Philippines. Upon his arrival in the Philippines, Manuel
confronted the Ramoses who promised to deploy him to other
projects.
- After the lapse of more than one year without being deployed
to other projects of petitioners, Manuel filed a complaint with
the POEA against petitioners for illegal dismissal. He alleged
that while he was employed as driver of Shary Limousine in its
branch at Jeddah he was stopped, and his driver's license
sought for inspection, by Saudi Arabian police. He showed the
police two documents given to him by his employer, Shary
Limousine who made him believe that these pertained to a
driver's temporary license. However, Manuel was informed that
the documents were not valid for a drivers license. Together
with eleven other drivers, they brought the matter before their
superiors. Three days after bringing the matter to his superior,
respondent was ordered to pack his things. He was taken to
Riyadh and from there, repatriated to the Philippines. Upon
respondent's arrival in the Philippines, he requested the
Ministry of Foreign Affairs for a translation of what purported to
be his driver's license. When translated it was only a
certification of employment with Shary Limousine in its branch
at Jeddah.
- Petitioners alleged that Manuels dismissal was for a valid and
just cause. Petitioners alleged that Manuel was dismissed
because of disobedience, absenteeism, refusal to work and
banding together to engage in concerted activities against the
employer.
POEA rendered judgment in favor of Manuel, directing
petitioners to pay him $6,900.00 or its peso equivalent. Upon
appeal, the NLRC affirmed said decision.
- According to petitioners, because of the probationary status
of the employment of Manuel, he may be dismissed at any
time. Furthermore, this agreement was contained in paragraph
four (4) of the employment contract signed by Manuel.
ISSUE
WON Manuel was illegally dismissed
HELD
YES
- There is no dispute that as a probationary employee, Manuel
had but a limited tenure. Although on probationary basis,
however, he still enjoys the constitutional protection on security
of tenure. During his tenure of employment therefore, or before
his contract expires, he cannot be removed except for cause as
provided for by law.
- The alleged causes for which private respondent was
dismissed (disobedience, absenteeism, refusal to work, etc.)
were not established. Respondent NLRC found that the
purported temporary licenses to drive issued to Manuel and his
co-drivers by their employer-the Shary Rent a Car/Limousine,
turned out to be mere certifications to the effect that they are
Filipino citizens who are holders of given passport numbers and
that they were sent to work with the Shary Limousine Branch in
Jeddah. It is for this reason that after being accosted twice at

Labor Law 1
checkpoints by Saudi police, who informed complainant and his
co-drivers that the alleged temporary licenses were not valid,
they brought the matter first to their Lebanese superior and
then to the Philippine Embassy. - - Further, records show that
Manuel reported for work regularly and even rendered regular
overtime services; that he did not even attempt to join a strike
or any other form of mass action while working in Jeddah,
because he knew that the laws in Jeddah are very strict and
being a foreigner he did not have the courage to join much less
lead a strike which is prohibited there; that he and his coworkers merely inquired from the Philippine Embassy why they
were allowed to drive without licenses; and that their action
prompted the Philippine Embassy to write their employer, which
is perfectly in order as it was designed to protect them in
foreign soil.

MANAGERIAL EMPLOYEE
INTERORIENT MARITIME ENTERPRISES INC V NLRC
(TAYONG)
235 SCRA 268
FELICIANO; August 11, 1994
NATURE
PETITION for reviewof a decision of the National Labor Relations
Commission
FACTS
- Captain Rizalino Tayong, a licensed Master Mariner with
experience in commanding ocean-going vessels, was employed
on 1989 by petitioners for 1 yr as stated in his employment
contract. He assumed command of petitioners vessel at the
port of Hongkong. His instructions were to replenish bunker and
diesel fuel, to sail to South Africa and there to load 120,000
metric tons of coal. However, while in HK and unwarding cargo,
he received a weather report that a storm would hit HK, so
precautionary measures were taken to secure the safety of the
vessel and its crew, considering that the vessels turbocharger was leaking and the vessel was 14 yrs old. He
also followed-up the requisition by the former captain for
supplies of oxygen and acetylene necessary for the weldingrepair of the turbo-charger and the economizer.
-The vessel then sailed from HK for Singapore. Captain Tayong
reported a water leak from M.E. Turbo Chapter No. 2
Exhaust gas casing so he was instructed to black off the
cooling water and maintain reduced RPM unless
authorized by the owners. However, the vessel stopped in
mid-ocean for 6 hrs and 45 minutes due to a leaking
economizer. He was instructed to shut down the economizer
and use the auxiliary boiler instead.
- The Chief Engineer reminded Captain Tayong that the oxygen
and acetylene supplies had not been delivered. He then
informed the shipowner that the departure of the vessel for
South Africa may be affected because of the delay in the
delivery of the supplies. The shipowner advised Captain Tayong
to contact its technical director who would provide a solution for
the supply of said oxygen and acetylene. The technical director
recommended to Captain Tayong that by shutting off the water
to the turbo charger and using the auxiliary boiler, there should
be no further problem. Captain Tayong agreed to the
recommendation of the technical director, but communicated
his reservations regarding proceeding to South Africa without
the requested supplies. So the shipowner advised him to wait
for the supplies.
- Finally, the vessel arrived at South Africa. However, Captain
Tayong was instructed to turn-over his post to the new captain,
and was repatriated to the Philippines after serving petitioners
for around 2 wks. He was not informed of the charges against
him, and was just sent a letter after arriving in the Philippines.
He therefore instituted a complaint for illegal dismissal before

A2010

- 192 -

Disini

the POEA, claiming his unpaid salary for the unexpired portion
of the written employment contract, plus attorneys fees.
- POEA: dismissed complaint, there was valid cause for his
untimely repatriation (the company alleged that due to Captain
Tayongs refusal to sail immediately to South Africa, the vessel
was placed off-hire by the charterers, and the charterers
refused to pay the charter hire or compensation corresponding
to 12 hours, amounting to US $15,500.00.They fired Captain
Tayong for lost of confidence; POEA believed that the Captains
concern for the oxygen and acetylene was not legitimate as
these supplies were not necessary or indispensable for running
the vessel.)
- NLRC: reversed and set aside POEA decision because Captain
Tayong had not been afforded an opportunity to be heard and
that no substantial evidenced was adduced to establish the
basis for petitioners loss of trust or confidence. Captain had
acted in accordance with his duties to maintain the
seaworthiness of the vessel and to insure the safety of the ship
and crew.
ISSUE
WON Captain Tayong was arbitrarily dismissed and without
cause as reasonably established in an appropriate investigation
(whether or not Captain Tayong had reasonable grounds
to believe that the safety of the vessel and the crew
under his command or the possibility of substantial
delay at sea required him to wait for the delivery of the
supplies needed for the repair of the turbo-charger and
the economizer before embarking on the long voyage
from Singapore to South Africa)
HELD
YES
Ratio It is well settled in this jurisdiction that confidential
and managerial employees cannot be arbitrarily
dismissed at any time, and without cause as reasonably
established in an appropriate investigation. Such
employees, too, are entitled to security of tenure, fair
standards of employment and the protection of labor
laws.
Reasoning
- Captain Tayong was denied any opportunity to defend himself.
Petitioners curtly dismissed him from his command and
summarily ordered his repatriation to the Philippines without
informing him of the charge or charges against him, and much
less giving him a chance to refute any such charge. In fact, it
was only 2 months after his repatriation that Captain Tayong
received a telegram dated 24 October 1989 from Inter-Orient
requiring him to explain why he delayed sailing to South Africa.
- NLRCs conclusion was supported by substantial evidence: The
official report of the technical director, which stated that a
disruption in the normal functioning of the vessels turbo
charger and economizer had prevented the full or regular
operation of the vessel and that he was the one who
recommended the reduction of RPM during the voyage to South
Africa instead of waiting in Singapore for the supplies that
would permit shipboard repair of the malfunctioning machinery
and equipment, supported NLRCs conclusion that Captain
Tayong did not arbitrarily and maliciously delay the voyage to
South Africa.
- Captain Tayong's decision (arrived at after consultation with
the vessel's Chief Engineer) to wait seven (7) hours in
Singapore for the delivery on board the Oceanic Mindoro of the
requisitioned supplies needed for the welding-repair, on board
the ship, of the turbo-charger and the economizer equipment of
the vessel, did not constitute merely arbitrary, capricious or
grossly insubordinate behavior on his part. In the view of the
NLRC, that decision of Captain Tayong did not constitute a legal
basis for the summary dismissal of Captain Tayong and for
termination of his contract with petitioners prior to the
expiration of the term thereof.
Obiter
- The captain of a vessel is a confidential and managerial
employee within the meaning of the above doctrine. A

Labor Law 1
master or captain, for purposes of maritime commerce, is one
who has command of a vessel. A captain commonly performs
three (3) distinct roles: (1) he is a general agent of the
shipowner; (2) he is also commander and technical director of
the vessel; and (3) he is a representative of the country under
whose flag he navigates. Of these roles, by far the most
important is the role performed by the captain as commander
of the vessel; for such role (which, to our mind, is analogous to
that of "Chief Executive Officer" [CEO] of a present-day
corporate enterprise) has to do with the operation and
preservation of the vessel during its voyage and the protection
of the passengers (if any) and crew and cargo. In his role as
general agent of the shipowner, the captain has authority to
sign bills of lading, carry goods aboard and deal with the freight
earned, agree upon rates and decide whether to take cargo.
The ship captain, as agent of the shipowner, has legal
authority to enter into contracts with respect to the
vessel and the trading of the vessel, subject to
applicable limitations established by statute, contract or
instructions and regulations of the shipowner. To the
captain is committed the governance, care and
management of the vessel. Clearly, the captain is vested
with both management and fiduciary functions.
- Indeed, if the ship captain is convinced, as a
reasonably prudent and competent mariner acting in
good faith that the shipowner's or ship agent's
instructions (insisted upon by radio or telefax from their
officers thousand of miles away) will result, in the very
specific
circumstances
facing
him,
in
imposing
unacceptable risks of loss or serious danger to ship or
crew, he cannot casually seek absolution from his
responsibility, if a marine casualty occurs, in such
instructions. 23
- Compagnie de Commerce v. Hamburg: xxx where by the force
of circumstances, a man has the duty cast upon him of taking
some action for another, and under that obligation adopts a
course which, to the judgment of a wise and prudent man, is
apparently the best for the interest of the persons for whom he
acts in a given emergency, it may properly be said of the
course so taken that it was in a mercantile sense necessary to
take it."
- ON management prerogative: that prerogative is nevertheless
not to be exercised, in the case at bar, at the cost of loss of
Captain Tayong's rights under his contract with petitioner's and
under Philippine law.
Disposition petitioners having failed to show grave abuse of
discretion amounting to loss or excess of jurisdiction on the part
of the NLRC in rendering its assailed decision, the Petition for
Certiorari is hereby DISMISSED, for lack of merit. Costs against
petitioners

E. MANAGEMENT RIGHTS AND SECURITY


OF TENURE
MANAGEMENT RIGHTS AND SECURITY
OF TENURE
COLEGIO DE SAN JUAN DE LETRAN V ASSN OF
EMPLOYEES AND FACULTY OF LETRAN
340 SCRA 587
KAPUNAN; September 18, 2000
NATURE
Petition for review on certiorari
FACTS
- Private respondent Ambas, the newly elected president of the
Association of Employees and Faculty of Letran (Union) wanted
to continue the renegotiation of its CBA with petitioner Colegio

A2010

- 193 -

Disini

de San Juan de Letran (Letran) for the last 2 years of the CBAs
5 year lifetime. However, petitioner claimed the CBA was
already prepared for signing by the parties. The CBA was
submitted to a referendum by the union members, who rejected
it.
- Petitioner accused the union officers of bargaining in bad faith
before the NLRC which decided in favor of petitioner but was
later reversed on appeal with the NLRC.
- The Union notified the National Conciliation and Mediation
Board (NCMB) of its intention to strike on the grounds of
petitioners refusal to bargain. Later, the parties agreed to
disregard the unsigned CBA and start negotiating a new 5 year
CBA for which the Union submitted its proposals. Ambas
protested a recent changing of her schedule and petitioner sent
the Union a letter dismissing Ambas for alleged insubordination
after which the Union amended its notice of strike to include the
said dismissal.
- Both parties again discussed the ground rules for the CBA
renegotiation but petitioner stopped the negotiations after
purportedly receiving information that a new group of
employees (ACEC) filed a petition for certification election,
giving rise to the issue of majority representation of the
employees.
- The Union finally went on strike and the Sec. of Labor and
Employment assumed jurisdiction, ordering those on strike to
return to work and for petitioner to accept them under the same
terms before the strike. All were readmitted except Ambas. The
Sec. issued an order declaring petitioner guilty of unfair labor
practice and directing the reinstatement of Ambas with
backwages. Letrans MFR was denied and the CA affirmed the
Sec.s decision, hence this petition.
ISSUES
1. WON petitioner is guilty of unfair labor practice by refusing
to bargain with the union
2.
WON the termination of the Ambas amounts to an
interference of the employees right to self-organization
HELD
1. YES
- Petitioner is guilty of unfair labor practice by its stern refusal
to bargain in good faith with respondent union.
- Article 252 defines collective bargaining as the performance of
a mutual obligation to meet and convene promptly and
expeditiously in good faith for the purpose of negotiating an
agreement. The Union, in sending its proposals during the 2nd
CBA negotiations, kept up its end of the bargain while Letran
devised ways and means to prevent the negotiation.
- Letran also failed to make a timely reply to the Unions
proposals (no counter-proposal a month later), violating Article
250 which requires such a reply within 10 days upon receipt of
a written notice of said proposals. Letrans refusal to reply is an
indication of bad faith, showing a lack of sincere desire to
negotiate.
- In a last ditch effort, Letran suspended the bargaining process
on the ground that it allegedly received information that ACEC
had filed a petition for certification election. The mere filing of a
petition for certification election does not ipso facto justify the
suspension of negotiations when there is no legitimate
representation issue raised; also, such an action for intervention
had already prescribed.
2. YES
- While we recognize the right of the employer to terminate the
services of an employee for just cause, the dismissal of
employees must be made within the parameters of law and
pursuant to the tenets of equity and fair play and must be
exercised in good faith. It must not amount to interfering with,
restraining or coercing employees in the exercise of their right
to self-organization as it would amount to unlawful labor
practice under Article 248.
-It would appear that Letran terminated Ambas in order to strip
the union of a leader who would fight for her co-workers rights
at the bargaining table and frustrate their desire to form a new
CBA. The charge of insubordination was a mere ploy to give a

Labor Law 1
color of legality to the action to dismiss her. Management may
have the prerogative to discipline its employees for
insubordination but when it interferes with employees right to
self-organization, it amounts to union-busting which is a
prohibited act.
Disposition petition is DENIED for lack of merit

SAN MIGUEL BREWERY SALES FORCE UNION V


OPLE
170 SCRA 25
GRIO-AQUINO; February 8, 1989
FACTS
- A collective bargaining agreement was entered into by
petitioner San Miguel Corporation Sales Force Union and the
private respondent, San Miguel Corporation. One provision of
the CBA was employees within the appropriate bargaining unit
shall be entitled to a basic monthly compensation plus
commission based on their respective sales."
- Few months after the said CBA, the company introduced a
marketing scheme known as the "Complementary Distribution
System" (CDS) whereby its beer products were offered for sale
directly to wholesalers through San Miguel's sales offices.
- The labor union filed a complaint for unfair labor practice in
the Ministry of Labor, with a notice of strike on the ground that
the CDS was contrary to the existing marketing scheme
whereby the Route Salesmen were assigned specific territories
within which to sell their stocks of beer, and wholesalers had to
buy beer products from them, not from the company. It was
alleged that the new marketing scheme violates Section 1,
Article IV of the collective bargaining agreement because the
introduction of the CDS would reduce the take-home pay of the
salesmen and their truck helpers for the company would be
unfairly competing with them.
ISSUES
1. WON the CDS violates the collective bargaining agreement
2. WON it is an indirect way of busting the union
HELD
1. NO
- CDS is a valid exercise of management prerogatives:
Ratio Except as limited by special laws, an employer is free to
regulate, according to his own discretion and judgment, all
aspects of employment, including hiring, work assignments,
working methods, time, place and manner of work, tools to be
used, processes to be followed, supervision of workers, working
regulations, transfer of employees, work supervision, lay-off of
workers and the discipline, dismissal and recall of work.
- So long as a company's management prerogatives are
exercised in good faith for the advancement of the employer's
interest and not for the purpose of defeating or circumventing
the rights of the employees under special laws or under valid
agreements, this Court will uphold them
2. NO
Ratio Nothing in the record as to suggest that the unilateral
action of the employer in inaugurating the new sales scheme
was designed to discourage union organization or diminish its
influence, but rather it is undisputable that the establishment of
such scheme was part of its overall plan to improve efficiency
and economy and at the same time gain profit to the highest.
While it may be admitted that the introduction of new sales plan
somewhat disturbed the present set-up, the change however
was too insignificant as to convince this Office to interpret that
the innovation interferred with the worker's right to selforganization.
Reasoning
- Petitioner failed to consider is the fact that corollary to the
adoption of the assailed marketing technique is the effort of the
company to compensate whatever loss the workers may suffer

A2010

Disini

- 194 -

because of the new plan over and above than what has been
provided in the collective bargaining agreement. To us, this is
one indication that the action of the management is devoid of
any anti-union hues."
Disposition Dismissed

F.
GUIDELINE
PENALTIES

ON

IMPOSITION

OF

VALIAO V CA
[PAGE 11]
FARROL V CA (RCPI)
325 SCRA 331
YNARES-SANTIAGA; February 10, 2000
FACTS
- Wenifrado Farrol was the station cashier of RCPI Cotabato City
Station.
- There was a P50K cash shortage in the branchs Peragram
Petty Cash Funds. Farrol was required to explain the cash
shortage. He paid to P25K to RCPI
- He was then required to explain why he should not be
dismissed. Petitioner wrote to the Field Auditor stating that the
missing funds were used for the payment of the retirement
benefits earlier referred by the Branch Manager and that he
already paid P25k. After he made 2 more payments of the cash
shortage, he was placed under preventive suspensions. He still
made 2 payments of the balance.
- RCPI then sent Farrol a letter informing him of the termination
of his services for alleging that part of the cash shortage was
used for payment of salaries and retirement benefits, disregard
of
policies
involving
statistical
reports,
malversation/misappropriation (which is a ground for dismissal),
and loss of trust and confidence.
- Unaware of the termination letter, he requested his
reinstatement since his preventive suspension had expired.
Ferrol even manifested his willingness to settle the case. RCPI
informed him that his employment had already been
terminated. The conflict was sent to the grievance committee.
Two years later, it was submitted for voluntary arbitration.
- VA ruled in favor of Farrol. RCPI filed a petition for certiorari
before the CA which reversed VA decision. CA also dismissed
MFR.
- Farrol now filed a petition for review on certiorari on the
ground that his dismissal was illegal because he was not
afforded due process and that he cannot be held liable for the
loss of trust and confidence reposed in him.
ISSUE
WON he was illegally terminated
HELD
YES
- BOP resides on the employer to prove that there was valid
cause for dismissal, and that he was afforded the opportunity to
be heard and defend himself.
- For the 1st notice, RCPI required petitioner to explain why he
failed to account for the shortage. The 2nd notice was that
informing Farrol of his termination. it does not clearly cite the
reasons for dismissal, nor were there facts and circumstances in
support thereof.
- Even assuming there was a breach of trust and confidence,
there was no evidence that Farrol was a managerial employee.
The term trust and confidence is restricted to managerial
employees.
- RCPI alleges that under its rules, petitioners infarction is
punishable by dismissal. However, employers rules cannot
preclude the state from inquiring whether strict and rigid
application or interpretation would be too harsh to the

Labor Law 1
employee. This is Farrols 1st offense, to which the Court holds
that dismissal is too harsh and grossly disproportionate.
Disposition CA is REVERSED and SET ASIDE and new one
entered REINSTATING the decision of the Voluntary Arbitrator
subject to the MODIFICATION that petitioners separation pay be
recomputed to include the period within which backwages are
due. For this purpose, this case is REMANDED to the Voluntary
Arbitrator for proper computation of backwages, separation
pay, 13th month pay, sick leave conversion and vacation leave
conversion.

VH MANUFACTURING INC V NLRC (GAMIDO)


322 SCRA 417
DE LEON; January 19, 2000
NATURE
Before us is a petition for certiorari
FACTS
- Since November 5, 1985 Gamido was employed in VH
Manufacturings business of manufacturing liquefied petroleum
gas (LPG) cylinders. He served as a quality control inspector
with the principal duty of inspecting LPG cylinders for any
possible defects. His service with the company was abruptly
interrupted on February 14, 1995, when he was served a notice
of termination of his employment.
- His dismissal stemmed from an incident on February 10, 1995
wherein VHs company President, Alejandro Dy Juanco,
allegedly caught private Gamido sleeping on the job. On that
same day, private respondent was asked through a written
notice from the petitioners Personnel Department to explain
within twenty-four (24) hours why no disciplinary action should
be taken against him for his violation of Company Rule 15-b
which provides for a penalty of separation for sleeping during
working hours. Without delay, private respondent replied in a
letter which reads:
"Sir, ipagpaumanhin po ninyo kung
nakapikit ako sa aking puwesto dahil hinihintay ko po ang niliha
hi Abreu para i quality pasensiya na po kung hindi ko po
namalayan ang pagdaan ninyo dahil maingay po ang painting
booth."
Notwithstanding his foregoing reply, he was
terminated.
- Feeling aggrieved, he filed a complaint for illegal dismissal,
praying for reinstatement to his position as quality control
inspector. Labor Arbiter declared that Gamidos dismissal is
anchored on a valid and just cause.
NLRC reversed the
decision.
ISSUE
WON Gamidos dismissal was too harsh a penaltly for his
violation of company rule 15-b
HELD
YES
- Basically, the reason cited for the dismissal of private
respondent is sleeping on the job in violation of Company Rule
15-b. But according to Gamido, he was not sleeping on the job
but was merely idle, waiting for the next cylinder to be checked.
- In view of the gravity of the penalty of separation, as provided
by the Company Rules and Regulation., in termination disputes,
the burden of proof is always on the employer to prove that the
dismissal was for a just and valid cause. What is at stake here is
not only the job itself of the employee but also his regular
income therefrom which is the means of livelihood of his family.
- A thorough review of the record discloses that, contrary to the
findings of the Labor Arbiter, petitioners claim that private

A2010

- 195 -

Disini

respondent slept on the job was not substantiated by any


convincing evidence other than the bare allegation of the
officer.
- Next, VHs reliance on the authorities it cited that sleeping on
the job is always a valid ground for dismissal, is misplaced. The
authorities cited involved security guards whose duty
necessitates that they be awake and watchful at all times
inasmuch as their function, to use the words in Luzon
Stevedoring Corp. v. Court of Industrial Relations, is "to protect
the company from pilferage or loss." Accordingly, the doctrine
laid down in those cases is not applicable to the case at bar.
- Finally, while an employer enjoys a wide latitude of discretion
in the promulgation of policies, rules and regulations on workrelated activities of the employees, those directives, however,
must always be fair and reasonable, and the corresponding
penalties, when prescribed, must be commensurate to the
offense involved and to the degree of the infraction. In the case
at bar, the dismissal meted out on private respondent for
allegedly sleeping on the job, under the attendant
circumstances, appears to be too harsh a penalty, considering
that he was being held liable for first time, after nine 9 of
unblemished service, for an alleged offense which caused no
prejudice to the employer, aside from absence of substantiation
of the alleged offense. Neither was it shown that private
respondents alleged negligence or neglect of duty, if any, was
gross and habitual. Thus, reinstatement is just and proper.
Disposition petition is hereby DISMISSED, and the challenged
Decision and Order of public respondent NLRC are AFFIRMED.

REYNO V MANILA ELECTRIC COMPANY


434 SCRA 660
SANDOVAL-GUTIERREZ; July 22, 2004
NATURE
Petition for review on certiorari under Rule 45 of the 1997 Rules
of Civil Procedure
FACTS
- Reyno was employed by MERALCO where he eventually
occupied the position of Assistant Squad Leader of Squad 12 at
the Inspection Department.
Petitioner and his team of
inspectors were in charge of monitoring and inspecting electric
meters installed at the premises of respondents customers;
ensuring the accuracy of the electric consumption recorded in
these meters; and reporting and apprehending violators who
use insidious schemes or devices to reduce their electric
consumption deliberately.
- Later, MERALCO implemented an incentive scheme aimed at
encouraging its inspectors to perform their duties zealously.
Under this incentive scheme, the inspector concerned shall be
paid an additional 30-minute overtime pay for every submitted
report of major violation/s committed by customers against
respondent.
- Roger Sacdalan, Senior Investigator of respondents Special
Presidential Committee (SPC), received several complaints
against Gilbert Villapa, Leader of Squad 12, about an illegal
connection.
- SPC conducted an investigation wherein members of Squad 12
were summoned to explain. However, they failed to establish
Villapas involvement in such illegal connection. Instead, their
declarations pointed to Reynos irregular performance of his
duties.
- This prompted SPC to conduct clarificatory hearing. But the

Labor Law 1
hearing was cancelled for failure of Reynos counsel to appear
despite notice. When the case was called for hearing as
scheduled, his counsel again failed to appear. He then opted to
proceed with the clarificatory hearing without the assistance of
his counsel.
- After evaluating the records on hand, the SPC found petitioner
guilty of dishonesty, serious misconduct and willful breach of
trust. Respondent then sent petitioner a notice terminating his
services.
- Reyno filed with the Labor Arbiter a complaint for illegal
dismissal and payment of overtime pay, premium pay for
holidays and rest days, damages and attorneys fees.
ISSUES
1. WON Reyno was deprived of his right to cross examine
witnesses before the Labor Arbiter
2. WON Reyno was illegally dismissed
HELD
1. NO
- His right to cross-examine the three witnesses, did not err as
it was not required to apply strictly the Rules of Evidence. At
any rate, MERALCO had valid reasons why it did not present
those three witnesses during the proceedings before the Labor
Arbiter
2. NO
- The standard of substantial evidence is satisfied where the
employer, as in this case, has reasonable ground to believe that
the employee is responsible for the misconduct and his
participation therein renders him unworthy of trust and
confidence demanded by his position. Reyno violated
MERALCOs Code of Employee Discipline and committed serious
misconduct in the performance of his duties have been proved
by the affidavits of petitioners own subordinates in Squad 12 of
which he was the Assistant Squad Leader. Moreover, MERALCO
had lost his trust and confidence in petitioner. Under Article
282 of the Labor Code, as amended, these are just causes for
his dismissal from the service.
- The longer an employee stays in the service of the company,
the greater is his responsibility for knowledge and compliance
with the norms of conduct and the code of discipline in the
company.
- An employees length of service with the company even
aggravates his offense. He should have been more loyal to
company from which he has derived his family bread and butter
for seventeen (17) years.
Disposition Petition is DENIED. The assailed Decision dated
January 17, 2001 and Resolution dated May 3, 2001 of the Court
of Appeals in CA-G.R. SP No. 53987 are hereby AFFIRMED.

FACTORS
ASSOCIATED LABOR UNION V NLRC
[PAGE 181]

PHILIPPINE LONG DISTANCE TELEPHONE V NLRC


(GABRIEL)
303 SCRA 9
QUISUMBING; February 11, 1999
NATURE
Appeal from the order of the NLRC
FACTS
- Private respondent, Enrique Gabriel, was foreman of petitioner
PLDT and was a supervisor with territorial responsibility for
Camp Crames First to 20th Avenue and portions of Project 4, all
located in Quezon City. On two occasions (September 5, 1989
and October 16, 1989) he ordered Medel Mercado and Juancho

A2010

- 196 -

Disini

Jocson to install two telephone lines each at Unit R, Facilities


Center Building, located at Shaw Boulevard, Mandaluyong.
- The ordered installations were investigated because (a) the
Facilities Center Building had no entrance cable facilities or
conduit wires for telephone connection, (b) Mandaluyong was
not within Gabriels area of jurisdiction, and (c) installers
Mercado and Jocson were not under his direct supervision.
- During the investigation, Gabriel. while acknowledging
responsibility for his action, claimed that his actuation was
motivated by the desire to provide customer satisfaction. He
also claimed that the telephones were installed after the
documents of approval were issued by PLDT. He dismissed from
service on September 3, 1990 on the ground that he committed
grave misconduct, breach of trust, and violations of company
rules and regulations.
- Gabriel filed an illegal dismissal complaint with the Labor
Arbiter on September 6, 1990. Said Arbiter affirmed the
dismissal but the same was reversed by the NLRC and ordered
PLDT to reinstate Gabriel to the position he held as at the time
of the complained dismissal, with full backwages, benefits, and
proportionate privileges. Hence the appeal.
ISSUE
WON Gabriel is guilty of serious misconduct and/or breach of
trust anent the irregular installation of the telephones
HELD
NO
- The facts of the case do not point to any misconduct or breach
of trust on the part of Gabriel. There was also no provision in
the written rule of PLDT which penalizes unwarranted
installation of telephone lines with dismissal. In any case, the
installations were approved by the company. There was also no
evidence that Gabriel profited personally with the transaction.
The dismissal of Gabriel is illegal.
Reasoning
- Dismissal is the ultimate penalty and should not be imposed if
the employee has been in service for a considerable length of
time and has not been the recipient of any disciplinary actions.
Where a penalty less punitive would suffice, whatever missteps
may have been committed by the worker ought not to be
visited with a consequence so severe such as dismissal. This
interpretation gives meaning and substance to the liberal and
compassionate spirit of the law as provided for in Article 4 of
the Labor Code which states that all doubts in the
implementation and interpretation of the provisions of the Labor
Code including its implementing rules and regulations shall be
resolved in favor of labor.
- Gabriel is not entirely faultless. As a supervisor, he is required
to act judiciously and to exercise his authority in harmony with
PLDTs policies. When he jeopardized the status of the rank and
file employees whom he ordered to by-pass the standard
operating procedures of the company, to the detriment of his
employer, he was not entirely blameless. The irregularity
attributable to him could not be disregarded. He must not be
rewarded, in fairness to the employers own legitimate concerns
such as company morale and discipline.
Disposition the resolution f the NLRC is affirmed subject to the
deletion of the other awards of unspecified benefits and
proportionate privileges.

Labor Law 1
DISMISSAL AS PENALTY
CEBU FILVENEER CORPORATION V NLRC
(VILLAFLOR)
286 SCRA 556
PUNO; February 24, 1998
FACTS
- Villaflor was the chief accountant of CFC. The top execs were
Italians: Cordaro (president), Kun (GM), Marinoni (Production
manager). Guillermo was the accounting clerk of Villaflor.
- Kun resigned from the company and asked for the liquidation
of his investment: P125k. Two weeks later, he asked Guillermo
for a blank check and a blank check voucher. Guillermo gave
him. Three days later, Villlaflor noticed that a check voucher
was missing. She asked Guillermo, who said that Mr. Kun has it.
Villaflor immediately informed Mr. Cordaro of what
happened. She also wrote to the bank demanding the return of
the encashed check.
Marinoni charged Villaflor of complicity in Kuns irregular
disbursement of company funds. Two days later, she was
prevented entry to the office by the security guards. Her office
drawer and safe were also forcibly opened upon order of
Marinoni. Villaflor reported the incident to the PNP.
- Marinoni suspended her for 30 days without pay for failure to
come to work for half a day (the day she was prevented entry).
The next day she was preventively suspended for 30 days
pending investigation of her involvement in Kuns booboo. The
company also printed a newspaper ad for an accountant.
Villaflor filed for illegal dismissal with the LA. LA decided in
her favor. NLRC affirmed.
ISSUE
WON Villaflor was illegally dismissed
HELD
YES
- Due to its far reaching implications, our Labor Code decrees
that an employee cannot be dismissed, except for the most
serious causes. Article 282 enumerates the causes for which the
employer may terminate an employee.
- Company says its loss of trust. The SC said that Villaflors
omission cannot be described as willful to justify dismissal. A
breach is willful if it is done intentionally, knowingly and
purposely. Petitioners merely proved the omission of the private
respondent but there is no evidence whatsoever that it was
done intentionally.
Company says shes grossly or habitually negligent in the
performance of her duties. The SC said that since she has not
been remiss in the performance of her duties in the past, she
cant be charged with habitual negligence. Neither is her
negligence gross in character. Gross negligence implies a
want or absence of or failure to exercise slight care or
diligence or the entire absence of care. It evinces a
thoughtless disregard of consequences without exerting
any effort to avoid them. She had not the slightest reason to
distrust Kun because he was the GM and appears to have
conducted himself well in the performance of his duties in the
past. At most, its error of judgment, not gross negligence.
Disposition NLRC decision affirmed.

GOLDEN THREAD KNITTING INDUSTIRES V NLRC


(MACASPAC)
304 SCRA 720
BELLOSILLO; March 11, 1999
NATURE
Petition to review decision of NLRC
FACTS

A2010

- 197 -

Disini

- several employees of Golden Thread Knitting Industries (GTK)


were dismissed for different reasons. 2 employees were
allegedly for slashing the companys products (towels), 2 for
redundancy, 1 for threatening the personnel manager and
violating the company rules, and 1 for abandonment of work.
- The laborers filed complaints for illegal dismissal. They allege
that the company dismissed them in retaliation for establishing
and being members of the Labor Union.
GTK, on the other hand, contend that there were valid causes
for the terminations. The dismissals were allegedly a result of
the slashing of their products, rotation of work, which in turn
was caused by the low demand for their products, and
abandonment of work. WRT to the cases involving the slashing
of their products and threats to the personnel manager, the
dismissals were in effect a form of punishment.
- The labor arbiter ruled partially in favor of GTK. He said that
there was no showing that the dismissals were in retaliation for
establishing a union. He, however, awarded separation pay to
some employees.
- NLRC, however, appreciated the evidence differently. It held
that there was illegal dismissal and ordered reinstatement.
ISSUE
WON there was illegal dismissal
HELD
YES
Ratio Dismissal is the ultimate penalty that can be meted to
an employee. It must therefore be based on a clear and not on
an ambiguous or ambivalent ground.
Reasoning
- WRT to the case involving slashing of towels, the employees
were not given procedural due process. There was no notice
and hearing, only outright denial of their entry to the work
premises by the security guards. The charges of serious
misconduct were not sufficiently proved.
- WRT to the employees dismissed for redundancy, there was
also denial of procedural due process. Hearing and notice were
not observed. Thus, although the characterization of an
employees services is a management function, it must first be
proved with evidence, which was not done in this case. the
company cannot merely declare that it was overmanned.
- WRT to the employee dismissed for disrespect, the SC
believed the story version of the company (which essentially
said that the personnel manager was threatened upon mere
service of a suspension order to the employee), but ruled that
the dismissal could not be upheld.
the dismissal will not be upheld where it appears that the
employees act of disrespect was provoked by the employer.
xxx the employee hurled incentives at the personnel
manager because she was provoked by the baseless
suspension imposed on her. The penalty of dismissal must be
commensurate with the act, conduct, or omission to the
employee.
- The dismissal was too harsh a penalty; a suspension of 1 week
would have sufficed.
GTK exercised their authority to dismiss without due regard
to the provisions of the Labor Code. The right to terminate
should be utilized with extreme caution because its
immediate effect is to put an end to an employee's present
means of livelihood while its distant effect, upon a
subsequent finding of illegal dismissal, is just as pernicious to
the employer who will most likely be required to reinstate the
subject employee and grant him full back wages and other
benefits.
Disposition Decision AFFIRMED

CENTRAL PANGASINAN ELECTRIC COOP INC V


MACARAEG
395 SCRA 720
PUNO; January 22, 2003

Labor Law 1
NATURE
Petition for review on certiorari
FACTS
- De Vera was employed as teller and Geronima Macaraeg as
cashier by Central Pangasinan Electric cooperative inc. They
accommodated and encashed two hundred eleven crossed
checks of Evelyn Joy Estrada (de Veras sister) amounting to
P6,945,128.95 payable to the cooperative despite the absence
of any transaction or any outstanding obligation with it. They
credited the checks as part of their collection and deposited the
same together with their cash collection to the coops account
at the Rural Bank of Central Pangasinan.
- The finance department noticed these checks which bounced
(insufficient funds).De Vera and Macaraeg were confronted with
the discovery. De Vera admitted that the checks were issued
by her sister and that she encashed them from the money
collected from petitioners customers.
- De Vera testified and admitted that she encashed the checks
of Evelyn Joy Estrada because the latter is her older sister.
Macaraeg admitted that she knew of the accommodations given
by respondent de Vera to her sister; that she allowed her
subordinate to do it because respondent de Vera is her kumare,
and that she knew that Mrs. Estradas checks were sufficiently
funded.
- On March 19, 1999, on the basis of the findings and
recommendation of Atty. Fernandez (presided over the
hearing), the General Manager issued to respondents separate
notices of termination for serious misconduct, and breach of
trust and confidence reposed on them by management.
- Respondents questioned their dismissal before the National
Conciliation and Mediation Board (NCMB),claiming that their
dismissal was without just cause and in violation of the
Collective Bargaining Agreement (CBA), which requires that the
case should first be brought before a grievance committee.
Eventually, the parties agreed to submit the case to a voluntary
arbitrator for arbitration.
- LA-ruled in favor of defendants and ordered their
reinstatement
CA-affirmed
ISSUES
1. WON the procedure leading to the termination of
respondents Maribeth de Vera and Geronima Macaraeg was in
violation of the provisions of the CBA
2. WON the respondents were validly dismissed
HELD
1. Issue is moot and academic
- The parties active participation in the voluntary arbitration
proceedings, and their failure to insist that the case be
remanded to the grievance machinery, shows a clear intention
on their part to have the issue of respondents illegal dismissal
directly resolved by the voluntary arbitrator.
2. YES
- The respondents were validly dismissed. Article 282(c) of the
Labor Code allows an employer to dismiss employees for willful
breach of trust or loss of confidence. Proof beyond reasonable
doubt of their misconduct is not required, it being sufficient that
there is some basis for the same or that the employer has
reasonable ground to believe that they are responsible for the
misconduct and their participation therein rendered them
unworthy of the trust and confidence demanded of their
position.
Reasoning
- the acts of the respondents were clearly inimical to the
financial interest of the petitioner. During the investigation,
they admitted accommodating Evelyn Joy Estrada by encashing
her checks from its funds for more than a year. They did so
without petitioners knowledge, much less its permission.
- there was willful breach of trust on the respondents part, as
they took advantage of their highly sensitive positions to violate
their duties.

A2010

- 198 -

Disini

- the acts of the respondents caused damage to the petitioner.


During those times the checks were illegally encashed,
petitioner was not able to fully utilize the collections, primarily
in servicing its debts.
- it is not material that they did not misappropriate any
amount of money, nor incur any shortage relative to the funds
in their possession. The basic premise for dismissal on the
ground of loss of confidence is that the employees concerned
hold positions of trust. The betrayal of this trust is the essence
of the offence for which an employee is penalized.
- the respondents held positions of utmost trust and confidence.
As teller and cashier, respectively, they are expected to possess
a high degree of fidelity.
They are entrusted with a
considerable amount of cash. Respondent de Vera accepted
payments from petitioners consumers while respondent
Macaraeg received remittances for deposit at petitioners bank.
They did not live up to their duties and obligations.

PHILIPS SEMICONDUCTORS V FADRIQUELA


[PAGE 77]

G. RULES MANAGERIALS AND RANK AND


RANK FILE EMPLOYEES
SALVADOR V PHILIPPINE MINING SERVICE CORP
395 SCRA 729
PUNO; January 22, 2003
FACTS
- JOSE V. SALVADOR was first employed by respondent in 1981.
He rose from the ranks and assumed the position of Plant
Inspection Foreman in 1991. He was tasked to: (1) supervise
plant equipment and facility inspection; (2) confirm actual
defects; (3) establish inspection standards and frequency; (4)
analyze troubles and recommend counter measures; and (5)
prepare weekly/monthly inspection schedule.[3]
- As early as March 1, 1985, respondent instituted the shift
boss scheme whereby the foreman from the Plant Section and
the foreman from the Mining Section rotate as shift boss
throughout their night shift to oversee and supervise both the
mining and plant operations. The shift boss was entrusted with
the care, supervision and protection of the entire plant.
- Aside from his employment with respondent, petitioner coowned and managed LHO-TAB Enterprises, with his partner
Ondo Alcantara. They were engaged in the manufacture and
sale of hollow blocks. On September 29, 1997, petitioners
employment relation with respondent was tainted with charges
of pilferage and violation of company rules and policy, resulting
to loss of confidence. Respondents evidence disclose that on
September 29, 1997, at about 9:30 a.m., Koji Sawa,
respondents Assistant Resident Manager for Administration,
was on his way back to his office in the plant. He and his driver,
Roberto Gresones, saw petitioner operating respondents
payloader, scooping fine ore from the stockpile and loading it
on his private cargo truck. As the truck was blocking the access
road leading to the stockyards gate, Sawas car stopped near
the stockpile and the driver blew the horn thrice. Petitioner did
not hear him because of the noise emanating from his operation
of the payloader. Sawas driver found a chance to pass through
when the payloader maneuvered to get another scoop from the
fine ore stockpile.
- As it was contrary to respondents standard operating
procedure for the plant foreman to operate the payloader, Sawa
went to the administration office to check the delivery receipt
covering the loading operation of petitioner that morning.
However, sales-in-charge Eduardo Guangco was in the wharf,
overseeing the loading of respondents product. Hence, it was
only in the afternoon that Sawa was able to verify the delivery
receipt covering petitioners loading transaction. The delivery
receipt showed that it was dolomite spillage that was purchased

Labor Law 1
by buyer Ondo Alcantara, not the fine ore that he saw petitioner
loading on his truck. The receipt also showed it was not the
respondent but Alcantara, the buyer, who was responsible for
loading the spillage he purchased from the plant.
- On the basis of the foregoing facts PMSC terminated Salvador
for pilferage of company property. Labor Arbiter and NLRC ruled
in favor of Salvador but CA reversed. Hence, this recourse.
ISSUES
1.
WON the charge of pilferage against petitioner was
supported by substantial evidence to warrant his dismissal from
the service
2. WON the employer was well within its rights in imposing a
harsh penalty considering the length of the employees service
HELD
1. YES
Ratio The settled rule in administrative and quasi-judicial
proceedings is that proof beyond reasonable doubt is not
required in determining the legality of an employers dismissal
of an employee, and not even a preponderance of evidence is
necessary as substantial evidence is considered sufficient.
Substantial evidence is more than a mere scintilla of evidence
or relevant evidence as a reasonable mind might accept as
adequate to support a conclusion, even if other minds, equally
reasonable, might conceivably opine otherwise. Thus,
substantial evidence is the least demanding in the hierarchy of
evidence.
Reasoning
- The Labor Code provides that an employer may terminate the
services of an employee for just cause and this must be
supported by substantial evidence. In the case at bar, our
evaluation of the evidence of both parties indubitably shows
that petitioners dismissal for loss of trust and confidence was
duly supported by substantial evidence.
2. NO
Ratio As a general rule, employers are allowed wider latitude
of discretion in terminating the employment of managerial
employees as they perform functions which require the
employers full trust and confidence.
Reasoning
- To be sure, length of service is taken into consideration in
imposing the penalty to be meted an erring employee.
However, the case at bar involves dishonesty and pilferage by
petitioner which resulted in respondents loss of confidence in
him. Unlike other just causes for dismissal, trust in an
employee, once lost is difficult, if not impossible, to regain.
Moreover, petitioner was not an ordinary rank-and-file
employee. He occupied a high position of responsibility. As
foreman and shift boss, he had over-all control of the care,
supervision and operations of respondents entire plant. It
cannot be over-emphasized that there is no substitute for
honesty for sensitive positions which call for utmost trust.
Fairness dictates that respondent should not be allowed to
continue with the employment of petitioner who has breached
the confidence reposed on him.
- In the case at bar, respondent has every right to dismiss
petitioner, a managerial employee, for breach of trust and loss
of confidence as a measure of self-preservation against acts
patently inimical to its interests. Indeed, in cases of this nature,
the fact that petitioner has been employed with the respondent
for a long time, if to be considered at all, should be taken
against him, as his act of pilferage reflects a regrettable lack of
loyalty which he should have strengthened, instead of betrayed.
Disposition The petition is DENIED.

A2010

- 199 -

Disini

CAOILE V NLRC (COCA-COLA BOTTLERS,


PHILIPPINES INC)
299 SCRA 76
QUISUMBING; November 24, 1998
NATURE
Special action for certiorari
FACTS
- Private respondent CCBPI, through the local plant
management, contracted the services of Mr. Redempto de
Guzman for the installation of a Private Automatic Branch
Exchange (PABX) housewiring in the plant premises for the sum
of P65,000.00. Since the project fell under the direct
supervision of petitioner, all cash advances by the contractor
were coursed through him.
- Mr. De Guzman, the contractor, requested for an initial cash
advance of P10,000.00. Petitioner caused the preparation of
the Payment Request Memo in the amount of P15,000.00 and
the issuance of a check in the same amount. After securing the
endorsement of the contractor, petitioner encashed the check
with the plant teller Mr. Dominador S. Pila and handed over
P10,000.00 to Mr. De Guzman while retaining the amount of
P5,000.00 for himself.
- The contractor requested for second and third cash advances
in the amounts of P5,000.00 and P10,000.00 respectively. As in
the first cash advance, petitioner caused the preparation of 2
checks in the amounts of P10,000.00 and P15,000.00
respectively. After securing the endorsements of the contractor
the requested cash advances while retaining for himself the
difference of P10,000.00.
- After the project was completed, the contractor requested
payment of the balance of the contract price in the amount of
P25,000.00. Petitioner caused the issuance of a check in the
amount of P24,350.00 (after deducting 1% of the total contact
price by way of witholding tax). Petitioner secured the
endorsement of the contractor, encashed the check with the
teller, then handed over to the contractor only P19,350.00 while
retaining fore himself the amount of P5,000.00.
- Upon completion of an additional project requested of the
contractor, petitioner caused the issuance a check, and after
securing the endorsement of the contractor, petitioner
encashed the check and delivered P8,000.00 to the contractor
and retained P500.00 for himself.
- Mr. de Guzman executed an affidavit exposing the fraudulent
acts perpetrated by petitioner, which prompted the company to
conduct an investigation.
- Petitioner was served a Notice of investigation. During the
investigation, petitioner admitted that the initials in the check
vouchers were his but denied having encashed the checks and
delivering the cash payments to the contractor.
- It was established through the testimony of Mrs. Macasinag
and Mr. Pila that petitioner personally withdrew the checks from
the GM Secretary and had them encashed with the teller after
Mr.de Guzman has endorsed the same.
- Mr. Mariano A. Limjap, Senior VP and Administration Director
issued a memo sustaining the findings and recommendation of
the local plant management for the termination of complainant
from his employ on the grounds of grave misconduct and
dishonesty considering that his position as EDP Supervisor is
bestowed with the highest trust and confidence by the
respondent as may be seen from the description of his duties
and responsibilities.
- As a consequence of his dismissal, petitioner filed a compliant
for illegal dismissal with damages
- Labor Arbiter rendered a decision finding that petitioner was
illegally dismissed
- Private respondents appealed to NLRC which reversed the
Labor Arbiter's decision. NLRC held that petitioner committed

Labor Law 1
acts constituting a breach of trust and confidence reposed on
him by his employer, thereby justifying his dismissal.
ISSUE
WON the NLRC committed grave abuse of discretion amounting
to lack or excess of jurisdiction in reversing and setting aside
the Labor Arbiter's decision finding private respondents guilty of
illegal dismissal
HELD
NO
Ratio Law and jurisprudence have long recognized the right of
employers to dismiss employees by reason of loss of trust and
confidence. As provided for in the Labor Code, "Art. 282. An
employer may terminate an employment for any of the
following causes: x x x (c) Fraud or willful breach of the trust
reposed in him by his employer or his duly authorized
representative. x x x." In the case of supervisors or personnel
occupying positions of responsibility, this Court has repeatedly
held that loss of trust and confidence justifies termination.
Obviously, as a just cause provided by law, this ground for
terminating employment, springs from the voluntary or willful
act of the employee, or "by reason of some blameworthy act or
omission on the part of the employee".
Reasoning
- Loss of confidence as a just cause for termination of
employment is premised from the fact that an employee
concerned holds a position of trust and confidence. But, in order
to constitute a just cause for dismissal, the act complained of
must be "work-related" such as would show the employee
concerned to be unfit to continue working for the employer.
- it must be noted the recent decisions of this Court has
distinguished the treatment of managerial employees from that
of rank-and-file personnel, insofar as the application of the
doctrine of loss of trust and confidence is concerned. Thus
with respect to rank-and-file personnel, loss of trust and
confidence as ground for valid dismissal requires proof
of involvement in the alleged events in question, and
that mere uncorroborated assertion and accusations by
the employer will not be sufficient. But, as regards as a
managerial employee, mere existence of a basis for
believing that such employee has breached the trust of
his employer would suffice for his dismissal. Hence, in the
case of managerial employees, proof beyond reasonable doubt
is not required, it being sufficient that there is some basis for
such loss of confidence, such as when the employer has
reasonable ground to believe that the employee concerned is
responsible for the purported misconduct, and the nature of his
participation therein renders him unworthy of the trust and
confidence demanded by his position.
- In the present case, petitioner is not an ordinary rank-and-file
employee. He is the EDP Supervisor tasked to directly
supervise the installation of the PABX housewiring project in
respondent company's premises. He should have realized that
such sensitive position requires the full trust and confidence of
his employer. Corollary, he ought to know that his job requires
that he keep the trust and confidence bestowed on him by his
employer unsullied.
Disposition Petition is DISMISSED for lack of merit.

G. TERMINATION OF EMPLOYMENT BY
EMPLOYEE

14.02 CAUSES
A. JUST CAUSES

A2010 - 200 B. WITHOUT JUST CAUSE

Disini

C. RESIGNATION
DEFINITION
HABANA V NLRC (HOTEL NIKKO)
298 SCRA 537
KAPUNAN; November 16, 1998
NATURE
Petition for certiorari seeking reversal of NLRC decision which
affirmed LA
FACTS
- On March 16, 1989, petitioner Antonio Habana was employed
by Hotel Nikko Manila Garden (Nikko) as Rooms Division
Director (RDD). One of his tasks as RDD was to conduct regular
and surprise inspection of all work areas to ensure quality of
performance. In the course of his employment, petitioner
encountered several problems: his frequent clashes with
Dolores Samson (his Senior Rooms Mgr); frequent absence and
tardiness; rampant violations of hotel rules due to his failure to
effectively manage his own division; and complaints regarding
the overall quality (or lack thereof) of service of Nikko. As a
result, private respondent Mr. Okawa, who replaced private
respondent Mr. Yokoo as the executive asst. for Sales, issued a
memorandum instructing petitioner, along with 2 others, to
conduct and report daily inspection of the guestrooms and
public areas. Petitioner sent a memorandum of protest claiming
that Mr. Okawas orders was a form of harassment to ease him
out of his position and illustrated in detail the other forms of
alleged harassment supposedly perpetrated by Mr. Okawa.
He, however, manifested that he had no intention to resign.
- But on May 2, 1990, petitioner went to the Hotels Comptroller
asking for his severance pay of P120,000 plus accrued benefits
of P11, 865.28. The check was not given to him until he
submitted his resignation letter (part of standard procedure).
He also executed an Affidavit of Quitclaim, along with his
resignation. The very next day, however, respondents received
a letter from petitioner (addressed to Mr. Okawa) who insisted
that he was forced to resign because he could no longer endure
Mr. Okawas acts of harassment against him. 2 weeks later,
petitioner filed a complaint for illegal dismissal and damages
against Hotel Nikko and its officers, including his direct
superiors, Yokoo and Okawa. The LA dismissed the complaint
finding that petitioner voluntarily resigned and that the alleged
acts of harassment were non-existent. On appeal, the NLRC
affirmed the LAs decision likewise finding that petitioner
voluntarily resigned as manifested by his act of negotiating for
a huge amount of separation pay. When his MFR was dismissed,
he came to the SC.
ISSUE
WON the resignation was forced upon Habana or he did so
voluntarily
HELD
The resignation was voluntary.
Ratio Voluntary resignation is the voluntary act of an employee
who finds himself in a situation where he believes that
personal reasons cannot be sacrificed in favor of the exigency
of the service and he has no other choice but to disassociate
himself from his employment.
Reasoning
- In this case, petitioner was clearly having trouble performing
his job, which undeniably carries immense responsibilities.
Notable too was petitioners failure to see eye to eye with his
immediate bosses, Mr. Yokoo and Mr. Okawa. Because of these
difficulties, it was quite reasonable for petitioner to think of, and

Labor Law 1
eventually, relinquishing his position voluntarily (and get a fat
sum as severance pay in the bargain) instead of waiting to be
fired.
- Petitioner laments that he was completely stripped of his
powers and functions as Director when Mr. Okawa tasked him
with inspecting the hotels guest and public areas. Conducting
these daily inspections, in effect, demoted him to a mere room
inspector one notch higher than a bellboy. He claims that the
humiliation he endured in going room to room, inspecting toilets
and garbage areas, was all part of a malicious scheme to harass
him out of his position. These orders were not borne out of
mere whim and caprice. They were made in response to the
complaints they were getting. Moreover, these measures
executed by the hotels top management were legitimate
exercise of management prerogatives.
- Petitioner asserts that private respondents coerced and
intimidated him to resigning through their collective acts of
harassment. Contrariwise, private respondents contend that it
was petitioner who approached them indicating his desire to
resign due to his difficulty in coping with his responsibilities and
his differences with his immediate boss, Mr. Okawa.
- Petitioner could not have been intimidated by private
respondents to quit. In his memorandum, petitioner
emphatically vowed not to resign despite private respondents
alleged acts of harassment. Surprisingly, however, after only a
few days he did quit alleging that he was forced and harassed
to do so. If petitioner was adamant in his intention not to be
coerced into leaving, how could he suddenly be forced to
resign? Petitioner glaringly contradicted himself. His excuse is
thus, unbelievable and unjustifiable.
- Moreover, the issue in this case is factual in nature and firm is
the principle that factual findings of the NLRC, particularly
when they coincide with those of the LA, are accorded respect,
even finality, and will not be disturbed for as long as such
findings are supported by substantial evidence. We have
painstaking reviewed the records of this case and we find no
justifiable reason to overturn the findings of both the LA and the
NLRC.
Disposition Petition is DISMISSED

REQUISITES
AZCOR MANUFACTURING V NLRC (CAPULSO)
303 SCRA 26
BELLOSILLO; February 11, 1999
NATURE
Petition for certiorari
FACTS
- Candido Capulso filed with the Labor Arbiter a complaint for
constructive illegal dismissal and illegal deduction of P50.00 per
day for the period April to September 1989.
The evidence presented by Capulso showed that he worked for
AZCOR as ceramics worker for more than two (2) years starting
from 3 April 1989 to 1 June 1991. From April to September 1989
the amount of P50.00 was deducted from his salary without
informing him of the reason therefor.
- In the second week of February 1991, upon his doctor's
recommendation, Capulso verbally requested to go on sick
leave due to bronchial asthma. It appeared that his illness was
directly caused by his job as ceramics worker where, for lack of
the prescribed occupational safety gadgets, he inhaled and
absorbed harmful ceramic dusts. His supervisor, Ms. Emily
Apolinaria, approved his request. Later, on 1 June 1991, Capulso
went back to petitioner AZCOR to resume his work after
recuperating from his illness. He was not allowed to do so by his
supervisors who informed him that only the owner, Arturo
Zuluaga, could allow him to continue in his job. He returned five
(5) times to AZCOR but when it became apparent that he would
not be reinstated, he immediately filed the instant complaint for
illegal dismissal.

A2010

- 201 -

Disini

- Capulso presented the following documentary evidence in


support of his claim: (a) His affidavit and testimony to prove
that he was terminated without just cause and without due
process; (b) Identification card issued by AZCOR which he
continued to use even after his supposed employment by
Filipinas Paso; (c) Certification of SSS premium payments; (d)
SSS Member Assistance Form wherein he stated that he worked
with AZCOR from March 1989 to April 1991; (e) Certification of
Employee Contribution with SSS; and, (f) Payslips issued by
AZCOR.
- AZCOR alleged that Capulso was a former employee of AZCOR
who resigned on 28 February 1990 as evidenced by a letter of
resignation and joined Filipinas Paso on 1 March 1990 as shown
by a contract of employment; in February 1991 Capulso
allegedly informed his supervisor, Ms. Emilia Apolinaria, that he
intended to go on terminal leave because he was not feeling
well; on 1 March 1991 he submitted a letter of resignation
addressed to the President of Filipinas Paso, Manuel Montilla;
and, in the early part of June 1991 Capulso tried to apply for
work again with Filipinas Paso but there was no vacancy.
- Petitioners submitted the following documentary evidence: (a)
Sworn Statement of Ms. Emilia Apolinaria and her actual
testimony to prove that respondent indeed resigned voluntarily
from AZCOR to transfer to Filipinas Paso, and thereafter, from
Filipinas Paso due to failing health; (b) Contract of Employment
between Filipinas Paso and respondent which took effect 1
March 1991; (c) Letter of resignation of respondent from AZCOR
dated 28 February 1990, to take effect on the same date; (d)
Undated letter of resignation of respondent addressed to
Filipinas Paso to take effect 1 March 1991; (e) BIR Form No. W-4
filed 6 June 1990; (f) Individual Income Tax Return of
respondent for 1990; and, (g) BIR Form 1701-B which was an
alphabetical list of employees of Filipinas Paso for the year
ending 31 December 1990.
- Labor Arbiter rendered a decision dismissing the complaint for
illegal dismissal for lack of merit, but ordered AZCOR and/or
Arturo Zuluaga to refund to Capulso P200.00 representing the
amount illegally deducted from his salary.
- NLRC modified the Labor Arbiter's decision by: (a) declaring
the dismissal of Capulso as illegal for lack of just and valid
cause; (b) ordering petitioners to reinstate Capulso to his
former or equivalent position without loss of seniority rights and
without diminution of benefits; and, (c) ordering petitioners to
jointly and solidarily pay Capulso his back wages computed
from the time of his dismissal up to the date of his actual
reinstatement.
- Petitioners' motion for reconsideration was denied by the
NLRC. Meanwhile, during the pendency of the case before this
Court, Capulso succumbed to asthma and heart disease, and
died.
- Petitioners insist that Capulso voluntarily resigned. They also
contend that they could not be held jointly and severally liable
for back wages since AZCOR and Filipinas Paso are separate
and distinct corporations with different corporate personalities;
and, the mere fact that the businesses of these corporations are
interrelated and both owned and controlled by a single
stockholder are not sufficient grounds to disregard their
separate corporate entities.
ISSUE
WON NLRC erred in finding that Capulso was illegally dismissed
and in holding petitioners jointly and solidarily liable to Capulso
for back wages
HELD
NO
- On resignation, requisites
Ratio To constitute a resignation, it must be unconditional
and with the intent to operate as such. There must be an
intention to relinquish a portion of the term of office
accompanied by an act of relinquishment.
- The fact that Capulso signified his desire to resume his work
when he went back to petitioner AZCOR after recuperating from
his illness, and actively pursued his case for illegal dismissal

Labor Law 1
before the labor courts when he was refused admission by his
employer, negated any intention on his part to relinquish his job
at AZCOR.
- a closer look at the subject resignation letters readily reveals
the following: (a) the resignation letter allegedly tendered by
Capulso to Filipinas Paso was identically worded with that
supposedly addressed by him to AZCOR; (b) both were predrafted with blank spaces filled up with the purported dates of
effectivity of his resignation; and, (c) it was written in English, a
language which Capulso was not conversant with considering
his low level of education. No other plausible explanation can be
drawn from these circumstances than that the subject letters of
resignation were prepared by a person or persons other than
Capulso. And the fact that he categorically disowned the
signatures therein and denied having executed them clearly
indicates that the resignation letters were drafted without his
consent and participation.
- Even assuming for the sake of argument that the signatures
were genuine, the resignation letters still cannot be given
credence in the absence of any showing that Capulso was
aware that what he was signing then were in fact
resignation letters or that he fully understood the
contents thereof.
- On illegal dismissal
> In illegal dismissal cases, the onus of proving that the
dismissal of the employee was for a valid and authorized cause
rests on the employer, and failure to discharge the same would
mean that the dismissal is not justified and therefore illegal.
- On joint and several liability
> The doctrine that a corporation is a legal entity or a person in
law distinct from the persons composing it is merely a legal
fiction for purposes of convenience and to subserve the ends of
justice. This fiction cannot be extended to a point beyond its
reason and policy. Where, as in this case, the corporate fiction
was used as a means to perpetrate a social injustice or as a
vehicle to evade obligations or confuse the legitimate issues, it
would be discarded and the two (2) corporations would be
merged as one, the first being merely considered as the
instrumentality, agency, conduit or adjunct of the other.
Disposition petition is DISMISSED. NLRC Decision is MODIFIED.
Petitioners AZCOR MANUFACTURING, INC., FILIPINAS PASO and
ARTURO ZULUAGA are ORDERED to pay, jointly and solidarily,
the heirs of private respondent Candido Capulso the amounts
representing his back wages, inclusive of allowances and other
benefits, and separation pay to be computed in accordance with
law.

METRO TRANSIT ORG V NLRC (GARCIA)


284 SCRA 308
BELLOSILLO; January 16 1998
FACTS
- Garcia had been working for Metro Transit (METRO) for almost
8 years as station teller. On April 22 1992, he called up his
immediate supervisor if he could go on LOA to go to Cebu to
look for his wife and children who suddenly left home without
his knowledge. After a few weeks of fruitless search he returned
to Manila.
- When he reported to the office on May 15 1992 Garcia was not
allowed to resume work but was directed to proceed to the
legal department of METRO where he would undergo
investigation. He was asked by Pili about his absence from
work. After he explained to Pili his predicament, Pili cut short
the inquiry and informed him right away that it would be better
for him to resign rather than be terminated for his absences.
Still in a state of extreme agitation and weighed down by a
serious family problem, Garcia at once prepared a resignation
letter. Then he left again for the province to look for his family.
But like his first attempt his effort came to naught. Soon after
the Personnel Committee of METRO approved his resignation.
- Garcia sought advice from his labor union and asked that the
union intervene in his case by bringing the matter of his forced
resignation before their grievance machinery for arbitration.

A2010

- 202 -

Disini

METRO rejected Garcia's plea that he be not considered


resigned from his employment. Garcia filed a complaint for
illegal dismissal. Labor Arbiter and NLRC ruled in favor of
Garcia.
- Petitioner: private respondent absented himself on 22 April
1992 without official leave and then later on freely and willingly
relinquished his employment because he was establishing his
own business.
ISSUE
WON Garcia resigned from his employment
HELD
NO
- An examination of the circumstances surrounding the
submission of the letter indicates that the resignation
was made without proper discernment so that it could
not have been intelligently and voluntarily done.
- What Pili did as petitioner's representative was to advise
Garcia, who at that time was thoroughly confused and bothered
no end by a serious family problem, that he had better resign or
face the prospect of an unceremonious termination from service
for abandonment of work. At that precise moment, the
employee could not be said to have fully understood what he
was doing, i.e., writing his resignation letter, nor could have
foreseen the consequences thereof, for it is established that as
soon as he came out of the investigation office he prepared his
resignation letter right then and there at a table nearby with no
time for reflection. It is noteworthy that shortly thereafter he
consulted his union president for help regarding his forced
resignation. This does not indicate by any means a resignation
that was knowingly and voluntarily done. On the contrary, it
shows that his writing and handing in the resignation
letter to petitioner were a knee-jerk reaction triggered
by that singular moment when he was left with no
alternative but to accede, having been literally forced
into it by being presented with the more unpleasant fate
of being terminated.
- the voluntariness of complaint's resignation can hardly be
believed if he was not forced by circumstances due to the
following:
- First he was already in the employ of respondent for almost
eight years with a high paying job and benefits; Second, no
offense or violation has been attributed to the complainant
during his period of employment; Third, the filing of this instant
complaint by the complainant for illegal dismissal negates or is
inconsistent with abandonment and voluntary resignation.
Lastly, there is no iota of evidence that complainant is indeed
engaged in business, and belies the contents of his resignation.
- Evidently the complainant was asked to make a choice
whether to tender his resignation or be terminated for his
absences which to our mind is anchored on justifiable grounds.
Such compulsion to make an unnecessary choice placed undue
and unjustifiable pressure on the employee who otherwise
would not have thought of leaving his position as Station Teller
if he had not been induced to do so. This being the case, the
resignation filed by the complainant did not become effective.

VOLUNTARY RESIGNATION
PHIL WIRELESS INC V NLRC (LUCILA)
310 SCRA 653
PARDO; July 20, 1999
NATURE
Petition for certiorari to set aside a decision of the NLRC
FACTS
- January 8, 1976 Phil. Wireless Inc. (Pocketbell) hired
respondent Doldwin Lucila as an operator/encoder. Three years
later, Lucila was promoted as Head Technical and Maintenance
Department of the Engineering Department. On September 11,

Labor Law 1
1987, he was promoted as Technical Services Supervisor and
later on October 1, 1990, he became Project Management
Superintendent.
- December 8, 1990 Lucila tendered his resignation.
- December 3, 1991 Lucila filed with the NLRC a complaint for
illegal/constructive dismissal.
- Lucila alleges that his promotion from Technical Services
Supervisor to Project Management Superintendent was actually
a demotion because it was demeaning, illusory and humiliating.
He based it on the fact that he was not given a
secretary/assistant and subordinates.
- June 29, 1992 Labor Arbiter Villarente declared that Lucila
actually resigned and dismissed the complaint for lack merit.
- June 15, 1993 NLRC reversed the findings of the Labor
Arbiter and ordered for Lucilas reinstatement with payment of
backwages or separation pay.
ISSUE
WON Lucila was constructively dismissed
HELD
NO
Ratio Constructive dismissal is an involuntary resignation
resorted to when continued employment is rendered
impossible, unreasonable or unlikely; when there is a demotion
in rank and/or diminution in pay; or when a clear discrimination,
insensibility or disdain by an employer becomes unbearable to
the employee.
Reasoning
- In this case, the Court ruled that Lucila voluntarily resigned
and was not pressured into doing so.
- Voluntary resignation is defined as the act of en employee
who finds himself in a situation where he believes that personal
reasons cannot be sacrificed in favor of the exigency of the
service and he has no other choice but to disassociate himself
from his employment.
- Lucilas basis for his demotion is inadequate as the Court
ruled that there is no demotion where there is no reduction in
position rank or salary as a result of such transfer.
Disposition The petition is hereby granted. The questioned
decision of the NLRC is set aside and the decision of the Labor
Arbiter is reinstated and affirmed. No costs.

PASCUA V NLRC
287 SCRA 554
PANGANIBAN; March 13, 1998
NATURE
Review on certiorari
FACTS
- The complainants are among the employees of Henry Lao at
the Tiongsan Super Bazaar. On August 7, 1991, Henry Lao
received a telephone call who informed him that one of his
sales ladies had just stolen a Karaoke, the previous night.
There, said saleslady made a confession, that, there were
others who were involved in the stealing of goods. She was
required by Henry Lao to write down their names. Violeta
Soriano and Susan Castillo were included in her list. The
eighteen (18) sales ladies who admitted their guilt resigned.
The remaining workers were placed under the watchful eyes of
respondent.
- On August 21, 1991, Lilia Pascua was caught repairing three
(3) pairs of pants that belonged to Mrs. Manaois and allegedly
were not bought at the Tiongsan Super Bazaar. Respondent
scolded Lilia Pascua for this offense, because it is against the
respondents policy that repair jobs of items not bought at the
bazaar should not be accepted. She was given a warning, that
this prohibition should be strictly followed. Lilia Pascua did not
report for work the next day. She went to see the respondents
bookkeeper for the computation of her separation pay.
Respondent paid her separation pay.

A2010

- 203 -

Disini

- On August 24, 1991, Victoria Santos was caught charging a


meter of a cloth for the price of a yard. For this offense, she
was suspended for a period of thirty (30) days. She never
returned to work since then.
- Mimi Macanlalay was employed on June 10, 1989.
Previously, she worked for Mrs. Tan. On September 19, 1991,
Mrs. Tan went to the Tiongsan Super Bazaar, and she saw Mimi
Macanlalay working as a cashier. Mrs. Tan informed Mr. Lao,
that Mimi Macanlalay was previously dismissed by her for
dishonesty. Mimi Macanlalay later on resigned.
- Violeta Soriano was employed on May 16, 1984. After the
August 7, 1991 incident, she was assigned as a cashier. She
was reverted back as a sales lady after a few weeks when Mr.
Lao learned, that, she had some knowledge of the schemes of
the resigned employees.
On November 9, 1991, Mr. Lao
required her to explain in writing, why she should not be the
subject matter of a disciplinary action, for her failure to fill up
her daily time record. Respondent reviewed her past records
and found out that, she was the subject matter of a disciplinary
action in the past. She was terminated [sic] on December 8,
1991.
- Susan De Castro refused to receive her salary on November
18, 1991, because she insisted on receiving more than what is
indicated in the payrolls. Respondent told her that if she is not
satisfied with her salary, she can find employment elsewhere.
She failed to report for work on the following day. In any case,
respondent states, that, she can be dismissed for lack of trust
and confidence, for her involvement in the pilferage of goods.
Petitioners filed at the Regional Arbitration Branch of the NLRC
separate complaints against Henry Lao for illegal dismissal and
claims for violation of labor standards pertaining to payment of
wages.
Subsequently, the labor arbiter ruled that the
dismissals were illegal and awarded back wages and separation
pay to petitioners.
- The NLRC, which modified the appealed decision and found
the termination of petitioners employment to be due either to
voluntary resignation or dismissals with just cause.
ISSUES
1.
WON petitioners employment terminated because of
resignation, abandonment or dismissal
2. WON petitioners employment terminated in accordance
with law
HELD
1. ILLEGAL DISMISSAL (except for Santos).
- Petitioner Pascua was aware of the close relationship between
Henry Lao and Mrs. Manaois. Thus, Pascua feared that, if she
turned down Mrs. Manaois request, she would be subjected to
public scolding by Lao. Thus, accommodation of the said
request may have been an act of disobedience of her
employers order, but hardly an instance of the wrongful and
perverse attitude that would warrant a penalty as grave as
dismissal. That after the incident, Henry Lao kept pushing me
by my shoulders as he repeatedly told me in a loud manner,
pakuwenta mo na ang separation pay mo at hindi ka na rin
makakabalik. Puntahan mo ang accountant. which made me
nervous and afraid especially that he kept on pushing me even
when I was already on top of the stairs; It is evident from the
above that Petitioner Pascua was forced to resign -- an act
which was tantamount to a dismissal, an illegal one at that.
- The NLRC could not explain the contradictions in Petitioner De
Castros case. If she had not been dismissed but was still an
employee of private respondent, then why did she file this case
for illegal dismissal? And even more perplexing: Why would
the NLRC conclude that reinstatement was no longer possible
because of the parties respective imputations of charges
against each other? Furthermore, the labor arbiters finding
that there was no evidence on record to establish her dismissal
is refuted by the uncontested allegations of Petitioner De
Castro.
- Prior to her employment at Tiongsan Super Bazaar Petitioner
Macanlalay had been a saleslady at Rommels which was owned
by a certain Mrs. Tan. On September 20, 1991, while she was

Labor Law 1
working as a cashier at Tiongsan, Mrs. Tan saw her; thereupon,
Mrs. Tan reported to Henry Lao that Petitioner Macanlalay had
previously been dismissed for alleged dishonesty. Petitioner
was then called by Lao and unceremoniously told: Kunin mo
na ang separation pay mo. Pa total mo na sa accountant. At
huwag ka ng magtrabajo dito. Clearly, she did not resign; she
was orally dismissed by Lao. It is this lack of clear, valid and
legal cause, not to mention due process, that made her
dismissal illegal, warranting reinstatement and the award of
back wages.
- The NLRC justified Petitioner Sorianos dismissal by alleging
that it was due to her failure to make regular entries in her daily
time records. We believe, however, that this alleged just
cause was convincingly disputed by Petitioner Soriano in her
letter dated November 9, 1991.
- We agree that Petitioner Santos voluntarily resigned. The
labor arbiter did not find Petitioner Santos to have been illegally
dismissed. Rather, after her suspension for charging for a
meter of cloth bought [at] the price of a yard, she offered to
resign. The solicitor general supports this by stating that even
the Labor Arbiter discovered this when he ruled that there
[was] no evidence on record to support Santos dismissal.
2. NO
Reasoning
- Basic is the doctrine that resignation must be voluntary and
made with the intention of relinquishing the office,
accompanied with an act of relinquishment. Based on the
evidence on record, we are more than convinced that
Petitioners Lilia Pascua, Mimi Macanlalay, Susan C. De Castro
and Violeta Soriano did not voluntarily quit their jobs. Rather,
they were forced to resign or were summarily dismissed without
just cause. Petitioners -- except Victoria L. Santos -- forthwith
took steps to protest their layoff and thus cannot, by any logic,
be said to have abandoned their work.
- In labor cases, the employer has the burden of proving that
the dismissal was for a just cause; failure to show this, as in the
instant case, would necessarily mean that the dismissal was
unjustified and, therefore, illegal. To allow an employer to
dismiss an employee based on mere allegations and
generalities would place the employee at the mercy of his
employer; and the right to security of tenure, which this Court is
bound to protect, would be unduly emasculated. Considering
the antecedents in the summary dismissals effected against
Petitioners Pascua, Macanlalay, De Castro and Soriano, the
causes asserted by private respondent are, at best, tenuous or
conjectural; at worst, they are mere afterthoughts.
- Under the Labor Code, as amended, the dismissal of an
employee which the employer must validate has a twofold
requirement: one is substantive, the other procedural. Not
only must the dismissal be for a just or an authorized cause as
provided by law (Articles 282, 283 and 284 of the Labor Code,
as amended); the rudimentary requirements of due process -the opportunity to be heard and to defend oneself -- must be
observed as well.
- Petitioners Pascua and Macanlalays acceptance of separation
pay did not necessarily amount to estoppel; nor did it connote a
waiver of their right to press for reinstatement, considering that
such acceptance -- particularly by Petitioner Pascua who had to
feed her four children -- was due to dire financial necessity.
Disposition REVERSED.

AZCOR MANUFACTURING INC V NLRC


[PAGE 197]
VALDEZ V NLRC (NELBUSCO INC)
286 SCRA 87
REGALADO; February 9, 1998
NATURE
Special civil action for certiorari
FACTS

A2010

- 204 -

Disini

- Sometime in December, 1986, petitioner was hired by private


respondent as a bus driver on commission basis, with an
average earning of P6,000.00 a month. On February 28, 1993,
the airconditioning unit of the bus which petitioner was driving
suffered a mechanical breakdown. Respondent company told
him to wait until the airconditioning unit was repaired.
Meanwhile, no other bus was assigned to petitioner to keep him
gainfully employed.
- Thereafter, petitioner continued reporting to his employer's
office for work, only to find out each time that the
airconditioning unit had not been repaired. Several months
elapsed but he was never called by respondent company to
report for work. Later, petitioner found out that the bus formerly
driven by him was plying an assigned route as an ordinary bus,
with a newly-hired driver.
- On June 15, 1993, petitioner filed a complaint against private
respondent for illegal dismissal, with money claims for labor
standard benefits, and for reimbursement of his bond and tire
deposit. He claimed that the reason why respondent company
did not allow him to drive again was due to his refusal to sign
an undated company-prepared resignation letter and a blank
affidavit of quitclaim and release.
- Private respondent, on the other hand, admitted that it told
petitioner to wait until the airconditioning unit of the bus was
repaired. However, private respondent alleged that after the
bus driven by the petitioner broke down due to his fault and
negligence, the latter did not report for work. He supposedly
informed the management later that he was voluntarily
resigning from his employment in order to supervise the
construction of his house. Consequent to his resignation,
petitioner demanded the return of his cash bond and tire
deposit. Respondent company required him to secure the
necessary management clearance and other pertinent papers
relative to his resignation. Instead of complying with those
requirements, petitioner filed the instant complaint.
ISSUE
WON petitioner was illegally dismissed because he did not
voluntarily resigned as claimed by respondents
HELD
- The reason for the stoppage of operation of the bus assigned
to petitioner was the breakdown of the airconditioning unit,
which is a valid reason for the suspension of its operation.
However, such suspension regarding that particular bus should
likewise last only for a reasonable period of time. The period of
six months was more than enough for it to cause the repair
thereof. Beyond that period, the stoppage of its operation was
already legally unreasonable and economically prejudicial to
herein petitioner who was not given a substitute vehicle to
drive.
- The so-called "floating status" of an employee should last only
for a legally prescribed period of time. When that "floating
status" of an employee lasts for more than six months, he may
be considered to have been illegally dismissed from the service.
Thus, he is entitled to the corresponding benefits for his
separation, and this would apply to the two types of work
suspension heretofore noted, that is, either of the entire
business or of a specific component thereof.
- It was not denied by private respondent that it tried to force
private respondent to sign an undated company-prepared
resignation letter and a blank undated affidavit of quitclaim and
release which the latter validly refused to sign. Furthermore,
the bus which petitioner used to drive was already plying a
transportation route as an ordinary bus and was being driven by
another person, without petitioner having been priorly offered
the same alternative arrangement.
- The other allegation of private respondent that petitioner
voluntarily resigned from work obviously does not deserve any
consideration. It would have been illogical for herein petitioner
to resign and then file a complaint for illegal dismissal.
Resignation is inconsistent with the filing of the said complaint.
- Resignation is defined as the voluntary act of an employee
who finds himself in a situation where he believes that personal

Labor Law 1
reasons cannot be sacrificed in favor of the exigency of the
service, and, that he has no other choice but to disassociate
himself from his employment. Resignation is a formal
pronouncement of relinquishment of an office. It must be made
with the intention of relinquishing the office accompanied by an
act of relinquishment.
- The cardinal rule in termination cases is that the employer
bears the burden of proof to show that the dismissal is for just
cause, failing in which it would mean that the dismissal is not
justified. This rule applies adversely against herein respondent
company since it has utterly failed to discharge that onus by
the requisite quantum of evidence.
- Under Article 279 of the Labor Code, as amended, an
employee who is unjustly dismissed from work shall be entitled
to reinstatement without loss of seniority rights and other
privileges and to his full back wages, inclusive of allowances,
and to other benefits or their monetary equivalent computed
from the time his compensation was withheld from him up to
the time of his actual reinstatement.
Disposition Decision of respondent National Labor Relations
Commission is SET ASIDE and the decision of the Labor Arbiter
REINSTATED

VALIDITY OF POLICY
MANILA BROADCASTING COMPANY V NLRC
(OLAIREZ, BANGLOY)
294 SCRA 486
MENDOZA; 1998
NATURE
Petition for certiorari to set aside the decision of the National
Labor Relations Commission, affirming the decision of the Labor
Arbiter which found private respondent to have been illegally
dismissed and which ordered him reinstated with damages.
FACTS
- Private respondent Samuel L. Bangloy was production
supervisor and radio commentator of the DZJC-AM radio station
in Laoag City. The radio station is owned by petitioner Manila
Broadcasting Company.
- On February 28, 1992, private respondent applied for leave of
absence for 50 days, from March 24 to May 13, 1992, in order
to run for Board Member in Ilocos Norte under the Kilusang
Bagong Lipunan (KBL). He made his application pursuant to
11(b) of R.A. No. 6646 which provides:
Sec. 11(b) . . . Any mass media columnist, commentator,
announcer, or personality who is a candidate for any elective
public office shall take a leave of absence from his work as
such during the campaign period.
- After a week, private respondents application was returned to
him, together with a copy of an office memorandum of Eugene
Jusi, Assistant Vice-President for Personnel and Administration,
to Atty. Edgardo Montilla, Executive Vice-President and General
Manager of the FJE Group of Companies, in which it was stated
that as a matter of company policy, any employee who files a
certificate of candidacy for any elective national or local office
would be considered resigned from the company.
- It would appear that private respondent nonetheless ran in the
election but lost. On May 25, 1992, he tried to return to work,
but was not allowed to do so by petitioner on the ground that
his employment had been terminated.
- Private respondent filed a complaint for illegal dismissal
against petitioner before the Department of Labor and
Employment.
ISSUES
1. WON the company policy that any employee who files a
certificate of candidacy for any elective national or local office
would be considered resigned from the company valid

A2010

Disini

- 205 -

2. WON the company policy was made known to employees


before it was sought to be applied to private respondent
HELD
1. YES
- the policy is valid and justified.
2. NO
- There are a number of circumstances which raise some doubts
whether the company policy was strictly enforced.
Ratio Although 11(b) of R.A. No. 6646 does not require mass
media commentators and announcers such as private
respondent to resign from their radio or TV stations but only to
go on leave for the duration of the campaign period, we think
that the company may nevertheless validly require them to
resign as a matter of policy.
- The policy is justified on the following grounds:
1) Working for the government and the company at the same
time is clearly disadvantageous and prejudicial to the rights
and interest not only of the company but the public as well. In
the event an employee wins in an election, he cannot fully
serve, as he is expected to do, the interest of his employer.
The employee has to serve two (2) employers, obviously
detrimental to the interest of both the government and the
private employer.
2) In the event the employee loses in the election, the
impartiality and cold neutrality of an employee as broadcast
personality is suspect, thus readily eroding and adversely
affecting the confidence and trust of the listening public to
employers station.
These are valid reasons for petitioner. No law has been cited by
private respondent prohibiting a rule such as that in question.
Disposition Decision AFFIRMED

14.03
NO
TERMINATION

PERFORMANCE OF MILITARY OR
CIVIC DUTY
C. TERMINATION OF EMPLOYMENT BY
EMPLOYER
1. PRELIMINARY MATTERS

14.04 BASIS OF
REQUIREMENTS

RIGHT

AND

BASIS
GUTIERREZ V SINGER SEWING MACHINE
411 SCRA 512
QUISUMBING; September 3, 2003
NATURE
review is the decision of the Court of Appeals
FACTS
- Petitioner Mario Gutierrez was initially hired by Singer Sewing
Machine Company as Audit Assistant on contractual basis in
1993. He became an Accounts Checker on probationary status
on February 8, 1994. Thereafter, he acquired regular status as
Asset Auditor on March 1, 1995, receiving a monthly salary of
P4,455, until September 9, 1996, when he was dismissed from
employment. Singer premised the petitioners termination on
the following incidents:
- On August 1, 1996, at around 3:15 p.m., Ms. Emelita Garcia,

Labor Law 1
Personnel Supervisor of Singer, caught Gutierrez and three
other Asset Auditors, watching a video tape inside the
Asset/Legal Department Office. Despite Ms. Garcias reminder
that it was no longer break time and that the other occupants of
the room might be disturbed, Gutierrez and company ignored
Ms. Garcia and continued to watch the video. The following day,
August 2, 1996, Ms. Evangeline Que-Ilagan, Administration
Manager of Singer, noticed a sign posted at the door of the
Asset/Legal Department Office, which read MAIPARIT TI
UMISBO DITOY. When she asked who placed the sign at the
door, Gutierrez admitted responsibility. When Ms. Que-Ilagan
asked what it meant, Gutierrez answered, BAWAL ANG UMIHI
DITO (No Urinating Here). Ms. Que-Ilagan then asked if
Gutierrez had seen anyone urinate at the door where the sign
was posted and the latter replied in the negative. Ms. QueIlagan then asked why he placed such a sign, to which Gutierrez
replied, Gusto ko, eh (It is my pleasure). She admonished him
not to do the same thing again and requested him to remove
the sign, but Gutierrez refused to do so.
- Later that same day, August 2, 1996, Gutierrez personally
explained his side to the Asset Manager, Mr. Leonardo Consunji,
at the latters office. Gutierrez claimed that he only admitted to
the posting of the sign in order to take the cudgels for a coemployee. He also explained that their use of the video
equipment was upon the orders of their supervisor, Mr. Romeo
C. Ninada. The latter wanted to test the quality of their video
players. Mr. Consunji brought the matter to the attention of Mr.
Ninada. The latter promptly issued a Memo dated August 6,
1996, requiring Gutierrez to explain his side. Gutierrez then
informed Mr. Ninada that he had already discussed the matter
with Mr. Consunji. In his letter to Mr. Consunji dated August 21,
1996, Mr. Ninada opined, [T]he case does not deserve to be
devoted with too much time and effort as he considered it a
minor offense.
- Nevertheless, Mr. Consunji issued a Memo dated August 28,
1996, informing Gutierrez of the latters violation of company
rules and regulations, specifically citing the following:
> Part V-B.9 Use of Companys time, materials, equipment and
other assets for personal use or business; and
> Part V-B.18 Acts of vandalism such as defacing or destroying
Company documents and records; posting, altering or removing
any printed matter, announcements or signs in the Bulletin
Boards unless specifically authorized.
- Under the Company Code of Discipline, these infractions were
classified as 4th Degree Offenses with the corresponding
sanction of dismissal. In the same Memo, Gutierrez was
directed to explain in writing why the aforesaid penalty should
not be imposed on him. He was given until August 30, 1996, to
comply with the directive. As Gutierrez insisted that he had
previously verbally explained his side to Mr. Consunji, no written
explanation was submitted by him.
- On September 9, 1996, another Memo was issued by Mr.
Consunji, worded as follows:
> After a thorough investigation of the incident and after having
found your explanations to be unsatisfactory and due to your
refusal to comply with my memo to you dated August 28, 1996
which constitutes willful defiance or disregard of Company
authority, the management deems it fitting and proper to
impose upon you the penalty of dismissal effective immediately
upon receipt hereof.
- On September 19, 1996, petitioner filed a motion/request for
reconsideration with Singer, but the latter stood pat on its
decision to dismiss him.
- Thus, petitioner filed the complaint for illegal dismissal with
claims for damages before the Labor Arbiter, docketed as NLRC
NCR Case No. 00-10-06201-96. In a decision dated August 13,
1997, Labor Arbiter Renato A. Bugarin dismissed the complaint
for lack of merit.
- Aggrieved, Singer filed a petition for certiorari with this Court,
which in turn was referred, by resolution dated December 2,
1998, to the Court of Appeals.The Court of Appeals reversed the
NLRC, thereby upholding and reinstating the decision of the
Labor Arbiter. Gutierrez now comes to the Court via a petition
for review on certiorari seeking to reverse and set aside the

A2010

- 206 -

Disini

decision of the Court of Appeals, with a prayer for moral


damages and attorneys fees.
ISSUE
WON the appellate court erred in reversing the NLRC which
declared respondents guilty of illegal dismissal of the petitioner
from his employment
HELD
YES
Ratio We agree with the NLRC that petitioners dismissal from
employment was unjustified and illegal. Petitioners dismissal
was based on his alleged violation of two company rules and
regulations, namely: (1) acts of vandalism; and (2) use of
companys time, materials, equipment and other assets for
personal use/business. These acts were found by the Labor
Arbiter to constitute serious misconduct or willful disobedience
under paragraph (a) of Article 282 of the Labor Code. The
Labor Arbiter characterized Gutierrez undesirable or
unreasonable behavior and unpleasant deportment with his
fellow employees, all the more his supervisors, as within the
scope of the analogous just causes for termination under
paragraph (e) of the same article.
- Singer averred that petitioners defiance of the reasonable
rules and regulations being implemented by Singer was enough
reason for his dismissal. Singer emphasized that the two
violations of company rules and regulations on the two
consecutive days, were manifestations that petitioner was
challenging the authorities of Singer.
In its impugned decision, however, the NLRC stated:
- We agree with the complainant that the questioned poster
contained an innocuous and harmless statement, which when
translated in tagalog means Bawal Umihi Dito and that such
posting cannot be interpreted as an act of vandalism. The
affidavit of Ms. Ilagan, in relation with such poster, is not
sufficient to establish complainants guilt of vandalism.The
complainant likewise justified his action in relation to his act of
watching video films during office hours by arguing that he,
together with four (4) other co-employees, were asked by their
immediate supervisor, Mr. Romy Ninada to test the video tape
player. Such claim was not denied by Mr. Ninada, who could
have been easily required by the respondents to do so. Mr.
Ninada was the logical officer to negate the claim of the
complainant that he was authorized to test the quality of the
VHS and CTV 143 to guarantee the excellency (sic) of
respondent firms products.
- Though no admission was made that the use of the video
player was upon the orders of the immediate supervisor of
Gutierrez, Mr. Ninada himself considered the same to be a
minor infraction, not worth the time and effort of the company
spent on the matter.
- We might add that, as contended by petitioner, the act of
posting the sign does not fall squarely within the scope of the
cited company rules and regulations, Part V-B.18, on vandalism.
The rule prohibits unauthorized posting in the Bulletin Board,
while the present case involved posting of a sign at one of the
office doors, a different matter. We must also stress that, even
on the assumption that Gutierrez in fact committed the cited
infractions, in our view they are not major violations but only
minor ones which do not merit the supreme penalty of dismissal
from employment. Time and again, this Court has underscored
the need for restraint in the dismissal of workers:
- Extreme caution should be exercised in terminating the
services of a worker for his job may be the only lifeline on which
he and his family depend for survival in these difficult times.
That lifeline should not be cut off except for a serious, just and
lawful cause, for, to a worker, the loss of his job may well mean
the loss of hope for a decent life for him and his loved ones.
- In the present case, the penalty of dismissal appears in our
view unjustified, much too harsh and quite disproportionate to
the alleged infractions. Not only were the alleged violations
minor in nature, in this case the evidence adduced to prove
them did not fairly show they fall exactly within the rules and
regulations allegedly violated. Otherwise stated, the evidence

Labor Law 1
did not square fully with the charges. That is why the Labor
Arbiter found only analogous causes which, in our view do not
sufficiently justify the extreme penalty of termination.
- The penalty imposed on the erring employee ought to be
proportionate to the offense, taking into account its nature and
surrounding circumstances. In the application of labor laws, the
courts and other agencies of the government are guided by the
social justice mandate in our fundamental law.
- To be lawful, the cause for termination must be a serious and
grave malfeasance to justify the deprivation of a means of
livelihood. This is merely in keeping with the spirit of our
Constitution and laws which lean over backwards in favor of the
working class, and mandate that every doubt must be resolved
in their favor.
- To conclude, the Court of Appeals erred in reversing the
decision of the NLRC which declared respondents guilty of illegal
dismissal.

MANILA TRADING AND SUPPLY CO INC V ZULUETA


69 PHIL 485
LAUREL; January 30, 1940
NATURE
Petition for Certiorari
FACTS
- On July 7, 1938, the Secretary of Labor apprised the Court of
Industrial Relations of a labor dispute existing between the
petitioner company and its employees who were members of
the Philippine Labor Union
- A preliminary hearing was held after which, on August 6, 1938
the respondent court entered an order requiring the company,
inter alia not to dismiss any of its employees and laborers
except for good cause and with its permission.
- Subsequently, on June 30, 1939, one of the gatekeepers of
the petitioners, Filomeno Ramollo, was suspended for a breach
of duty. The breach consisted in that as gatekeeper of the
petitioner he permitted, contrary to instructions, one of the
customers to pass thru the exit gate without paying for the
work done on the car. Before this, it is also alleged that he
refused to work in the setting up department of the company
when ordered by his superior.
- The Philippine Labor Union submitted a petition requesting
the reinstatement of the suspended laborer, to which an answer
was filed by the company.
- In its order of July 28, 1939, the respondent court found that
the laborer was guilty of the breach imputated to him, but,
deciding that his suspension from June 30 to July 28, 1939 was
a sufficient punishment, ordered his immediate reinstatement.
- The petitioner moved for reconsiderations, but the respondent
Court of Industrial Relations, sitting in banc, denied the motion.
ISSUE
WON the Court of Industrial Relations can order the readmission
of a laborer who has been found derelict in the performance of
his duties
HELD
NO
- The right of an employer to freely select or discharge his
employees, is subject to regulation by the State. An employer
cannot legally be compelled to continue with the employment
of a person who admittedly was guilty of misfeasance or
malfeasance towards his employer, and whose continuance in
the service of the latter is patently inimical to his interest. The
law, in protecting the rights of the laborer, authorizes neither
oppression nor self-destruction of the employer. There may, of
course, be cases where the suspension or dismissal of an
employee is whimsical or unjustified or otherwise illegal
scrutinized carefully and the proper authorities will go to the
core of the controversy and not close their eyes to the real
situation.

A2010

Disini

- 207 -

Disposition Writ of Certiorari granted

AGABON V NLRC
[PAGE 35]
PLDT V TOLENTINO
438 SCRA 555
CORONA; September 21, 2004
FACTS
- Arturo R. Tolentino Tolentino was employed in petitioner PLDT
for 23 years.
- He started in 1972 as an installer/helper and, at the time of his
termination in 1995, was the division manager of the Project
Support Division, Provincial Expansion Center, Meet Demand
Group.
- His division was in charge of the evaluation, recommendation
and review of documents relating to provincial lot acquisitions.
Sometime in 1995, Jonathan de Rivera, a supervisor directly
under respondent Tolentino, was found to have entered into an
internal arrangement with the sellers of a parcel of land which
he recommended for acquisition under PLDTs expansion
program. Quirino Donato, the attorney-in-fact of the landowner,
executed an affidavit disclosing his internal arrangement with
de Rivera.
- Donatos affidavit revealed that all follow-up calls regarding
the transaction were to be directed to the office of respondent
and de Rivera. Upon being apprised of this internal
arrangement, PLDT dismissed de Rivera. After he was
dismissed, de Rivera submitted a sworn statement to PLDT
implicating respondent as the person behind the anomalous
internal arrangement. Respondent, in an affidavit, denied this
and pointed out that his authority to approve real estate
acquisitions was limited to land valued below P200,000.
- Petitioner PLDT sent a notice of dismissal, effective October 27,
1995, to respondent Tolentino. Attached to this notice was a
handwritten note from Nicanor E. Sacdalan, Vice-President of
the Provincial Expansion Center, Meet Demand Group, giving
respondent Tolentino the option to resign. Petitioner did not
grant respondents request for a formal hearing but delayed the
implementation of his dismissal. On December 4, 1995,
petitioner informed respondent that his dismissal was already
final and effective on December 5, 1995.
- Respondent then filed a complaint for illegal dismissal, moral
and exemplary damages and other monetary claims against
petitioner PLDT in January, 1996. The labor arbiter found that
petitioner PLDT failed to prove and substantiate the charges
against respondent
- On appeal, the NLRC reversed the labor arbiters decision on
the ground that respondent was a managerial employee and
that loss of trust and confidence was enough reason to dismiss
him.
- Respondents petition for certiorari was referred by this Court
to the Court of Appeals which rendered the assailed decision
reinstating the decision of the labor arbiter, that is, ordering
respondents reinstatement.
ISSUE
WON the Court of Appeals erred in ruling that the dismissal was
not founded on clearly established facts sufficient to warrant
separation from employment
HELD
NO
- The petition is without merit. PLDTs basis for respondents
dismissal was not enough to defeat respondents security of
tenure.
- There is no dispute over the fact that respondent was a
managerial employee and therefore loss of trust and confidence
was a ground for his valid dismissal. The mere existence of a
basis for the loss of trust and confidence justifies the dismissal
of the employee because:

Labor Law 1
[w]hen an employee accepts a promotion to a managerial
position or to an office requiring full trust and confidence, she
gives up some of the rigid guaranties available to ordinary
workers. Infractions which if committed by others would be
overlooked or condoned or penalties mitigated may be visited
with more severe disciplinary action. A companys resort to
acts of self-defense would be more easily justified.
- Proof beyond reasonable doubt is not required provided there
is a valid reason for the loss of trust and confidence, such as
when the employer has a reasonable ground to believe that the
managerial employee concerned is responsible for the
purported misconduct and the nature of his participation
renders him unworthy of the trust and confidence demanded by
his position.
- However, the right of the management to dismiss must be
balanced against the managerial employees right to security of
tenure which is not one of the guaranties he gives up. This Court
has consistently ruled that managerial employees enjoy security
of tenure and, although the standards for their dismissal are less
stringent, the loss of trust and confidence must be substantial
and founded on clearly established facts sufficient to warrant
the managerial employees separation from the company.
Substantial evidence is of critical importance and the burden
rests on the employer to prove it. Due to its subjective nature, it
can easily be concocted by an abusive employer and used as a
subterfuge for causes which are improper, illegal or unjustified.
- In the case at bar, this Court agrees with the Court of Appeals
that the petitioners dismissal was not founded on clearly
established facts sufficient to warrant separation from
employment. The factual findings of the court a quo on the issue
of whether there was sufficient basis for petitioner PLDT to
dismiss respondent Tolentino are binding on this Court. In the
exercise of the power of review, the factual determinations of
the Court of Appeals are generally conclusive and binding on the
Supreme Court.
- The evidence relied upon by petitioner PLDT de Riveras
sworn statement and Donatos affidavit does not, in our view,
establish respondent Tolentinos complicity in the internal
arrangement engineered by his subordinate de Rivera.
- To be sure, respondent Tolentino was remiss in his duties as
division manager for failing to discover the internal
arrangement contrived by his subordinate. However, dismissal
was not the proper sanction for such negligence. It was not
commensurate to the lapse committed, especially in the light of
respondents unblemished record of long and dedicated service
to the company. In Hongkong Shanghai Bank Corporation vs.
NLRC, we had occasion to rule that:
The penalty imposed must be commensurate to the depravity
of the malfeasance, violation or crime being punished. A
grave injustice is committed in the name of justice when the
penalty imposed is grossly disproportionate to the wrong
committed.
[D]ismissal is the most severe penalty an employer can
impose on an employee. It goes without saying that care
must be taken, and due regard given to an employees
circumstances, in the application of such punishment.
- Certainly, a great injustice will result if this Court upholds
Tolentinos dismissal.
An employee illegally dismissed is entitled to full backwages
and reinstatement pursuant to Article 279 of the Labor Code,
as amended by RA 6715.
- Although a managerial employee, respondent should be
reinstated to his former position or its equivalent without loss of
seniority rights inasmuch as the alleged strained relations
between the parties were not adequately proven by petitioner
PLDT which had the burden of doing so. In Quijano vs. Mercury
Drug Corporation, the Court ruled that strained relations are a
factual issue which must be raised before the labor arbiter for
the proper reception of evidence. In this case, petitioner PLDT
only raised the issue of strained relations in its appeal from the
labor arbiters decision. Thus, no competent evidence exists in
the records to support PLDTs assertion that a peaceful working
relationship with respondent Tolentino was no longer possible.
In fact, the records of the case show that PLDT, through VP

A2010

- 208 -

Disini

Sacdalan, gave respondent Tolentino the option to resign.[18]


Such a deferential act by management makes us doubt PLDTs
claim that its relations with respondent were strained. The
option to resign would not have been given had animosity
existed between them.
- Furthermore, respondent was dismissed in December, 1995
when petitioner PLDT was still under the Cojuangco group. PLDT
has since then passed to the ownership and control of its new
owners, the First Pacific group which has absolutely nothing to
do so with this controversy. Since there are no strained
relations between the new management and respondent,
reinstatement is feasible.
Disposition The petition was denied.

PEREZ V MEDICAL CITY GENERAL HOSPITAL


484 SCRA 138
AZCUNA; March 6, 2006
NATURE
Petition for certiorari
FACTS
- September 9, 1999:Prompted by reports of missing medicines
and supplies in the Emergency Room/Trauma Room (ER/TR) and
upon the suggestion of one of the Hospitals staff nurses,
Medical City General Hospital, opened 22 lockers of employees
assigned to the ER/TR. The Hospital found four lockers with
items belonging to it. The employees corresponding to the
lockers (Dominador Perez, Celine Campos, Lailanie Espiritu and
Mateo Butardo) were directed to submit written explanations as
to why these items were inside their lockers.
- Perez, Campos and Butardo submitted their written
explanations, while Espiritu opted to resign. An administrative
hearing was held where the three employees who responded
were represented by a union counsel. At the end of the
proceedings, the charge against Butardo was dismissed while
Perez and Campos, herein petitioners, were found to have
violated category seven of the company rules, a serious
infraction meriting dismissal. The Hospital offered them the
opportunity to voluntarily resign with separation pay, under a
clause provided in the Collective Bargaining Agreement. They
refused and the Hospital dismissed them from the service.
- January 19, 2000: petitioners filed a complaint for illegal
dismissal with the NLRC.
- Labor Arbiter found respondents guilty of illegal dismissal and
ordered the reinstatement of petitioners with backwages and
without loss of seniority rights. NLRC reversed the Labor
Arbiters decision and dismissed the complaint. CA affirmed.
Hence, this petitiom.
- Petitioners maintain that they have sufficiently accounted for
the presence of these items inside their lockers and that the
evidence presented against them is insufficient to show that
they are guilty of misappropriating company property.
Moreover, assuming ex gratia argumenti that there was
violation of company rules, the penalty of dismissal would be
too harsh considering their long years of dedicated service to
the Hospital.
ISSUES
1. WON there was sufficient basis to hold that petitioners
misappropriated hospital property
2. WON dismissal was the appropriate penalty
HELD
1. YES

Labor Law 1
- The Supreme Court is not a trier of facts, and this rule applies
with greater force in labor cases. Hence, the factual findings of
the NLRC are generally accorded not only respect but even
finality if supported by substantial evidence and especially
when affirmed by the CA. However, a disharmony between the
factual findings of the Labor Arbiter and the NLRC opens the
door to a review by this Court.
- Contrary to the position taken by the Labor Arbiter, the
Hospitals dismissal of petitioners did not rest on speculative
inferences. Petitioners themselves have admitted that
properties belonging to the Hospital were found inside their
lockers. As to how these items got inside the lockers, petitioners
acknowledged having placed them there against company
rules. In view of these admissions, there is ample evidence to
support a charge for pilferage unless petitioners can
satisfactorily explain their possession.
- It was made clear to all hospital staff that hospital equipment
should only be kept in the supplies locker.
2. NO
- The power to dismiss an employee is a recognized prerogative
that is inherent in the employers right to freely manage and
regulate his business. An employer cannot be expected to
retain an employee whose lack of morals, respect and loyalty to
his employer or regard for his employers rules and appreciation
of the dignity and responsibility of his office has so plainly and
completely been bared. An employer may not be compelled to
continue to employ a person whose continuance in service will
patently be inimical to his interest. The dismissal of an
employee, in a way, is a measure of self-protection.
- Nevertheless, whatever acknowledged right the employer has
to discipline his employee, it is still subject to reasonable
regulation by the State in the exercise of its police power. Thus,
it is within the power of this Court not only to scrutinize the
basis for dismissal but also to determine if the penalty is
commensurate to the offense, notwithstanding the company
rules.
- In this case, the Court agrees with the Labor Arbiter that
dismissal would not be proportionate to the gravity of the
offense considering the circumstances present in this case.
During Perez and Campos' long tenure (19 and 7 years,
respectively) with the Hospital, it does not appear that they
have been the subject of disciplinary sanctions and they have
kept their records unblemished. Moreover, the Court also takes
into account the fact that petitioners are not managerial or
confidential employees in whom greater trust is placed by
management and from whom greater fidelity to duty is
correspondingly expected.
- The reinstatement of petitioners is in line with the social
justice mandate of the Constitution. Nevertheless, the Court
does not countenance the wrongful act of pilferage but simply
maintains that the extreme penalty of dismissal is not justified
and a lesser penalty would suffice. Under the facts of this case,
suspension would be adequate. Without making any doctrinal
pronouncement on the length of the suspension in cases similar
to this, the Court holds that considering petitioners nonemployment since January 2000, they may be deemed to have
already served their period of suspension. Consequently, the
Labor Arbiters order of reinstatement is upheld, with the
deletion of the award of backwages, so as not to put a premium
on acts of dishonesty.
Disposition Petition partially granted.

REQUIREMENTS
SUBSTANTIVE AND PROCEDURAL DUE
PROCESS
FUJITSU COMPUTER PRODUCTS OF THE PHILS V CA
(DE GUZMAN, ALVAREZ)
454 SCRA 737

A2010

- 209 -

Disini

CALLEJO SR; April 8, 2000


NATURE
A petition for review assailing the Decision of the Court of
Appeals in reversing the decision of the National Labor
Relations Commission (NLRC).
FACTS
- Petitioner Fujitsu Computer Products Corporation of the
Philippines (FCPP) is a corporation organized and existing under
Philippine laws engaged in the manufacture of hard disc drives,
MR heads and other computer storage devices for export.
- Respondent Victor de Guzman began working for FCPP on
September 21, 1997 as Facilities Section Manager. As of 1999,
he was also holding in a concurrent capacity the position of
Coordinator ISO 14000 Secretariat. Allan Alvarez, on the other
hand, was employed as a Senior Engineer on April 21, 1998. He
was assigned at the Facilities Department under the supervision
of respondent De Guzman.
- The garbage and scrap materials of FCPP were collected and
bought by the Saros Trucking Services and Enterprises (Saros).
On January 15, 1999, respondent De Guzman as Facilities
Section Manager, for and in behalf of FCPP, signed a Garbage
Collection Agreement with Saros, and the latters signatory
therein was its owner and general manager, Larry Manaig.
- De Guzman served as middleman between Sta. Rosa Bible
Baptist Church and Saro. The Church was looking for scrap
metal, and was willing to buy the purlins at P3. The scrap metal
was then delivered from FCPP to Sta. Rosa Bible Baptist Church.
- Ernesto Espinosa, HRD and General Affairs Director of FCPP,
received a disturbing report from Manaig. Manaig reported that
respondent De Guzman had caused the anomalous disposal of
steel [purlins] owned by FCPP. Two of Manaigs employees,
Roberto Pumarez and Ma. Theresa S. Felipe, executed written
statements detailing how respondent De Guzman had ordered
the steel purlins to be brought out. Thereafter, petitioner
Espinosa sent a two-page Inter-Office Memorandum dated July
24, 1999 to respondent De Guzman, effectively placing him
under preventive suspension.
- On July 28, 1999, respondent Alvarez sent an e-mail message
to his co-employees, expressing sympathy for the plight of
respondent De Guzman. Respondent Alvarez used a different
computer, but the event viewer system installed in the
premises of petitioner FCPP was able to trace the e-mail
message to him. Respondent Alvarez submitted a written
Explanation dated September 29, 1999 where he apologized,
readily admitted that he was the sender of the e-mail message
in question, and claimed that he acted alone with his own
conviction. He alleged, however, that he was only expressing
his sentiments, and that he was led by his desire to help a
friend in distress.
- Respondent Alvarez was informed that his services were
terminated on the ground of serious misconduct effective
August 13, 1999. Respondent De Guzmans employment was,
thereafter, terminated effective August 23, 1999 through an
Inter-Office Memorandum.
- The respondents then filed a complaint for illegal dismissal
against the petitioners with prayer for reinstatement, full
backwages, damages and attorneys fees before the NLRC.
Labor Arbiter Antonio R. Macam ruled in favor of FCPP, stating
that it was justified in terminating the employment of the
respondents. According to the Labor Arbiter, respondent De
Guzman, a managerial employee, was validly dismissed for loss
of trust and confidence. Citing a number of cases,[24] the Labor
Arbiter stressed that where an employee holds position of trust
and confidence, the employer is given wider latitude of
discretion in terminating his services for just cause.
- The NLRC sustained the ruling of the Labor Arbiter and
dismissed the respondents appeal for lack of merit. The NLRC
also affirmed the Labor Arbiters finding that respondent De
Guzman, a managerial employee who was routinely charged
with the custody and care of the petitioners property, was
validly dismissed on the ground of willful breach of trust and

Labor Law 1
confidence. In so far as the dismissal of respondent Alvarez was
concerned, the Commission held that the circumstances
surrounding the sending of the clearly malicious and
premeditated e-mail message constituted no less than serious
misconduct. Hence, respondent Alvarezs dismissal was also
justified under the circumstances.
- The CA reversed the ruling of the NLRC and held that the
respondents were illegally dismissed.
According to the
appellate court, the non-payment of the scrap steel purlins by
the Sta. Rosa Bible Baptist Church (Sta. Rosa) to Saros was not
a valid cause for the dismissal of respondent De Guzman.
Contrary to the findings of the Labor Arbiter, respondent De
Guzman did not betray the trust reposed on him by his
employer, as the transaction involving the sale of scrap steel
purlins was between Sta. Rosa and Saros. Anent the dismissal
of respondent Alvarez, the CA ruled that his act of
sympathizing and believing in the innocence of respondent De
Guzman and expressing his views was not of such grave
character as to be considered serious misconduct which
warranted the penalty of dismissal.
ISSUES
1. WON De Guzman is guilty of breach of confidence, thus
warranting dismissal
2. WON Alvarez committed serious misconduct in sending the
e-mail
HELD
1. NO
- De Guzman is not guilty of breach of confidence.
Ratio To be a valid ground for dismissal, loss of trust and
confidence must be based on a willful breach of trust and
founded on clearly established facts. A breach is willful if it is
done intentionally, knowingly and purposely, without justifiable
excuse, as distinguished from an act done carelessly,
thoughtlessly, heedlessly or inadvertently. It must rest on
substantial grounds and not on the employers arbitrariness,
whims, caprices or suspicion; otherwise, the employee would
eternally remain at the mercy of the employer. In order to
constitute a just cause for dismissal, the act complained of must
be work-related and shows that the employee concerned is unfit
to continue working for the employer.
Reasoning
- The term trust and confidence is restricted to managerial
employees. In this case, it is undisputed that respondent De
Guzman, as the Facilities Section Manager, occupied a position
of responsibility, a position imbued with trust and confidence.
- The Court had the occasion to reiterate in Nokom v. National
Labor Relations Commission the guidelines for the application of
the doctrine of loss of confidence:
Loss of confidence should not be simulated;
> It should not be used as a subterfuge for causes which are
improper, illegal or unjustified;
> It may not be arbitrarily asserted in the face of overwhelming
evidence to the contrary; and
> It must be genuine, not a mere afterthought to justify earlier
action taken in bad faith.
- The scrap metals, including the steel purlins, were already
classified as scrap materials and ready for disposal. No less
than the written statements of the witnesses for the petitioners
confirm this.
- No fraud or bad faith could be attributed to respondent De
Guzman, as evinced by his readiness to disclose his
participation in the transaction between Saros and Sta. Rosa.
- Loss of trust and confidence as a just cause for termination of
employment is premised on the fact that the employee
concerned is invested with delicate matters, such as the
handling or care and protection of the property and assets of
the employer. After such scrap materials are weighed, loaded
onto a truck and carried out of the company premises, the
petitioner FCPP can no longer be considered the owner thereof,
and ceases to exercise control over such property. In this case
however, Saros, as the new owner of the scrap materials in

A2010

- 210 -

Disini

question, including the steel purlins, was free to contract with


anyone as it wished.
- A condemnation of dishonesty and disloyalty cannot arise from
suspicions spawned by speculative inferences. Because of its
subjective nature, this Court has been very scrutinizing in cases
of dismissal based on loss of trust and confidence because the
same can easily be concocted by an abusive employer. Thus,
when the breach of trust or loss of confidence theorized upon is
not borne by clearly established facts, as in this case, such
dismissal on the ground of loss of confidence cannot be
allowed.
2. NO
- Alvarez did not commit serious misconduct in sending the email.
Ratio Misconduct has been defined as improper or wrong
conduct.
It is the transgression of some established and
definite rule of action, a forbidden act, a dereliction of duty,
willful in character, and implies wrongful intent and not mere
error of judgment. The misconduct to be serious must be of
such grave and aggravated character and not merely trivial and
unimportant.
Reasoning
- For misconduct or improper behavior to be a just cause for
dismissal, (a) it must be serious; (b) must relate to the
performance of the employees duties; and (c) must show that
the employee has become unfit to continue working for the
employer.
- The Court finds that respondent Alvarezs act of sending an email message as an expression of sympathy for the plight of a
superior can hardly be characterized as serious misconduct as
to merit the penalty of dismissal.
- There is no showing that the sending of such e-mail message
had any bearing or relation on respondent Alvarezs
competence and proficiency in his job. To reiterate, in order to
consider it a serious misconduct that would justify dismissal
under the law, the act must have been done in relation to the
performance of his duties as would show him to be unfit to
continue working for his employer.
Disposition Petition is denied. Decision of the CA is affirmed,
with costs against the petitioners.

ARIOLA V PHILEX MINING CORP


446 SCRA 514
CARPIO; August 9, 2005
NATURE
Petition for review of the decision of the CA finding the
retrenchment of the petitioners to be valid
FACTS
- Petitioners are former supervisors of respondent Philex Mining
Corp. Philex sustained financial losses in its operations and
adopted several measures including reducing personnel
through early voluntary retirement and retrenchment programs
to save costs. The labor union representing the rank-and-file
employees and the union representing the supervisory
employees signed a MOA with Philex prescribing the criteria for
retrenchment.
- Petitioners, with 6 other supervisors and 49 rank-and-file
employees, received from Philex termination notices informing
them of their retrenchment. Philex paid them separation pay,
and all of them signed Deeds of Release and Quitclaim in
Philexs favor. Claiming that Philex dismissed them illegally,
these supervisors and rank-and-file employees separately
submitted for voluntary arbitration the legality of their
separation from service.
The rank-and-file employees case
- The rank-and-file employees case was referred to Arbitrator
Valdez. Valdez ruled in the employees favor, declared their
dismissal illegal, and ordered their reinstatement. He held that
Philex failed to prove its claim of financial losses and that the
criteria for retrenchment in the rank-and-files MOA were

Labor Law 1
arbitrary and inconsistent with the CBA then in force. The CA
reversed Valdezs finding on Philexs financial condition and
held that Philex had a valid reason to undertake retrenchment.
Nevertheless, the appellate court affirmed Valdezs ruling that
Philex is liable for illegal dismissal because the criteria for
retrenchment in the rank-and-files MOA were inequitable.
Philex further appealed to this Court, which denied Philexs
petition.
The supervisory employees case
- The supervisors case was referred to Arbitrator Advincula,
who issued an order to reinstate petitioners and their cocomplainants, after Philex failed to timely file its Position Paper.
On Philexs motion, Advincula admitted Philexs Position Paper
and Supplementary Position Paper. He rendered judgment
finding sufficient basis or just cause for Philex to undertake a
retrenchment.
Advincula also held that petitioners were barred from
questioning their separation from service because they availed
of the early retirement program and executed the Deeds of
Release and Quitclaim releasing Philex from further liability.
Petitioners appealed to the CA, which denied the petition for
lack of merit. The appellate court no longer ruled on the validity
of Philexs retrenchment program because it treated its decision
in the rank-and-file employees case as the law of the case on
that issue.
ISSUES
1. WON petitioners retired or whether Philex dismissed them
from service
2. WON petitioners dismissal was illegal
HELD
1. NO
Ratio If the intent to retire is not clearly established or if the
retirement is involuntary, it is to be treated as a discharge.
Reasoning
- Although there is no dispute that petitioners received varied
amounts denominated as retirement gratuity, the records
show that Philex paid these amounts because of petitioners
retrenchment. Under Philexs Retirement Gratuity Plan,
retirement gratuity is paid not only to retiring employees but
also to those who, like petitioners, are dismissed for cause
beyond their control such as retrenchment. Philex treated the
retirement gratuity as petitioners basic separation pay as
indicated in Deeds of Release and Quitclaims petitioners signed.
Significantly, Philex paid petitioners such separation pay after
notifying them of their retrenchment.
Obiter
- In the letter addressed to petitioner Biete, Roxas of Philex
Retirement Trust informed Biete that he was entitled to receive
retirement gratuity because his separation, as a result of the
retrenchment program, is for cause beyond his control. Biete
submitted Roxas letter to the CA after that court had rendered
its decision. However, at that time, petitioners did not yet file
their MFR. Considering the import of the letter, it was error for
the CA not to have considered the letter in resolving petitioners
MFR. There can be no denial of due process where the party
claiming to be aggrieved is the one who is guilty of not
disclosing to the court the vital document that contains the
most conclusive evidence regarding the matter in dispute.
Philex cannot feign ignorance of this letter.
2. YES
Ratio A substantive defect invalidates a dismissal because the
ground for dismissal is negated by such defect, rendering the
dismissal without basis.
Reasoning
- Philexs financial condition justified petitioners retrenchment.
What Philex failed to do was implement its retrenchment
program in a just and proper manner. Its failure to use a
reasonable and fair standard in the computation of the
supervisors demerits points is not merely a procedural but a
substantive defect which invalidates petitioners dismissal.
When the defect is procedural, the dismissal remains valid

A2010

- 211 -

Disini

because the basis of the dismissal is not in any way affected by


such defect.
Disposition The petition is GRANTED. The decision of the CA is
SET ASIDE. We ENTER another judgment finding petitioners to
have been illegally dismissed and ordering Philex to reinstate
petitioners with full backwages, provided that the amounts
petitioners received shall be deducted therefrom. If
reinstatement is no longer possible, Philex shall pay backwages
as computed above plus separation pay.

PHILIPPINE NATIONAL BANK V CABANSAG


460 SCRA 514
PANGANIBAN; June 21, 2005
NATURE
Petition for review on certiorari
FACTS
- Florence Cabansag arrived in Singapore as a tourist. She
applied for the Singaore branch of PNB. At that time, PNB had 2
types of employees: 1) employees hired in Manila and assigned
in Singapore 2) locally hired.
- Ruben Tobias, the general manager of the bank, found her
qualified and recommended her to the President of the bank in
Manila. The latter approved
- Cabansag then applied for an Employment pass with the
Ministry of Manpower of the Government of Singapore. She was
issued said pass.
- On December 7, 1998, she was offered a temporary
appointment, as Credit Officer, wherein she was to be on
probation for 3 months. Cabansag accepted the position and
assumed office. In the meantime, the Philippine Embassy in
Singapore processed the employment contract of Florence O.
Cabansag and, on March 8, 1999, she was issued by the
Philippine Overseas Employment Administration, an Overseas
Employment Certificate, certifying that she was a bona fide
contract worker for Singapore
- On April 15, 1999, she was asked to resign. Tobias said that it
was a cost cutting measure. He likewise said that the PNB
branch would be transformed into a remittance office.
Cabansag then asked Tobias that she be furnished with a
Formal Advice from the PNB Head Office in Manila. However,
Tobias flatly refused. Cabansag did not submit any letter of
resignation.
- On April 16,1999, Tobias again demanded that she submit a
resignation letter. She was warned that he will be dismissed if
she does not. Cabansag asked for more time in order for her to
look for another job. Cabansag said that she should be out by
May15, 1999.
- However, on April 19, 1999, Tobias again asked that Cabansag
submit her letter of resignation. Cabansag refused. The next
day she was terminated.
- NLRC ruled in favor of Cabansag. CA affirmed.
ISSUES
1. WON the NLRC has jurisdiction over the case at bar
2. WON the arbitration of the NLRC in the National Capital
Region is the most convenient venue or forum to hear and
decide the instant controversy
3. WON Cabansag was illegally dismissed
HELD
1. YES
- As enunciated in A217 of the Labor Code, labor arbiters clearly
have original and exclusive jurisdiction over claims arising from
employer-employee relations, including termination disputes
involving all workers, among whom are overseas Filipino
workers
- When Cabansag obtained an employment pass from the
Singapore Ministry of Manpower, it did not imply a waiver of
ones national labor laws. The permit only grants one a status
as a worker in the issuing country. She also applied for an
Overseas Employment Certificate from the POEA through the

Labor Law 1
Philippine Embassy in Singapore. This entitles her to all benefits
and processes under our statutes
- Moreover, petitioner admits that it is a Philippine corporation
doing business through a branch office in Singapore.
Significantly, respondents employment by the Singapore
branch office had to be approved by Benjamin P. Palma Gil,[19]
the president of the bank whose principal offices were in Manila.
This circumstance militates against petitioners contention that
respondent was locally hired; and totally governed by and
subject to the laws, common practices and customs of
Singapore, not of the Philippines. Instead, with more reason
does this fact reinforce the presumption that respondent falls
under the legal definition of migrant worker.
2. YES
- The law gives her two choices:
(1) at the Regional Arbitration Branch (RAB) where she resides
or
(2) at the RAB where the principal office of her employer is
situated
3. YES
- Cabansag was already a regular employee at the time she was
terminated, since her 3 months probationary period has already
ended.
- The twin requirements of notice and hearing constitute the
essential elements of procedural due process, and neither of
these elements can be eliminated without running afoul of the
constitutional guarantee
- In dismissing employees, the employer must furnish them two
written notices:
1) one to apprise them of the particular acts or omissions for
which their dismissal is sought; and
2) the other to inform them of the decision to dismiss them. As
to the requirement of a hearing, its essence lies simply in the
opportunity to be heard.
- Respondent was not notified of the specific act or omission for
which her dismissal was being sought. Neither was she given
any chance to be heard, as required by law. At any rate, even if
she were given the opportunity to be heard, she could not have
defended herself effectively, for she knew no cause to answer
to
- All that petitioner tendered to respondent was a notice of her
employment termination effective the very same day, together
with the equivalent of a one-month pay. This Court has already
held that nothing in the law gives an employer the option to
substitute the required prior notice and opportunity to be heard
with the mere payment of 30 days salary.
- Moreover, Articles 282,[26] 283[27] and 284[28] of the Labor
Code provide the valid grounds or causes for an employees
dismissal. The petitioner has not asserted any grounds as a
valid reason for terminating the employment of respondent
Disposition Petition denied

GENUINO ICE CO INC V MAGPANTAY


493 SCRA 195
AUSTRIA-MARTINEZ; June 27, 2006
NATURE
Review on certiorari
FACTS
- Alfonso Magpantay (respondent) was employed as a machine
operator with Genuino Ice Company, Inc. (petitioner). On
November 18, 1996, respondent filed against petitioner a
complaint for illegal dismissal with prayer for moral and
exemplary damages. In his Position Paper, respondent alleged
that he was dismissed from service effective immediately by
virtue of a memorandum, after which he was not allowed
anymore to enter the company premises. Respondent bewailed
that his termination from employment was done without due
process.Petitioner countered that he was not illegally dismissed,
since the dismissal was based on a valid ground, i.e., he led an
illegal strike at petitioners sister company, Genuino Agro
Industrial Development Corporation, which lasted from

A2010

- 212 -

Disini

November 18 to 22, 1995, resulting in big operation losses on


the latters part. Petitioner also maintained that respondents
dismissal was made after he was accorded due process.
- Petitioner initially claimed that respondents acts were
tantamount to serious misconduct or willful disobedience, gross
and habitual neglect of duties, and breach of trust.
Subsequently, petitioner amended its position paper to include
insubordination among the grounds for his dismissal, since it
came out during respondents cross-examination, and the
matter was reported only after the new personnel manager
assumed his position in August 1996.
- Labor Arbiter of the National Labor Relations Commission
(NLRC) dismissed the case for lack of merit finding that
petitioner had valid cause to dismiss respondent. Labor
Arbiters Decision affirmed. Motion for reconsideration of the
NLRC Decision was denied. Special civil action for certiorari with
the CA was filed. Petitioner filed its Comment, contending that
the petition was filed out of time, considering that contrary to
respondents claim that the NLRC Resolution dated August 31,
1999 was received on December 20, 1999, it was actually
received on September 15, 1999, as shown in the registry
return card.
Petitioner also reiterated its arguments that
respondent was dismissed for cause and with due process.
- CA rendered the assailed Decision granting the petition and
declaring respondents dismissal as illegal. Petitioner filed a
motion for reconsideration which the CA denied.
ISSUES
1. WON the petition was filed by petitioner out of time
2. WON he was illegally dismissed (and on what ground)
3. WON there was due process under Section 2 (d), Rule 1, Book
VI of the Omnibus Rules Implementing the Labor Code provides
for the standards of due process
HELD
1. NO
- The New Rules of Procedure of the NLRC provides the rule for
the service of notices and resolutions in NLRC cases, to wit:
Sec. 4. Service of notices and resolutions. a) Notices or
summons and copies of orders, resolutions or decisions shall
be served on the parties to the case personally by the bailiff
or the duly authorized public officer within three (3) days
from receipt thereof by registered mail; Provided, that where
a party is represented by counsel or authorized
representative, service shall be made on such counsel or
authorized representative;
- The presumption is that the decision was delivered to a person
in his office, who was duly authorized to receive papers for him,
in the absence of proof to the contrary. It is likewise a
fundamental rule that unless the contrary is proven, official
duty is presumed to have been performed regularly and judicial
proceedings
regularly
conducted,
which
includes
the
presumption of regularity of service of summons and other
notices. The registry return of the registered mail as having
been received is prima facie proof of the facts indicated therein.
Thus, it was necessary for respondent to rebut that legal
presumption with competent and proper evidence. Records
show that Ducut is not an employee of the FEU Legal Aid
Bureau, but is connected with the Computer Services
Department. The FEU Legal Aid Bureau has its own personnel
which include Ms. dela Paz who is the one authorized to receive
communications in behalf of the office. It has been ruled that a
service of a copy of a decision on a person who is neither a
clerk nor one in charge of the attorneys office is invalid. The CA
was correct in ruling that the reckoning period should be the
date when respondents counsel actually received the NLRC
Resolution dated August 31, 1999, which was on December 20,
1999. Petitioner, however, pointed out that a certain Ruby D.G.
Sayat received a copy of their Motion for Reconsideration filed
by registered mail on August 16, 2000. Respondent contended
that at the time Sayat received the motion, she was then
detailed at the office and was authorized to receive said
pleading, and that it was an isolated and exceptional instance.
On this matter, the FEU Acting Postmaster certified that Sayat is

Labor Law 1
a permanent employee of the FEU Legal Aid Bureau. As such,
she is authorized to receive communications in behalf of the
office and need not possess an express authority to do so. More
importantly, the Court has consistently frowned upon the
dismissal of an appeal on purely technical grounds. While the
right to appeal is a statutory, not a natural right, it is,
nonetheless, an essential part of our judicial system. Courts
should proceed with caution so as not to deprive a party of the
right to appeal, but rather, ensure amplest opportunity for the
proper and just disposition of a cause, free from the constraints
of technicalities.
2. NO, on the ground of habitual neglect of duties but YES on
the ground of insubordination. The Court sustained the CAs
finding that respondents four-day absence does not amount to
a habitual neglect of duty; however, the Court found that
respondent was validly dismissed on ground of willful
disobedience or insubordination.
- FOR HABITUAL NEGLECT OF DUTY: Neglect of duty, to be a
ground for dismissal, must be both gross and habitual. Gross
negligence connotes want of care in the performance of ones
duties. Habitual neglect implies repeated failure to perform
ones duties for a period of time, depending upon the
circumstances. On the other hand, fraud and willful neglect of
duties imply bad faith on the part of the employee in failing to
perform his job to the detriment of the employer and the
latters business. Thus, the single or isolated act of negligence
does not constitute a just cause for the dismissal of the
employee. Thus, the Court agrees with the CA that respondents
four-day absence is not tantamount to a gross and habitual
neglect of duty. As aptly stated by the CA, (W)hile he may be
found by the labor courts to be grossly negligent of his duties,
he has never been proven to be habitually absent in a span of
seven (7) years as GICIs employee. The factual circumstances
and evidence do not clearly demonstrate that petitioners
[respondent] absences contributed to the detriment of GICIs
operations and caused irreparable damage to the company.
- FOR INSUBORDINATION OR WILLFUL DISOBEDIENCE: On this
point, the CA opined that petitioner included insubordination as
a mere after-thought. It noted that petitioner seemed to be
irresolute in stating the cause of respondents dismissal, as in
its Position Paper, it originally relied on respondents four-day
absence or participation in the illegal strike as a cause for
dismissal but later on amended its Position Paper to include
insubordination. Thus, the CA did not make any factual finding
or conclusion in its Decision vis--vis petitioners allegation of
respondents insubordination.
While its perception may be true, it should not have deterred
the CA from making any resolution on the matter. For one,
respondent was able to argue against petitioners allegation of
insubordination before the Labor Arbiter and the NLRC. For
another, it was respondent himself who raised the subject
before the CA, wherein he stated in his Petition. Further, the
proceedings before the Labor Arbiter and the NLRC are nonlitigious in nature. As such, the proceedings before it are not
bound by the technical niceties of the law and procedure and
the rules obtaining in courts of law, as dictated by Article 221 of
the Labor Code:
ART. 221. Technical rules not binding and prior resort to
amicable settlement. In any proceeding before the
Commission or any of the Labor Arbiters, the rules of
evidence prevailing in courts of law or equity shall not be
controlling and it is the spirit and intention of this Code that
the Commission and its members and the Labor Arbiters shall
use every and all reasonable means to ascertain the facts in
each case speedily and objectively and without regard to
technicalities of law or procedure, all in the interest of due
process. This rule applies equally to both the employee and
the employer. In the interest of due process, the Labor Code
directs labor officials to use all reasonable means to ascertain
the facts speedily and objectively, with little regard to
technicalities or formalities. What is essential is that every
litigant is given reasonable opportunity to appear and defend
his right, introduce witnesses and relevant evidence in his
favor, which undoubtedly, was done in this case. Willful

A2010

- 213 -

Disini

disobedience, or insubordination as otherwise branded in this


case, as a just cause for dismissal of an employee,
necessitates the concurrence of at least two requisites: (1)
the employee's assailed conduct must have been willful, that
is, characterized by a wrongful and perverse attitude; and (2)
the order violated must have been reasonable, lawful, made
known to the employee and must pertain to the duties which
he had been engaged to discharge. Company policies and
regulations are generally valid and binding on the parties and
must be complied with until finally revised or amended,
unilaterally or preferably through negotiation, by competent
authority. For misconduct or improper behavior to be a just
cause for dismissal, the same must be related to the
performance of the employees duties and must show that he
has become unfit to continue working for the employer. In the
case at bench, petitioner informed respondent, through a
Memorandum dated November 14, 1995, that he was being
transferred to its GMA, Cavite operations effective November
20, 1995.
- Due to his refusal to report to the Cavite plant, petitioner
reiterated its order transferring respondent in its Memorandum
dated November 24, 1995, where respondent was also warned
that his failure to report to the Cavite plant will be considered
as an absence without leave (AWOL) and insubordination.
Respondent was required to comply with the order within 24
hours from receipt, otherwise, disciplinary action will be
imposed on respondent. Respondent replied with a request that
he remain in the Otis plant since a transfer to the Cavite plant
will entail additional expenditure and travel time on his part.
Petitioner again wrote respondent inviting him to appear before
the Plant Level Investigation on December 11, 1995 for the
latter to be able to clarify his reasons for refusing the transfer.
Finally, petitioner issued its Memorandum dated December 12,
1995 informing respondent of its decision to terminate his
services. The rule is that the transfer of an employee ordinarily
lies within the ambit of the employers prerogatives. The
employer exercises the prerogative to transfer an employee for
valid reasons and according to the requirement of its business,
provided the transfer does not result in demotion in rank or
diminution of the employees salary, benefits and other
privileges. In this case, petitioners order for respondent to
transfer to the GMA, Cavite Plant is a reasonable and lawful
order was made known to him and pertains to his duties as a
machine operator. There was no demotion involved or
diminution of salary, benefits and other privileges, and in fact,
petitioner was even willing to provide respondent with
monetary allowance to defray whatever additional expenses he
may incur with the transfer. Such being the case, respondent
cannot adamantly refuse to abide by the order of transfer
without exposing himself to the risk of being dismissed. Hence,
his dismissal was for just cause in accordance with Article 282
(a) of the Labor Code. Consequently, respondent is not entitled
to reinstatement or separation pay and backwages.
3. YES
- Simply stated, the employer must furnish the employee a
written notice containing a statement of the cause for
termination and to afford said employee ample opportunity to
be heard and defend himself with the assistance of his
representative, if he so desires, and the employee must be
notified in writing of the decision dismissing him, stating clearly
the reasons therefor.
- The CA found that petitioner failed to observe the twin
requirements of notice and hearing, stating that its
Memorandum dated December 13, 1995 does not squarely
meet the standards of due process.
The circumstances
surrounding respondents dismissal, however, prove the
contrary. The CA failed to take into account that prior to the
Memorandum dated December 13, 1995, petitioner sent
respondent several memoranda apprising him of the possible
implications of his refusal to comply with the order of transfer.
Thus, in its Memorandum dated November 24, 1995, petitioner
notified respondent that his continued non-compliance with the
order of transfer might bring about disciplinary action.
Respondent replied to this memorandum, stating the reasons

Labor Law 1
for his refusal, i.e., additional expenses, longer travel time, and
union concerns. Petitioner sent another Memorandum on
December 9, 1995, asking respondent to appear on December
11, 1995, for further clarification of his reasons for refusing the
transfer.
Despite the meeting, and since respondent,
apparently, stubbornly refused to heed petitioners order, it was
then that the Memorandum dated December 13, 1995 was
issued to respondent informing him of the managements
decision to terminate his services. Clearly, respondents right
to due process was not violated.
Disposition petition is GRANTED. The CA Decision dated
August 3, 2000 and Resolution dated March 16, 2001 are SET
ASIDE, and the NLRC Decision dated June 30, 1999 is
REINSTATED.

14.05 JUST CAUSES


SUBSTANTIVE DUE PROCESS
GROUNDS FOR TERMINATION
A. SERIOUS MISCONDUCT
DEFINITION AND ACTS
VALIAO V CA
[PAGE 11]
VILLAMOR GOLF CLUB V PEHID
472 SCRA 36
CALLEJO; October 4, 2005
NATURE
Petition for review on certiorari of CA decision
FACTS
- Rodolfo Pehid was employed by the Villamor Golf Club (VGC)
as an attendant in the mens locker room, and, thereafter, he
became the Supervisor-in-Charge. His subordinates included
Superal, Parilla, Mendoza, Velasquez, Casabon, Buenaventura
and Modelo. Pehid and these employees agreed to establish a
common fund from the tips they received from the customers,
guests and members of the club for their mutual needs and
benefits. Each member was to contribute the amount of P100
daily. The contributions of the employees had reached the
aggregate amount of P17,990 based on the logbook maintained
in the locker room. This agreement was not known to the VGC
management.
- An audit of the Locker Room Section of the golf club was
conducted stating, among others, that based on the information
relayed, there was an undeclared and unrecorded aggregate
amount of P17,990 for the fund from May 98 to October 98.
Further, not one in the said section admitted custody of such
amount and there was no record that the money had been
distributed among those employed in the locker room. In said
report, Capuyan recommended that an investigation be
conducted to determine the whereabouts of said amount and
who was accountable therefor.
- After the requisite formal investigation by the Administrative
Board of Inquiry, Pehid received order that his employment was
terminated. Based on its findings, Pehid committed gross
misconduct in the performance of his duties in violation of
Paragraph
IV-E(d) of the VGC Rules and Regulations. He
was also informed that he committed acts of dishonesty which
caused and tend to cause prejudice to the club for
misappropriating the common fund of P17,990.00 for his
personal benefit.
- Pehid filed a complaint for illegal dismissal, unfair labor
practice, separation pay/retirement benefits, damages and

A2010

- 214 -

Disini

attorneys fees against petitioners VGC. LA ruled in favor of


Pehid saying that his dismissal was illegal. NLRC set aside and
reversed the decision of LA.
- CA set aside and reversed NLRC decision. The CA declared
that Paragraph IV-E(a) and (d) of the VGC Rules1 expressly
provide that the funds referred to therein are funds of the club
and that the P17,990 did not form part of such fund but
belonged to the locker room personnel. The CA also declared
that the management of the VGC had no personal knowledge
about the funds and, in fact, had not sanctioned its existence.
Moreover, VGC was not prejudiced by the loss of the fund.
Hence, this petition by VGC.
Petitioners contentions:
> That when confronted with the letter-complaint against him,
Pehid admitted that his accountability arose from the proceeds
of the sale of the golf club and golf shares entrusted to him,
which he used for his personal needs without the knowledge of
the persons concerned;
> That there is substantial evidence that Pehid was the
custodian of fund belonging to the members of the locker room
and that his misappropriation of the same constituted gross
misconduct;
> That it is an act of manifest dishonesty within the context of
Paragraph IV-E(d) of the Rules of Conduct of the club, in relation
to A282(e) of the Labor Code, tending to prejudice the VGC
> That, based on the substantial evidence Pehid
misappropriated the fund as his co-employees in the locker
room even positively identified him as the custodian thereof;
and
> that Pehids failure to account for and distribute the common
fund which the locker personnel had established for their
mutual aid and benefit is a manifest dishonesty falling within
the scope of the proviso
Respondents arguments:
> That he was dismissed without just cause and due process of
law;
> that there was no basis or evidence to show that he had
custody of the common fund which was used for his own
benefit;
> that he incurred the ire of his superiors for testifying in
support of Tansiongco, a former Director of Personnel who was
dismissed by VGC; and
> that one of Tansiongcos accusers was the brother of
Velasquez, one of the locker boys who complained against him.
ISSUES
1. WON CA decision is contrary to law and jurisprudence and
therefore reversible
2. WON the incident of the case shall fall within the provision of
Article 282 paragraph (e) of the Labor Code
HELD
1. NO
- Company policies and regulations are, unless shown to be
grossly oppressive or contrary to law, generally valid and
binding and must be complied with by the parties unless finally
revised or amended, unilaterally or preferably through
negotiation.
However, while an employee may be validly
dismissed for violation of a reasonable rule or regulation
adopted for the conduct of the companys business, an act
allegedly in breach thereof must clearly and convincingly fall
within the express intendment of such order.
- The CA was correct in ruling that the NLRC had overlooked and
misapplied certain facts and circumstances of substance, which,
if properly appreciated, would affect the disposition of the case.
- Theres no doubt that funds alleged to have been embezzled
by the petitioner, belonged to the personnel of respondent VGC
and not to respondent VGC. Under the afore-quoted VGC rule
(see footnote), the dishonesty of an employee to be a valid
cause for dismissal must relate to or involve the
1

E. Dishonesty
1.
The following shall constitute violation of this section.
a)
Misappropriation or malversation of Club funds.
d)
All other acts of dishonesty which cause or tend to cause prejudice to VGC

Labor Law 1
misappropriation or malversation of the club funds, or cause or
tend to cause prejudice to VGC. The substantial evidence on
record indicates that the P17,990, which was accumulated from
a portion of the tips given by the golfers from May 1998 to
October 1998 and was allegedly misappropriated by the
respondent as the purported custodian thereof, did not belong
to VGC but to the forced savings of its locker room personnel.
Hence, VGC was not prejudiced. So it is within law and
jurisprudence that CA reversed NLRC ruling.
2. NO
Ratio
The principle in statutory construction of ejusdem
generis: Where general words follow an enumeration of persons
or things, by words of a particular and specific meaning, such
general words are not to be construed in their widest extent,
but are to be held as applying only to persons or things of the
same kind or class as those specifically mentioned.
Reasoning
- Based on the grounds of termination provided under A282 of
the Labor Code and the VGC Rules and Regulations, the
common denominator thereof to constitute gross misconduct as
a ground for a valid termination of the employee, is that it is
committed in connection with the latters work or employment.
In the instant case, as previously pointed out, the alleged
petitioners misappropriation or malversation was committed,
assuming it to be true, against the common funds of the Locker
Room personnel, which did not belong nor sanctioned by
respondent VGC. A fortiori, respondent VGC was not prejudiced
or damaged by the loss or misappropriation thereof.
Obiter
- Important for our purposes in the outline: Serious
misconduct as a valid cause for the dismissal of an employee
is defined as improper or wrong conduct; the transgression of
some established and definite rule of action, a forbidden act, a
dereliction of duty, willful in character, and implies wrongful
intent and not mere error in judgment. To be serious within the
meaning and intendment of the law, the misconduct must be of
such grave and aggravated character and not merely trivial or
unimportant. However serious such misconduct, it must be in
connection with the employees work to constitute just cause
for his separation. The act complained of must be related to the
performance of the employees duties such as would show him
to be unfit to continue working for the employer.
Disposition Petition is DENIED for lack of merit. CA decision
AFFIRMED.

LAKPUE V BELGA
473 SCRA 617
YNARES-SANTIAGO; October 20, 2005
FACTS
- Petitioner Tropical Biological Phils., Inc. (Tropical), a subsidiary
of Lakpue Group of Companies, hired on March 1, 1995
respondent Ma. Lourdes Belga (Belga) as bookkeeper and
subsequently promoted as assistant cashier. On March 19,
2001, Belga brought her daughter to the Philippine General
Hospital (PGH) for treatment of broncho-pneumonia. On her
way to the hospital, Belga dropped by the house of Marylinda O.
Vegafria, Technical Manager of Tropical, to hand over the
documents she worked on over the weekend and to give notice
of her emergency leave.
- While at the PGH, Belga who was pregnant experienced labor
pains and gave birth on the same day. On March 22, 2001, or
two days after giving birth, Tropical summoned Belga to report
for work but the latter replied that she could not comply
because of her situation. On March 30, 2001, Tropical sent
Belga another memorandum ordering her to report for work and
also informing her of the clarificatory conference scheduled on
April 2, 2001. Belga requested that the conference be moved
to April 4, 2001 as her newborn was scheduled for check-up on
April 2, 2001. When Belga attended the clarificatory conference

A2010

- 215 -

Disini

on April 4, 2001, she was informed of her dismissal effective


that day.
ISSUE
WON Belga was illegally dismissed
HELD
YES
- Tropical terminated Belga on the following grounds: (1)
Absence without official leave for 16 days; (2) Dishonesty, for
deliberately concealing her pregnancy; (3) Insubordination, for
her deliberate refusal to heed and comply with the memoranda
sent by the Personnel Department on March 21 and 30, 2001
respectively
- Tropical cites the following paragraphs of Article 282 of the
Labor Code as legal basis for terminating Belga:
Article 282. Termination by employer. An employer may
terminate an employment for any of the following causes:
(a) Serious misconduct or willful disobedience by the
employee of the lawful orders of his employer or
representative in connection with his work;
(c)
Fraud or willful breach by the employee of the trust
reposed in him by his employer or duly authorized
representative
- We have defined misconduct as a transgression of some
established and definite rule of action, a forbidden act, a
dereliction of duty, willful in character, and implies wrongful
intent and not mere error in judgment. Such misconduct,
however serious, must, nevertheless, be in connection with the
employees work to constitute just cause for his separation
- Her absence for 16 days was justified considering that she had
just delivered a child, which can hardly be considered a
dereliction of duty or wrongful intent on the part of Belga.
-Tropical harps on the alleged concealment by Belga of her
pregnancy. This argument, however, begs the question as to
how one can conceal a full-term pregnancy. We agree with
respondents position that it can hardly escape notice how she
grows bigger each day. While there may be instances where
the pregnancy may be inconspicuous, it has not been
sufficiently proven by Tropical that Belgas case is such
- The charge of disobedience for Belgas failure to comply with
the memoranda must likewise fail. Disobedience, as a just
cause for termination, must be willful or intentional. In the
instant case, the memoranda were given to Belga two days
after she had given birth. It was thus physically impossible for
Belga to report for work and explain her absence, as ordered
- Tropical avers that Belgas job as Treasury Assistant is a
position of responsibility since she handles vital transactions for
the company. It adds that the nature of Belgas work and the
character of her duties involved utmost trust and confidence.
- In order to constitute a just cause for dismissal, the act
complained of must be work-related such as would show the
employee concerned to be unfit to continue working for the
employer. More importantly, the loss of trust and confidence
must be based on the willful breach of the trust reposed in the
employee by his employer. A breach of trust is willful if it is
done intentionally, knowingly and purposely, without justifiable
excuse, as distinguished from an act done carelessly,
thoughtlessly, heedlessly or inadvertently
- Belga was an assistant cashier whose primary function was to
assist the cashier in such duties as preparation of deposit slips,
provisional receipts, post-dated checks, etc. As correctly
observed by the Court of Appeals, these functions are
essentially clerical.

COCA-COLA BOTTLERS PHIL INC V KAPISANAN NG


MALAYANG MANGGAGAWA SA COCA-COLA
452 SCRA 480

Labor Law 1
CALLEJO; February 28, 2005
NATURE
This is a petition for review of the Resolution1 the Court of
Appeals reversing the Resolution of the National Labor Relations
Commission
FACTS
Petitioner Coca-Cola Bottlers Phil., Inc. is a domestic
corporation engaged in the manufacture, sale and distribution
of softdrinks.
- On July 1, 1982, the petitioner hired Florentino Ramirez as
"driver-helper" with the following duties: (a) as driver, he
checks the trucks oil, water, wheels, etc.; (b) as helper, he is
charged of loading and unloading trucks load; putting bottles in
the coolers and displays company products to each outlet or
customers store.2
- Ramirez became a member of the respondent Kapisanan ng
Malayang Manggagawa Sales Force Union, the bargaining
representative of the rank- and-file employees of the petitioner
company. In 1996, he was the "shop steward" of the union at
the companys Batangas Sales Office.
- Sometime in October 1996, it happened that the route
salesman for Route M11 was unavailable to make his usual
routes. Since Ramirez had been driving for the route salesman
for so long, the petitioner company decided to assign him as
temporary replacement of the regular route salesman for routes
M11, AMC and LPR.
- Thereafter, in a Letter dated December 5, 1996, the Officerin-Charge of the Batangas Sales Office, Victor C. dela Cruz,
informed the Officer-in-Charge of DSS-District 44, Rolando
Manzanares, that a review of the copies of the invoices relating
to the transactions of Ramirez in Rt. M11 revealed the following
discrepancies: (a) the number of cases delivered to customers;
(b) empty bottles retrieved from them, and (c) the amounts in
Sales Invoices Nos. 3212215, 3288587, 3288763, 3288765 and
3288764
- Ramirez received a Memorandum from District Office Nos. 44
and 45 requiring him to report to the said office starting
December 5, 1996 until such time that he would be notified of
the formal investigation of the charges against him.
- During the formal investigation conducted by a panel of
investigators on December 20, 1996, Ramirez was not
represented by counsel. He also manifested that he was
waiving his right to be represented by counsel when the
members of the panel asked him about it.
- Ramirez was then asked to explain the discrepancies subject
of the charges
- On February 11, 1997, Ramirez received a notice from the
company informing him that his services were being
terminated; his employment was terminated effective February
12, 1997.
- On March 17, 1997, Ramirez and the union filed a Complaint
for unfair labor practice and illegal dismissal against the
company with the Arbitration Branch of the NLRC.
- Ramirez likewise claimed that he was denied of his right to
due process, based on the following grounds: Firstly, individual
complainant was dismissed without having been first issued a
"notice of dismissal" which supposedly should contain the
charges against him, which would be made as basis for his
termination. Secondly, individual complainant was dismissed
without affording him an ample opportunity to defend himself,
as he was not notified in advance of the subject of the
administrative investigation. Thirdly, individual complainant
was terminated without just and valid cause, and in gross
violation of his right to due process.
Lastly, individual
complainant was terminated by respondents in utter bad faith,
as the decision on the said termination was arrived at, without
any just and valid cause. Simply put, respondents simply acted
oppressively, malevolently, and with grave abuse of
prerogatives.7

A2010

- 216 -

Disini

- Petitioner company alleged that the dismissal of Ramirez was


based on the facts unearthed during the formal investigation,
and that he was guilty of serious misconduct, a valid ground for
termination of employment. Even if he was occupying the
position of route driver/helper, he was nevertheless performing
the functions and duties of a route salesman, and, as such, he
not only committed fraud, but also willfully breached the trust
and confidence reposed on him by the petitioner company.
According to the petitioner company, considering the
sanctions imposed on Ramirez for prior breaches of company
rules, his dismissal from employment was with basis. The
petitioner company also insisted that Ramirez was accorded his
right to due process: he was notified of the charges against him,
was subjected to a formal investigation during which he was
allowed to explain the discrepancies, and was notified of the
outcome thereof, as well as the bases of the termination of his
employment.
- On July 31, 1998, the Labor Arbiter rendered judgment
dismissing the complaint for lack of merit. The LA found that
based on the evidence, there was a justifiable basis for the
dismissal of Ramirez. According to the LA, it was of no moment
that the official designation of Ramirez was "driver-helper,"
since he committed the infractions while he was performing the
functions of an "acting salesman." The LA further found that
due process had been complied with.9
- Ramirez appealed the decision to the NLRC
- On September 20, 1999, the NLRC rendered a Resolution
affirming the decision of the LA.
- Upon the denial of his motion for reconsideration, Ramirez
filed a petition for certiorari under Rule 65 of the Rules of Court
with the Court of Appeals
- In a Decision dated October 25, 2000, the CA dismissed the
petition. It ruled that the petitioners designation at the time of
the infraction was of no moment; when he agreed to be an
"acting salesman" for Route M11, AMC and LPR, he actually
performed the duties of a salesman, and in so doing, assumed
the responsibilities of the position. The CA further ratiocinated
that notwithstanding Ramirezs lack of training, he had
assumed and performed the duties of a salesman; hence, he
was obligated to do so with due care, dedication, and with due
regard to the exercise of the degree of diligence to prevent the
commission of any serious error, mistake or blunder on his part.
- The petitioner filed a motion for the reconsideration of the
decision
- This time, the CA found merit in petitioners cause
ISSUE
WON respondent Florentino Ramirez was dismissed by the
petitioner without just or valid cause
HELD
- with just cause, but too severe penalty
The respondent, by his acts and omissions, committed
irregularities in the performance of his duties. However the
penalty imposed on respondent by the petitioner company was
too severe. In order to effect a valid dismissal of an employee,
the law requires that there be just and valid cause as provided
in Article 282 and that the employee was afforded an
opportunity to be heard and to defend himself. Pursuant to
Article 282 of the Labor Code, an employees services can be
terminated for the following just causes:
(a) Serious misconduct or willful disobedience by the
employee of the lawful orders of his employer or
representative in connection with his work;
(b) Gross and habitual neglect by the employee of his duties;
(c) Fraud or willful breach by the employee of the trust
reposed in him by his employer or duly-authorized
representative.
(d) Commission of a crime or offense by the employee
against the person of his employer or any immediate member
of his family or his duty-authorized representative; and
(e) Other causes analogous to the foregoing.
- In termination disputes, the burden of proof is always on the
employer to prove that the dismissal was for a just and valid

Labor Law 1
cause. Considering the nature of the charges and the penalties
therefore, the petitioner is bound to adduce clear and
convincing evidence to prove the same.
- It is recognized that company policies and regulations, unless
shown to be grossly oppressive or contrary to law, are generally
valid and binding on the parties and must be complied with
until finally revised or amended, unilaterally or preferably
through negotiation, by competent authority. The Court has
upheld a companys management prerogatives so long as they
are exercised in good faith for the advancement of the
employers interest and not for the purpose of defeating or
circumventing the rights of the employees under special laws or
under valid agreements. For misconduct or improper behavior
to be a just cause for dismissal, the same must be related to the
performance of the employees duties and must show that he
has become unfit to continue working for the employer.
- In cases when an employer may dismiss an employee on the
ground of willful disobedience, there must be concurrence of at
least two requisites: (1) the employees assailed conduct must
have been willful or intentional, the willfulness being
characterized by a wrongful and perverse attitude; and (2) the
order violated must have been reasonable, lawful, made known
to the employee and must pertain to the duties which he had
been engaged to discharge.
- That the individual petitioner has not been specifically trained
as salesman is undisputed. In acting as a salesman, he was
tasked with a duty involving trust and specialized skills for
which he was never trained. His alleged failure to comply
strictly with all the procedures, of which he was unfamiliar, was
to be expected. Yet Ramirez was penalized as a full-fledge
salesman, not as a driver-helper who was forced to perform the
functions of acting salesman or perhaps risk being charged with
insubordination. Then it was not just any penalty meted out to
him, as if there is only one punishment possible for him: the
supreme sanction of dismissal.
- Perhaps, individual petitioner should first have been given a
mere warning, then a reprimand or even a suspension, but
certainly not outright dismissal from employment. One must
keep in mind that a workers employment is property in the
constitutional sense, and he cannot be deprived thereof without
due process and unless it was commensurate to his acts and
degree of moral depravity.
- In order to validly dismiss an employee on the ground of loss
of trust and confidence under Article 282 of the Labor Code of
the Philippines, the following guidelines must be followed:
1. The loss of confidence must not be simulated;
2. It should not be used as a subterfuge for causes which are
illegal, improper or unjustified;
3. It may not be arbitrarily asserted in the face of
overwhelming evidence to the contrary;
4. It must be genuine, not a mere afterthought, to justify
earlier action taken in bad faith; and
5. The employee involved holds a position of trust and
confidence.
- Considering the factual backdrop in this case, we find and so
rule that for his infractions, the respondent should be meted a
suspension of two (2) months.
Disposition PARTIALLY GRANTED

GENUINO ICE CO INC V MAGPANTAY


[PAGE 206]
PREMIERE DEVT BANK V MANTAL
485 SCRA 234
YNARES-SANTIAGO; March 23, 2006
NATURE
Petition for review on certiorari seeking to annul and set aside
the Decision of the Court of Appeals in CA-G.R. SP No. 80975
dated January 17, 2005 and its Resolution dated April 7, 2005
holding the petitioner Premiere Development Bank liable for
illegal suspension and illegal dismissal, ordering it to reinstate

A2010

- 217 -

Disini

respondent Elsie Escudero Mantal to her former position and to


pay her full backwages from date of suspension and dismissal
until actual reinstatement, half month salary and half month
13th month pay, as well as attorneys fees.
FACTS
- Respondent is a regular employee of petitioners Cubao
branch, serving as accounting clerk since July 17, 1996. On
November 24, 2000, the branch manager, Rosario Detalla,
instructed respondent: "Elsie, baka may mag-confirm sa Bank
Guarantee ng GIA Fuel, sabihin mo OKAY NA, may kulang pa
lang dokumento."
- Later that day, Emmie Crisostomo of Filpride Energy
Corporation inquired whether GIA Fuel and Lubricant Dealer has
a credit line or maintains an account with petitioner Bank which
respondent confirmed after checking the files on the computer.
Crisostomo also inquired if the bank guarantee signed by
Detalla is in order, and likewise respondent replied in the
affirmative. However, upon verification from petitioners head
office, Crisostomo was informed that the bank guarantee was
spurious.
- On the same day, respondent was summoned to the head
office and was required to write down what she knew about the
subject bank guarantee. Respondent also received a
memorandum placing her under preventive suspension
effective immediately for a period of 30 days. During the
investigation, Detalla admitted issuing the falsified bank
guarantee.
- On December 21, 2000, Detalla tendered her irrevocable
letter of resignation. Respondent was asked to execute a
resignation letter on December 22, 2000, but she declined. The
following day, respondent received a Notice of Termination
dated December 22, 2000.
- Respondent filed a complaint for illegal suspension, illegal
dismissal, unpaid salary and 13th month pay, moral and
exemplary damages. The Labor Arbiter rendered a decision
holding petitioner liable for illegal suspension and illegal
dismissal and ordering the reinstatement of respondent to her
former position, with full backwages, half month salary and half
month 13th month pay, and attorneys fees. NLRC reversed the
labor arbiters decision, and dismissed the complaint for lack of
merit. The motion for reconsideration having been denied,
respondent appealed to the Court of Appeals which affirmed the
Labor Arbiter.
ISSUE
WON respondent was validly suspended and dismissed from her
position as accounting clerk
HELD
NO
Ratio Misconduct is improper or wrongful conduct. It is the
transgression of some established and definite rule of action, a
forbidden act, a dereliction of duty, willful in character, and
implies wrongful intent and not mere error in judgment. Under
Article 282 of the Labor Code, the misconduct, to be a just
cause for termination, must be of such grave and aggravated
character, not merely of a trivial or unimportant nature. For
serious misconduct to warrant the dismissal of an employee, it
(1) must be serious; (2) must relate to the performance of the
employees duty; and (3) must show that the employee has
become unfit to continue working for the employer.
Reasoning
- Respondent did what was expected of her as an employee of
the bank. Before answering the telephone inquiry, respondent
verified the existence of the GIA Fuel and Lubricant Dealer
account through the bank computer. If ever she was negligent,
it would only constitute a single or isolated act which is not a
just cause for the dismissal of the respondent from her
employment.
In addition, although respondents position as accounting clerk
involves a high degree of responsibility requiring trust and
confidence, carrying with it the duty to observe proper company
procedures in the fulfillment of her job as it relates closely to

Labor Law 1
the financial interests of the company, the charge against her is
not reasonably connected to her job of opening of savings,
current and/or time deposits and the payment of withdrawals.
The duty and ultimately, the responsibility of approving
transactions relating to bank guarantees lie with the branch
manager and the management personnel of the petitioners
head office. Thus, in Metropolitan Bank and Trust Company v.
Barrientos, the Court held that respondent therein was not
liable of misconduct for allowing the opening of fictitious
accounts, because he was merely a cashier and had no
authority to approve new accounts and had no way of knowing
the anomalous transactions.
Disposition petition is DENIED. The Decision of the Court of
Appeals in CA-G.R. SP No. 80975 dated January 17, 2005 finding
petitioner guilty of illegal dismissal and ordering the
reinstatement of respondent to her former position, with full
backwages, inclusive of allowances and to the other benefits or
their monetary equivalent from the time her compensation was
withheld up to her actual reinstatement, plus attorneys fees,
and the Resolution dated April 7, 2005 denying the motion for
reconsideration, are AFFIRMED.

MOLINA V PACIFIC PLANS INC


484 SCRA 498
CALLEJO; March 10, 2006
NATURE
Petitions for Review on Certiorari assailing the decision and
resolution of the CA reversing the decision of the NLRC.
FACTS
- The accident occurred on July 9, 1912.
- Because of injuries, plaintiff spent 10 days in the hospital. The
first 4-5 days he couldnt leave his bed. After being discharged,
he received medical attention from a private practitioner for
several days.
- Plaintiff testified that he had down no work since the accident,
that his earning capacity was P50/month
- He described himself as being well at the end of July; the trial
took place September 19
- Plaintiff sold distillery products and had about 20 regular
customers who purchased in small quantities, necessitating
regular, frequent deliveries
- It took him about 4 years to build up the business he had at
the time of the accident, and since the accident, he only kept 4
of his regular customers.
- The lower court refused to allow him any compensation for
injury to his business due to his enforced absence therefrom.
ISSUE
How to determine the amount of damages to award plaintiff
HELD
- The judgment of the lower court is set aside, and the plaintiff
is awarded the following damages; ten pesos for medical
expenses; one hundred pesos for the two months of his
enforced absence from his business; and two hundred and fifty
pesos for the damage done to his business in the way of loss of
profits, or a total of three hundred and sixty pesos. No costs will
be allowed in this instance.
Reasoning
- Actions for damages such as the case at bar are based upon
article 1902 of the Civil Code: "A person who, by act or
omission, causes damage to another where there is fault or
negligence shall be obliged to repair the damage so done." Of
this article, the supreme court of Spain, in considering the
indemnity imposed by it, said: "It is undisputed that said
reparation, to be efficacious and substantial, must rationally
include the generic idea of complete indemnity, such as is
defined and explained in article 1106 of the said (Civil) Code."
- Art 1106. Indemnity for losses and damages includes not only
the amount of the loss which may have been suffered, but also

A2010

- 218 -

Disini

that of the profit which the creditor may have failed to realize,
reserving the provisions contained in the following articles.
- Art 1107. The losses and damages for which a debtor in good
faith is liable, are those foreseen or which may have been case
is will gradually increase. The injury to plaintiff's business
begins where these profits leave off, and, as a corollary, there is
where defendant's liability begins. Upon this basis, we fix the
damages to plaintiff's business at P250.
- Before us is a Petition for Review on Certiorari assailing the
Decision and Resolution of the Court of Appeals (CA) in CA-G.R.
SP No. 81298 reversing the Decision of the National Labor
Relations Commission (NLRC) in NLRC-NCR (South) Case No. 3007-03393-01.
Pacific Plans, Inc. (PPI) is a domestic corporation engaged in the
business of selling pre-need plans, such as educational,
pension, and memorial plans. It maintains regional offices
throughout the Philippines. At the time material to this case,
Metro Manila regional offices were divided into two sales
divisions - the South Sales Division and the North Sales Division.
Metro Manila VI was part of the North Sales Division. Among the
corporate officers of PPI were Geoffrey Martinez, Executive VicePresident for Finance; Luciano Abia, Senior Assistant VicePresident, Metro Manila Marketing Division; and Atty. Manuel
Reyes, the Head of the Legal Department. Roy Padiernos then
occupied the position of Regional Manager of Metro Manila VI.
- PPI solicited subscribers and buyers of its pre-need plans
through clusters of sales associates. One of them was Ruth
Padiernos, wife of Roy Padiernos.
Sometime in October 1994, PPI hired Agripino Molina as
Regional Manager of Metro Manila VI, replacing Roy Padiernos
who was promoted as First Vice-President for Marketing
Operations. As Regional Manager, Molina performed both
administrative and marketing functions, whose duties and
responsibilities included the following:
a. formulating and recommending short and long range
marketing plans for the Region and executing approved
plans;
b. generating new and conserving existing pre-need plan
businesses;
c. motivating, training, and developing a dedicated and
effective counselor force;
d. conducting researches to determine sales potentials and
share of the market, pricing, and profitability of Company's
products, competition and the directing of product
development for the Region;
e. hiring and terminating counselors, unit managers or group
managers in accordance with policies previously laid out;
f. recommending the creation of additional positions or
termination of services of any employee within the Region;
g. recommending promotions or changes in salaries of
personnel within the Region and lateral shifts of supervisor,
their assistants, understudies of positions of equal rank;
h. training and developing understudies for each position
within the Region to provide immediate replacement
whenever vacated;
i. changing methods and procedures not affecting the other
Regions, provided, however, that radical changes should first
be cleared with [the] superior;
j. controlling the operations of the Region and establishing a
system of periodic work reporting;
k. coordinating the Regions activities with those of the other
Regions;
l. keeping [the] superior informed of [the] Region's activities
and specially of [the] decision on matters for which he may
be held responsible;
m. realizing the Companys objective for service, growth, and
profit;
n. establishing and maintaining harmonious and dignified
relationship with plan holders, counselors, employees, the
public, government instrumentalities, other pre-need plan
companies; [and]
o. further enhancing the prestige of the Company and
maintaining its position of leadership in its field.

Labor Law 1
- Since Metro Manila VI was consistently on top in terms of
nationwide sales and productivity, Molina was promoted
Assistant Vice-President with the same functions as those of a
regional manager of the same sales region.
- Caritas Health Shield, Inc. (Caritas for brevity), a health
maintenance organization (HMO) engaged in selling health and
hospitalization plans, was established on December 16, 1998.
Geoffrey Martinez resigned as Executive Vice-President of PPI
and became the President and Chief Executive Officer of
Caritas. Among the incorporators and members of the Board of
Directors were Luciano Abia and Atty. Manuel Reyes. Molina
was hired as Assistant Vice-President and Marketing Head of
Area 10. His wife, Fe Molina, was the head of a sales agency of
Caritas.
- In the meantime, from February 2000, there was a
considerable decrease in the sales output production of PPIs
Metro Manila Region VI.
- On March 21, 2000, Molina received a Memorandum from PPI,
through its Senior Assistant Vice-President for Human Relations,
Patricio A. Picazo, informing him that, based on written reports,
he committed the following: 1) recruiting and pirating activities
in favor of Caritas, in particular, initiating talks and enticing
associates to join Caritas, and a number of associates have
already signed up; 2) he called for a meeting with his associates
sometime in November 1999, and solicited contributions from
them for the bill but later asked for reimbursement from the
company; and 3) acts of misdemeanor on several occasions,
such as coming to the office under the influence of liquor,
initiating a smear campaign against PPI, and other acts inimical
to the companys interest. Molina was also required to submit,
on March 23, 2000, a written explanation why he should not be
held administratively liable for said acts which, it opined, might
constitute conduct unbecoming of an officer, conflict of interest,
and breach of trust and confidence. Molina was also informed
that he was preventively suspended pending formal
investigation effective immediately until April 24, 2000.
- In a letter addressed to Picazo dated March 22, 2000, Molina
categorically denied the acts attributed to him. He, however,
requested that he be furnished with copies of the alleged
written reports to enable him to prepare the required written
explanation. However, instead of acceding to the request of
copies of the written reports, Picazo wrote a letter dated April 3,
2000, citing the particulars of the charges against Molina, thus:
I. Conflict of Interest
1. Recruiting and pirating activities in favor of Caritas
Health Shield, Inc.
* You have acted as conduit for Caritas in
recruiting/pirating Mr. Restie Acosta on March 04, 2000 and
Ms. Eppie Acosta on March 06, 2000.
*Your failure to stop and/or tolerating your wife's activities
in recruiting for Caritas Ms. Lennie Gatmaitan who belongs
to Ms. Celeste Villena, a PPI GA.
II. Misappropriation of Funds
1. Solicitation of associates' personal funds in the amount
of P200.00 per person, to which 12 persons contributed for
a total P2,400.00, for payment of official function during
the meeting held at Barrio Fiesta last November 27, 1999.
Amount solicited was subsequently reimbursed from the
company but not returned to the associates concerned.
III. Dereliction of Duties
1. You failed to prevent associates from leaving the
company in favor of competitors, thus causing
demoralization among your sales associates.
2. You even encouraged associates to transfer to Caritas.
IV. Conduct unbecoming of a Company Officer
1. Often reporting to office under the influence of liquor.
2. Sowing intrigue in the case of Vilma del Rosario which
almost caused her early retirement from the company and
transfer to Caritas.
3. Sowing intrigues between Mr. Roy Padiernos and Mr.
Abia.
4. Showing disrespect to immediate superior, Mr. Roy
Padiernos, by shouting at him and walking out in one of the

A2010

- 219 -

Disini

meetings called by him after the retirement of Atty.


Haceta.
- During the investigation the following day, April 4, 2000,
Molina reiterated his request to be provided with a copy of the
written reports. Picazo denied the request in a Memorandum
dated April 6, 2000, and reiterated his order for Molina to
submit his written explanation on April 11, 2000, and to address
his concerns during the investigation scheduled on April 14,
2000. Molina failed to submit any written explanation. On April
24, 2000, PPI issued a Memorandum advising Molina that he
would be reinstated in the payroll effective April 25, 2000
without requiring him to report for work during the pendency of
his investigation.
- Molina filed a "Motion to Dismiss Complaints and Motion for
Full Reinstatement" on May 2, 2000. He asserted that the
charges should be dismissed since he was compelled to prepare
a written explanation on the basis of "summarized specific
acts," denying him the right to be informed of the exact charges
and to confront those who made written reports against him. As
to the issue of reinstatement, he alleged that he should be
allowed to report for work, conformably with Rule XIV, Section 4
of the Implementing Rules of the Labor Code.
- On May 11, 2000, Picazo wrote Molina that his motion to
dismiss the charges would be resolved after the investigation.
He was warned that his non-appearance at the investigation
would be considered a waiver of his right to be heard.
- On the same day, May 11, 2000, Abia issued an inter-office
Memorandum announcing the appointment of Sercy F. Picache
as the Officer-In-Charge (OIC) for Metro VI and XVI effective May
6, 2000.
- Molina and his counsel attended the May 19, 2000
investigation and filed a Motion to Suspend Proceedings,
praying that the administrative investigation be deferred until
the resolution of the "prejudicial" issues raised in his previous
motion.
- When Picazo failed to respond, Molina filed, on June 1, 2000, a
complaint for damages with a prayer for a temporary
restraining order and preliminary injunction based on Article 19
of the New Civil Code. PPI filed a Motion to Dismiss, maintaining
that the courts have no jurisdiction over matters arising from
employee-employer relationship. The trial court denied the
motion as well as PPIs motion for reconsideration.
- Meanwhile, in letter dated June 13, 2000, Molina was notified
of the termination of administrative investigation. PPI
considered his failure to submit a written explanation as a
waiver of his right to be heard, and as such, the investigating
committee had evaluated the evidence at hand and submitted
its recommendations to the "higher management" for decision.
Also, it confirmed the denial of his Motion to Suspend
Proceedings
- On June 23, 2000, the trial court issued an Order granting
Molina's prayer for temporary restraining order, which was later
made permanent per its Order dated July 12, 2000. The motion
for reconsideration filed by PPI on July 26, 2000 was likewise
denied. Thereafter, it filed a petition for certiorari before the CA,
assailing the writ of preliminary injunction issued by the RTC
and its order denying the motion to dismiss the complaint. On
July 16, 2001, the CA rendered judgment in favor of PPI and
nullified the writ of preliminary injunction issued by the RTC as
well as the order denying the motion of PPI for the dismissal of
the complaint.
- On July 30, 2001, PPI resolved to dismiss Molina from
employment on its finding that the latter violated its standard
operating procedure.
- Molina forthwith filed a complaint with the NLRC against PPI
and Alfredo C. Antonio, Patricio A. Picazo, and Certerio B. Uy, in
their capacity as President, Senior Assistant Vice-President of
Human
Resources
Development,
and
Division
Head,
respectively, for illegal dismissal and illegal suspension with
claim for monetary benefits.
- In his Position Paper, Molina principally argued that he was
denied the right to due process due to the failure of PPI to
furnish him a copy of the written reports of the sales associates
and co-employees, the basis of the accusations against him.

Labor Law 1
Since an OIC for his position was already appointed even before
all his pending motions were resolved, he surmised that there
were really no such reports, and that the alleged accusations
were merely concocted in order to replace him with someone
close to Picazo. Molina maintained that since he was denied the
opportunity to dispute the authenticity and substantive
contents of the reports, his alleged violations of company rules
and policies were hearsay and, therefore, lacked probative
value. Besides, the termination of his employment was made
without the 30-day prior notice; his dismissal from employment
took effect immediately, only six days after PPI received the CA
decision decreeing that the NLRC has the rightful jurisdiction
over the case. Thus, he prayed for the following relief:
1. Total Money Claims
a) Salary with (overriding) commission from March 21 to
April 24, 2000 - suspended w/o pay - P45,000.00
(P25,000[.00] mo. salary & P20,000[.00] [overriding])
b) Unpaid (overriding) commission from April 25, 2000 to
present - P400,000[.00]
c) Unpaid salary from August 1, 2001 to present P125,000[.00]
d) One mo. salary for every yr. of service in lieu of
reinstatement - 7 years = P175,000.00
2. Leave Credits - P100,000.00 for 7 years
3. Profit Bonus for Year 2000 & 2001 - P400,000.00
4. Moral Damages - P300,000.00
5. Exemplary Damages - P500,000.00
6. Actual Damages - for lifetime medical attendance and
medicines at 16 more years life expectancy - P1,249,384.00
7. Attorney's Fees - P300,000.00
8. Amount debited from complainant's ATM [as partial
payment for hospitalization expenses incurred by him which
PPI had advanced] - P12,000.00
9. Retention of complainant's car, as additional penalty for
illegal dismissal.
- For its part, PPI stressed that Caritas was its competitor in the
pre-need plans business, and that Molina and his wife recruited
and enticed some of the sales associates of PPI to work for
Caritas, in violation of its policy against conflict of interest.
Some of these sales associates were the spouses Eppie and
Restie Acosta, Lenita Gatmaitan, Lolita Casaje, Lydia Magalso,
Lydia San Miguel, and Alice Halili, and including Vilma del
Rosario, the secretary of Roy Padiernos. PPI, likewise, averred
that Molina had the habit of coming to the office under the
influence of liquor; he constantly shouted to lady employees
and solicited money from his sales associates in connection
with an official company function without returning the same
after PPI reimbursed him for the expenses incurred;
disseminated intrigues and created divisiveness among the
employees and PPIs senior officers; and disrespected
Padiernos, his superior, by shouting at him during one of the
meetings with other senior officers, and walked out of the
meeting afterwards. Supporting its claims that Molina
committed breach of trust, serious misconduct, fraud, and gross
neglect of duty by reason thereof, PPI appended to its position
paper the statements/affidavits of Marivic Uy, Ruth and Roy
Padiernos, Eppie and Restie Acosta, Celeste Villena, and Vilma
del Rosario.
- On the claim of Molina that he was denied due process, PPI
averred that he was given sufficient opportunity to present his
personal submissions before finally issuing the notice of
dismissal but Molina persistently refused to submit his
explanation. PPI further argued that he was not entitled to the
payment of 13th and 14th month salaries, overriding
commission, profit bonus, actual, moral or exemplary damages,
and attorneys fees. PPI maintained that, under Article 217(a) of
the Labor Code, as amended, and the ruling of this Court in
Baez v. Valdevilla, Molina should be held liable for P1,000,000
as moral damages and an amount not less than P428,400.00 for
the salary he received during the time when the restraining
order/ writ of injunction was erroneously enforced.
- In his Reply, Molina averred that the affidavits submitted by
PPI were antedated since he was never furnished copies of said
reports/affidavits despite demands. PPI even failed to present

A2010

- 220 -

Disini

the reports/affidavits before the RTC where his complaint for


damages against PPI and its officers was pending. He and Roy
Padiernos had been at odds because the latter appointed his
brother and wife as agency manager and group manager of PPI
to which he objected. Molina averred that the P200.00 collected
from each of the employees of PPI during their luncheon
meeting was a voluntary contribution, and that they spent
P4,000.00, more than the amount collected from the
employees. He contended that he had no motive to recruit sales
associates or employees of PPI to be employed by Caritas
because the depletion of sales associates would diminish his
effectiveness as an area manager, including his overriding
commission, profit bonus and fringe benefits. He admitted that
he may have raised his voice in the heat of arguing a point
during meetings, but averred that it should not be considered
as disrespect or misdemeanor.
- Molina further emphasized that Caritas was not a competitor
of PPI, as the former was engaged in selling health care and is
supervised by the Department of Health (DOH), while the latter
is into the business of selling pre-need plans and supervised by
the Securities and Exchange Commission (SEC). Finally, he
averred that the so-called "associates" of PPI were not actually
employees but "independent journeymen" who derived income
on commission basis, free to engage in any kind of selling
activities not in direct competition with PPI.
- Molina admitted having had drinking sessions with Certerio Uy,
Ilustre Acosta and Reynaldo Villena, who provided the hard
liquor and pulutan, but only after office hours. He claimed that
his officemates mistook him for being drunk when he went to
his office even after office hours because of his "mestizo
complexion."
- In its response, PPI averred that, based on the sales data, the
acts of Molina caused demoralization of the sales associates,
resulting in a sudden decrease of the region's output from
P343,009,643.00 in 1998 to P263,099,773.00 in 1999, and
P228,752,090.00 in 2000. PPI insisted that he should be held
liable for not less than P507,348.00, P2,000,000, and
P1,000,000 as actual, moral and exemplary damages, and
attorney's fees, respectively, and P273,600.00 which was the
balance on his car plan agreement with PPI.
- In his Rejoinder and Sur-Rejoinder Molina submitted the
affidavit of Geoffrey Martinez, who belied the reports of Uy,
Villena, Del Rosario, and the spouses Padiernos and Acosta. He
also appended the affidavits of Natividad Gatchalian, San
Miguel, Gatmaitan, and Magalso, who all disputed, in one way
or another, Molina's alleged violations. To counter the
imputations of conflict of interest, Molina also alleged that Abia
and Atty. Reyes were incorporators of Caritas, and that Villena
had in her possession a license to sell Caritas products. With
regard to the declining sales output of his region, Molina
attributed the same to the Asian regional crisis that hit the
Philippines sometime in 1997. He noted, however, that the
same records revealed that despite the financial bane, Metro VI
still managed to be on top from 1998 up to 2000 in terms of its
sales relative to the other regions.
- Molina denied any liability for the car plan, claiming that he
already settled the obligation when PPI demanded full payment
as, in fact, all the papers related thereto, including the Release
of Mortgage, were already in his possession.
- In its Sur-Rejoinder, PPI stressed its claim that Caritas was a
business competitor, as may be inferred from the benefits
available under its health care agreement and the pre-need
contract of PPI. Particularly with regard to the pension plan
contract, it noted the following similarities: (a) Caritas also
provides Term Life Insurance, Accidental Death Insurance,
Credit Life Insurance, and Waiver of Installment Due to
Disability; (b) there are similarities in the provisions on contract
price, grace period, cancellation, reinstatement, and transfer
and termination; and (c) unlike other health care programs that
provide a one-year coverage, renewable every year thereafter,
Caritas offers a continuous five year coverage and sells the
same in units payable in five-year installment basis, with
maturity period and guaranteed return of investment in the
form of Full-Term Medical Expense Fund computed at

Labor Law 1
P10,000.00 for every unit purchased with increment of 10%
yearly after the maturity period, which may be withdrawn in
cash by its member. It stressed that this was similar to the
pension program offered by PPI which was also sold in per unit
basis, payable by installment in certain number of years or
lump sum payment, and upon maturity also gives P10,000.00
pension benefit per unit purchased by the plan holder. With
respect to the alleged interest of Atty. Reyes with Caritas, PPI
adduced in evidence a Deed of Sale to prove that as early as
February 1999 he had already divested his stockholdings in
Caritas.
- On November 18, 2002, Labor Arbiter Roma C. Asinas
rendered a Decision dismissing the complaint and the
counterclaims for lack of merit. The labor arbiter ruled that
Molina was lawfully dismissed from his employment for serious
misconduct in office and fraud or willful breach of trust and
confidence. It declared that Molinas mere denial of the charges
against him did not overthrow the overwhelming evidence
against him tending to show that he committed the allegations
against him. Moreover, his wife was then an agency manager of
Caritas, and some PPI sales associates were with Caritas
because they were recruited by Molina. The labor arbiter also
ruled that other employees of respondent attested to the fact
that they were being recruited and enticed by the complainant
to join Caritas. This act of pirating constituted serious
misconduct in office, fraud or willful breach of trust and
confidence, which are just causes for termination of
employment under Article 282 of the Labor Code, as amended.
As such, PPI could not legally be compelled to continue Molinas
employment due to breach of trust.
- The labor arbiter likewise held that Molina was afforded his
right to due process, but that he refused to give an answer to
the charges leveled against him, and instead insisted that he be
furnished a copy of the alleged reports against him. Since he
was given ample opportunity to answer the charges and explain
his side during the investigation, and a formal or trial-type
hearing is not at all times essential, Molinas right to due
process was not violated. The labor arbiter stressed that the
requirements of due process are satisfied where the parties are
afforded fair and reasonable opportunity to explain their side of
the controversy at hand.
- Molina appealed the decision to the NLRC, which rendered
judgment in his favor. The NLRC reversed the decision of the
Labor Arbiter and ordered Molinas immediate reinstatement to
his former position as Assistant Vice President without demotion
in rank and salary; and the payment of his backwages from
August 1, 2001 up to his actual reinstatement, and other
accrued monetary benefits. However, the NLRC denied all other
claims for damages.
- According to the NLRC, the charges of coming to the office
under the influence of liquor and making PPI reimburse the
expenses already paid by Molina's co-employees were not
supported by the records. The "loss of trust and confidence"
had no factual basis since the alleged acts of Molina did not
result to any loss in favor of PPI.
- Anent Molinas recruitment activities, the NLRC ratiocinated
that PPI failed to show that Caritas was a competitor of PPI.
Caritas caters to the health care needs of its clients while PPI to
the
pre-need
(pension,
educational,
and
memorial)
requirements of its plan holders. Any similarity between PPI and
Caritas extra features like term life insurance, accidental death
insurance, credit life insurance, and waiver of installment due to
disability, did not ipso facto make Caritas a competitor of PPI.
Thus, there was no conflict of interest in Molinas act of trying to
recruit counselors for Caritas to help his wife. Moreover, PPI
failed to establish that recruiting for Caritas affected Molinas
decisions in the performance of his duties with PPI. According to
the NLRC, the drop in the sales and productivity of
complainants area of responsibility may be due to market
forces and depressed economic condition at that time; absent
any clear and convincing proof, it cannot be attributed to the
alleged acts of Molina which constituted willful breach of trust
or confidence.

A2010

- 221 -

Disini

- PPI filed a motion for reconsideration, and appended a Letter


dated June 13, 2002 from the SEC to Caritas, indicating that its
HMO Plan was similar to the previous plans offered by pre-need
companies, hence, under the regulatory suspension of the SEC;
another letter of SEC ordering Caritas to immediately desist
from selling its HMO plan with the full term medial expense
fund; and the letter of Caritas, through counsel, endorsing the
objectionable features of the HMO plan.
- The NLRC, however, was not persuaded, and resolved to deny
PPIs motion in its Order dated September 30, 2003. On
November 19, 2003, the NLRC declared its Decision final and
executory as of November 14, 2003.
- PPI filed a Petition for Certiorari with the CA for the nullification
of the decision and resolution of the NLRC and the
reinstatement of the decision of the Labor Arbiter.
- On August 13, 2004, the CA rendered a decision reversing the
Decision and Resolution of the NLRC, and reinstating the
November 18, 2002 Decision of the Labor Arbiter. Later, the CA
denied Molinas Motion for Reconsideration in its Resolution
dated September 27, 2004.
- The issues for resolution are the following: whether the
decision of the NLRC was already final and executory when PPI
filed its petition for certiorari in the CA; and whether the NLRC
committed grave abuse of discretion amounting to excess or
lack of jurisdiction in issuing the assailed decision and
resolution.
- On the first issue, we find and so hold that the decision of the
NLRC had become final and executory when PPI filed its Petition
for Certiorari in the CA. PPI received a copy of the NLRC
Decision on July 11, 2003 and filed the Motion for
Reconsideration thereof on July 18, 2003, which motion was
denied on September 30, 2003. Under Rule VII, Section 2 of the
NLRC Omnibus Rules of Procedure, the decision of the NLRC
becomes final and executory after ten (10) calendar days from
receipt of the same. PPI received a copy of the NLRC decision
on November 30, 2003; hence, such decision became final and
executory on December 3, 2003. Nonetheless, the Court ruled
in St. Martin Funeral Home v. NLRC that, although the 10-day
period for finality of the NLRC decision may have elapsed as
contemplated in the last paragraph of Section 223 of the Labor
Code, the CA may still take cognizance of and resolve a petition
for certiorari for the nullification of the decision of the NLRC on
jurisdictional and due process considerations. Indeed, the
remedy of the aggrieved party from an adverse decision of the
NLRC is to timely file a motion for reconsideration as a
precondition for any further or subsequent remedy, and if the
motion is denied, such party may file a special civil action in
accordance with law and jurisprudence considering that these
matters are inseparable in resolving the main issue of whether
the NLRC committed grave abuse of discretion.
- The Labor Arbiter and the NLRC act in quasi-judicial capacity in
resolving cases after hearing and on appeal, respectively. On
the presumption that they have already acquired expertise in
their jurisdiction, which is confined on specific matters, their
findings of facts are oftentimes accorded not only with respect
but even finality if supported by substantial evidence. However,
in spite of the statutory provision making "final" the decision of
the NLRC, the Court has taken cognizance of petitions
challenging such decision where there is a clear showing that
there is want of jurisdiction, grave abuse of discretion, violation
of due process, denial of substantial justice, or erroneous
interpretation of law.
- In this case, the Labor Arbiter declared that there is
substantial evidence on record warranting the dismissal of
petitioner as Assistant Vice President for serious misconduct in
office, fraud or willful breach of trust and confidence. The NLRC
disagreed with the Labor Arbiter and reversed the latters
findings. The CA, for its part, concurred with the findings of the
Labor Arbiter. In view of the discordance between the findings
of the Labor Arbiter and the CA on one hand, and the NLRC on
the other, there is a need for the Court to review the factual
findings and the conclusions based on the said findings. As this
Court held in Diamond Motors Corporation v. Court of Appeals:

Labor Law 1
- A disharmony between the factual findings of the Labor Arbiter
and the National Labor Relations Commission opens the door to
a review thereof by this Court. Factual findings of administrative
agencies are not infallible and will be set aside when they fail
the test of arbitrariness. Moreover, when the findings of the
National Labor Relations Commission contradict those of the
labor arbiter, this Court, in the exercise of its equity jurisdiction,
may look into the records of the case and reexamine the
questioned findings
- Article 282 of the Labor Code of the Philippines provides:
Art. 282. Termination by employer. An employer may
terminate an employment for any of the following causes:
a. Serious misconduct or willful disobedience by the
employee of the lawful orders of his employer or
representative in connection with his work;
b. Gross and habitual neglect by the employee of his duties;
c. Fraud or willful breach by the employee of his duties of the
trust reposed in him by his employer or duly authorized
representative;
d. Commission of a crime or offense by the employee against
the person of his employer or any immediate member of his
family or his duly authorized representative; and
e. Other causes analogous to the foregoing.
- Misconduct has been defined as improper or wrong conduct;
the transgression of some established and definite rule of
action; a forbidden act, a dereliction of duty, unlawful in
character and implies wrongful intent and not mere error of
judgment. The misconduct to be serious must be of such grave
and aggravated character and not merely trivial and
unimportant. Such misconduct, however, serious, must
nevertheless, be in connection with the employees work to
constitute just cause for his separation.
- The loss of trust and confidence, in turn, must be based on the
willful breach of the trust reposed in the employee by his
employer. Ordinary breach will not suffice. A breach of trust is
willful if it is done intentionally, knowingly and purposely
without justifiable excuse, as distinguished from an act done
carelessly, thoughtlessly, heedlessly or inadvertently.
The Court has laid down the guidelines for the application of the
doctrine for loss of confidence, thus:
1. the loss of confidence must not be simulated;
2. it should not be used as a subterfuge for causes which are
illegal, improper or unjustified;
3. it may not be arbitrarily asserted in the face of
overwhelming evidence to the contrary;
4. it must be genuine, not a mere afterthought, to justify
earlier action taken in bad faith; and
5. the employee involved holds a position of trust and
confidence.
In Samson v. Court of Appeals, the Court enumerated the
conditions for one to be considered a managerial employee:
(1) Their primary duty consists of the management of the
establishment in which they are employed or of a department
or subdivision thereof;
(2) They customarily and regularly direct the work of two or
more employees therein;
(3) They have the authority to hire or fire other employees of
lower rank; or their suggestions and recommendations as to
the hiring and firing and as to the promotion or any other
change of status of other employees are given particular
weight.
- As a general rule, employers are allowed wide latitude of
discretion in terminating the employment of managerial
personnel. The mere existence of a basis for believing that such
employee has breached the trust and confidence of his
employer would suffice for his dismissal.
- In this case, petitioner was not a mere employee of
respondent. He was the Assistant Vice-President with the same
functions of a regional manager of the same sales region, Metro
Manila VI. Taking into account his job description, he was one of
the top managers of the respondent, tasked to perform key and
sensitive functions in the interest of his employer and, thus,
bound by the more exacting work ethic.

A2010

- 222 -

Disini

- We find, however, that the charge of misappropriation of funds


was not proven with substantial evidence. As gleaned from the
handwritten statement of Ilustre Acosta, the General Manager
of the Springs and Blessings General Agency under Metro
Manila VI, it appears that, aside from him and petitioner, there
were 10 other attendees during the luncheon conference on
November 27, 1999 at the Barrio Fiesta, Cubao, Quezon City.
Petitioner received the amount of only P2,386.00 from
respondent to pay for the cost of the luncheon for the
conference, based on Petty Cash Voucher signed by petitioner, 74
but the conferees spent more than P4,000.00. Upon petitioners
suggestion, the conferees agreed to contribute P200.00 each,
or the total amount of P2000.00 to answer for the difference.
Petitioner had no obligation to return the contributions of the
conferees, nor was he liable for said amount. Significantly,
except for Ilustre Acosta, the other attendees in the conference
never complained against petitioner or requested him to return
their respective contributions of P200.00.
- Regarding the charge that the petitioner peddled false and
malicious informations against Abia and Padiernos, Abia has not
executed any affidavit to confirm paragraph 9 of the affidavit of
Roy Padiernos. As admitted by del Rosario, the informations
allegedly relayed to her by the petitioner pertaining to Roy
Padiernos were confirmed by Zita Domingo.
- The petitioner does not deny having had a heated exchange of
words with Roy Padiernos in the course of a meeting. However,
such incident does not constitute proof that the petitioner
thereby showed disrespect to Roy Padiernos, nor a valid cause
for petitioners dismissal. It does happen that in the course of
exchange of views during a meeting, participants may become
so assertive to the point of being overbearing or unyielding and
in the process lose their temper, on their sincere belief of being
right. There is no evidence on record that petitioner committed
the same or similar acts thereafter.
- To prove its charge of conduct unbecoming of a company
officer, more specifically of drinking alcoholic beverages within
the premises of the company during office hours or going to
work drunk, respondent relied on the statement/affidavit of
Celeste Villena, the Agency Manager of the Wondrous and
Miraculous General Agency under Metro Manila VI; and Marivic
Uy, the General Manager of the DMBP General Agency under
Metro Manila VI. Both claimed that they always saw petitioner
drunk during office hours, most especially during cut-offs when
many sales counselors were present. Petitioner admitted having
had drinking sessions with Certerio Uy, the husband of Marivic
Uy, Ilustre Acosta and Reynaldo Villena, the husband of Celeste
Villena, and who, according to petitioner, provided the hard
liquor and the pulutan. He, however, denied reporting to office
drunk and insisted that he reported for work sober.
- We are inclined to give credence to petitioners claim, noting
that in her handwritten letter-report to Norman Gonzales dated
March 10, 2000, Villena made no mention of the petitioner
going to office drunk. It was only in her affidavit dated January
16, 2002 that Villena made such declaration. Villena did not
explain her failure to report the matter to Gonzales on March
10, 2000, and why she made the charge for the first time in her
Affidavit dated January 16, 2002. Uy is the wife of no less than
Certerio Uy, the Senior Vice-President of the Manila North Sales
Division of respondent. If petitioners "drinking problem" had
any ring of truth, she should have immediately reported the
matter to her husband or to other officials concerned. Uys
unexplained silence until March 10, 2000 thus renders her
report implausible.
- Respondent avers that petitioner served directly as agent of
Caritas, a business competitor of the respondent, when he
connived with his wife in recruiting Sales Associates of the
Metro Sales Division VI to transfer to Caritas as sales associates.
Respondent claims that, by his acts, petitioner failed to
dedicate his full time on the job with respondent and prevented
said sales associates from doing the same. Aside from violating
its policy against conflict of interest, petitioners acts adversely
affected his decisions in the performance of his duties and
obligations to respondent.

Labor Law 1
- Loyalty of an employee to his employer consists of certain
very basic and common sense obligations. An employee must
not, while employed, act contrary to the employers interest.
The scope of the duty of loyalty that an employee owes to his
employer may vary with the nature of their relationship.
Employees occupying a position of trust and confidence owe a
higher duty than those performing low-level tasks. Assisting an
employees competitor can even constitute a breach of the
employees duty of loyalty. An employees self-dealing may
breach that duty. However, it has been ruled that
- A reality of contemporary life is that many families will consist
of two wage earners, one wage earner with two jobs, or both.
For some employees, particularly those earning low or modest
incomes, second sources of income are an economic necessity.
For them, a second job or "moonlighting" is the only way to
make ends meet. Conversely, employers need the assurance
that employees will not disserve them by furthering their own
interests or those of competitors at the employers expense.
- A slight assistance to a direct competitor could constitute a
breach of the employees duty of loyalty. However, when
competition is indirect or minimal, the employer may be
required to show that the employee received substantial
assistance from the competitor. If an employee usurped a
corporate opportunity or secretly profited from a competitive
activity, the employer may receive the value of the lost
opportunity or the secret profit.
- An employees skill, aptitude, and other subjective knowledge
obtained in the course of employment are not the property of
his employer. However, an employee occupying a managerial
position or office is obliged to protect the trade secret of his
employer consisting of formula, process, device or compilation
which it uses in its business and gives it an opportunity to
obtain an advantage over competitors who do not know of such
trade secret. However, the rule does not apply to a matter of
public knowledge or of general knowledge within the industry.
Moreover, an employer has a protectible interest in the
customer relationships of its former employee established
and/or nurtured while employed by the employer, and is
entitled to protect itself from the risk that a former employee
might appropriate customers by taking unfair advantage of the
contract developed while working for the employer. While
acting as an agent of his employer, an employee owes the duty
of fidelity and loyalty. Being a fiduciary, he cannot act
inconsistently with his agency or trust. He cannot solicit his
employers customers or co-employees for himself or for a
business competitor of his employer. If such employee or officer
connives with and induces another to betray his employer in
favor of a business competitor of his employer, he is held
accountable for his mischief.
- In this case, we are not persuaded that Caritas is the business
competitor of respondent. The evidence on record shows that
while Abia, the Senior Vice-President of respondents Metro
Manila Marketing, is one of the incorporators of Caritas and is
even a member of the Board of Directors, respondent did not
dismiss him from employment. The Head of the Legal Division
of the respondent, Atty. Reyes, was also an incorporator of
Caritas and a member of its Board of Directors, and although he
appears to have sold his shares to Herminigildo C. Belen for
P127,312.34, he only did so on March 7, 1999. There is no
evidence on record whether the transfer of such shares of
stocks has already been reflected in the books of Caritas.
Celeste Villena, one of the Sales Associates of respondent, is
herself licensed by Caritas to sell plans for the latter. Villena has
likewise not been prohibited from selling pre-need plans for
Caritas. Fe Molina, who is the head of a sales agency of Caritas,
is also a sales agency head of respondent. Petitioner, his wife,
and Villena were not charged nor meted any sanction by the
respondent for conflict of interest. Petitioner was the Assistant
Vice-President, Marketing Head, Area 10, of Caritas, and for a
while, without any protest from respondent. If Caritas is a
business competitor of the respondent, it should have meted
sanctions not only on petitioner but also on Abia, Reyes, Fe
Molina and Villena as well.

A2010

- 223 -

Disini

- The truth of the matter is that, as averred by Caritas President


Geoffrey Martinez, Caritas is engaged in health care and
hospitalization package, whereas respondent sells educational,
pension, and pre-need plans. Caritas is an HMO and is directly
supervised by the DOH, while respondent is under the
supervision of the SEC. The so-called sales associates of the
respondent are non-salaried employees and are paid on
commission basis only. Their commissions are based on their
individual initiative and industry. That the contracts executed
by the beneficiaries of both corporations have similar provisions
regarding
contract
price,
grace
period,
cancellation,
reinstatement, transfer and termination, do not constitute proof
that Caritas and respondent are business competitors. There is
also no proof that the two corporations compete with each
other in the same or similar business; in fact, the business of
Caritas and that of the respondent complement each other.
- Respondent relied on the declarations of Ruth Padiernos,
Spouses Eppie and Ilustre Acosta, Celeste Villena, and Marivic
Uy to prove its charge that Fe Molina pirated sales associates
working for respondent and that petitioner tolerated the
actuations of his wife and even connived with her.
- The Court finds, however, that the evidence adduced by
respondent insufficient to warrant the petitioners dismissal
from employment.
- Ruth Padiernos, wife of Roy Padiernos, averred in her written
statement dated March 8, 2000, that as far back as July 1999,
she had a conference with her husband and Abia where she
reported that petitioner connived with his wife in pirating sales
associates. She was assured that something would be done to
arrest the problem.90 However, Ruth Padiernos failed to name
any such sales associate who was recruited by Fe Molina. There
is likewise no evidence that Abia ever confronted petitioner
relative to the charge. Roy Padiernos confronted petitioner, but
the latter denied the charge. Since then, no further action was
taken against the petitioner by respondent, until the letter of
Picazo dated March 21, 2000 was sent to him. Roy Padiernos
did not explain why he executed his affidavit regarding the
matter almost three years later, only on January 18, 2002. In an
Affidavit dated January 18, 2002, it was made to appear that
Ruth Padiernos claimed that petitioners wife, the Unit Manager
of the Ark Group under Metro Manila Sales Group VI and also an
Agency Manager of Caritas, recruited sales associates under
respondent to work for Caritas, and that petitioner did the
same; and that she (Padiernos) learned that almost all the
productive Sales Associates in Metro Manila VI were already
connected with Caritas, using "different names." Although
notarized, the affidavit has no probative weight because it was
unsigned.
- Celeste Villena, for her part, declared in her handwritten
statement dated March 10, 2000 that Fe Molina recruited Lenie
Gatmaitan to join Caritas and that she confronted petitioner.92
In her Affidavit dated January 16, 2002, she alleged that
petitioner and his wife, Fe Molina, recruited Gatmaitan to join
Caritas. However, the signature of the notary public does not
appear in said affidavit. For his part, Ilustre Acosta, averred in
his handwritten statement dated March 11, 2000, that on March
4, 2000, petitioner informed him that Geoffrey Martinez called
petitioner to inquire if petitioner would have no objection for
him (Ilustre) to be with Caritas and that petitioner replied that
he had no objection if that was Ilustres decision. Ilustre
maintained this claim in his Affidavit dated January 16, 2002.
Eppie Acosta, the wife of Ilustre Acosta, averred in her
handwritten statement of March 12, 2000, that on March 6,
2000, petitioner commented about their low sales production,
and she retorted that he was the cause, hence, may have
grudges against him. Petitioner replied that he and his wife did
not interfere with each others business dealings, and that
petitioner even declared "Mare, for all you know, ikaw na lang
ang hindi nag-ca-Caritas." She reiterated her claim in her
affidavit dated January 16, 2000. Marivic Uy averred that the
wife of petitioner had been pirating sales associates of
respondent since 1999 to join Caritas and that she tried to
recruit Morena Siasoco, one of the Group Managers. Petitioner

Labor Law 1
failed to stop his wife, but rather tolerated her actuations. She
reiterated her claim in her Affidavit dated January 16, 2002
- However, there is no evidence on record to prove that
respondent expressly prohibited its Sales Associates from
selling for Caritas. Neither is there evidence on record to prove
that Caritas prohibited its sales associates from selling pre-need
plans of respondent.
- Respondent likewise failed to present the affidavits of Siasoco,
Casaje, Magalso, San Miguel and Halili. In contrast to the
evidence of respondent, Gatchalian, San Miguel, Siasoco, and
Gatmaitan executed their respective affidavits declaring that
neither petitioner nor his wife ever recruited them.99 They
admitted that they sold plans for Caritas, but without any
prodding from petitioner and his wife. Geoffrey Martinez
declared, in his affidavit, that Siasoco, San Miguel, Casaje,
Magalso, and Halili joined Caritas voluntarily and individually,
through him, and he was not aware that petitioner and his wife
recommended them to Caritas. Lenita Gatmaitan called him and
inquired if she could join Caritas, and he replied in the
affirmative. He never called petitioner concerning Ilustre
Acosta; on the contrary, it was the latter who called to inquire if
he was entitled to a discount if he purchased a Caritas health
plan. He talked to Vilma Del Rosario and convinced her to apply
as Branch Manager of Caritas, which she did, but backed out
later on.
Disposition IN LIGHT OF ALL THE FOREGOING, the instant
petition is hereby GRANTED. The August 13, 2004 Decision and
September 27, 2004 Resolution of the Court of Appeals are
REVERSED AND SET ASIDE. The decision and resolution of the
NLRC are reinstated.

WILLFUL DISOBEDIENCE
MICRO SALES OPERATION NETWORK V NLRC
472 SCRA 328
QUISUMBING; October 11, 2005
NATURE
For review on certiorari of the Resolutions the CA dismissing
petitioners special civil action for certiorari against the NLRC
Resolution, which affirmed the Labor Arbiters Decision finding
petitioners herein liable for illegal dismissal.
FACTS
- Micro Sales Operation Network is a domestic corporation
engaged in local transportation of goods by land. Petitioner
Willy Bendol was the companys operations manager at the
time of the controversy.
- Private respondents Larry Hermosa, Leonardo de Castro, and
Ramil Basinillo were employed by the company as driver,
warehouseman, and helper, respectively. Hermosa was hired on
November 17, 1997, de Castro on February 1, 1996, and
Basinillo on February 4, 1998.
- Hermosa failed to promptly surrender the ignition key of the
companys vehicle after discharging his duties. Such failure was
allegedly contrary to the companys standard operating
procedure. Thus, he was asked to explain within 24 hours why
disciplinary action should not be meted on him. He explained
that he kept the ignition key because the vehicle was stalled
when its battery broke down. Unsatisfied with Hermosas
explanation, the company dismissed him on January 9, 1999.
- LA found that private respondents were illegally dismissed.
NLRC affirmed the Labor Arbiters decision. It also denied
petitioners motion for reconsideration.
CA dismissed the petition for being defective in form.
ISSUES
1. WON the private respondents were unjustly dismissed
2. WON there was willful disobedience on the part of the
private respondents, justifying their dismissal

A2010

- 224 -

Disini

HELD
1. YES
- Hermosa was unjustly dismissed
2. NO
- For willful disobedience to be a valid cause for dismissal, the
following twin elements must concur: (1) the employee's
assailed conduct must have been willful, that is, characterized
by a wrongful and perverse attitude; and (2) the order violated
must have been reasonable, lawful, made known to the
employee and must pertain to the duties which he had been
engaged to discharge.
- Both elements are lacking. We find no hint of perverse attitude
in Hermosas written explanation. On the contrary, it appears
that the alleged company procedure for leaving the ignition key
of the companys vehicles within office premises was not even
made known to him. Petitioners failed to prove Hermosa
willfully disobeyed the said company procedure. At any rate,
dismissal was too harsh a penalty for the omission imputed to
him.
Disposition NLRC Resolution affirming the Labor Arbiters
Decision, finding petitioners liable for illegal dismissal, is
AFFIRMED.

BASCON V CA (METRO CEBU COMMUNITY


HOSPITAL)
422 SCRA 122
QUISUMBING; February 5, 2004
FACTS
- ELIZABETH BASCON and NOEMI COLE, petitioners, were
employees of Metro Cebu Community Hospital, Inc. (MCCH) and
members of the Nagkahiusang Mamumuo sa Metro Cebu
Community Hospital (NAMA-MCCH), a labor union of MCCH
employees. Bascon had been employed as a nurse by MCCH
since May 1984. At the time of her termination from
employment in April 1996, she already held the position of
Head Nurse. Cole had been working as a nursing aide with
MCCH since August 1974. Both were dismissed for allegedly
participating in an illegal strike.
- The controversy arose from an intra-union conflict between
the NAMA-MCCH and the National Labor Federation (NFL), the
mother federation of NAMA-MCCH. In November 1995, NAMAMCCH asked MCCH to renew their CBA, which was set to expire
on December 31, 1995. NFL, however, opposed this move.
Mindful of the apparent intra-union dispute, MCCH decided to
defer the CBA negotiations until there was a determination as to
which union had the right to negotiate a new CBA. Believing
that their union was the certified CBA agent, NAMA-MCCH
staged a series of mass actions inside MCCHs premises starting
February 27, 1996. The DOLE in Region 7 issued two
certifications stating that NAMA-MCCH was not a registered
labor organization. This finding, however, did not deter NAMAMCCH from filing a notice of strike with the Region 7 Office of
the National Conciliation and Mediation Board (NCMB). Said
notice was, however, disregarded by the NCMB for want of legal
personality of the union.
- MCCH notified the petitioners that they were to be
investigated for their activities in the mass actions. Petitioners,
however, denied receiving said notices. In a notice dated April
8, 1996, MCCH ordered petitioners to desist from participating
in the mass actions conducted in the hospital premises with a
warning that non-compliance would result in the imposition of
disciplinary measures. Petitioners again claimed they did not
receive said order. Bascon and Cole were then served notices
terminating their employment effective April 12, 1996 and April
19, 1996, respectively.
- The Labor Arbiter found the termination to be valid and legal.
The Labor Arbiter held that petitioners were justly dismissed
because they actually participated in the illegal mass action. It
also concluded that petitioners received the notices of hearing,
but deliberately refused to attend the scheduled investigation.

Labor Law 1
- The NLRC reversed the ruling and ordered the reinstatement
of petitioners with full back wages. First, it found that
petitioners merely wore armbands for union identity, per
instruction of their union officials. Said wearing of armbands
while nursing patients, is a constitutional right, which cannot be
curtailed if peacefully carried out. Second, it ruled that the
placards complained of by MCCH did not contain scurrilous,
indecent or libelous remarks. Finally, it concluded that, in a
belated but crude attempt to camouflage the illegal dismissal of
petitioners, MCCH merely fabricated the notices allegedly sent
to petitioners. On the charge of gross insubordination, it ruled
that petitioners were not guilty, because the elements had not
been sufficiently proven, to wit: (1) reasonableness and
lawfulness of the order or directive, (2) sufficiency of knowledge
on the part of the employee of such order, and (3) the
connection of the order with the duties which the employee had
been engaged to discharge.
- MCCH filed a special civil action for certiorari before the CA.
The CA granted the petition but ordered payment of separation
pay.
ISSUES
1. WON petitioners were validly terminated for allegedly
participating in an illegal strike
2. WON petitioners were validly terminated for gross
insubordination to the order to stop wearing armbands and
putting up [of] placards
HELD
1. NO
Ratio While a union officer can be terminated for mere
participation in an illegal strike, an ordinary striking employee
must have participated in the commission of illegal acts during
the strike. There must be proof that they committed illegal acts
during the strike. But proof beyond reasonable doubt is not
required. Substantial evidence, which may justify the imposition
of the penalty of dismissal, may suffice.
Reasoning
- Article 264 (a) of the Labor Code provides in part that:
Any union officer who knowingly participates in illegal strike
and any worker or union officer who knowingly participates in
the commission of illegal acts during a strike may be declared
to have lost his employment status
- The CA found that petitioners actual participation in the illegal
strike was limited to wearing armbands and putting up
placards. There was no finding that the armbands or the
placards contained offensive words or symbols. Thus, neither
such wearing of armbands nor said putting up of placards can
be construed as an illegal act. In fact, per se, they are within
the mantle of constitutional protection under freedom of
speech. Evidence shows that various illegal acts were
committed by unidentified union members in the course of the
protracted mass action. And we commiserate with MCCH,
patients, and third parties for the damage they suffered. But we
cannot hold petitioners responsible for acts they did not
commit. The law, obviously solicitous of the welfare of the
common worker, requires, before termination may be
considered, that an ordinary union member must have
knowingly participated in the commission of illegal acts during a
strike.
2. NO
Ratio Willful disobedience of the employers lawful orders, as a
just cause for dismissal of an employee, envisages the
concurrence of at least two requisites: (1) the employee's
assailed conduct must have been willful, that is, characterized
by a wrongful and perverse attitude; and (2) the order violated
must have been reasonable, lawful, made known to the
employee and must pertain to the duties which he had been
engaged to discharge.

Reasoning

A2010

- 225 -

Disini

- Article 282 of the Labor Code provides in part:


An employer may terminate an employment for any of the
following causes: (a) Serious misconduct or willful
disobedience by the employee of the lawful orders of his
employer or representative in connection with his work.
- We find lacking the element of willfulness characterized by a
perverse mental attitude on the part of petitioners in disobeying
their employers order as to warrant the ultimate penalty of
dismissal. Wearing armbands and putting up placards to
express ones views without violating the rights of third parties,
are legal per se and even constitutionally protected. Thus,
MCCH could have done well to respect petitioners right to
freedom of speech instead of threatening them with disciplinary
action and eventually terminating them.
- Neither are we convinced that petitioners exercise of the right
to freedom of speech should be taken in conjunction with the
illegal acts committed by other union members in the course of
the series of mass actions. It bears stressing that said illegal
acts were committed by other union members after petitioners
were already terminated, not during the time that the latter
wore armbands and put up placards.
- Finally, even if willful disobedience may be properly
appreciated, still, the penalty of dismissal is too harsh. Not
every case of willful disobedience by an employee of a lawful
work-connected order of the employer may be penalized with
dismissal. There must be reasonable proportionality between,
on the one hand, the willful disobedience by the employee and,
on the other hand, the penalty imposed. In this case, evidence
is wanting on the depravity of conduct and willfulness of the
disobedience on the part of petitioners, as contemplated by
law. Wearing armbands to signify union membership and
putting up placards to express their views cannot be of such
great dimension as to warrant the extreme penalty of dismissal,
especially considering their long years of service and the fact
that they have not been subject of any disciplinary action in the
course of their employment with MCCH.
Disposition Petition is GRANTED. The Decision of the CA is
REVERSED. MCCH is hereby ordered to reinstate petitioners
without loss of seniority rights and other privileges and to pay
them full back wages, inclusive of allowances, and other
benefits computed from the time they were dismissed up to the
time of their actual reinstatement.

R TRANSPORT CORP V EJANELRA


[PAGE 55]

B. GROSS AND HABITUAL NEGLECT OF


DUTIES
REQUISITES
JUDY PHILIPIINES V NLRC
289 SCRA 755
MARTINEZ; April 29, 1998
NATURE
Special civil action for certiorari to annul NLRC decision
FACTS
- Virginia Antiola was employed as an assorter of baby infant
dresses by Judy Philippines, Inc. in its export business. She was
directed by her supervisor, to sort out baby infant dresses
pursuant to an instruction sheet.
- Petitioner required Antiola to explain in writing why she should
not be meted disciplinary sanctions for her erroneous
assortment and packaging of 2,680 dozens of infant wear. She
admitted her error and asked for forgiveness. Antiolas
supervisor and the packer also received a memo requiring them
to explain why they should not be penalized. Both submitted
their explanations.

Labor Law 1
- Petitioner found Antiola guilty of negligence and she was
dismissed from employment. The supervisor was suspended for
one month on the ground of negligence through command
responsibility. The packer was found innocent on the ground
that when she undertook the packing of the infant wear, the
same were already sealed in black plastic bags and could no
longer be checked.
- The National Federation of Labor Union (NAFLU), in behalf of
Antiola, filed a complaint for unfair labor practice and illegal
dismissal against Judy Philippines, Inc. They alleged that the
dismissal was unjustified because the infant wear erroneously
assorted by Antiola should not have been shipped to the buyer
had the companys supervisor and the buyers quality
comptroller exercised due diligence in the performance of their
duties in ensuring that the goods were properly assorted.
- Labor arbiter held that the dismissal was lawful on the ground
of fault and negligence causing an irreparable damage to the
goodwill of the petitioners business, especially considering that
the latter is an export oriented entity
- NLRC held that to qualify as a valid cause for dismissal under
Art. 282(b) of the Labor Code, neglect must not only be gross, it
should be Gross and habitual neglect in character. NLRC
ordered petitioner to reinstate Antiola, with one year
backwages
ISSUES
1. WON the appeal before the NLRC had been seasonably made
2. WON the offense committed by Antiola constitute a just
cause for dismissal under article 282 of the labor code.
HELD
1. YES
- Under Article 223 of the Labor Code, as amended, the period
to appeal to the Commission is ten calendar days, to wit:
Article 223. Appeal. - Decisions, awards, or orders of
the Labor Arbiter are final and executory unless
appealed to the Commission by any or both parties
within ten (10) calendar days from receipt of such
decisions, awards or orders.
- It is admitted that Antiola received the labor arbiters decision
on May 2, 1990. She filed her appeal on May 14, 1990, a
Monday.
- In subsequent cases, We ruled that if the tenth day to perfect
an appeal from the decision of the Labor Arbiter to the NLRC
falls on a Saturday, the appeal shall be made on the next
working day as embodied in Section 1, Rule VI of the NLRC
Rules of Procedure promulgated on January 14, 1992. This
conclusion recognizes the fact that on Saturdays the offices of
NLRC and certain post offices are closed.
- Even assuming arguendo that the appeal was filed beyond the
period allowed by law, technical rules of procedure in labor
cases are not to be strictly applied if the result would be
detrimental to the working man.
2. NO
- Gross negligence implies a want or absence of or failure to
exercise slight care or diligence, or the entire absence of care.
It evinces a thoughtless disregard of consequences without
exerting any effort to avoid them.
- Article 282 (b) of the Labor Code requires that xxx such
neglect must not only be gross, it should be Gross and
habitual neglect in character.
- The employers obligation to give his workers just
compensation and treatment carries with it the corollary right to
expect from the workers adequate work, diligence and good
conduct.
- Considering however that private respondent worked with the
company for 4 years with no known previous bad record, the
ends of social and compassionate justice would be better
served if she was merely suspended from work rather than
terminated.
- Petitioner should be reinstated but not awarded backwages.
RA 6715, which provides that an illegally dismissed employee is
entitled to full backwages, inclusive of allowances, and to his
other benefits or their monetary equivalent computed from the

A2010

Disini

- 226 -

time his compensation was withheld from him up to the time of


his actual reinstatement, has no retroactive effect.
Disposition NLRC decision AFFIRMED but MODIFIED in that
petitioner. is ordered to pay private respondent Virginia Antiola
backwages for a period of three years, without qualification or
deduction.

CHAVEZ V NLRC
[PAGE 59]
CHALLENGE SOCKS CORP V CA (NLRC, ANTONIO
ET AL)
474 SCRA 356
YNARES-SANTIAGO; November 8, 2005
NATURE
CERTIORARI under RULE 45
FACTS
- CHALLENGE SOCKS CORP (CSC) hired Elvie Buguat as knitting
operator.
- In the course of her employment, she incurred absences and
tardiness without prior approval and had been neglectful of her
duties.
- May 25, 1998: she failed to check the socks she was working
on causing excess use of yarn and damage to the socks
design.
- She was suspended for 5 days and warned that a repetition of
the same act would mean dismissal from the service.
- February 2, 1999: she committed the same infraction and was
given a warning.
- Despite the previous warnings, Buguat continued to be
habitually absent and inattentive to her task.
- March 1, 1999: she again failed to properly count the bundle
of socks assigned to her.
- March 2, 1999: CSC terminated her services on grounds of
habitual absenteeism without prior leave, tardiness and neglect
of work.
- Thereafter, Buguat filed a complaint for illegal dismissal.[8]
- LA: Buguat was illegally dismissed; ordered CSC to reinstate
her without loss of seniority rights and benefits, but w/o
backwages; ruled that mistake in counting bundles of socks is
tolerable and should be punished by suspension only.
- NLRC: adopted the findings of LA. Denied CSC's Appeal and
MR.
- CA: reversed and set aside LAs and NLRCs decisions; CSC
was ordered to pay BUGUAT full backwages; remanded to the
Regional LA for the computation of the backwages.
- CA also noted that petitioner failed to comply with the twinnotice requirement in terminating an employee hence, the
dismissal was considered ineffectual.
ISSUE
WON Buguats termination is valid
HELD
YES
Reasoning
- One of the just causes for terminating an employment under
Article 282 of the Labor Code is gross and habitual neglect by
the employee of her duties. This cause includes gross
inefficiency, negligence and carelessness. Such just causes is
derived from the right of the employer to select and engage his
employees.
- As a knitting operator, Elvie was required to check the socks
she was working on and to count the bundles of socks she had
to pack to be forwarded to the Looping Section.
- Her repeated commission of the same offense could be
considered willful disobedience. Elvie, despite the suspension
and warning, continued to disregard the company rules and
regulations.

Labor Law 1
- Habitual neglect implies repeated failure to perform ones
duties for a period of time. Buguats repeated acts of absences
without leave and her frequent tardiness reflect her indifferent
attitude to and lack of motivation in her work. Her repeated
and habitual infractions, committed despite several warnings,
constitute gross misconduct. Habitual absenteeism without
leave constitute gross negligence and is sufficient to justify
termination of an employee.
- Her repeated negligence is not tolerable; neither should it
merit the penalty of suspension only.
- The record of an employee is a relevant consideration in
determining the penalty that should be meted out.
- An employees past misconduct and present behavior must be
taken together in determining the proper imposable penalty.
The totality of infractions or the number of violations committed
during the period of employment shall be considered in
determining the penalty to be imposed upon an erring
employee. The offenses committed by him should not be taken
singly and separately but in their totality. Fitness for continued
employment cannot be compartmentalized into tight little
cubicles of aspects of character, conduct, and ability separate
and independent of each other.
- It is the totality, not the compartmentalization, of such
company infractions that Buguat had consistently committed
which justified her dismissal.
- Terminating an employment is one of petitioners
prerogatives.
- Management has the prerogative to discipline its employees
and to impose appropriate penalties on erring workers pursuant
to company rules and regulations.
- The Court has upheld a companys management prerogatives
so long as they are exercised in good faith for the advancement
of the employers interest and not for the purpose of defeating
or circumventing the rights of the employees under special laws
or under valid agreements.
- In the case at bar, petitioner exercised in good faith its
management prerogative as there is no dispute that Buguat
had been habitually absent, tardy and neglectful of her work, to
the damage and prejudice of the company. Her dismissal was
therefore proper.
- The law imposes many obligations on the employer such as
providing just compensation to workers, observance of the
procedural requirements of notice and hearing in the
termination of employment. On the other hand, the law also
recognizes the right of the employer to expect from its workers
not only good performance, adequate work and diligence, but
also good conduct and loyalty. The employer may not be
compelled to continue to employ such persons whose
continuance in the service will patently be inimical to his
interests.
- The employer has the burden of proving that the dismissed
worker has been served two notices: (1) one to apprise him of
the particular acts or omissions for which his dismissal is
sought, and (2) the other to inform him of his employers
decision to dismiss him.
- A review of the records shows that private respondent was
served a written termination notice on the very day she was
actually dismissed from the service. It was not shown that CSC
notified Elvie in advance of the charge or charges against her
nor was she given an opportunity to refute the charges made
against her.
- Agabon v. National Labor Relations Commission: Upheld as
valid the dismissal for just cause even if there was no
compliance with the requirements of procedural due process.
While the procedural infirmity cannot be cured, it should not
invalidate the dismissal. However, the employer should be held
liable for non-compliance with the procedural requirements of
due process.
Disposition CAS DECISION IS AFFIRMED; backwages is
DELETED; Nominal damages (for violation of Buguats statutory
due process) in the amount of P30,000.00.

Disini

A2010 - 227 GROSS AND HABITUAL NEGLIGENCE


DEFINED
VALIAO V CA
[PAGE 11]
REYES V MAXIMS TEA HOUSE
398 SCRA 288
QUISUMBING; February 27, 2003
NATURE
Peition for review on certiorari of a decision of the Court of
Appeals
FACTS
- Respondent Maxim's Tea House (hereinafter Maxim's for
brevity) had employed Reyes as a driver since October 1995.
He was assigned to its M.H. del Pilar Street, Ermita, Manila
branch. His working hours were from 5:00 P.M. to 3:00 A.M.,
and among his duties was to fetch and bring to their respective
homes the employees of Maxim's after the restaurant closed for
the day.
- In the wee hours of the morning of September 27, 1997,
petitioner was driving a Mitsubishi L300 van and was sent to
fetch some employees of Savannah Moon, a ballroom dancing
establishment in Libis, Quezon City. Petitioner complied and
took his usual route along Julia Vargas Street in Pasig City. He
was headed towards Meralco Avenue at a cruising speed of 50
to 60 kilometers per hour, when he noticed a ten-wheeler truck
coming his way at full speed despite the fact that the latter's
lane had a red signal light on. Petitioner maneuvered to avoid a
collision, but nonetheless the van he was driving struck the
truck. As a result, petitioner and seven of his passengers
sustained physical injuries and both vehicles were damaged
- The management of Maxim's required petitioner to submit,
within forty-eight hours, a written explanation as to what
happened that early morning of September 27, 1997. He
complied but his employer found his explanation unsatisfactory
and as a result he was preventively suspended for thirty (30)
days. Subsequently, Maxim's terminated petitioner for cause.
- Feeling that the vehicular accident was neither a just nor a
valid cause for the severance of his employment, petitioner
filed a complaint for illegal dismissal docketed as NLRC NCR
Case No. 00-12-08773-97. In his decision, the Labor Arbiter
found that petitioner was grossly negligent in failing to avoid
the collision. Instead of filing the requisite pleading for appeal,
petitioner filed a "Motion for Partial Reconsideration" with the
NLRC. The NLRC opted to treat petitioner's motion as an appeal.
The NLRC reversed the decision of the Labor Arbiter on the
ground that there was no negligence on petitioner's part.
Respondents moved for reconsideration of the foregoing
decision, but said motion was denied by the Commission in its
resolution
- Respondents then filed a special civil action for certiorari with
the Court of Appeals, The appellate court decided in favor of the
employer and its manager. Hence, the instant case.
ISSUE
WON petitioners dismissal from employment is valid and legal
HELD
NO
- The issue of whether a party is negligent is a question of fact.
As a rule, the Supreme Court is not a trier of facts and this
applies with greater force in labor cases. But where the findings
of the NLRC and the Labor Arbiter are contradictory, as in this
case, the reviewing court may delve into the records and
examine for itself the questioned findings.
- Under the Labor Code, gross negligence is a valid ground for
an employer to terminate an employee. Gross negligence is
negligence characterized by want of even slight care, acting or

Labor Law 1
omitting to act in a situation where there is a duty to act, not
inadvertently but willfully and intentionally with a conscious
indifference to consequences insofar as other persons may be
affected. In this case, however, there is no substantial basis to
support a finding that petitioner committed gross negligence.
- In sustaining the Labor Arbiter's finding that petitioner was
grossly negligent, the appellate court stressed that the cited
episode was the second vehicular accident involving petitioner,
and as such it "may clearly reflect against [his] attitudinal
character as a driver." The Court notes, however, that the
Commission found that in the first vehicular accident involving
petitioner "he was the victim of the reckless and negligent act
of a fellow driver." An imputation of habitual negligence cannot
be drawn against petitioner, since the earlier accident was not
of his own making.
The test to determine the existence of negligence is as follows:
Did petitioner in doing the alleged negligent act use that
reasonable care and caution which an ordinarily prudent person
would use in the same situation? It is not disputed that
petitioner tried to turn left to avoid a collision. To put it
otherwise, petitioner did not insist on his right of way,
notwithstanding the green light in his lane. Still, the collision
took place as the ten-wheeler careened on the wrong lane.
Clearly, petitioner exerted reasonable effort under the
circumstances to avoid injury not only to himself but also to his
passengers and the van he was driving. To hold that petitioner
was grossly negligent under the circumstances goes against the
factual circumstances shown. It appears to us he was more a
victim of a vehicular accident rather than its cause.
- There being no clear showing that petitioner was culpable for
gross negligence, petitioner's dismissal is illegal.
Disposition Petition granted.

CEBU FILVENEER CORPORATION V NLRC


[PAGE 194]
CITIBANK NA V GATCHALIAN
240 SCRA 212
PUNO; January 18, 1995
FACTS
- Petitioner bank received thirty-one (31) applications from
alleged APBCI employees for the issuance of Citibank credit
cards, popularly known as Mastercard.
- A Citibank employee verified by phone the data which
appeared on the application forms. It was Florence Verendia, as
secretary of the APBCI General Manager, who answered the
check calls. The applications were then approved and the
corresponding new and unsigned credit cards were issued.
Petitioner bank's policy is for new and unsigned credit cards to
be released only to the cardholders concerned or their duly
authorized representatives. However, a Citibank employee may
himself take delivery of new and unsigned credit cards after
accomplishing a Card Pull-Out Request Form wherein the
employee assumes the responsibility of delivering the same to
the cardholder concerned.
- Supnad (an employee of bank) and Verendia, conspired
together to get the fictitious cards. They got seven cards from
bank employee Llonillo. As a result, the two (Supnad and
Verendia) used the cards in commercial establishments causing
injury to the bank in the amount of 200k.
- the Bank found out about this and conducted an investigation
- Investigation resulted in the decision to terminate Llonilla and
to file charges against Verendia and Supnad
-the labor arbiter ruled that Llonilla be reinstated based on
evidence that what Llonilla did was not gross negligence
ISSUE
WON Llonillas negligence was gross
HELD

A2010

- 228 -

Disini

YES
- Gross negligence implies a want or absence of or failure to
exercise slight care or diligence, or the entire absence of care.
It evinces a thoughtless disregard of consequences without
exerting any effort to avoid them. The evidence on record
succinctly established the gross negligence of respondent
Llonillo. She admitted that the first time she was asked by
Verendia to pick up one of the newly approved and unused
credit cards, she immediately acceded. Yet at that time, she
had not personally met nor previously seen Verendia. When
asked how she came to know to whom she would give the card,
respondent Llonillo responded that Verendia described herself
over the phone and that was how she was able to identify
Verendia when she first met her. Thus, on the basis of a mere
description over the telephone, respondent Llonillo delivered
the credit cards to Verendia.
- Furthermore, not only is her negligence gross, it was also
habitual it being found out that she picked up the newly
approved credit cards on five (5) separate occasions and
delivered the same to Verendia and the latter's messenger.
Certainly, these repetitive acts and omissions bespeak of
habituality.
- Company says shes grossly or habitually negligent in the
performance of her duties. The SC said that since she has not
been remiss in the performance of her duties in the past, she
cant be charged with habitual negligence. Neither is her
negligence gross in character. Gross negligence implies a want
or absence of or failure to exercise slight care or diligence or
the entire absence of care. It evinces a thoughtless disregard of
consequences without exerting any effort to avoid them. She
had not the slightest reason to distrust Kun because he was the
GM and appears to have conducted himself well in the
performance of his duties in the past. At most, its error of
judgment, not gross negligence.
Disposition NLRC decision affirmed.

CHUA V NLRC (SCHERING-PLOUGH CORP ET AL)


453 SCRA 244
MELO; March 11, 2005
NATURE
Petition for review on certiorari of a decision and resolution of
the CA
FACTS
- On June 1, 1995, Dennis Chua was hired as a Professional
Medical Representative by Schering-Plough Corporation (SPC),
and thereafter became a regular employee on December 1,
1995.
- As a Professional Medical Representative, he was tasked to
promote SPC and its products to physicians, hospitals,
paramedics, including trade and government outlets in his
assigned territory.
- One of the petitioners duties was to submit a Daily Coverage
Report (DCR) every Monday, or at least to mail the same to the
Field Operations Manager. Furthermore, he was required to
have call cards signed by any of the eighty (80) doctors under
his coverage to show that he indeed visited them and handed
out promotional items. This system enabled the SPC to know
how many doctors the petitioner had visited in a week and the
number of call cards he was required to submit.
- Respondent Roberto Z. Tada, Field Operations Manager of the
corporation for the Bicol Region, noticed that the petitioner filed
his DCRs late, and in batches at that. Specifically, a batch of
DCRs up to January 10, 1997 was filed only on March 13, 1997,
while another batch was filed only on March 18, 1997. The
petitioner also failed to submit the DCRs for the period covering
February 10, 1997 to April 7, 1997. Respondent Tada also
found some discrepancies in the DCRs submitted by the
petitioner.
- On April 6, 1997, respondent Tada confronted the petitioner
regarding the said discrepancies, to which Tada merely replied,

Labor Law 1
Pagbigyan mo na lang ako, boss. Tulungan mo na lang ako,
boss.
- On April 8, 1997, Tada went to the petitioners residence and
confiscated all the paraphernalia used by the latter for his
fieldwork, including the call cards and medicine samples. The
car assigned to the respondent was likewise confiscated.
- On April 9, 1997, the petitioner filed an application for a
three-day sick leave, but indicated therein that he was going
on leave only for two (2) days, from April 10 to 11, 1997.
However, after the lapse of his applied leave of absence, the
petitioner failed to report for work.
- On April 15, 1997, the petitioner had already filed a complaint
for illegal dismissal with the National Labor Relations
Commission (NLRC) against the SPC, Epitacio Titong, Jr. (as
President and General Manager), Danny T. Yu (as Division
Manager) and Roberto Z. Tada (as Field Operations Manager
- On April 16, 1997, the petitioner received a telegram from the
SPC instructing him to report to the office on April 18, 1997 and
to see respondent Danny T. Yu who was the Division Manager.
The petitioner, however, failed to comply.
- On April 18, 1997, respondent Tada sent a Memorandum to
the petitioner requiring the latter to explain the late submission
of DCRs, insufficiency of the information on the call cards, etc.
- The same letter informed the petitioner that he was under
preventive suspension effective April 11, 1997 while the case
was under investigation.
- On May 8, 1997, while the case for illegal dismissal was
pending resolution before the arbitration branch of the NLRC,
the SPC sent another letter to the petitioner, informing him that
his employment was terminated.
- On September 30, 1998, Labor Arbiter Ramon Valentin C.
Reyes rendered a Decision declaring the petitioners dismissal
from employment as illegal. The Labor Arbiter held that the
SPC failed to establish any ground for the petitioners dismissal
and ordered the SPC to reinstate him.
- SPC appealed the decision of the Labor Arbiter to the NLRC.
- On October 19, 1999, the NLRC issued a Resolution, finding
respondent to have validly dismissed complainant.
- The petitioner filed a motion for reconsideration of the said
resolution, but the same was dismissed.
- The petitioner sought relief from the CA, which affirmed, in
toto, the resolution of the NLRC, and consequently denied the
petitioners MFR
ISSUE
WON petitioners dismissal form employment was illegal
HELD
NO
- The petitioners termination from employment was anchored
on the following: (a) gross and habitual neglect; (b) serious
misconduct; and (c) willful disobedience to the lawful orders of
the employer. Thus, it all boils down to the filing of the
requisite DCRs due every Monday. As found by both the NLRC
and the CA, the petitioner failed to file the DCRs on time on
several occasions, and instead filed them in batches.
Furthermore, the petitioner failed to submit the DCRs for
February 10, 1997 to April 7, 1997. Considering that about
ninety percent (90%) of the petitioners work as a medical
representative entails fieldwork, such DCRs were vital to his job;
the DCRs were the primary basis upon which the petitioners
employer could track his accomplishments and work progress.
Without the said DCRs, the employer would have no basis to
determine if the petitioner was actually performing his assigned
tasks or not.
- In the same light, the petitioner also failed to submit several
doctors call cards, and submitted others which were
incomplete; that is, undated although signed by the doctors. It
must be stressed that the said call cards were also vital to the
petitioners fieldwork. The requirement of asking the doctors to
affix their signatures in the call cards, the date of the visit, as
well as the samples and promotional items, if any, given to the
doctors, enabled the SPC to verify whether such doctors were
indeed visited by the petitioner.

A2010

- 229 -

Disini

- Gross negligence under Article 282 of the Labor Code, as


amended, connotes want of care in the performance of ones
duties, while habitual neglect implies repeated failure to
perform ones duties for a period of time, depending upon the
circumstances. Clearly, the petitioners repeated failure to
submit the DCRs on time, as well as the failure to submit the
doctors call cards constitute habitual neglect of duties.
Needless to state, the foregoing clearly indicates that the
employer had a just cause in terminating the petitioners
employment.
***But because there was a violation of the petitioners
statutory right to two notices prior to the termination of his
employment for a just cause, he is entitled to nominal damages
of P30,000.00, absent sufficient evidence to support an award
for actual or moral damages. (In line with the ruling in Agabon)
Disposition The decision of the Court of Appeals is affirmed
with modification that petitioner is entitled to above stated
award for nominal damages..

GENUINO ICE CO INC V MAGPANTAY


[PAGE 206]
PREMIER DEVT BANK V MANTAL
[PAGE 210]

SIMPLE NEGLIGENCE
PAGUIO TRANSPORT CORP V NLRC (MELCHOR)
294 SCRA 657
PANGANIBAN; August 28, 1998
NATURE
Petition for review of NLRC decision
FACTS
- Complainant Wilfredo Melchor was hired by respondent
company as a taxi driver under the "boundary system. He was
to drive the taxi unit assigned to him on a 24-hour schedule per
trip every two 2 days, for which he used to earn an average
income from P500 to P700 per trip, exclusive of the P650
boundary and other deductions.
- He was involved in a vehicular accident along Quirino Ave
when he accidentally bumped a car. He was allegedly advised
to stop working and have a rest. When reported for work, he
was told that his service was no longer needed.
- He then filed complaint for illegal dismissal.
- Paguio maintained that Melchor was not illegally dismissed
since there was no employer-employee relationship. (no control,
no payment of compensation) Even if EER existed,
complainant's termination arose out of a valid cause since he
was already involved in 3 accidents.
- NLRC ruling: there was illegal dismissal
ISSUES
1. WON an employer-employee relationship exists
2. WON dismissal was for a just cause
3. WON Melchor was afforded due process
4. WON doctrine of strained relations applies
HELD
1. YES
Ratio The relationship of taxi owners and taxi drivers is the
same as that between jeepney owners and jeepney drivers
under the "boundary system." This relationship is that of
employer-employee and not of lessor-lessee. The fact that the
drivers do not receive fixed wages but get only the excess of
that so-called boundary they pay to the owner/operator is not
sufficient to withdraw the relationship between them from that
of employer and employee.
Reasoning

Labor Law 1
- He was considered an employee because he was engaged to
perform activities which were usually necessary or desirable in
the usual trade of the employer.
- This is different from lease of chattels, wherein the lessor loses
complete control over the chattel leased. In the case of jeepney
owners/operators and jeepney drivers, the former exercise
supervision and control over the latter.
2. NO
Ratio Employer has the burden of proving that the dismissal of
an employee is for a just cause. The failure of the employer to
discharge this burden means that the dismissal is not justified
and that the employee is entitled to reinstatement and
backwages.
- Mere involvement in an accident, absent any showing of fault
or recklessness on the part of an employee, is not a valid
ground for dismissal.
3. NO
Ratio The twin requirements of notice and hearing are
essential elements of due process. The employer must furnish
the worker two written notices: (1) one to apprise him of the
particular acts or omissions for which his dismissal is sought
and (2) the other to inform him of his employer's decision to
dismiss him. The essence of due process lies simply in an
opportunity to be heard, and not always and indispensably in an
actual hearing.
4. NO
Ratio Strained relations must be demonstrated as a fact.
- The doctrine on "strained relations" cannot be applied
indiscriminately since every labor dispute almost invariably
results in "strained relations"; otherwise, reinstatement can
never be possible simply because some hostility is engendered
between the parties as a result of their disagreement.
Reasoning
- Paguios allegation that private respondent was incompetent
and reckless in his manner of driving, which led to his
involvement in three vehicular accidents, is not supported by
the records. No evidence was properly submitted by petitioner
to prove or give credence to his assertions.

C. FRAUD WILLFUL BREACH OF TRUST


SANTOS V SAN MIGUEL CORPORATION
399 SCRA 172
SANDOVAL-GUTIERREZ; March 14, 2003
NATURE
Petition for review on certiorari
FACTS
- Petitioner Carmelita Santos was appointed Finance Director of
respondent SMCs Beer Division for Luzon Operations.
On September 15, 1987, SMC issued a Memorandum prohibiting
the encashment of personal checks at respondent's Plants and
Sales Offices. Thereafter, SMC noticed that petitioner encashed
her 3 personal checks in various Metro Manila Sales Offices.
SMC commenced an audit investigation. Petitioner received
from respondent an inter-office memorandum requiring her to
explain in writing why no disciplinary action should be taken
against her in view of her unauthorized encashment of her 3
personal checks at respondent's sales offices.
- Petitioner admitted that she encashed three personal checks
at respondent's sales offices but claimed that such act was not
irregular since all personnel in respondent's Beer Division were
allowed to encash their personal checks at any sales office upon
clearance from the region management concerned. She stated
that her encashment of personal checks had prior clearance.
She further clarified that only two of the three checks she
encashed were dishonored for insufficiency of funds, but she
promptly funded the checks upon receipt of notice of such
dishonor, thereby causing no damage to respondent.

A2010

- 230 -

Disini

Meanwhile, the audit results revealed that, aside from


petitioner's reported encashment of 3 personal checks, she had
previously encashed 50 personal checks in varying amounts,
which were not endorsed by the Sales Operations Manager or
the Region Finance Officer. Additionally, petitioner encashed 2
other personal checks. After receiving such report, SMC formed
an Investigating Panel to conduct a full-blown investigation.
- The Investigating Panel found the encashment by petitioner of
her personal checks with the region/sales offices as highly
irregular transactions to the detriment of the Company. It
recommended that Santos be terminated from employment.
In a memorandum, SMC adopted the findings of the
Investigating Panel and informed petitioner of her termination
from employment for abuse of position as Finance Director,
engaging in highly irregular transactions to the detriment of the
company and employer's loss of trust and confidence.
- The complaint filed by petitioner against SMC for illegal
dismissal was dismissed by the Labor Arbiter for lack of merit.
The NLRC reversed the Labor Arbiters decision. Upon an MR
filed by SMC, the NLRC dismissed the complaint filed by Santos.
Hence, this recourse.
ISSUE
WON SMC dismissed the petitioner from employment without
just cause
HELD
NO
- Under the Labor Code, a valid dismissal from employment
requires that: (1) the dismissal must be for any of the causes
expressed in Article 282 of the Labor Code and (2) the
employee must be given an opportunity to be heard and to
defend himself.Article 282(c) of the same Code provides that
"willful breach by the employee of the trust reposed in him by
his employer" is a cause for the termination of employment by
an employer. This ground should be duly established.
Substantial evidence is sufficient as long as such loss of
confidence is well-founded or if the employer has reasonable
ground to believe that the employee concerned is responsible
for the misconduct and her act rendered her unworthy of the
trust and confidence demanded of her position. It must be
shown, though, that the employee concerned holds a position of
trust. The betrayal of this trust is the essence of the offense for
which an employee is penalized.
- Petitioner argues that her position as Finance Director of
respondent's Beer Division is not one of trust but one that is
merely functional and advisory in nature. She possesses no
administrative control over the plants and region finance
officers, including cashiers. She reports to two superiors.
Petitioner's argument is misplaced. As Finance Director, she is
in charge of the custody, handling, care and protection of
respondent's funds. The encashment of her personal checks
and her private use of such funds, albeit for short periods of
time, are contrary to the fiduciary nature of her duties.
- Moreover, petitioner has functional control over all the plant
and region finance officers, including cashiers, within the Luzon
Operations Area. In fact, she is the highest ranking managerial
employee for the finance section of the Luzon Beer Division
Operations. Obviously, her position is a factor in abetting the
encashment of her personal checks.
- Indeed, there is substantial ground for respondent's loss of
confidence in petitioner. She does not deny encashing her
personal checks at respondent's sales offices and diverting for
her own private use the latter's resources. The audit
investigation accounted for all the checks she encashed, some
of which were dishonored for insufficiency of funds. The
Investigating Panel concluded that petitioner not only encashed
her personal checks at respondent's sales offices, but also used
company funds to temporarily satisfy her insufficient accounts.
This Court has held that misappropriation of company funds,
although the shortages had been fully restituted, is a valid
ground to terminate the services of an employee of the
company for loss of trust and confidence.

Labor Law 1
- Petitioner contends that there is a prolonged practice of other
payroll personnel, including persons in managerial levels, who
encashed personal checks but remained unpunished by
respondent. She asserts that her administrative superiors even
encouraged her to encash her checks at the nearest sales office
since her appearance at the bank for encashment would entail
undue digression from her daily work routine.
- Prolonged practice of encashing personal checks among
respondent's payroll personnel does not excuse or justify
petitioner's misdeeds. Her willful and deliberate acts were in
gross violation of respondent's policy against encashment of
personal checks of its personnel, embodied in its Memorandum.
She cannot feign ignorance of such memorandum as she is
duty-bound to keep abreast of company policies related to
financial matters within the corporation. Equally unmeritorious
are her claims that the acts complained of are regular, being
with the knowledge and consent of her superiors, Francisco
Gomez de Liano and Ben Jarmalala, and that she is being
charged because she resisted the sexual advances of her
superior. Suffice it to state that she could have proved these
matters during the investigation had she attended the
proceedings.

LAKPUE DRUG INC V BELGA


[PAGE 208]

LOSS OF CONFIDENCE REQUISITES


JARDINE DAVIES INC V NLRC (REYES)
311 SCRA 289
QUISUMBING; July 28, 1999
FACTS
- Petitioner, the exclusive distributor of Union 76 lubricating
oil, engaged the services of a private investigation agency due
to reports that petitioners products, particularly Union 76, were
being illegally manufactured, blended, packed and distributed.
Upon confirmation of the investigator through a surveillance
report and having secured a search warrant, petitioner seized
some of the fake items found in the apartment of private
respondent, a former sales representative of petitioner.
- a criminal complaint for unfair competition violating Article
189 of the RPC (repealed by Section 239 of the Intellectual
Property Code) was filed against Reyes along with
administrative charges for serious misconduct inimical to the
interest of petitioner. He was advised to go on an indefinite
leave which later led to his termination.
- the materials seized were released in view of a petition filed
by Reyes younger brother Donato, who convinced the court
that the materials belonged to him and that he was legally
engaged in the business of general merchandising (Lubrix
Conglomerate) reselling oil and lubricant products to the public.
- with that, Reyes sued petitioner for illegal dismissal but the
complaint was dismissed by the Labor Arbiter as he thought
otherwise. Upon appeal with the NLRC, the decision was
reversed on the ground that there was no cogent reason for
petitioner to lose trust and confidence in private respondent,
there being no shadow of an act amounting to serious
misconduct, fraud or breach of trust.
- petitioners MFR was denied, hence this petition.
ISSUE
WON there was reason for petitioner to lose trust and
confidence in private respondent and justify his dismissal
HELD
NO
Ratio The right of an employer to dismiss employees on
account of loss of trust of confidence must not be exercised
arbitrarily and without showing just cause, so as not to render

A2010

Disini

- 231 -

the employees constitutional right to security of tenure


nugatory.
Reasoning
- Article 282 provides that an employer may terminate an
employment for fraud or willful breach by the employee of the
trust reposed in him by his employer. It is settled that loss of
confidence as a just cause for termination must be premised on
the fact that an employee concerned holds a position of trust
and confidence, as in this case. And in order to constitute just
cause, the act complained of must be work-related. Proof
beyond reasonable doubt is not required, so long as there is
some basis for the loss of confidence, but basis thereof must
still be clearly and convincingly established, arising from
particular proven facts which the employer bears to prove.
- in the instant case, the surveillance report of the private
investigator was unreliable as the conclusions therein were
mere deductions not supported by substantial corroborating
evidence. Petitioner also failed to controvert proof presented by
private respondent that the reselling of the oil was in support of
petitioners marketing policy. It was also odd that petitioners
agents did not submit the alleged fake merchandise to be
tested in their labs, virtually affirming the articles were genuine,
having been purchased from petitioners dealers.
- another confirmation that petitioner lacked basis for its
distrust of private respondent was the release of the seized
articles, with Donato even presenting receipts to prove they
were purchased from authorized dealers.
- Considering this, private respondent was illegally dismissed.
As such, he is entitled to backwages. Since he was terminated
before the effectivity of RA 6715, he is entitled to only 3 years
of backwages, and not full backwages as would be granted now.
Because the antagonism and imputations of the criminal act
strained the parties relationship, reinstatement would not be
feasible. Instead, a more equitable disposition would be an
award of separation pay.
Disposition instant petition is DENIED for lack of merit

PLDT V TOLENTINO
[PAGE 202]
DELA CRUZ V NLRC
[PAGE 100]
PHILIPPINE NATIONAL CONSTRUCTION
CORPORATION V MATIAS
458 SCRA 148
PANGANIBAN; May 6, 2005
FACTS
- Rolando Matias was employed by Construction and
Development Corporation of the Philippines (CDCP) as Chief
Accountant and Administrative Officer. During his employment
with the company, various parcels of land situated at Don
Carlos Bukidnon were placed in the names of certain employees
as trustees for the purpose of owning vast tracts of land more
than the limit a corporation can own which were primarily
intended for CDCP agricultural businesses.
By internal
arrangement documents transferring back the properties to the
corporation were executed. A piece of land was registered in
the name of Matias.
- CDCP was later converted a government owned or controlled
corporation, and the name of CDCP was changed to Philippine
National Construction Corporation (PNCC). Under a new set up,
PNCC offered a retrenchment program and on December 31,
1984 Matias availed of the said program.
- Sometime in 1985, the Conjuangco Farms owned by Mr.
Danding Conjuangco acquired CDCP Farms Corporation wh[ich]
took over the operations of said farms. Not long after, or in
1989, CDCP Farms Corporation ceased to operate.

Labor Law 1
- In July 1992, two former CDCP employees, namely Reynaldo
Tac-an and Luciano Tadena went to the house of Matias and
brought with them duly accomplished documents and Special
Power of Attorney for his signature and informed him that the
lands in Bukidnon under his name with all the others were
invaded by squatters, and that the said land were covered by
the Comprehensive Agrarian Reform Program (CARP) where
Matias name was included in the list of landowners. Matias
reluctantly signed the document and after six months, he
signed an acknowledgment receipt of P100,000.00.
- The original title registered in the name of Matias was
cancelled and a new title was issued. The transfer of said
parcel of land was made possible because Rolando Matias and
Elena Esmeralda Matias received managers checks from the
Land Bank of the Philippines in the amount of P102,355.96 and
P219.22 and bond worth P203,478.48 as payment of Land
Transfer Acquisition.
- On August 12, 1996, Matias was rehired by PNCC as Project
Controller in Zambales PMMA Project.
- Not long after, Mr.Alday, Head of the Realty Management
Group of PNCC invited Matias to his office and showed him a
listing of parcels of land in the name of different persons with
the corresponding status including the latters name. On the
basis of the listing, Mr. Alday told Matias that the transfer of the
property registered in the latters name was not yet
consummated by the LBP and then requested Matias to execute
a Deed of Assignment in favor of PNCC pertaining to the said
property, which Matias did and guaranteed in writing that the
parcel of land is free from any lien or encumbrance.
- On April 20, 1998, a memorandum was issued to Matias by
PNCC directing the former to explain in writing why none of the
following actions, falsification, estafa, dishonesty, and breach of
trust and confidence, should be taken against him in connection
with the Deed of Assignment. PNCC alleges that respondent
fraudulently breached its trust and confidence when, without its
knowledge and consent, he disposed of the Bukidnon property;
though actually belonging to petitioner, that property had
purportedly been merely placed in trust under his name.
Thereafter, he assigned the same property to petitioner,
allegedly despite his full knowledge that the title had already
been transferred -- with his active planning and participation -to the Republic of the Philippines .
- In due time, Matias submitted his written explanation.
However, he was later advised that he was terminated from the
service on the ground of loss of trust and confidence. Hence,
Matias filed a complaint for illegal dismissal and money claims
against PNCC alleging that the dismissal on the ground of loss
of
trust
and
confidence
was
without
basis.
ISSUE
WON the dismissal of Matias on the ground of loss of trust and
confidence was without basis
HELD
YES
Ratio: To constitute a valid cause to terminate employment,
loss of trust and confidence must be proven clearly and
convincingly by substantial evidence. To be a just cause for
terminating employment, loss of confidence must be
directly to the duties of the employee to show that he or she is
woefully unfit to continue working for the employer.
Reasoning
- Undeniably, the position of project controller -- the position of
respondent at the time of his dismissal -- required trust and
confidence, for it related to the handling of business
expenditures or finances.
However, his act allegedly
constituting breach of trust and confidence was not in any way
related to his official functions and responsibilities as controller.
In fact, the questioned act pertained to an unlawful scheme
deliberately engaged in by petitioner in order to evade a
constitutional and legal mandate.
- It has oft been held that loss of confidence should not be used
as a subterfuge for causes which are illegal, improper and
unjustified. It must be genuine, not a mere afterthought to

A2010

- 232 -

Disini

justify an earlier action taken in bad faith. Be it remembered


that at stake here are the sole means of livelihood, the name
and the reputation of the employee. Thus, petitioner must
prove an actual breach of duty founded on clearly established
facts sufficient to warrant his loss of employment.
- We stress once more that the right of an employer to dismiss
an employee on account of loss of trust and confidence must
not be exercised whimsically. To countenance an arbitrary
exercise of that prerogative is to negate the employees
constitutional right to security of tenure. In other words, the
employer must clearly and convincingly prove by substantial
evidence the facts and incidents upon which loss of confidence
in the employee may be fairly made to rest; otherwise, the
latters dismissal will be rendered illegal.

CRUZ V CA (NLRC, CITYTRUST BANK)


494 SCRA 226
AUSTRIA-MARTINEZ; July 12, 2006
NATURE
Special civil action for certiorari under Rule 65 PROC seeking to
annul CA decision affirming NLRC decision and resolution .
FACTS
- Felix Cruz was an employee of Ciytrust Banking Corporation.
He held a confidential position of Micro Technical
Support Officer, whose duties include: evaluating and
recommending requests for Micro Computers received
by the bidding committee, further evaluating and
accepting of bids done by the Technical Commitee. He
was recognized with awards and citations due to his
good performance.
- There were feedbacks and informations that there were
irregularities in the bidding process and purchase of the
computers. A special investigation was conducted which found
that there were unauthorized and unreported commissions and
rebated given out by one of its computer suppliers (MECO) for
purchases made by Citytrust.
- Citytrust sent a show cause memorandm to Cruz placing him
under preventive suspension and directing him to appear in an
administrative hearing by the Ad Hoc Committee. The
committee found him guilty of fraud, serious misconduct, gross
dishonesty and serious violation of the bank policies. For the
resultant loss of confidence, Citytrust terminated Cruz from
employment.
- Cruz filed before the Labor Arbiter an action for illegal
dismissal and damages for being denied due process and
hastily dismissed. LA decision favored Cruz.
- Citytrust appealed to the NLRC, setting aside LA decision and
dismissing the case fro lack of merit. Cruz filed MFR, but was
denied for lack of merit.
- Cruz filed petition for Certiorari with SC, which was referred to
the CA for appropriate action and disposition.
- CA dismissed the petition. It held that although the signature
of the petitioner does not appear in the check vouchers, other
pieces of evidence prove that he benefited from the proceeds of
the checks issued and that there is substantial evidence to hold
the petitioner liable for soliciting; that his acts constituted a
willful breach of the suppliers trust and confidence; that the
dismissal was the result of a thorough investigation and
hearing.
ISSUES
1. WON CA committed grave abuse of discretion
2. WON he denied due process
HELD
1. NO
- Petitioner failed to prove such.
- Petitioner was dismissed on the ground, among others, of loss
of trust and confidence. Loss of trust and confidence, as a valid
ground for dismissal, must be substantiated by evidence.

Labor Law 1

A2010

- WRT to rank-and-file personnel, loss of trust and confidence


requires proof of involvement in the alleged events in question.
But as regards a managerial employee, the mere existence of a
basis fro believing that such employee has breached the trust
of his employer would suffice for his dismissal. Proof beyond
reasonable doubt is not required, it being sufficient that there is
some basis for such loss of confidence such when the employer
has reasonable ground to believe that the employee concerned
id responsible for the purported misconduct, and the nature of
his participation renders him unworthy of the trust and
confidence demanded by his position.
- Art 282 ( c) LC states that the loss of trust and confidence
must be based on willful breach. It should be done intentionally,
knowingly and purposely without justifiable excuse. It must not
be indiscriminately used as a shield by the employer against a
claim that the dismissal of an employee was arbitrary. And, in
order to constitute a just cause for dismissal, the act
complained of must be work-related and shows that the
employee concerned is unfit to continue working for the
employer. In addition, loss of confidence is premised on the fact
that the employee concerned holds a position of responsibility,
trust and confidence or that the employee concerned is
entrusted with confidence with respect to delicate matters. The
betrayal of this trust is the essence of the offense for which an
employee is penalized
- Cruzs job entails the observance of proper company
procedures. His functions are also extended to all branches
nationwide, involving high degree of responsibility requiring a
substantial amount of trust and confidence.
- Petitioners acceptance of commissions and rebates from
MECO, without knowledge and consent from Citytrust, and
without said rebates being reported and turned over to the
latter, are acts which can be considered willful breach of the
trust and confidence reposed by Citytrust on him.
- An employer cannot be compelled to retain an employee who
s guilty of acts inimical to the interests of the employer.
2. NO, he was not denied due process.
- The basic requirement of notice and hearing in termination
cases is for the employer to inform the employee of the specific
charges against him and to hear his side and defenses. This
does not mean a full adversarial proceeding. The parties may
be heard through pleadings, written explanations, position
papers, memorandum or oral argument.
In all of these
instances, the employer plays an active role by providing the
employee with the opportunity to present his side and answer
the charges in substantial compliance with due process.
- The fact alone that he was not able to confront the witnesses
against him during the investigation conducted by Citytrust
does not mean that he was denied his right to due process.
What is frowned upon is the absolute lack of notice and hearing.
- Citytrust complied with the first requirement of notice when it
informed petitioner through a letter of the charges against him,
directing him to explain in writing why his employment should
not be terminated and to appear in a hearing to be conducted
by the company to give him further opportunity to explain his
side. Citytrust also complied with the second requirement of
notice when it sent a memorandum informing him of his
dismissal from employment and the reasons therefore.
Dispositon instant petition is DISMISSED for lack of merit.

BREACH OF
CONFIDENCE

TRUST

LOSS

OF

CENTRAL PANGASINAN ELEC CORP V MACARAEG


[PAGE 195]

POSITION, TRUST AND CONFIDENCE


SANTOS V SAN MIGUEL CORP
[PAGE 219]

- 233 -

Disini

PANDAY V NLRC (LUZON MAHOGANY TIMBER


INDUSTRIES INC)
209 SCRA 122
GUTIERREZ; May 20, 1992
NATURE
Petition seeking the review of the order rendered by the NLRC
authorizing the separation from the service of Panday to the
payment by the private respondent of separation pay
equivalent to one-half month salary for every year of service. It
likewise ordered the payment of the complainant's 13th month
pay for 1977 but dismissed his claim for living allowance for
lack of merit.
FACTS
- Panday was hired by Luzon Mahogany Timber Industries since
Aug. 23, 1973. Sometime in Dec. 1977, Panday was called by
Martin Gaw, the owner-manager who instructed him to cut off
the living allowance of the employees. Panday requested that a
memorandum to this effect be made so that he would not be
blamed by the workers. Gaw got angry and shouted "what for
is the memorandum? I am telling you to do so." He then
butted, "Ano ba talaga Naning ang ibig mong sabihin? Sa
tuwing magsasalita ka, panay ka "policy" ng companya at
panay ka records".
- From the time of that incident, Panday was deprived of free
light. He was no longer given any accounting work. His per
diem was abruptly cut off. All that was left for him to do was the
simple clerical job of registering or paying SSS premiums. Still
complainant continued to bear it out. In 1979, however he was
totally divested of all his duties and he was compelled to
approach Manager Martin Gaw to clear up matters.
Gaw
referred him to Mr. Gerry Lumban who was supposed to give
him some work to do. It turned out, however, that no such
instructions were given to said Mr. Lumban.
- In 1979, Panday filed a request for vacation leave with pay for
15 days from April 14-30, 1979. On that same day he brought
his son to Manila for medical treatment and stayed there up to
the end of the month. Upon his return to Isabela, he asked for
his salaries only to learn that his application for leave was
disapproved.
- Hence, he filed this case for illegal dismissal, non-payment of
13th month pay for 1977, emergency allowance under P.D. 525
since 1975 up to 1977 and unpaid wages for April 16-30, 1979.
- The Office of the Minister found and ruled that Panday was
constructively dismissed from the service. Luzon Timber was
ordered to comment on the petition but failed to so. Thus, the
actual findings are affirmed. The only complaint of Panday
which remains is his claim that Deputy Minister Vicente
Leogardo, Jr. should have ordered his reinstatement
with backwages.
ISSUE
WON Pandays prayer for reinstatement should have been
granted
HELD
NO
- Panday, as branch accountant occupied a position involving
trust and confidence and in the light of the estranged relation
between the complainant and the respondent that may not
permit the full restoration of an employment relationship based
on trust and confidence, we have to allow termination of the
employer-employee relationship but upon the payment of
separation pay equivalent to one-half (1/2) month for every
year of service rendered.
- The case of Lepanto Consolidated Mining Co. v. Court of
Appeals provides a definition of a "position of trust and
confidence". It is one where a person is "entrusted with
confidence on delicate matters, or with the custody,

Labor Law 1
handling, or care and protection of the employer's
property"
- A few examples were given by the Court in the case of GlobeMackay Cable and Radio Corporation v. National Labor
Relations Commission and Imelda Salazar, G.R. No. 82511,
March 3, 1992, to illustrate the principle:
- where the employee is a Vice-President for Marketing and as
such, enjoys the full trust and confidence of top management
- or is the Officer-In-Charge of the extension office of the bank
where he works
- or is an organizer of a union who was in a position to
sabotage the union's efforts to organize the workers in
commercial and industrial establishments
- or is warehouseman of a non-profit organization whose
primary purpose is to facilitate and maximize voluntary gifts
by foreign individuals and organizations to the Philippines
- or is a manager of its Energy Equipment Sales
- Credit and Collection Supervisor (Tabacalera Insurance Co.
v. National Labor Relations Commission)
- If the respondent had been a laborer, clerk or other rank-andfile employee, there would be no problem in ordering her
reinstatement with facility. An officer in such a key position as
Vice President for Marketing(or as Chief Accountant as in the
present case) can work effectively only if she enjoys the full
trust and confidence of top management.
- The case of Metro Drug Corp. v. National, Labor Relations
Commission, aptly describes the difference in treatment
between the positions of trust on one hand and mere clerical
positions on the other. It states:
Managerial personnel and other employees occupying
positions of trust and confidence are entitled to security of
tenure, fair standards of employment, and the protection of
labor laws. However, the rules on termination of employment,
penalties for infractions, and resort to concerted action are
not necessarily the same as those for ordinary employees.
A special and unique employment relationship exists between
a corporation and its cashiers. More than most key positions,
that of cashier calls f or the utmost trust and confidence. . . .
When an employee accepts a promotion to a managerial
position or to an office requiring full trust and confidence she
gives up some of the rigid guaranties available to ordinary
workers. Infractions which if committed by others would be
overlooked or condoned or penalties mitigated may be visited
with more severe disciplinary action. A company's resort to
acts of self-defense would be more easily justified. It would
be most unfair to require an employer to continue employing
as its cashier a person whom it reasonably believes is no
longer
capable
of
giving
full
and
wholehearted
trustworthiness in the stewardship of company funds.
- Reinstatement in the present case is no longer possible not
only because of the strained relationship between the employee
and the employer but also because of the length of time that
has passed from the date the incident occurred to its resolution.
Instead of reinstating the employee, this Court has in several
cases awarded separation pay although the employee was
found to be illegally dismissed.
The following reasons have been advanced by the Court for
denying reinstatement
- reinstatement can no longer be effected in view of the long
passage of time
- because of the realities of the situation
- that it would be inimical to the employer's interest
- that reinstatement may no longer be feasible
- that it will not serve the best interests of the parties
involved
- that the company would be prejudiced and by the workers'
continued employment
- that it will not serve any prudent purpose as when
supervening facts have transpired which make execution on
that score unjust or inequitable
Disposition the prayer for reinstatement is DENIED but the
order rendered by Deputy Minister Vicente Leogardo, Jr. dated
May 29, 1984 is modified to cover five (5) years backwages.
The order is AFFIRMED in other respects.

A2010

- 234 -

Disini

CRUZ V COCA-COLA BOTTLERS PHILS INC


460 SCRA 340
YNARES-SANTIAGO; June 15, 2005
FACTS
- Cruz has been working for respondent companys plant in
Calamba, Laguna, as a driver/helper since June 1983. At times,
however, Cruz gets designated as Acting Salesman for
companys soft drinks and other beverages. On July 25, 1998,
petitioner was assigned as acting salesman of Route DA1,
covering the small barangays. Together with his helper, Mr.
Pablito Aguila, Cruz loaded their truck with CCBPI products.
After the required verification and confirmation of the products
loaded on the truck by the Checker and the guard at the gate,
Cruz proceeded to leave the plant vicinity.
- After gate inspection, however, Cruz drove back inside the
plant on the pretext of refueling. While waiting in line to refuel,
Cruz allegedly asked Aguila to load an additional thirty cases of
assorted canned soft drinks as plus load. Aguila reminded
him about the required documents but he merely stated Ayos
na and continued with the refueling of the truck.
- On his second exit from the plant premises, Cruz did not slow
down for the mandatory inspection even as the security guards
at the gate flagged him down.
- One of the guards pursued the truck and when he caught up
with petitioner at the Walter Mart Shopping Mall in Barangay
Real, Calamba, the latter could not produce the proper
documents for the extra thirty cases loaded on his truck. He
was then directed to return to the plant and unload the
products. At this point, it was confirmed that Cruz did not
actually secure any paper for the added products nor did he
follow the established procedure before taking out the extra
cases.
- Cruz admitted the incident but alleged that he forgot to secure
the requisite documents for the products. On August 5, 1998,
an investigation was conducted on the alleged violations
committed by petitioner. On August 19, 1998, respondent
company terminated the services of petitioner effective upon
receipt of the memorandum.
ISSUE
WON Cruz was validly dismissed
HELD
YES
- The Labor Arbiter, the NLRC and the Court of Appeals were
unanimous in their findings that petitioner was guilty of
dishonest acts but differed only on the propriety of the penalty
imposed upon petitioner.
- After a careful evaluation of the evidence on record of this
case, we found no compelling reason to disturb the unanimous
findings of the Court of Appeals, the NLRC and the Labor
Arbiter.
- Several factors militate against petitioners claim of good
faith. Petitioners length of service, which spans almost fifteen
years, works against his favor in this case. We have held that
the longer an employee stays in the service of the company,
the greater is his responsibility for knowledge and compliance
with the norms of conduct and the code of discipline in the
company. Considering that petitioner has worked at respondent
company for a long period of time, one expects that securing
the LOGP or TGP would be automatic for him.
- Faced with the overwhelming evidence presented by
respondents on one hand and the mere general denial of
petitioner on the other, the invocation of the protective mantle
of the law in favor of labor cannot be upheld in this case. This
principle cannot be adopted where there is clear and convincing
evidence of the truth. While this court endeavors to live up to

Labor Law 1
its mandate that the workingmans welfare should be the
primordial and paramount consideration, it cannot do so if it will
be at the expense of justice and will result in the oppression or
self-destruction of the employer. The interests of both the
employers and employees are intended to be protected and not
one of them is given undue preference.
- Termination of employment by reason of loss of confidence is
governed by Article 282(c) of the Labor Code, which provides
that an employer can terminate the employment of the
employee concerned for fraud or willful breach by an employee
of the trust reposed in him by his employer or duly authorized
representative. Loss of confidence, as a just cause for
termination of employment, is premised on the fact that the
employee concerned holds a position of responsibility, trust and
confidence. He must be invested with confidence on delicate
matters such as the custody, handling, care and protection of
the employers property and/or funds.
- Admittedly, the company rules violated by Cruz are
punishable, for the first offense, with the penalty of suspension.
However, company has presented evidence showing that Cruz
has a record of other violations from as far back as 1986. To be
sure, the nature of petitioners offenses is downright inimical to
the interests of respondent company. By virtue of his job, Cruz
is entrusted with the property and funds, which belong to
respondent company. His actions on that fateful day highlight,
not only his consistent and deliberate defiance of company
rules and regulation, but also his duplicity in handling
respondent companys properties. It would appear that
company had tolerated his work ethic far too long. We
therefore find that it was justified in terminating petitioner after
the flagrant dishonesty he committed.
Disposition Instant petition is DENIED. Dismissal of petitioner
is declared valid but respondent company is ORDERED to pay
petitioner the amount of P20,000.00 as nominal damages for
non-compliance with statutory due process.

GUIDELINES
VITARICH CORP V NLRC (RECODO)
307 SCRA 509
BELLOSILLO; May 20, 1999
NATURE
Special Civil action in the SC. Certiorari
FACTS
- Private respondent, Isagani Recodo, started working at
Vitarich as an Accounting clerk. He gradually moved up the
organization ladder until he was made Sales Manager for
Western Visayas in 1988. He was dismissed in October 15, 1992
for alleged violation of a memorandum dated August 4, 1992
and also for violation of company policies relating to credit
extensions and cash advances. He was also terminated for loss
of trust and confidence.
- Apparently, his new boss, Onofre Sebastian, was under
pressure from senior management to address and correct all
the problems he had inherited from his predecessor. The
problems included high account receivable level in the sales
territory of Recodo. The two had a meeting sometime middle of
July to address the problems, including the A/R level of one Rex
Cordova.
- The August 4 Memorandum referred to contains instructions to
Recodo to ground salesmen with thirty say overdue A/R so that
the levels of said A/R can be regularized. Apparently, Recodo
received the said memo garbled and had to verify its contents
on September 5, 1992. In the meantime, he postponed the
grounding of Cordova until August 20 to bring about the
desired reduction. The reduction hoped for in fact happened
when Cordovas A/R went down from Pesos 800,000 to just
Pesos 250,000. Huffing and puffing, Sebastian was asked to

A2010

- 235 -

Disini

explain why he should not be terminated for failure to ground


Cordova in compliance with the August 4 memo of Sebastian.
- Recodo complied
with the order to explain and an
investigation was conducted by the Head of Personnel, a certain
Enriquez. In his report, Enriquez found that there was no
defensible ground for terminating Recodos services in the
absence of documented warnings given to Recodo to justify any
loss of trust and confidence in him. Nonetheless, Recodo was
terminated on October 15, 1992.
- Private respondent filed a complaint for illegal termination,
non-payment of managerial bonus, and for moral and
exemplary damages. The Labor Arbiter ruled illegal dismissal.
The NLRC initially overturned the ruling but on appeal by
Recodo, the finding of the Labor Arbiter was upheld. Hence this
action.
ISSUE
WON the NLRC committed a grave abuse of discretion in finding
in favor of Recode
HELD
NO
. In rectifying its previous appreciation and assessment of
Recodos dismissal, the NLRC did not commit any abuse of
discretion. A careful scrutiny of the records reveal that the
decision of the Labor Arbiter is suffused with established facts
and a correct understanding of them.
Reasoning
- While it may be true that there was a delay on the part of
Recodo in implementing his superiors order with regard
Cordovas grounding, the question is whether the delay
constitutes disobedience and whether this disobedience was
willful to merit loss of confidence. The SC, in AHS Philippines,
Inc. vs. CA, explained that willful disobedience of the
employers lawful orders, as a just cause for dismissal of an
employee, envisages the concurrence of at least two requisites:
a. the employees assailed conduct must be willful or
intentional, the willfulness being characterized by a
wrongful and perverse attitude;
b. the order violated must have been reasonable, lawful,
made known to the employee and must pertain to the
duties
which he had been engaged to discharge.
- In the case at bar, the non-compliance by Recodo was not an
open defiance but as one of the discretions which he had to
take under the circumstances in his capacity as sales manager.
As it turned out, the result both Recodo and Sebastian hoped
for was achieved by not immediately grounding Cordova.
- While an employer is allowed wide latitude to dismiss
employees on loss of trust and confidence, still the loss thereof
must have some basis and must be proved by the employer
otherwise the social justice policy of the labor lawsand the
constitution will be for naught. The guidelines for the
application of the doctrine of loss of confidence are:
a. loss of confidence should not be simulated
b. it dhould not be used as subterfuge for causes which
are improper, illegal, or unjustified
c. it should not be arbitrarily asserted in the face of
overwhelming evidence to the contrary
d. it must be genuine, not a mere afterthought to justify
earlier action taken in bad faith
Disposition the resolution f the NLRC is affirmed with the
modification that corresponding back wages of respondent be
forthwith updated and released to him.

COCA-COLA BOTTLERS PHIL INC V KAPISANAN NG


MALAYANG MANGGAGAWA SA COCA-COLA
[PAGE 209]

WILLFUL BREACH
ATLAS CONSOLIDATED MINING & DEVELOPMENT
CORP V NLRC (VILLACENCIO)

Labor Law 1
290 SCRA 479
PUNO; May 21, 1998
NATURE
petition for certiorari under Rule 65 of the Revised Rules of
Court of Decision dated December 27, 1994 of NLRC which
ordered the payment of separation pay and backwages to
private respondent Isabelo O. Villacencio, and its Resolution
dated August 18, 1995 denying petitioner's Motion for
Reconsideration.
FACTS
- private respondent Isabelo O. Villacencio worked with
petitioner ACMDC from January 23, 1970 to February 2, 1990.
He started as an ordinary laborer/helper in the Mill Department.
In 1973, he became supervisor of the Tailings Disposal
Department. In 1982, he was elevated as a junior staff of the
department. Finally, he was promoted general foreman of the
Tailings Disposal and Water Supply Department with a monthly
salary of P7,440.00. He held this position until his services were
terminated on February 2, 1990.
- As general foreman, Villacencio was the second-to-the-highest
man in the department which has a field office located in
Magdugo, Toledo City. Under Villacencio were some fifty nine
(59) workers whom he supervised through regular field
inspections. Villacencio was assigned a service jeep and a
service motorcycle which he used alternately. He was given the
privilege to withdraw the necessary fuel/gasoline for the
vehicles at the Transport Department located inside the main
compound of ACMDC.
- September 8, 1989 - Engineer Sanchez of the Services Division
wrote a memorandum requesting that Villacencio be
investigated for alleged anomalies at the Magdugo Tailings
Field Office. Villacencio was charged before the Special
Investigation Board with acts of malfeasance consisting of:
1. withdrawal of company-owned gasoline for the refueling
of his personal jeep;
2. use of company personnel on company time as well as
company-owned materials for the assembly of a jeep not
belonging to the company; and
3.
granting of authority to non-company personnel to
withdraw company-owned stocks.
- January 1990 - He was summoned and investigations were
conducted. the Special Investigation Board found Villacencio
guilty of the charge of withdrawing on various dates a total of
192 liters of company-owned gasoline which he used to refuel
his private jeep and of the charge of using company personnel
on company time in the assembly of his jeep. The third charge
was dismissed for insufficiency of evidence. Villacencio was
dismissed from work on February 2, 1990.
- February 19, 1990 - Villavicencio lodged a complaint against
ACMDC before the Regional Arbitration Cebu City, for illegal
dismissal with prayer for reinstatement and backwages plus
damages. The case was assigned to Labor Arbiter Reynoso A.
Belarmino.
- Meanwhile, ACMDC initiated a criminal complaint against
Villacencio for the misappropriation of 192 liters of gasoline
amounting to P1,086.72. An Information for Estafa was filed
against Villacencio before the Municipal Trial Court of Toledo
City. After trial, he was found guilty and sentenced to prision
correccional as maximum, and to pay ACMDC the amount of
P1,086.72 for the misappropriated gasoline.
- Villacencio appealed his conviction to RTC Toledo City. For
failure of the prosecution to establish the guilt of Villacencio
beyond reasonable doubt, the appellate court acquitted him
- August 9, 1993 - Labor Arbiter Belarmino rendered a Decision
dismissing Villacencio's complaint of illegal dismissal for lack of
merit.
- December 27, 1994 - NLRC reversed the Labor Arbiter's
decision.
- Both parties filed their respective Motion for Reconsideration.
ACMDC's motion assailed the public respondent's decision for
allegedly misapprehending the Labor Arbiter's decision. On the

A2010

- 236 -

Disini

other hand, Villacencio's motion prayed for reinstatement and


award of backwages in addition to separation pay.
- August 18, 1995 NLRC rendered a Resolution granting
Villacencio's prayer for backwages and denying ACMDC's
motion.
ISSUES
1. WON NLRC acted with grave abuse of discretion amounting
to lack of jurisdiction in reversing the Decision of the Labor
Arbiter and holding Villavicencios dismissal illegal
2. WON there is willful breach of trust
HELD
1. NO
- In illegal dismissal cases, the employer bears the burden of
proof to show that the dismissal is for a just or authorized
cause. The charges against private respondent are: (1)
withdrawal of 192 liters of gasoline from company stocks for his
private use; and (2) knowingly allowing company personnel to
work on company time in the assembly of a privately-owned
jeep. To prove the first charge, petitioner presented the Tenders
Logbook showing the unsigned entries of gasoline withdrawals
allegedly made by Villavicencio . Wilfredo Caba and Bienvenido
Villacencio also testified that Villavicencio refused to sign the
entries when requested to do so.
(1) The evidence for the Villavicencio shows that during his
more than twenty (20)-year stint with petitioner, he received
several awards and commendations for his contribution in the
areas of production, services and smooth operation of his
department. The management recognized his ability in handling
his subordinates and in protecting company assets in relation to
his assigned duties. As a stickler for company rules, he never
held back on issuing warnings, admonitions and even
suspensions against erring subordinates. Consequently, he
earned the ire of some of his subordinates. Among them were
Wilfredo Caba., June Climaco, Felix Gonzales and Bienvenido
Villacencio. In sum, Villavicencios position is that the logbook
entries do not prove that he received the 192 liters of gasoline
since his signature does not appear therein and that the
witnesses presented by the petitioner to explain the absence of
his signature in the logbook entries were motivated by
vengeance since he offended their feelings when he disciplined
them and denied their requests for promotion.
- The Standard Guidelines of ACMDC require that all withdrawals
of consumable items and the borrowing of company materials
and equipments should be recorded in the Tender's Logbook by
the tender on duty and should be signed by the withdrawing
party. The tender on duty is also required to immediately report
to his supervisor any discrepancy, error or irregularity. Needless
to stress, the best evidence of any withdrawal is the Tender's
Logbook. In the case at bar, the gasoline withdrawal entries
were made by tenders Caba and Villacencio. Villavicencios
signature does not appear in the logbook, thus, there is no
proof that he actually withdrew and received the gasoline.
(2) The Authorization to Work Overtime dated May 14, 1989
indicates that A. Saavedra, A. Sepada and V. Rago were among
those authorized to work overtime 'to assist in emergency
repair of busted 280 CIC Tailings Line' on that date. The same
does not show or affirm petitioner's contention that said
workers were not actually authorized or did not actually perform
the required work but were at the Magdugo Field Office working
on private respondent's personal jeep. On the contrary, the
Authorization to Work Overtime appears regular on its face, as
in fact, the same bears the imprimatur indicated by the
signature not only of private respondent alone but of three (3)
other officers: the Supervisor, J.V. Climaco, Jr., the Department
Head, J. N. Tecson, and the Division Manager, C. N. Sanchez. If
ever there was an irregularity, these officers would likewise
have to be answerable to the company, instead of letting
private respondent bear the burden alone.
2. NO
- We reject the ruling of the Labor Arbiter that since
Villavicencio neglected to inspect the logbook and thus failed to
discover the irregularity, he committed breach of trust.

Labor Law 1
Ratio Settled is the rule that under Article 283(c) of the Labor
Code, the breach of trust must be willful. A breach is willful if it
is done intentionally, knowingly and purposely, without
justifiable excuse, as distinguished from an act done carelessly,
thoughtlessly, heedlessly or inadvertently. It must rest on
substantial grounds and not on the employer's arbitrariness,
whims, caprices or suspicion; otherwise, the employee would
eternally remain at the mercy of the employer. It should be
genuine and not simulated; nor should it appear as a mere
afterthought to justify earlier action taken in bad faith or a
subterfuge for causes which are improper, illegal or unjustified.
It has never been intended to afford an occasion for abuse
because of its subjective nature. Private respondent explained
that he failed to inspect the logbook for about two (2) months
before its disappearance because he was preoccupied with
some emergency works brought about by a storm. With the
foregoing explanation, it cannot be said that Villavicencios
failure was willful.
Disposition the assailed Decision and Resolution of public
respondent NLRC are AFFIRMED.

COVERAGE
FUJITSU COMPUTER PRODUCTS CORP V CA
[PAGE 204]

PROOF
RAMATEK PHILS V DE LOS REYES
474 SCRA 129
CARPIO; October 25, 2005
NATURE
petition for review resolutions of CA (denying appeals for being
filed out of time)
FACTS
- Anelia de los Reyes was employed by Ramatek as a
comptroller. Subsequently, Ramatek entered into a subcontracting agreement with Sicar Micro-Electronics Corp, of
which Anelias husband Nestor was a major stockholder,
treasurer, and COO.
- Some time after, Sicar filed a civil action for damages against
the Ramatek officials for the unilateral termination of their
contract. Later, the chairman of the board of directors of
Ramatek informed Anelia that she should file a leave of absence
while the case was ongoing. Afterwards, the chairman emailed
Anelia, requesting her to tender her voluntary resignation from
the company. The email said in part: IT IS WITH GREAT
REGRET THAT I MUST INFORM YOU OF MY REACTION TO THE
SICAR AFFAIR. YOUR CONNECTION IN THIS MATTER HAS
CAUSED ME TO LOSE MY FAITH AND TRUST IN YOU. IT IS A
MAJOR CONFLICT OF INTEREST SITUATION.
- In a letter dated a month later, the company required Anelia to
explain within 72 hours some of her allegedly questionable
transactions. Such included awards of work bids to bidders who
did not give the lowest bids, purchase of equipment not at the
lowest prices, and failure to submit company documents
despite demand.
- Anelia did not answer (she failed to claim the letter sent by
Ramatek through registered mail) nor did she appear in the
administrative investigation. Ramatek, soon after, terminated
Anelias employment for committing anomalies amounting to
breach of trust and confidence. Anelia filed for illegal
suspension and illegal dismissal.
LA ruled in favor of Anelia. NLRC affirmed. Appeal to CA (by
Ramatek) was denied for being filed out of time.
ISSUES

A2010

- 237 -

Disini

1. WON the appeal was filed out of time


2. WON dismissal based on loss of trust and confidence was
valid
HELD
1. NO
Ratio The latest amendment to Rule 65, ROC allows filing of an
appeal within 60 days after the notice of denial of a motion for
reconsideration.
Reasoning
- The amended rule now reads:
- Sec. 4. When and where petition filed. The petition shall be
filed not later than sixty (60) days from notice of the judgment,
order or resolution. In case a motion for reconsideration or new
trial is timely filed, whether such motion is required or not, the
sixty (60) day period shall be counted from notice of the denial
of the said motion.
- In the present case, the petition filed in the Court of Appeals
was indeed filed beyond the 60-day period if computed from the
time the notice of judgment was received and interrupted only
by the filing of the motion for reconsideration. However, if the
60-day period is reckoned from the receipt of the notice
denying the motion for reconsideration, as provided under
Circular No. 56-2000, then the petition for certiorari was filed on
time.
2. NO
Ratio That the dismissal was based on loss of trust and
confidence was not sufficiently proven by evidence. Ramateks
evidence are insubstantial and inadequate to support a
conclusion that Anelia engaged in anomalous transactions.
Since the company had the burden of proving the same, said
dismissal cannot be held valid.
Reasoning
- The SC upholds the findings of the Labor Arbiter that Anelia
was able to prove that the charges against her were false and
baseless.
Despite the gravity of the charges, there is nothing
competent in the records to substantiate the same. Xxx
Ramatek has the burden to prove just cause, but it failed to
undertake the burden. On the other hand, complainant
explained to the satisfaction of this Office that the charges
against her are utterly false and baseless.
- Ramatek having failed to substantiate their charges against
Anelia with competent and credible evidence, this Office
perceives that the primordial inspiration for her dismissal was
the filing by her husband of a civil suit against the company
officials, a matter which respondents cannot legally use against
complainant to deprive her of her tenurial rights. This is
because the suit was not filed by Anelia against Ramatek or its
officials but by her husband. There is no showing that the filing
of the suit was a joint decision by the couple or was instigated
by complainant as to charge complainant with disloyalty or a
conflict of interests. Moreover, it appears that Anelias husband
was merely asserting and exercising his right to seek redress in
the courts, a matter which Ramatek should not begrudge Anelia
about. Finally, the case was amicably settled by the parties
such that there can be no rational justification for respondents
to dismiss Anelia just because a plaintiff in the civil suit
happened to be her husband.
- Loss of confidence as a ground for dismissal does not require
proof beyond reasonable doubt. The law requires only that
there be at least some basis to justify it. Thus, there must be
some evidence to substantiate the claim and form a legal basis
for loss of confidence. The employer cannot exercise arbitrarily
and without just cause the right to dismiss an employee for loss
of trust and confidence.
Disposition Resolutions of CA set aside. Decision of NLRC
AFFIRMED.

LACK OF DAMAGE

Labor Law 1
CADIZ V CA (PHILIPPINE COMMERCIAL BANK
[EQUITABLE PCIBANK])
474 SCRA 232
TINGA; October 25, 2005
NATURE
Certiorari
FACTS
- Cadiz, Bongkingki and Gloria were employed as signature
verifier, bookkeeper, and foreign currency denomination
clerk/bookkeeper-reliever, respectively, in the main office
branch (MOB) of Philippine Commercial International Bank
(respondent bank).
- Cadiz reserved S/A No. 1083-4 in July 1987 as reflected on
respondent banks new account register.
- Foreign denominated checks payable to other payees were
diverted into the said account.
- The various deposit slips, covering the said checks, did not
bear the machine validation of any of the tellers-in-charge.
- Petitioner Cadiz agreed to pay Alqueza the equivalent amount
of $600.00 but it was made to appear that Alfiscar paid the said
amount.
- In view of these findings, petitioners were served with showcause memoranda asking them to explain the lapses.
- Finding their explanations unsatisfactory, petitioners were
terminated from employment.
LA-adjudged that petitioners were illegally dismissed and
ordered their reinstatement and payment of backwages.
- NLRC-reversed
- CA-affirmed reversal by NLRC
ISSUES
1. WON petitioners were validly dismissed (with just cause and
were afforded due process)
2.
WON petitioners should be relieved of any liability
considering that respondent bank did not suffer a pecuniary
loss
HELD
1. YES
- Petitioners had surreptitiously diverted funds deposited by
depositors to S/A No. 1083-4 which was under their control and
disposition.
- Their behavior in the course of the discharge of their duties is
clearly malfeasant, and constitutes ground for their termination
on account of just cause.
- respondent bank complied with the two-notice rule prescribed
in Article 277(b) of the Labor Code. Petitioners were given all
avenues to present their side and disprove the allegations of
respondent bank. An informal meeting was held between the
branch manager of MOB, the three petitioners and Mr. Gener,
the Vice-President of the PCIB Employees Union.
2. NO
- In University of the East v. NLRC the court held that lack of
material or pecuniary damages would not in any way mitigate a
persons liability nor obliterate the loss of trust and confidence.
- In the case of Etcuban v. Sulpicio Lines, this Court definitively
ruled that:
. . . Whether or not the respondent bank was financially
prejudiced is immaterial. Also, what matters is not the
amount involved, be it paltry or gargantuan; rather the
fraudulent scheme in which the petitioner was involved,
which constitutes a clear betrayal of trust and confidence. . . .

D. COMMISSION OF A CRIME
E. ANALOGOUS CAUSES
QUARELSOME BOSSY

A2010

- 238 -

Disini

CATHEDRAL SCHOOL OF TECHNOLOGY V NLRC


(VALLEJERA)
214 SCRA 551
October 13, 1992
NATURE
Petition for certiorari of a decision of NLRC.
FACTS
- Starting as an aspirant to the Congregation of the Religious of
Virgin Mary (RVM), VALLEJERA worked on a volunteer basis as a
library aide of CST, an educational institution run by the RVM
sisters. Eventually she became a regular employee of CST,
again as library aide.
- It was around such regular employment, however, that trouble
developed. The sisters began receiving complaints from
students and employees about VALLEJERA's difficult personality
and sour disposition at work. On one occasion, VALLEJERA was
summoned to the Office of the Directress by SISTER
APOLINARIA, shortly after the resignation of the school's Chief
Librarian on account of irreconcilable differences with
VALLEJERA, for the purpose of clarifying the matter. SR
APOLINARIA also informed VALLEJERA of the negative reports
received by her office regarding the latter's frictional working
relationship with co-workers and students and reminded
VALLEJERA about the proper behavior in the interest of peace
and harmony in the school library. VALLEJERA resented the
observations about her actuations and was completely
unreceptive to the advice given by her superior. She reacted
violently to SR APOLINARIA and angrily offered to resign,
repeatedly saying, "OK, I will resign. I will resign." Thereafter,
without waiting to be dismissed from the meeting, she stormed
out of the office.
- On separate occasions thereafter, CST and SR APOLINARIA
(PETITIONERS, for brevity) sent people to convince VALLEJERA
to settle her differences with the former. VALLEJERA remained
adamant in her refusal to submit to authority. Eventually, SR
APOLINARIA, by letter, informed VALLEJERA to look for another
job as the school had decided to accept her resignation.
VALLEJERA filed a complaint for illegal dismissal. An issue arose
as to whether there was lawful cause for her dismissal.
ISSUE
WON there was there lawful cause for VALLEJERAs dismissal
HELD
YES
Ratio
The reason for which VALLEJERAs services were
terminated, namely, her unreasonable behavior and unpleasant
deportment in dealing with the people she closely works with, is
analogous to the other "just causes" enumerated under
ART.282, Labor Code.
- PETITIONERS' averments on VALLEJERAs disagreeable
character as "quarrelsome, bossy, unreasonable and very
difficult to deal with," are supported by testimonies of several
co-employees and students of CST. In fact, her overbearing
personality caused the chief librarian to resign, Furthermore,
the complaints about her objectionable behavior were
confirmed by her reproachable actuations during her meeting
with SR APOLINARIA, when VALLEJERA, upon being advised of
the need to improve her working relations with others,
obstreperously reacted and unceremoniously walked out on her
superior, and arrogantly refused to subsequently clear up
matters or to apologize therefor. To make matters worse, she
ignored the persons sent by PETITIONERS to intervene in an
effort to bring the matter to a peaceful resolution. The conduct
she exhibited on that occasion smacks of sheer disrespect and
defiance of authority and assumes the proportion of serious
misconduct or insubordination, any of which constitutes just
cause for dismissal from employment.

Labor Law 1
- As CST is run by a religious order, it is but expected that good
behavior and proper department, especially among the ranks of
its own employees, are major considerations in the fulfillment of
its mission. Under the circumstances, the sisters cannot be
faulted for deciding to terminate VALLEJERA whose presence
"has become more a burden rather than a joy" and had proved
to be disruptive of the harmonious atmosphere of the school.
Disposition
NLRC decision that VALLEJERA was illegally
dismissed, SET ASIDE.

HEAVYLIFT MANILA INC V CA (GALAY, NLRC)


473 SCRA 541
QUISUMBING; October 20, 2005
NATURE
A petition for certiorari
FACTS
- Petitioner Heavylift, a maritime agency, thru a letter signed by
Josephine Evangelio, Admin. and Finance Manager of Heavylift,
informed respondent Ma. Dottie Galay, Heavylift Insurance and
Provisions Assistant, of her low performance rating and the
negative feedback from her team members regarding her work
attitude. The letter also notified her that she was being relieved
of her other functions except the development of the new
Access program.
- Subsequently, Galay was terminated for alleged loss of
confidence.
- Thereafter, she filed with the Labor Arbiter a complaint for
illegal dismissal and nonpayment of service incentive leave and
13th month pay against petitioners.
- Petitioners alleged that Galay had an attitude problem and did
not get along with her co-employees for which she was
constantly warned to improve. Petitioners aver that Galays
attitude resulted to the decline in the companys efficiency and
productivity.
- The Labor Arbiter found that Galay was illegally terminated for
petitioners failure to prove that she violated any company
regulation, and for failure to give the proper notice as required
by law.
- Petitioner appealed to the NLRC. The latter, however, denied
the appeal for lack of merit and affirmed the decision of the
Labor Arbiter.
- CA denied the motion for lack of justifying circumstances, and
because the attached board resolution was issued after the
petition was filed (petitioners failed to: state the full names and
actual addresses of all the petitioners; attach the copies of all
pleadings and supporting documents; properly verify the
petition; and certify against forum-shopping)
ISSUES
1. WON petitioners were denied due process with the CAs
dismissal of the petition on technical grounds
2. WON attitude problem is a valid ground for the termination
of an employee.
3. If issue 2 is in the affirmative, WON this was sufficiently
proved
4. WON the procedural requirements for an effectual dismissal
were present
5. WON the awards of service incentive pay and 13th month
pay were proper
HELD
1. YES
Ratio The Rules of Court are designed for the proper and
prompt disposition of cases. In not a few instances, we relaxed
the rigid application of the rules to afford the parties
opportunity to fully ventilate their cases on the merits. In that
way, the ends of justice would be better served. Additionally,
verification of a pleading is a formal, not a jurisdictional
requisite. It is intended to secure an assurance that what are

A2010

- 239 -

Disini

alleged in the pleading are true and correct and not the product
of the imagination or a matter of speculation, and that the
pleading is filed in good faith.
2. YES
Ratio An employee who cannot get along with his coemployees is detrimental to the company for he can upset and
strain the working environment. Without the necessary
teamwork and synergy, the organization cannot function well.
Thus, management has the prerogative to take the necessary
action to correct the situation and protect its organization.
When
personal
differences
between
employees
and
management affect the work environment, the peace of the
company is affected. Thus, an employees attitude problem is a
valid ground for his termination. It is a situation analogous to
loss of trust and confidence that must be duly proved by the
employer. Similarly, compliance with the twin requirement of
notice and hearing must also be proven by the employer.
3. NO
Ratio We are not convinced that in the present case,
petitioners have shown sufficiently clear and convincing
evidence to justify Galays termination. Though they are correct
in saying that in this case, proof beyond reasonable doubt is not
required, still there must be substantial evidence to support the
termination on the ground of attitude. The mere mention of
negative feedback from her team members, and the letter, are
not proof of her attitude problem. Likewise, her failure to refute
petitioners allegations of her negative attitude does not
amount to admission. Technical rules of procedure are not
binding in labor cases. Besides, the burden of proof is not on
the employee but on the employer who must affirmatively show
adequate evidence that the dismissal was for justifiable cause.
4. NO
Ratio The letter did not constitute the required notice. It did
not inform her of the specific acts complained of and their
corresponding penalty. Additionally, the letter never gave
respondent Galay an opportunity to explain herself, hence
denying her due process.
5. YES
Ratio Apropos the award of service incentive pay and 13th
month pay, we find that they were properly prayed for by
Galay. These were subsumed in the complaint and under the
position papers general prayer of such other relief as are just
and equitable under the law.
Disposition Decision of the Labor Arbiter and the Resolution of
the NLRC are hereby affirmed.

PROBABLE CAUSE
STANDARD ELECTRIC MANUFACTURING CORP V
STANDARD ELECTRIC EMPLOYEES UNION
CALLEJO; August 25, 2005
NATURE
Petition for review on certiorari to review the CA decision
annulling the NLRC Resolution which affirmed the LA decision
FACTS
- Rogelio Javier, a radio machine operator, employee of
Standard Electric Manufacturing Corp. (SEMC) and member of
the Standard Electric Employees Union (Union), failed to report
for work and failed to report the reason for his absence. This
failure to report for work and failure to report the reason
therefor happened several times until he was later found to
have been arrested and detained for the charge of rape.
- Javier informed SEMC (through a letter and through his
counsel) that he was detained for the charge of rape which is
why he failed to report for work. He requested that SEMC defer
the implementation of its intention to dismiss him. The SEMC
denied his request and issued a Memorandum terminating his
employment for having been absent without leave (AWOL) for
more than 15 days and for committing rape.

Labor Law 1
- Javier, after the RTC granted his demurrer to evidence and
ordered his release from jail, reported for work but the SEMC
refused to accept him back. A grievance meeting between the
Union, Javier and the SEMC was later held, but the SEMC
refused to re-admit Javier. The Union and Javier then filed a
complaint for illegal dismissal against SEMC before the NLRC,
alleging that since his detention for rape was non-existent, the
termination of his employment was illegal. SEMC averred that
Javiers prolonged absences caused irreparable damage to its
orderly operation and that it could not afford to wait for Javiers
indefinite return from detention, if at all.
- The LA dismissed the complaint but ordered SEMC to pay
Javier P71, 760 as separation pay. On appeal, the NLRC affirmed
the LAs ruling (held that Javier was given a chance to explain
his side), and later denied a subsequent MFR. Javier and the
Union then filed a petition for certiorari with the CA, which
reversed the findings of both the LA and the NLRC and ordered
the reinstatement of Javier to his former position. The appellate
court cited Magtoto v NLRC and City Govt of Makati v Civil
Service as precedents and declared that it was not Javiers
intention to abandon his job; his incarceration reasonably
justified his failure to report for work and negated the theory
that he was on AWOL. The CA also held that Javier could not be
terminated on the ground of commission of a crime, as he was
acquitted of the rape charges. Hence, despite the fact that
Javier was allegedly afforded the opportunity to explain his side
(the basis of the LA and NLRC decisions), the same was
unnecessary since there was no just or authorized cause for the
dismissal. The MFR by SEMC was denied by the CA, hence, this
recourse.
ISSUE
WON the CA erred in holding that the termination was illegal
HELD
NO
- The CA was correct in holding that the termination was illegal
and correctly applied the Magtoto case.
Ratio Separation from employment founded on a false or nonexistent cause is illegal
Reasoning
- In the Magtoto case, Alejandro JONAS Magtoto was arrested.
He was charged with violation of Arts 136 and 138 of the RPC.
Although Magtoto informed his employer and pleaded that he
be considered on leave until released, his employer denied
the request. About seven months after his arrest, Magtoto was
released after the City Fiscal dismissed the criminal charges for
lack of evidence. On the same date, he informed his employer
of his intent to start working again but the employer rejected
the offer. In ruling that his termination was illegal, the SC ruled:
The employer tries to distance itself from the detention by
stressing that the petitioner was dismissed due to prolonged
absence. However, Mr. Magtoto could not report for work
because he was in a prison cell. The detention cannot be
divorced from prolonged absence. One caused the other.
Since the causes for the detention, which in turn gave the
employer a ground to dismiss the petitioner, proved to be
non-existent, we rule that the termination was illegal and
reinstatement is warranted.
- Respondent Javier was dismissed by the petitioner for: (a)
being AWOL from July 31, 1995 up to January 30, 1996; and (b)
committing rape. However, on demurrer to evidence, Javier
was acquitted of the charge. With Javiers acquittal, the cause
of his dismissal from his employment turned out to be nonexistent.
- A non-existent cause for dismissal was explained in Pepito v.
Secretary of Labor (96 SCRA 454):
... A distinction, however, should be made between a
dismissal without cause and a dismissal for a false or nonexistent cause. In the former, it is the intention of the
employer to dismiss his employee for no cause whatsoever,
in which case the Termination Pay Law would apply. In the
latter case, the employer does not intend to dismiss the
employee but for a specific cause which turns out to be false

A2010

- 240 -

Disini

or non-existent. Hence, absent the reason which gave rise to


his separation from employment, there is no intention on the
part of the employer to dismiss the employee concerned.
Consequently, reinstatement is in order. And this is the
situation here. Petitioner was separated because of his
alleged involvement in the pilferage in question. However, he
was absolved from any responsibility therefor by the court.
The cause for his dismissal having been proved non-existent
or false, his reinstatement is warranted. It would be unjust
and unreasonable for the Company to dismiss petitioner after
the latter had proven himself innocent of the cause for which
he was dismissed.
- The petitioner acted with precipitate haste in terminating
respondent Javiers employment on the ground that he had
raped the complainant therein. Respondent Javier had yet to be
tried for the said charge. In fine, the petitioner prejudged him,
and preempted the ruling of the RTC. Petitioner had, in effect,
adjudged Javier guilty without due process of law. While it may
be true that after the preliminary investigation of the complaint,
probable cause for rape was found and respondent Javier had to
be detained, these cannot be made as legal bases for the
immediate termination of his employment.
Disposition petition DISMISSED for lack of merit. CA decision is
AFFIRMED with MODIFICATION. Petitioner is ordered to reinstate
Rogelio Javier to his former position or, if no longer possible, a
substantially equivalent position without loss of seniority rights
and other privileges appurtenant thereto, with full backwages
from the time it refused to allow his reinstatement on May 24,
1996 until actually reinstated; or, if reinstatement is no longer
feasible, to pay him separation pay equivalent to one (1) month
salary for every year of service.

CONVICTION MORAL TURPITUDE


IRRI V NLRC (MICOSA)
221 SCRA 760
NOCON; May 12, 1993
NATURE
Petition for certiorari
FACTS
- International Rice Research Institute (IRRI) is an international
organization recognized by the Philippine government and
accorded privileges, rights and immunities normally granted to
organizations of universal character. In 1977, it hired Nestor
Micosa, who thereby became bound by IRRI Employment Policy
and Regulations, the Miscellaneous Provisions of which states:
"C. Conviction and Previous Separation.
XXX
'2. An employer who has been convicted of a (sic) criminal
offense involving moral turpitude may be dismissed from the
service.'"
- On February 6, 1987, Micosa stabbed to death Reynaldo
Ortega inside a beer house in Laguna. He was accused of
homicide. During the pendency of the criminal case, Micosa
voluntarily applied for inclusion in IRRI's Special Separation
Program. However, IRRI's Director General expressed deep
regret that he had to disapprove Micosa's application for
separation because of IRRI's desire to retain the skills and
talents that persons like him possess.
- Trial court found Micosa guilty of homicide, but appreciated in
his favor the mitigating circumstances of incomplete selfdefense and voluntary surrender, and no aggravating
circumstance. Subsequently, Micosa applied for suspension of
his sentence under the Probation Law.
- On February 8, 1990, IRRI's Director General personally wrote
Micosa that his appointment as laborer was confirmed, making
him a regular core employee whose appointment was for an
indefinite period and who "may not be terminated except for
justifiable causes as defined by the pertinent provisions of the
Philippine Labor Code."

Labor Law 1
- On March 30, 1990, IRRIs HR head wrote Micosa urging him to
resign from employment in view of his conviction in the case for
homicide.
- Laguna Parole and Probation Office No. II wrote IRRI informing
the latter that said office found Micosa's application for
probation meritorious as he was evaluated "to possess
desirable social antecedents in his life."
- Micosa informed IRRI that he had no intention of resigning
from his job.
- IRRIs HR head replied to Micosa's letter insisting that the
crime for which he was convicted involves moral turpitude and
informing him that he is thereby charged of violating Section IAA, Par VII, C-2 of the Institute's Personnel Manual (quoted
above).
- Micosa explained to IRRI that the slaying of Ortega arose out
of his act of defending himself from unlawful aggression; that
his conviction did not involve moral turpitude and that he opted
not to appeal his conviction so that he could avail of the
benefits of probation, which the trial court granted to him.
- Micosa sought the assistance of IRRI's Grievance Committee
who recommended to the Director General, his continued
employment. However, IRRI issued a notice to Micosa that the
latter's employment was to terminate effective May 25, 1990.
- Micosa then filed a case for illegal dismissal. Labor Arbiter
found the termination was illegal and ordered his reinstatement
with full backwages from the date of his dismissal up to actual
reinstatement. NLRC affirmed decision.
Petitioners claims:
> Micosa's conviction of homicide, which is a crime involving
moral turpitude, is a valid ground for his dismissal under the
Miscellaneous Provisions of IRRI's Employment Policy
Regulations. IRRI has the prerogative to issue rules and
regulations including those concerning employee discipline and
that its employees are bound by the aforesaid personnel
manual
- While IRRI admits that Micosa's interests in his employment
and means of livelihood are adversely affected; that a
convict should not be discriminated against in society and that
he should be given the same opportunities as those granted to
other fellow citizens, it claims that one's right is deemed
superior than that of another. It believes that it has a superior
right to maintain a very high degree or standard not only to
forestall any internal problem hampering operations but also to
prevent even the smallest possibility that said problems could
occur considering that it is an international organization with
concomitant obligation to the host country to avoid creating
disturbance or give occasion for such disturbance.
ISSUE
WON a conviction of a crime involving moral turpitude is a
ground for dismissal from employment
HELD
NO, it is not one of the causes enumerated in the Labor Code.
- Article 282 of the Labor Code enumerates the just causes
wherein an employer may terminate an employment.
Conviction of a crime involving moral turpitude is not one of
these justifiable causes. Article 282 (c) or (d) may not be
applied by analogy. Analogous causes must have an element
similar to those found in the specific just cause enumerated
under Article 282.
- Under Article 282 (c) fraud or willful breach by the employees
of the trust reposed in him by his employer or duly authorized
representative refers to any fault or culpability on the part of
the employee in the discharge of his duty rendering him
absolutely unworthy of the trust and confidence demanded by
his position. The breach of trust must be related to the
performance of the employee's function.
- Commission of a crime by the employee under Article 282 (d)
refers to an offense against the person of his employer or any
immediate member of his family or his duly authorized
representative.
- The commission of the crime of homicide was outside the
perimeter of the IRRI complex, thus, the conviction of Micosa for

A2010

- 241 -

Disini

homicide was not work-related, his misdeed having no relation


to his position as laborer and was not directed or committed
against IRRI or its authorized agent.
- IRRI failed to show how the dismissal of Micosa would be in
consideration of the safety and welfare of its employees, its
reputation and standing in the community and its special
obligations to its host country. Micosa's service record is
unblemished. IRRI's Director General even expressed his
confidence in him when he disapproved his application for
special separation and decided to promote him to the status of
a regular core employee, with the commensurate increases in
benefits. In addition, the employees at IRRI's Grievance
Committee interceded favorably in behalf of Micosa when they
recommended his retention despite his conviction showing that
the very employees which IRRI sought to protect did not believe
that they were placing their very own lives in danger with
Micosa's retention.
- Likewise, Micosa, although found guilty as charged, was also
found worthy of probation. This means that there existed no
undue risk that Micosa will commit another crime during his
period of probation and that his being placed on probation
would be to the benefit of society as a whole.
- Even under IRRI's Employment Policy and Regulations, the
dismissal of Micosa on the ground of his conviction for homicide
cannot be sustained. The miscellaneous provisions of said
personnel manual mentions of conviction of a crime involving
moral turpitude as a ground for dismissal. IRRI simply assumed
that conviction of the crime of homicide is conviction of a crime
involving moral turpitude.
- Moral turpitude has been defined in Can v. Galing citing In
Re Basa and Tak Ng v. Republic as everything which is done
contrary to justice, modesty, or good morals; an act of
baseness, vileness or depravity in the private and social duties
which a man owes his fellowmen, or to society in general,
contrary to justice, honesty, modesty or good morals.
As to what crime involves moral turpitude, is for the Supreme
Court to determine. The conclusion of IRRI that conviction of the
crime of homicide involves moral turpitude is unwarranted
considering that the said crime which resulted from an act of
incomplete self-defense from an unlawful aggression by the
victim has not been so classified as involving moral turpitude.
- The facts of the incident show that Micosa's intention was not
to slay the victim but only to defend his person. The
appreciation in his favor of the mitigating circumstances of selfdefense and voluntary surrender, plus the total absence of any
aggravating circumstance demonstrate that Micosa's character
and intentions were not inherently vile, immoral or unjust.
- Corollary issue: WON conviction of homicide involves
moral turpitude
Homicide may or may not involve moral turpitude depending on
the degree of the crime. Moral turpitude is not involved in
every criminal act and is not shown by every known and
intentional violation of statute, but whether any particular
conviction involves moral turpitude may be a question of fact
and frequently depends on all the surrounding circumstances.
Moral turpitude is somewhat a vague and indefinite term, the
meaning of which must be left to the process of judicial
inclusion or exclusion as the cases are reached.
Disposition petition is DISMISSED for lack of merit.

OANIA V NLRC (PHILEX MINING)


244 SCRA 668
ROMERO; June 1, 1995
FACTS
- Alfredo Oania, a welder, and Aurelio Caluza and Santiago Biay,
miners, were employed by Philex Mining Corporation (Philex) in
Benguet. They were accused of mauling their co-worker, Felipe
Malong, at the gasoline area within the company compound.
- Philex conducted investigation regarding the incident. After a
formal hearing wherein petitioners were duly notified and
accorded the opportunity to be heard, the company arrived at
the decision to terminate their employment on the ground that

Labor Law 1
petitioners violated Art I, par 1 of the company rules and
regulations2
- Malong instituted a criminal complaint (frustrated murder) vs.
petitioners. But later, Malong desisted from pursuing the
criminal case because he said his conscience bothered him.
- With Malong's affidavit of desistance, petitioners sought
reconsideration of their dismissal from employment. Philex
refused. Petitioners filed complaints for illegal dismissal before
the labor arbiter.
- Labor Arbiter: The termination of employment of petitioners
was not justified was based on findings that there was no proof
that the mauling of Malong was "caused by a dispute involving
their employment" with private respondent (which, the Labor
Arbiter believed, was the only dispute clearly prohibited by the
company rule).
- Petitioners had been illegally dismissed from employment.
Philex to reinstate them to their former positions or
substantially equivalent positions and to pay each of them one
year's backwages.
- NLRC: Reversed. there is prima facie evidence that the
complainants injured physically a co- employee under
circumstance(s) which constitute an infraction of specific
company rules; and that the respondent had valid cause to
terminate their employment."
ISSUES
1. WON the mauling comes under Art 1 of the company rules
and regulations
2. WON there was illegal dismissal
HELD
1. YES
- The provision in question obviously covers situations where
any company employee inflicts or attempts to inflict physical
harm or injury upon any person. There are two separate
instances contemplated here. The first part of the sentence
conceives of a situation wherein such injury was done "on the
job site on company time or property," regardless of the reason.
What is material is the venue. The second half of the sentence
deals with a situation where an employee attempts to inflict or
actually inflicts bodily injury upon another "anywhere at
anytime," regardless of the venue, as long as it arose in
connection with a dispute "involving one's employment." The
site matters not; what is crucial in the subject matter, i.e. it
should have something to do with the employee's job. Clearly,
the commas in the sentence may be dispensed with without
sacrificing the intent behind the provision.
2. YES
- Violation of a company rule prohibiting the infliction of harm or
physical injury against any person under the particular
circumstances provided for in the same rule may be deemed
analogous to "serious misconduct" stated in Art. 282 (a).
(H)owever, there is no substantial evidence definitely pointing
to petitioners as the perpetrators of the mauling of Malong.
What is an established fact is that, after investigation, private
respondent dismissed them and, thereafter, a criminal
complaint was filed against petitioners. It is of record that
Malong desisted from suing the perpetrators before the regular
courts. In criminal cases, an affidavit of desistance may create
serious doubts as to be the liability of the accused
- On the issue of the legality of the dismissal, two requisites
must concur to constitute a valid dismissal: (a) the dismissal
must be for any of the causes expressed in Art. 282 of the Labor
Code, and (2) the employee must be accorded due process,
basic of which are the opportunity to be heard and to defend
himself.

LIM V NLRC (PEPSI-COLA FAR EAST TRADE DEVT)


259 SCRA 485
2

"Inflicting or attempting to inflict bodily injury on the job-site on company time or


property for any reason, or attempting to inflict or inflicting bodily injury anywhere at
anytime, in any dispute involving one's employment

A2010

- 242 -

Disini

DAVIDE JR; July 26, 1996


NATURE
Petition for certiorari
FACTS
- Pepsi is a manufacturer of concentrates sold to Pepsi-Cola
Bottlers Co. Inc. Petitioner Sixta Lim had been employed with
the Pepsi Group since January 1, 1981, working as a secretary
for Pepsi Bottling Co. Pepsi employed Lim as a secretary on
June 15, 1983.
- At the time of her dismissal she was a staff accountant.
> She assisted and worked closely with the Plant Accountant
to carry out the accounting department's tasks necessary to
ensure an accurate, timely, and coordinated compilation of
data for each accounting transaction.
> Her work involved cost accounting production, cost
accounting financial reporting, payroll reporting, statutory
reporting and preparation of daily trade accounts receivable
reports, petty cash fund custodianship, and check
preparation.
- Pepsi regularly evaluated its employees' performance using
following ratings: Marginal (obviously well below the
acceptable level for the position), Fair Below (shows
noticeable need for improvement), Commendable (fully
meeting the performance requirements of the position),
Superior (noticeably better than required performance) and
Distinguished Outstanding (obviously far above an
acceptable job).
- Lims overall performance appraisals rated as follows: (a) "S"
(Superior) as of May 1, 1984; (b) "C" (Commendable) for the
period for December 1, 1987 to August 31,1988; and (c) "U' (C
minus), quantified as 81.10% for the period from September 1,
1988 to May 31, 1989.
- In 1989, Pepsi changed its rating scale to: Significantly
Above Target (SA, exceeds position requirements by a wide
margin; exceptional), Above Target (AT, usually exceeds
position requirements), On Target (OT, meets and sometimes
exceeds position requirements), Below Target (BT, meets
some or many but not all position requirements) and
Significantly Below Target (SB, below position requirements
by a wide margin; unacceptable).
- July 1, 1989 to December 31, 1989 Lim received an overall
rating of BT.
> This was heavily influenced by her ratings in production
reporting which made up 40% of her final rating.
Her
supervisor noted several discrepancies which could have
been avoided had Sixta been more diligent in her work.
> In cost accounting and financial reporting (20% of the
rating), Lim also was given a BT. Her supervisor noted that
she did not seem to be aware of the importance of the
reports she issued and her work always needed to be
reviewed. She also needed a systematic workplan.
> For the remaining 60%, she was given an OT. Overall, she
was given a BT.
- Lim questioned the change in the rating style as well as the
ratings and appraisals given to her by her supervisors. She
asserted her previous positive ratings and expressed disbelief
over the sudden decline of her ratings.
Pepsi conducted
another evaluation and Lims overall rating was a BT. Following
that evaluation, she was given a report which outlined the areas
where she could improve.
- Lim then wrote Mr. Mihara of Pepsi Co. in Japan and Mihara
replied, saying that he would discuss the matters with her upon
arrival in the Philippines. Pepsi, however, did not wait for
Mihara and offered to pay Lims termination benefits if she
resigned.
- Lim refused to do so and on May 6, 1991, she was informed
that she was terminated as an employee of Pepsi. On May 14,
1991, she filed a complaint for illegal dismissal with the Labor
Arbiter.
The Labor Arbiter decided matters in her favor,
ordering Pepsi to reinstate Lim to her former position or to pay
her separation pay, 13th month and backwages.

Labor Law 1
- The NLRC reversed the Labor Arbiters ruling.
Petitioners Claim
> Lims BT performance appraisal was sufficient ground to
dismiss her under Article 282 (b) of the Labor Code.
Respondents Comments
> Lim argues her alleged inefficiency was not among the just
causes prescribed by law for the dismissal of an employee and
even assuming that such dismissal was justified, she was still
entitled to separation benefits of P268,000.00 in accordance
with company policy plus damages and attorney's fees.
ISSUE
WON Lims alleged gross inefficiency was an adequate ground
for her dismissal
HELD
NO
Ratio "Gross inefficiency" is closely related to "gross neglect,"
for both involve specific acts of omission on the part of the
employee resulting in damage to the employer or to his
business. The Court has ruled that failure to observe prescribed
standards of work, or to fulfill reasonable work assignments due
to inefficiency may constitute just cause for dismissal.
Reasoning
- Pepsi had not characterized as "gross inefficiency" whatever
failures, shortcomings, or deficiencies may have been
attributable to the petitioner.
- Lim obtained an unfavorable rating, but not to the extent,
under the company's standards, to warrant even a probationary
measure which is given to the lowest rating of Significantly
Below Target (SB).
- In Pepsi's brochure entitled Managing Performance for the
90's, a BT rating does not merit dismissal from the service; as a
matter of fact, the lower rating - Significantly Below Target (SB)
- is not even a ground for termination of employment, but may
only justify putting the employee "on probation, telling the said
employee that improvement is necessary.
- If the company truly found the petitioner's "inefficiency" to be
of such a gross character, then it should have rated her even
lower than SB, since the latter only requires that the employee
be put on probation.
- Pepsi also violated the petitioner's right to due process. Prior
to the issuance of her termination letter, Pepsi never called
Lims attention to any alleged "gross inefficiency" on her part.
Likewise, she was never warned of possible disciplinary action
due to any alleged "gross inefficiency." The evaluation report
merely indicated her areas for improvement.
Disposition
the instant petition is GRANTED.
Private
Respondent Pepsi-Cola Far East Trade Development Co., Inc. is
ordered to reinstate petitioner Sixta C. Lim to her position as
Staff Accountant without loss of seniority rights, and to pay her
(a) backwages from the time she was illegally dismissed until
she was effectively reinstated, less whatever she may have
received through payroll reinstatement and whatever amount
she may have earned from employment elsewhere during the
period of her illegal dismissal, and (b) other monetary benefits
that may be due her from the date of her illegal dismissal until
such effective reinstatement.

F. OTHER JUST CAUSES CLAIMED BY


EMPLOYER
1. ABANDONMENT
DEFINED
NUEVA ECIJA ELECTRIC COOP (NEECO) II V NLRC
461 SCRA 169

A2010

- 243 -

Disini

CHICO-NAZARIO; June 23, 2005


NATURE
Petition for review
FACTS
- Petitioner NEECO II staunchly asserts that since its new GM
assumed office on 01 March 1995, the GM never saw private
respondent Eduardo Cairlan report for work prompting the
former to issue a memorandum dated 22 November 1995,
which required private respondent to explain in writing why he
was not reporting for duty. Private respondent was likewise
directed in the said memo to report to its main office at
Calipahan, Talavera, Nueva Ecija. For failure of the private
respondent to comply with the said memorandum, Mr. dela
Cruz directed a certain Mr. Marcelo to conduct an
investigation on the whereabouts of the petitioner. It was then
that NEECO II uncovered that private respondent was at that
time already working with the Provincial Government of Nueva
Ecija as driver allegedly under an assumed name of Eduardo
Caimay. For these reasons, petitioner contended that it was
left with no other alternative but to terminate private
respondents services.
- Petitioners GM terminated private respondents services on
ground of abandonment. Immediately thereafter, private
respondent talked with the GM regarding this matter and the
latter promised him that the issue would be brought to the
attention of NEECOs Board of Directors for appropriate action.
But nothing came out of the GMs promise prompting private
respondent to institute a Complaint for illegal dismissal with
prayer for reinstatement and payment of backwages since the
NEECOs Board of Directors did not act upon his termination.
- The Labor Arbiter rendered a Decision declaring that private
respondent was illegally dismissed on the following grounds:
First, petitioners assertion that it required private respondent
to explain in writing why he was not reporting for duty as driver
assigned at Quezon Service Center merited scant consideration
since a copy of the alleged memorandum dated 22 November
1995, purportedly as its Annex A, was nowhere to be found in
the record of the case. Second, petitioners contention that
private respondent Cairlan was later discovered to be working
with the Provincial Government of Nueva Ecija under an
assumed name of Eduardo Caimay remained unsubstantiated
as petitioner failed to adduce independent evidence that said
Eduardo Caimay and private respondent Eduardo Cairlan are
one and the same person. Third, the Labor Arbiter held that the
private respondent was denied his right to due process since
the letter of termination dated 15 January 1996 stated that said
termination is retroactively effected on 1 January 1996. Finally,
according to the Labor Arbiter, petitioner failed to corroborate
its claim that private respondent was guilty of dereliction of
duty.
Public respondent NLRC dismissed for lack of merit. The NLRC
affirmed in toto the decision of Labor Arbiter. Hence this
petition.
ISSUES
1. WON petitioner was accorded due process
2.
WON petitioner is guilty of illegally dismissing private
respondent
HELD
1. YES
Ratio The rules of evidence prevailing in courts of law or
equity shall not be controlling and it is the spirit and intention of
this Code that the Commission and its members and the Labor
Arbiters shall use every and all reasonable means to ascertain
the facts in each case speedily and objectively and without
regard to technicalities of law or procedure, all in the interest of
due process.
Reasoning
- The Labor Arbiter shall motu proprio determine whether there
is need for a formal trial or hearing.

Labor Law 1
- Under Section 4, Rule V of the New Rules of Procedure of the
NLRC, the Labor Arbiter is given the latitude to determine the
necessity for a formal hearing or investigation, once the
position papers and other documentary evidence of the parties
have been submitted before him. The parties may ask for a
hearing but such hearing is not a matter of right of the parties.
The Labor Arbiter, in the exercise of his discretion, may deny
such request and proceed to decide the case on the basis of the
position papers and other documents brought before him
without resorting to technical rules of evidence as observed in
regular courts of justice.
- In the present case, a scrupulous study of the records reveals
that the Labor Arbiter did not abuse his discretion conferred
upon him by the Rules in not conducting a formal hearing. On
this, the findings of the Court of Appeals, consistent with that of
the NLRC and the Labor Arbiter, ought to be sustained.
2. YES
Ratio Abandonment3 is the deliberate and unjustified refusal of
an employee to resume his employment; it is a form of neglect
of duty; hence, a just cause for termination of employment by
the employer under Article 282 of the Labor Code, which
enumerates the just causes for termination by the employer:
i.e., (a) serious misconduct or willful disobedience by the
employee of the lawful orders of his employer or the latters
representative in connection with the employees work; (b)
gross and habitual neglect by the employee of his duties; (c)
fraud or willful breach by the employee of the trust reposed in
him by his employer or his duly authorized representative; (d)
commission of a crime or offense by the employee against the
person of his employer or any immediate member of his family
or his duly authorized representative; and (e) other analogous
causes.
Reasoning
- Private respondents alleged abandonment of work through his
employment with the Provincial Government of Nueva Ecija was
not clearly established and proven. The evidence submitted by
petitioner to buttress its allegation that private respondent
abandoned his work consists merely of indexes of payments to
employees under the name Eduardo Caimay without any further
evidence showing that Eduardo Caimay and private respondent
Eduardo Cairlan is one and the same person. The best
evidence that could have established the allegation that
Eduardo Caimay and private respondent Eduardo Cairlan is one
and the same person is Eduardo Caimays Personal Data Sheet
which definitely would have the pertinent personal information
about him and a picture that would identify him and not a
testimony of a representative from the Provincial Government
of Nueva Ecija, as adverted to by petitioner to justify its motion
for a trial type hearing.
- Worse, private respondent received his notice of termination
only on 15 January 1996 which termination is effective as early
as 01 January 1996, all in gross violation of the requirements
provided for by law.
- Further negating petitioners contention of abandonment, as
noted by the Labor Arbiter, is private respondents letter dated
04 March 1996 addressed to Mr. Danilo dela Cruz reiterating the
formers plea for reconsideration of his dismissal. This letter
depicts private respondents fervor and yearning to continue
working with petitioner the very antithesis of abandonment
Disposition AFFIRMED.

GABUAY V OVERSEA PAPER SUPPLY INC


436 SCRA 514
CALLEJO; August 13, 2004
NATURE
Petition for review of the decision of the Court of Appeals
FACTS
3

The elements of abandonment are: (a) failure to report for work or absence without
valid or justifiable reason; and (b) a clear intention to sever the employer-employee
relationship, with the second element as the more determinative factor manifested
by some overt acts (Tomas Lao Construction v. NLRC, 278 SCRA 716 [1997]).

A2010

- 244 -

Disini

- The respondent Oversea Paper Supply, Inc. is a domestic


corporation engaged in the business of selling paper products.
On different dates, the respondent corporation hired the
petitioners for the positions machine operators, driver and
helpers.
- On April 7, 1999, the respondent corporations sales and
operations manager, James C. Tan, required all employees to fill
up and submit their bio-data not later than April 17, 1999 so
that their 201 files could be updated. All the employees
complied except for the petitioners.
Petitioners William
Lacambra and Rodolfo Gabuay even failed to report for work
starting April 19 and 21, 1999, respectively.
- Thereafter, the respondent corporation required petitioners to
explain why they refused to submit their updated bio-data and
requiring each of them to (a) return to work, and (b) explain
why they were absent. Despite the receipt of such notices, the
petitioners, except for Reynante Lacambra, did not reoport back
to work.
- On April 21, 1999, petitioner Rodolfo Gabuay filed a complaint
for illegal dismissal, payment of separation pay, accumulated
vacation and sick leave, and reinstatement with full backwages
before the arbitration branch of the National Labor Relations
Commission (NLRC). On April 26, 1999, petitioners William
Lacambra, Reynante Lacambra, Rolando Vicente and Tomacito
Tabuli filed a similar complaint.
- The petitioners alleged that they were barred from reporting
for work after they refused to fill up their bio-data for the
respondent corporation. They also claimed that they were not
paid vacation and sick leave benefits; that their 13th month pay
for 1996 to 1998 was underpaid; and, that the respondents
violated their right to security of tenure and payment of
separation pay.
ISSUE
WON the petitioners were legally dismissed by reason of
abandonment of work
HELD
- As correctly ruled by the Labor Arbiter, the NLRC and the CA,
the petitioners were not illegally dismissed. Even after the
petitioners received notices from the respondent corporation
requiring them to report for work and to explain their
unauthorized absences and failure to submit their updated biodata, they still failed to report for work. It can then be inferred
that the petitioners had abandoned their work. Indeed, the
factors considered for finding a valid abandonment are present
in the case at bar: the petitioners failure to report for work or
absence was without valid or justifiable cause, and their refusal
to report for work notwithstanding their receipt of letters
requiring them to return to work, show their clear intention to
sever the employer-employee relationship.
- Consistent with the finding that the petitioners abandoned
their work, the award of financial assistance in the form of
separation pay should be deleted. Separation pay is defined as
the amount that an employee receives at the time of his
severance and is designed to provide the employee with the
wherewithal during the period that he is looking for another
employment. Under the Labor Code, the award of separation
pay is sanctioned when termination was due to an authorized
cause, i.e., (a) installation of labor saving device, redundancy,
retrenchment to prevent losses, closure or cessation of
business operations not due to serious business losses or
financial reverses; and, (b) disease prejudicial to the health of
the employee and his fellow employees.
- Separation pay is, likewise, awarded in lieu of reinstatement if
it can be shown that the reinstatement of the employee is no
longer feasible, as when the relationship between employer and
employee has become strained. In some cases, it is awarded as
a measure of social justice. In the present case, the petitioners
were not dismissed, either legally or illegally; the petitioners
abandoned their jobs. They failed to return to work despite the
respondents directive requiring them to do so. There is, thus,
no room for the award of financial assistance in the form of
separation pay. To sustain the claim for separation pay under

Labor Law 1
the circumstances herein established would be to reward the
petitioners for abandoning their work.
Disposition Petition denied

REQUISITES
LEONARDO V NLRC (REYNALDO'S MKTG CORP)
333 SCRA 589
DE LEON JR; June 16, 2000
NATURE
Petitions for certiorari seeking the annulment of a Decision of
the public respondent, NLRC.
FACTS
- Petitioner AURELIO FUERTE was originally employed by private
respondent REYNALDO'S MARKETING CORPORATION on August
11, 1981 as a muffler specialist, receiving P45.00 per day. He
was appointed as supervisor in 1988and his compensation was
increased.
- DANILO LEONARDO was hired by private respondent on March
4, 1988 as an auto-aircon mechanic.
- FUERTE alleges that on January 3, 1992, he was instructed to
report at private respondent's main office where he was
informed by the company's personnel manager that he would
be transferred to its Sucat plant due to his failure to meet his
sales quota, and for that reason, his supervisor's allowance
would be withdrawn.
- For a short time, FUERTE reported for work at the Sucat plant;
however, he protested his transfer, subsequently filing a
complaint for illegal termination.
- LEONARDO abandoned his post following an investigation
wherein he was asked to explain an incident of alleged
"sideline" work which occurred on April 22, 1991. It would
appear that late in the evening of the day in question, the driver
of a red Corolla arrived at the shop looking for LEONARDO. The
driver said that, as prearranged, he was to pick up LEONARDO
who would perform a private service on the vehicle. When
reports of the "sideline" work reached management, it
confronted LEONARDO and asked for an explanation. According
to private respondent, LEONARDO gave contradictory excuses,
eventually claiming that the unauthorized service was for an
aunt.
- When pressed to present his aunt, it was then that LEONARDO
stopped reporting for work. He filed a complaint for illegal
dismissal some ten months after his termination.
ISSUES
1. WON the demotion of Fuerte by the private respondent is
proper
2. WON Fuerte's action constitutes abandonment
3. WON the dismissal of Leornado is justified
HELD
1. YES
- Private respondent's justification is well-illustrated in the
record. Complainant Fuerte's failure to meet his sales quota
which caused his demotion and the subsequent withdrawal of
his allowance is fully supported by Exhibit "4" of respondents'
position paper showing that his performance for the months of
July 1991 to November 1991 is below par.
Reasoning
- FUERTE nonetheless decries his transfer as being violative of
his security of tenure, the clear implication being that he was
constructively dismissed. We have held that an employer acts
well within its rights in transferring an employee as it sees fit
provided that there is no demotion in rank or diminution in pay.
11
The two circumstances are deemed badges of bad faith, and
thus constitutive of constructive dismissal. In this regard,
constructive dismissal is defined in the following manner:

A2010

- 245 -

Disini

an involuntary resignation resorted to when continued


employment becomes impossible, unreasonable, or unlikely;
when there is a demotion in rank or diminution in pay; or
when a clear discrimination, insensibility or disdain by an
12
employer becomes unbearable to the employee.
- However, this arrangement appears to us to be an allowable
exercise of company rights. An employer is entitled to impose
productivity standards for its workers, and in fact, noncompliance may be visited with a penalty even more severe
than demotion. Thus, the practice of a company in laying off
workers because they failed to make the work quota has been
recognized in this jurisdiction.
- In the case at bar, the petitioners' failure to meet the sales
quota assigned to each of them constitute a just cause of their
dismissal, regardless of the permanent or probationary status of
their employment. Failure to observe prescribed standards of
work, or to fulfill reasonable work assignments due to
inefficiency may constitute just cause for dismissal. Such
inefficiency is understood to mean failure to attain work goals
or work quotas, either by failing to complete the same within
the allotted reasonable period, or by producing unsatisfactory
results. This management prerogative of requiring standards
may be availed of so long as they are exercised in good faith for
the advancement of the employer's interest.
2. NO
- his actions do not constitute abandonment. The filing of a
complaint for illegal dismissal, as in this case, is inconsistent
with a charge of abandonment.
Ratio To constitute abandonment there must be (1) failure to
report for work or absence without valid or justifiable
reason; and (2) a clear intention, as manifested by some
overt acts, to sever the employer-employee relationship.
3. YES
- He was not terminated by the company but Leonardo
abandoned his position in light of the pending investigation
against him. Abandonment is a valid ground for dismissal.
- He protests that he was never accorded due process. This
begs the question, for he was never terminated; he only
became the subject of an investigation in which he was
apparently loath to participate. As testified to by Merlin P.
Orallo, the personnel manager, he was given a memorandum
asking him to explain the incident in question, but he refused to
receive it. In an analogous instance, we held that an employee's
refusal to sign the minutes of an investigation cannot negate
the fact that he was accorded due process.
Disposition Petition dismissed.

R.P. DINGLASAN CONSTRUCTION INC V ATIENZA


433 SCRA 263
PUNO; June 29 2004
NATURE
Special Civil Action in the Supreme Court. Certiorari
FACTS
- This is an appeal from the decision and resolution of the Court
of Appeals, dated January 17, 2001 and October 30, 2002,
respectively, upholding the finding of constructive dismissal
against petitioner.
- Petitioner R.P. Dinglasan Construction, Inc. provided janitorial
services to Pilipinas Shell Refinery Corporation (Shell
Corporation) in Batangas City. Private respondents Mariano
Atienza and Santiago Asi served as petitioners janitors
assigned with Shell Corporation since 1962 and 1973,
respectively. Private respondents claim that on July 7, 1994,
petitioner called for a meeting and informed private
respondents and three (3) other employees that their
employment with Shell Corporation would be terminated
effective July 15, 1994. They were told that petitioner lost the
bidding for janitorial services with Shell. Petitioner notified
respondents that they may reapply as helpers and redeployed
in other companies where petitioner had subsisting contracts
but they would receive only a minimum wage.
Private

Labor Law 1
respondents refused as the offer would be a form of demotion
--- they would lose their seniority status and would not be
guaranteed to work at regular hours.
- In December 1994, private respondents filed a complaint
against petitioner for non-payment of salary with the district
office of the Department of Labor and Employment (DOLE) in
Batangas City.
In February 1995, during the conciliation
proceedings with the DOLE, petitioner sent notices to
respondents informing them that they would be reinstated with
Shell Corporation as soon as they submit their barangay
clearance, medical certificate, picture and information sheet as
per the new identification badge requirements of Shell
Corporation. Thereafter, petitioner again met with private
respondents, who were then accompanied by the barangay
captain and a councilor, and the latter confirmed to the former
their willingness to be reinstated. Private respondents duly
submitted the documents required for their reinstatement.
- In May 1995, respondents demanded the payment of their
backwages starting from July 15, 1994. On June 1, 1995,
petitioner notified private respondents that they have been
declared absent without leave (AWOL) as they allegedly failed
to signify their intention to return to work and submit the badge
requirements for their reinstatement. On June 13, 1995, private
respondents wrote petitioner and insisted that they had
complied with the badge requirements. Accompanied by the
barangay officials, private respondents attempted to meet with
the officers of petitioner but the latter refused to dialogue with
them. As proof of their compliance with the Shell requirements,
private respondents submitted to the DOLE their x-ray results,
dated May 17 and 19, 1995 and their barangay certification,
dated May 13, 1995. The case was eventually referred to the
National Labor Relations Commission (NLRC) for compulsory
arbitration. Private respondents amended their complaint
charging petitioner with illegal dismissal and non-payment of
13th month pay, with a claim for payment of attorneys fees and
litigation expenses, and a prayer for reinstatement with
payment of full backwages from July 15, 1994.
- Petitioner gave a different version of the incident. It allegedly
informed respondents and the other affected employees that
they would be deployed to petitioners other principal
companies but that their work would be different. Except for
private respondents, all the affected employees accepted its
offer of redeployment and reported back to work. Respondents
failed to submit a resignation letter to signify their intention not
to return to work. Thereafter, during the pendency of the labor
case, petitioner in two (2) separate notices, informed private
respondents that they could be reinstated at Shell Corporation
with no diminution in their salary provided that they submit the
documents for the new identification badge requirement of
Shell Corporation. Private respondents, however, refused to
return to work until they were paid their backwages.
Consequently, petitioner was constrained to consider them as
having abandoned their work and to terminate their
employment on September 19, 1995. Petitioner, thus, justified
the dismissal of private respondents on the grounds of gross
and habitual neglect of duties and abandonment of work. On
September 3, 1998, labor arbiter Andres Zavalla rendered a
decision finding that private respondents were illegally
dismissed from service and ordering their reinstatement.
- On appeal, the decision of the labor arbiter was affirmed by
the NLRC. Without moving for reconsideration, petitioner
immediately filed a petition for certiorari before the Court of
Appeals but petitioner suffered the same fate.
On the
procedural aspect, the Court of Appeals ruled that the petition
could not prosper as petitioner failed to move for a
reconsideration of the NLRC decision. On the substantive
issues, the appellate court upheld the findings of the labor
arbiter and the NLRC that: (1) private respondents were
constructively dismissed as petitioners offer of reassignment
involved a diminution in pay and demotion in rank that made
their continued employment unacceptable; and, (2) private
respondents could not be considered to have abandoned their
work.

A2010

- 246 -

Disini

- As petitioners motion for reconsideration was denied,


petitioner filed this appeal
ISSUES
1. WON the respondents dismissal is justified
2. WON the Court of Appeals, contrary to existing law, erred in
dismissing the petition for certiorari and affirming the decision
of the NLRC insofar as the monetary award is concerned
HELD
1. Ratio In an illegal dismissal case, the onus probandi rests
on the employer to prove that its dismissal of an employee is
for a valid cause. In the case at bar, petitioner failed to
discharge its burden.
It failed to establish that private
respondents deliberately and unjustifiably refused to resume
their employment without any intention of returning to work.
- To constitute abandonment of work, two (2) requisites must
concur: first, the employee must have failed to report for work
or must have been absent without justifiable reason; and
second, there must have been a clear intention on the part of
the employee to sever the employer-employee relationship as
manifested by overt acts. Abandonment as a just ground for
dismissal requires deliberate, unjustified refusal of the
employee to resume his employment. Mere absence or failure
to report for work, after notice to return, is not enough to
amount to abandonment.
Reasoning
- In the case at bar, the evidence of private respondents
negates petitioners theory that they abandoned their work.
Firstly, private respondents reported back to petitioners office
a number of times expressing their desire to continue working
for petitioner without demotion in rank or diminution of salary.
This fact was established by the corroborating testimony of
barangay councilman Valentin Clerigo who, together with the
barangay captain, accompanied private respondents to
petitioners office at least ten (10) times to negotiate their
redeployment on more acceptable terms. Secondly, in seeking
reinstatement, private respondents also sought the intervention
of the DOLE to arbitrate the labor issue between the parties.
Thirdly, private respondents submitted the barangay clearances
and x-ray results required from them by petitioner for their
reinstatement as witnessed by the barangay officials. Lastly,
the records would bear that private respondents lost no time
and sought their reinstatement by filing an illegal dismissal case
against petitioner, which act is clearly inconsistent with a desire
to sever employer-employee relations and abandon their work.
All these overt acts on the part of private respondents negate
petitioners claim of abandonment of work and prove beyond
doubt their steadfast desire to continue their employment with
petitioner and be reinstated to their former position. Moreover,
petitioner failed to explain why it waited for 14 months from the
time private respondents allegedly did not return to work before
it dismissed them for being AWOL.
- We hold that private respondents were constructively
dismissed by petitioner. Constructive dismissal is defined as
quitting when continued employment is rendered impossible,
unreasonable or unlikely as the offer of employment involves a
demotion in rank and diminution of pay. In the case at bar,
petitioner committed constructive dismissal when it offered to
reassign private respondents to another company but with no
guaranteed working hours and payment of only the minimum
wage.
The terms of the redeployment thus became
unacceptable for private respondents and foreclosed any choice
but to reject petitioners offer, involving as it does a demotion in
status and diminution in pay. Thereafter, for six (6) months,
private respondents were in a floating status. Interestingly, it
was only after private respondents filed a complaint with the
DOLE that petitioner backtracked in its position and offered to
reinstate private respondents to their former job in Shell
Corporation with no diminution in salary. Eventually, however,
petitioner unilaterally withdrew its offer of reinstatement,
refused to meet with the private respondents and instead
decided to dismiss them from service.
2. On the second issue, petitioner cannot impugn for the first

Labor Law 1
time the computation of the monetary award granted by the
labor arbiter to private respondents.
Doctrine The settled rule is that issues not raised or ventilated
in the court a quo cannot be raised for the first time on appeal
as to do so would be offensive to the basic rules of fair play and
justice. The computation of monetary award granted to private
respondents is a factual issue that should have been posed at
the arbitration level when the award was first granted by the
labor arbiter who received and evaluated the evidence of both
parties, or, at the latest, raised by petitioner in its appeal with
the NLRC.
- Petitioner omitted to do any of these. All throughout the
proceedings below, from the labor arbiter to the NLRC, and
even in its petition before the Court of Appeals, petitioner
repeatedly pounded only on the sole issue of the validity of its
dismissal of private respondents. Thus, at this late stage of the
proceedings, it cannot ask the Court to review the bases and
verify the correctness of the labor arbiters computation of the
monetary award which it never assailed below. A first-hand
evaluation of the evidence of the parties upon which the
monetary award is based belongs to the labor arbiter. This
Court is not a trier of facts and factual issues are improper in a
petition for review on certiorari. Likewise, the Court notes that
in seeking reinstatement and payment of their monetary
claims, private respondents have traversed a long and
difficult path. This case has passed the DOLE, the labor
arbiter, the NLRC, the Court of Appeals and now this Court, with
the finding of illegal dismissal having been consistently affirmed
in each stage. Private respondents had been rendering janitorial
services as early as 1962 and, at the time of their dismissal,
were receiving a measly P4,000.00 monthly salary. It is time to
put a period to private respondents travail. If there is anything
that frustrates the search for justice by the poor, it is the
endless search for it.

CHAVEZ V NLRC
[PAGE 59]
FLOREN HOTEL V NLRC (CALIMLIM, RICO, ET AL)
458 SCRA 128
QUISUMBING; May 6, 2005
FACTS
- At the time of their termination, private respondents Roderick
A. Calimlim, Ronald T. Rico and Jun A. Abalos were working in
the hotel as room boys, private respondent Lito F. Bautista as
front desk man, and private respondent Gloria B. Lopez as
waitress. They all started working for the hotel in 1993, except
for Jun A. Abalos who started only in 1995.
- In the afternoon of June 6, 1998, petitioner Dely Lim randomly
inspected the hotel rooms to check if they had been properly
cleaned. When she entered Room 301, she found private
respondent Lito F. Bautista sleeping half-naked with the airconditioning on. Lim immediately called the attention of the
hotels acting supervisor, Diosdado Aquino, who had supervision
over Bautista. Lim admonished Aquino for not supervising
Bautista more closely, considering that it was Bautistas third
offense of the same nature.
When she entered Room 303, she saw private respondents
Calimlim and Rico drinking beer, with four bottles in front of
them. They had taken these bottles of beer from the hotels
coffee shop. Like Bautista, they had switched on the air
conditioning in Room 303.
- That same afternoon, Dely Lim prepared a memorandum for
Bautista, citing the latter for the following incidents: (1)
sleeping in the hotel rooms; (2) entertaining a brother-in-law for
extended hours during duty hours; (3) use of hotel funds for
payment of SSS loan without management consent; (4)
unauthorized use of hotels air-con; and (5) failure to pay cash
advance in the amount of P4,000.
- Dely Lim tried to give Bautista a copy of the memorandum but
Bautista refused to receive it. Bautista then went on absence

A2010

- 247 -

Disini

without leave. Calimlim and Rico, embarrassed by the incident,


went home. When they returned to work the next day, they
were served with a notice of suspension for one week.
- Like Bautista, they refused to receive the notice of suspension,
but opted to serve the penalty. Upon their return on June 15,
1998, they saw a memorandum dated June 13, 1998 on the
bulletin board announcing (a) the suspension as room boys of
Calimlim and Rico, or alternately, (b) returning to work on
probation as janitors for the following reasons: unsatisfactory
work, having a drinking spree inside the hotels rooms, cheating
on the Daily Time Record, being absent without valid reason,
leaving work during duty time, tardiness, and sleeping on the
job. The memorandum also included Calimlim and Ricos new
work schedule.
- Calimlim and Rico submitted handwritten apologies and
pleaded for another chance, before they went AWOL
- On June 25, 1998, Calimlim, Rico and Bautista filed separate
complaints, for illegal dismissal and money claims, before the
Labor Arbiter. Abalos and Lopez later also filed separate
complaints for underpayment of wages, non-payment of their
13th month pay, and service incentive leave pay. On July 7,
1998, after they stopped working, Abalos and Lopez amended
their complaints. They claimed that petitioners orally dismissed
them when they refused to withdraw their complaints.
- Petitioners alleged that they did not dismiss private
respondents but that private respondents had abandoned their
jobs.
- Private respondents filed a manifestation and motion dated
November 24, 1998, praying that petitioners be ordered to
reinstate them to their former positions since after all,
according to petitioners, they were not dismissed.
- Petitioners opposed the motion and argued that private
respondents cannot be reinstated since they were not illegally
dismissed but they had abandoned their jobs and management
simply considered them dismissed for abandonment.
- On March 19, 1999, the Labor Arbiter dismissed the complaints
but ordered petitioners to pay private respondents their
proportionate 13th month pay, and service incentive leave pay.
He likewise ordered petitioners to pay Calimlim and Rico
indemnity.
- The Labor Arbiter found that Calimlim, Rico, and Bautista did
not report for work and they did not show any order of
dismissal, thus constructively, they abandoned their work and
were not illegally dismissed. The Labor Arbiter also ruled that
Calimlim and Ricos demotion and reassignment were valid
exercises of management prerogatives. The reassignment was
intended to enable management to supervise them more
closely and, in any event, did not involve a diminution of wages.
The Labor Arbiter, however, held petitioners liable for indemnity
to Calimlim and Rico for not observing the twin notices rule.
- Private respondents appealed to the National Labor Relations
Commission
- On March 22, 2000, the NLRC rendered its decision. It
reversed the decision of the Labor Arbiter and ordered the hotel
management to immediately reinstate complainants-appellants
to their former positions without loss of seniority rights, with full
backwages and other benefits until they are actually
reinstated. In the event that reinstatement was no longer
possible, the respondent-appellees should pay herein private
respondents their separation pay in addition to the payment of
their full backwages; their incentive leave pay and their 13th
month pay, together with P1,000 to each of them as indemnity.
- Later, the NLRC also denied petitioners motion for
reconsideration. The petitioners appealed to the Court of
Appeals.
- On September 10, 2002, the Court of Appeals decided the
petition as follows: (1) The Court declares that the private
respondents Roderick A. Calimlim and Jose Abalos [should be
Ronald T. Rico] were illegally dismissed by petitioner Floren
Hotel/Ligaya Chu who is ORDERED to reinstate them to their
former positions without loss of [seniority] rights, with full
backwages and other benefits until they are actually reinstated;
but if reinstatement is no longer possible, Floren Hotel/Ligaya
Chu shall pay their separation pay in addition to their

Labor Law 1
backwages. (2) Declaring private respondents Lito Bautista, Jun
Abalos and Gloria Lopez to have abandoned their employment,
and, therefore, not entitled to either backwages nor separation
pay; and (3) ORDERING Floren Hotel/Ligaya Chu to pay all the
private respondents their 13th month pay and incentive leave
pay as computed in the Decision of the Labor Arbiter
ISSUES
1. WON the Court of Appeals erred in giving due course to the
petition for certiorari filed before the appellate court
2. WON the private respondents were illegally dismissed
3 WON the Court of Appeals erred in ordering petitioners to pay
Calimlim and Rico indemnity of P1,500
4 WON the appellate court erred in ordering petitioners to pay
all of private respondents their proportionate 13th month pay
and incentive leave pay
HELD
1. NO
- Acceptance of a petition for certiorari as well as the grant of
due course thereto is addressed to the sound discretion of the
court. Section 1, Rule 65 of the Rules of Court in relation to
Section 3, Rule 46 of the same rules does not specify the
precise documents, pleadings or parts of the records that
should be appended to the petition other than the judgment,
final order, or resolution being assailed. The Rules only state
that such documents, pleadings or records should be relevant
or pertinent to the assailed resolution, judgment or orders.
2. YES
- Petitioners claimed that all five private respondents were
guilty of abandoning their jobs. Thus, it was incumbent upon
petitioners to show that the two requirements for a valid
dismissal on the ground of abandonment existed in this case.
Specifically, petitioners needed to present, for each private
respondent, evidence not only of the failure to report for work
or that absence was without valid or justifiable reasons, but also
of some overt act showing the private respondents loss of
interest to continue working in his or her job.
- It was true that private respondents abandoned their jobs,
then petitioners should have served them with a notice of
termination on the ground of abandonment as required under
Sec. 2, Rule XIV, Book V, Rules and Regulation Implementing
the Labor Code, in effect at that time. Said Section 2 provided
that:
Notice of Dismissal. Any employer who seeks to dismiss a
worker shall furnish him a written notice stating the particular
acts or omission constituting the grounds for his dismissal. In
cases of abandonment of work, the notice shall be served at
the workers last known address.
But petitioners failed to comply with the foregoing
requirement, thereby bolstering further private respondents
claim that they did not abandon their work but were illegally
dismissed.
- None of the private respondents in this case had any intention
to sever their working relationship. Just days after they were
dismissed, private respondents Calimlim, Rico, Bautista, Abalos
and Lopez filed complaints to protest their dismissals. The wellestablished rule is that an employee who takes steps to protest
his layoff cannot be said to have abandoned his work. That
private respondents all desired to work in the hotel is further
shown by the fact that during the proceedings before the Labor
Arbiter, shortly after private respondents received petitioners
position paper where the latter averred that private
respondents were never terminated, private respondents filed a
manifestation and motion asking that petitioners be ordered to
allow them back to work. This is nothing if not an unequivocal
expression of eagerness to resume working.
3. YES (should have reinstated)
- Article 279 of the Labor Code gives to Calimlim and Rico the
right to reinstatement without loss of seniority rights and other
privileges or separation pay in case reinstatement is no longer
possible, and to his full backwages, inclusive of allowances and
other benefits. It was thus error for the Court of Appeals to
affirm the NLRC decision to award Calimlim and Rico indemnity

A2010

- 248 -

Disini

in addition to the measure of damages provided in Article 279.


The award of indemnity is a penalty awarded only when the
dismissal was for just or authorized cause but where the twinnotice requirement was not observed.
4. NO
- Petitioners did not question the propriety of the award of
proportionate 13th month pay and service incentive leave in the
Court of Appeals. They assailed the NLRC decision on only one
ground: Respondent NLRC committed grave abuse of
discretion in reversing the Labor Arbiters decision insofar as it
relates to the issues of illegal dismissal. Hence, the
correctness of the cited award in the NLRC ruling was never
brought before the appellate court and is deemed to have been
admitted by petitioners. It cannot therefore be raised anymore
in this petition. The decision of the NLRC as regards the award
of 13th month pay and service incentive leave pay became
binding on petitioners because the failure to question it before
the Court of Appeals amounts to an acceptance of the ruling. In
any event, the award appears to us amply supported by
evidence and in accord with law.
Disposition Assailed decision MODIFIED

INFERENCE
HDA. DAPDAP V NLRC (BARRIENTOS JR)
285 SCRA 9
BELLOSILLO; January 26, 1998
FACTS
- Nine workers of Hda. Dapdap I, a sugar farm in Victorias,
Negros Occidental, filed a complaint for illegal dismissal against
its owner Magdalena Fermin alleging that they had been
working in the farm since 1977 but were unjustly terminated,
without notice and without any valid ground, on 27 January
1992.
- The only reason for their dismissal was their refusal to return
the 6-hectare lot given to them for cultivation under an
"Amicable Settlement in connection with an illegal dismissal
case previously filed against the management of Hda. Dapdap I
by its workers.
- In addition, complainants charged Magdalena Fermin with
unfair labor practice for trying to bust the National Federation of
Sugar Workers Food and General Trades (NFSW-FGT) Union
which forged the 1986 "Amicable Settlement."
- Eight of the original complainants withdrew from the
complaint and returned to work on the ground that their
misunderstanding with management was already settled.
- Pedro Barrientos Jr. was left as the sole complainant who
amended the complaint by impleading Lumbia Agricultural and
Development Corporation (LADCOR), the real owner of Hda.
Dapdap I, as co-respondent with its President Magdalena
Fermin.
- LADCOR denied that complainant was terminated and alleged
that complainant voluntarily abandoned his work to transfer to
the adjacent farm of a certain Mr. Ramos.
- In addition, LADCOR alleged that it had a personality separate
and distinct from its president, Magdalena Fermin, hence the
latter could not be held personally liable for the alleged illegal
dismissal.
- The Labor Arbiter ruled in favor of complainant.] While
LADCOR was absolved from the charge of unfair labor practice
it was held liable for illegal dismissal on the ground that its
claim of voluntary abandonment by complainant of his work
was not credible in view of the immediate institution of the case
for illegal dismissal.
- LADCOR appealed to the NLRC.
- The NLRC affirmed the Labor Arbiter's decision in toto. The
defense that complainant voluntarily abandoned his work was
similarly rejected on the additional grounds that no notice of
dismissal was sent by LADCOR to complainant as required by
Sec. 2, Rule 14, Book V, of the Rules Implementing the Labor

Labor Law 1
Code and no concurrence of the intention to abandon on the
part of complainant and overt acts from which it could be
inferred that he was no longer interested in working for
LADCOR.
ISSUE
WON petitioner was illegally dismissed
HELD
YES
- The Court is not a trier of facts. Whether respondent
voluntarily abandoned his work issue of credibility best left to
the determination of the Labor Arbiter. Great respect and even
finality is accorded the conclusions of the Labor Arbiter and the
NLRC in accordance with the well-settled rule that findings of
fact of labor arbiters affirmed by the NLRC are binding on the
Supreme Court.
- Judicial review in such cases is limited only to issues of
jurisdiction or grave abuse of discretion amounting to lack of
jurisdiction.
- No such grave abuse of discretion was committed by the NLRC
as it correctly applied the consistent ruling in labor cases that a
charge of abandonment is totally inconsistent with the
immediate filing of a complaint for illegal dismissal.
- It is indeed inconceivable that an employee like herein
respondent who has been working at Hda. Dapdap I since 1977
and cultivating a substantial portion of a 6-hectare lot therein
for himself would just abandon his work in 1992 for no apparent
reason.
- Nor could intent to abandon be presumed from private
respondent's subsequent employment with another employer
as petitioner alleges.
The fact that the start of such
employment, i.e., after 1 March 1992 as petitioner alleges,
coincides with the date of the original complaint strongly
indicates that such employment was only meant to help
respondent and his family survive during the pendency of his
case.
- It has been said that abandonment of position cannot be
lightly inferred, much less legally presumed from certain
equivocal acts such as an interim employment.
Disposition Petition was dismissed.

SPECIFIC ACTS
PREMIERE DEVT BANK V NLRC (LABANDA)
293 SCRA 49
MARTINEZ; July 23, 1988
NATURE
Petition for certiorari
FACTS
- August 8, 1985: Ramon T. Ocampo, a depositor of Premiere
Devt Bank (PDB), issued a check in the amount of P6,792.66 in
favor of and for deposit to the account of Country Banker's
Insurance Corporation (CBISCO), also a depositor of PDB. On the
same day, after the check and the deposit slip were presented
to respondent Teodora Labanda, who was employed as teller at
PDB Taytay Branch, they were turned over to the Branch
cashier for verification of the fund balance and signature of the
drawer. There was a confirmation of the check and the same
was accepted by Labanda for deposit to the current account of
CBISCO.
- The check was posted by Manuel S. Torio, the Taytay Branch
bookkeeper. But instead of posting it to CBISCO's account, the
same was posted to the account of Ocampo treating it as "OnUs Check," that is, drawn against the Taytay Branch where the
check was deposited.
- January 13, 1986: the wife of Ocampo, together with the
auditor from CBISCO, went to PDB and complained to PDB
Chairman Dr. Procopio C. Reyes that her husband was being
held accountable for the amount. It was only then that PDB

A2010

- 249 -

Disini

discovered the misposting of the check issued by Ocampo,


resulting in the overstatement of his outstanding daily balance
by P6,792.66. The overstatement remained undetected until
Ocampo withdrew the money from PDB.
- Due to this incident, PDB Asst VPres Pacita M. Araos sent a
demand letter to Labanda requesting her to explain in writing
the misposting and erroneous crediting of the subject check in
issue as well as the circumstances surrounding the incident
within three (3) days from receipt thereof, and in case she fails
to do so, necessary action shall be taken against her.
- PDB Exec VPres Renato G. Dionisio, upon instructions of
Reyes, sent the internal auditors of the bank to investigate and
make a detailed report about the incident.
- January 22, 1986: the auditors came out with a report finding
Labanda and bookkeeper Torio primarily liable for the incident.
These findings prompted Dionisio to send a letter to Labanda
requiring her to shoulder 20% of the amount lost via salary
deduction. Labanda replied, objecting to such move, reasoning
out that she is the breadwinner in the family. She further asked
the bank to furnish her a copy of the audit report and requested
for a full-dress investigation. For this reason, petitioners held in
abeyance the salary deductions.
- March 13, 1986: Labanda was placed under preventive
suspension pending investigation of the incident. She was
requested to report on April 4, 1986 so that she can present her
side of the story. Labanda then wrote a letter to Reyes
requesting information on the duration of her suspension and at
the same time asking for an expeditious investigation. In
response thereto, she was informed that the period of her
suspension shall last until the investigation is completed and a
decision is made thereon.
- On the date of said inquiry, Labanda executed a statement.
However, she manifested before Atty. Revelo during the inquiry
that she will not sign any of the preliminary statements she
made unless the same is with the consent and advice of her
husband. She also told the inquiring officer that she could not
inform petitioners of the dates when she would be available for
investigation.
- April 8, 1986: another letter was sent to Labanda by Reyes
informing the former that her refusal to sign or authenticate
preliminary statements given on April 4, 1986 was a clear
indication of her unwillingness to cooperate or an effort to hide
something or suppress the truth.
- The dates of the hearing were rescheduled by petitioners
several times. The first rescheduled hearing was on April 14,
1986 where Labanda sent her lawyer bringing with him a letter
asking that she be given time to confer with her counsel for
which she was given until April 23. Notices were sent to inform
her of the rescheduled dates with warning that failure to attend
the same shall be taken as a tacit admission of her liability and
the case shall be resolved based on the evidence available. In
the meantime, Bookkeeper Torio admitted liability and was
allowed to resign.
- April 7, 1986: the bank officials received a letter from Labanda
through her counsel demanding payment of actual damages in
the amount of P50,000.00 for their alleged arbitrary, illegal and
oppressive acts. Petitioners did not heed the demand.
- May 23, 1986: Labanda filed a complaint for damages before
the court. Petitioners motion to dismiss, and subsequent
motion for reconsideration were both denied. The petition for
certiorari was also dismissed by CA, without prejudice to the
refiling of the complaint with the labor arbiter. The decision
became final and executory on July 30, 1987.
- Eight months from the finality of the CA decision and two
years from the alleged termination of her employment, Labanda
filed an illegal dismissal case before the Labor Arbiter on the
ground that her dismissal was without lawful cause and without
due process. After trial, the Labor Arbiter dismissed the labor
case, ruling that Labanda was not illegally dismissed, and that
she abandoned her job when she filed a complaint for
compensatory damages with the regular court.
- NLRC reversed the decision of the Labor Arbiter ruling that
Labandas indefinite preventive suspension amounted to
constructive dismissal. It ordered PDB to immediately reinstate

Labor Law 1
Labanda to her former position with backwages and other
benefits for a period not exceeding three (3) years without
qualifications and deductions computed on the amount of
P87,750.00. It denied the subsequent MFR.
ISSUES
1. WON there was legal cause in placing Labanda under
preventive suspension
2. WON the filing of an action for damages against one's
employer is tantamount to abandonment of job
3. WON PDB violated due process requirements in dismissing
Labanda
4. WON Labandas action is barred by laches
HELD
1. NO
- Labanda's preventive suspension is without valid cause since
she was outrightly suspended by petitioner. As of the date of
her preventive suspension on March 13, 1986 until the date
when the last investigation was rescheduled on April 23, 1986,
more than 30 days had expired. The preventive suspension
beyond the maximum period amounted to constructive
dismissal.
- The question of whether or not an employee has abandoned
his/her work is a factual issue, not reviewable by this Court.
2. NO
- Labanda did not abandon her job. To constitute abandonment,
two elements must concur: (1) the failure to report for work or
absence without valid or justifiable reason, and (2) a clear
intention to sever the employer-employee relationship, with the
second element as the more determinative factor and
being manifested by some overt acts. Abandoning one's job
means the deliberate, unjustified refusal of the employee to
resume his employment and the burden of proof is on the
employer to show a clear and deliberate intent on the part of
the employee to discontinue employment.
- The law, however, does not enumerate what specific overt
acts can be considered as strong evidence of the intention to
sever the employee-employer relationship. An employee who
merely took steps to protest her indefinite suspension and to
subsequently file an action for damages, cannot be said to have
abandoned her work nor is it indicative of an intention to sever
the employer-employee relationship. Her failure to report for
work was due to her indefinite suspension.
Petitioner's
allegation of abandonment is further belied by the fact that
Labanda filed a complaint for illegal dismissal. Abandonment of
work is inconsistent with the filing of said complaint.
3. YES
- The twin requirements of notice and hearing constitute the
essential elements of due process which are set out in Rule XIV,
Book V of the Omnibus Rules Implementing the Labor Code.
- Granting arguendo that there was abandonment in this case, it
nonetheless cannot be denied that notice still has to be served
upon the employee sought to be dismissed, as the second
sentence of Section 2 of the pertinent implementing rules
explicitly requires service thereof at the employee's last known
address.
While it is conceded that it is the employer's
prerogative to terminate the services of an employee,
especially when there is a just cause therefor, the requirements
of due process cannot be taken lightly. The law does not
countenance the arbitrary exercise of such a power or
prerogative when it has the effect of undermining the
fundamental guarantee of security of tenure in favor of the
employee.
4. NO
- Laches is the failure for an unreasonable and unexplained
length of time to do that which in exercising due diligence,
could or should have been done earlier. It is negligence or
omission to assert a right within a reasonable time, warranting
the presumption that the party entitled to assert it either has
abandoned or has declined to assert it. A party cannot be held
guilty of laches when he has not incurred undue delay in the
assertion of his rights.
- Under the law, an illegal dismissal case is an action predicated

A2010

- 250 -

Disini

on the injury to the rights of the dismissed employee which


prescribes in four (4) years. On April 4, 1988 or eight months
from the finality of the Court of Appeals' decision and two years
from the alleged termination of employment by respondent
Labanda, she filed her complaint with the Labor Arbiter which is
within the four-year reglementary period. She did not sleep on
her rights for an unreasonable length of time.
- SolGen: Labanda never intended to abandon her job. First,
after her indefinite suspension, she requested that the "fulldressed" investigation be done at the quickest time possible,
and appealed to petitioner Reyes to consider that she was the
breadwinner in the family. Second, she actively fought for her
right to security of tenure by filing first with the RTC an action
for damages, and later with the Labor Arbiter a complaint for
illegal dismissal. Moreover, Labanda's inability to report for
work was not voluntary but was rather the result of her
indefinite suspension, which in reality was a constructive
dismissal. Petitioners never took the initiative to notify Labanda
to report back to work or charge the latter with abandonment of
work. These show that Labanda did not abandon her job but
was illegally dismissed from employment without due process
of law.
Disposition Petition is DISMISSED. The challenged NLRC
Resolution is AFFIRMED.

1. LOANS
BORROWING MONEY
MEDICAL DOCTORS INC V NLRC (MAGLAYA, ELOA)
136 SCRA 1
MAKASIAR; April 24, 1985
NATURE
An appeal of the decision of the NLRC.
FACTS
- Evelyn Eloa (complainant) was given a probationary
appointment as Clerk by the Makati Medical Center from July
16, 1975 to January 15, 1976, and assigned at the Out-Patient
Charity Department of said Medical Center.
- Two of the conditions embodied in the appointment:
'Comply with all existing policies, rules and regulations and
those that may be adopted or promulgated in the future
deemed necessary in the internal affairs of the employer;
'If at anytime during the probationary employment of the
employee her services are judged to be unsatisfactory, the
employer may terminate such employment.'
- The termination or dismissal was and is predicated mainly on
the fact that Evelyn Elona borrowed P50 from one of the
patients, Mrs. Leticia Lavapiez, allegedly in violation of
respondent's policies, rules and regulations against solicitation
of any consideration from indigent patients. The borrowing took
place at Mrs. Lavapiezs house and after she was discharged
from the OPCD. The amount of P50 that was borrowed was also
returned, remitted or paid by complainant to Mrs. Lavapiez
- Eloa worked in this capacity of clerk continuously until
February 14, 1976 when she was dismissed or terminated.
- NLRC: Borrowing money and paying the same is not an act of
dishonesty, of immorality, of illegality, or of omissions
punishable by law as to be a ground for dismissal as in this
case. We so hold that the Rules and Regulations & Policies of
respondent Medical Center are whimsical, capricious, arbitrary
and oppressive The facts and the law point unerringly to her
side. She has completed her probationary period. Her
employment contract is not covered by an apprenticeship
agreement stipulating a longer period.
ISSUE
WON Eloa was justly dismissed on sole reason of borrowing
money from the patients

Labor Law 1
HELD
NO, Eloa was not dismissed justly.
Ratio Borrowing money is neither dishonest, nor immoral nor
illegal, much less criminal.
Reasoning
- Private respondent paid the money she borrowed from the
hospital patient. She was even recommended for permanent
appointment from her probationary status, from clerk to
secretary, by her immediate superior, Sis. Consolacion Briones.
- It may be added that she must have been compelled to borrow
P50.00 from her patient because of economic necessity, which
circumstance should evoke sympathy from this Court, the very
constitutional organ mandated by the fundamental law to
implement the social justice guarantee for the protection of the
lowly, efficient and honest employee, who is economically
disadvantaged, like herein petitioner.
Disposition Petition is dismissed, and decision of the labor
arbiter is affirmed, with the modification that backwages should
cover three (3) years.

SEPARATE OPINION
AQUINO [dissent]

- Nicolas A. Zarate, the chief of the public information


assistance division, apprised the Makati Medical Center of
Evelyn's conduct. Zarate alleged that Evelyn "has the habit of
borrowing money from OPD patients of that hospital." Evelyn
allegedly borrowed P100 from Leticia Lavapiez after she
delivered a baby. She attempted to borrow money from Teofila
Luzon and tried to ask for lunch from another patient, Mrs.
Fabian. A copy of the denunciation was furnished Mayor
Nemesio Yabut.
- To have more time for investigating the charge, Evelyn's
probationary appointment was extended by one month or up to
February 15, 1976. After due investigation, Consolacion
Briones, the supervisor of the Outpatient Charity Department,
submitted a report exonerating Evelyn. The Barangay
Secretariat of Makati also recommended Evelyn's exoneration.
- Eloa should not be reinstated or placed under permanent
status because, as correctly observed by Commissioner
Villatuya of NLRC, she was dismissed when she was still a
probationary employee. It is true that the probationary status
does not exceed six months but under the peculiar
circumstances of this case Evelyn's probationary or temporary
status was extended for one month due to the investigation.
This may well be considered an exceptional case. Evelyn is not
the kind of employee who can invoke security of tenure.

PEARL S. BUCK FOUNDATION V NLRC


182 SCRA 446
GUTIERREZ; February 21, 1990
NATURE
Appeal from the decision of the NLRC as well as the resolution
denying the motion for reconsideration
FACTS
- Petitioner Pearl S. Buck Foundation, Inc. extends financial,
education and medical assistance to indigent "Amerasian"
youth through funds provided by individuals and church groups
in the US. Private respondent Rubini Gosiaco Querimit was
employed by the petitioner as a case worker in the Olongapo
City branch. One of the wards assigned to Mrs. Querimit as such
case worker was Richard Aliarte, Amerasian son of Andrea
Aliarte.
- It appears that Mrs. Querimit borrowed P300 from Andrea
Aliarte. It is not clear from the records when she paid said debt
but Mrs. Querimit once again borrowed P3,000.00 from Aliarte,
who requested assistance from petitioner for the collection of
the indebtedness. Mrs. Querimit paid the amount allegedly only
after the petitioner had exerted incessant pressure on her.

A2010

- 251 -

Disini

Thereafter, she received a letter dated from the petitioner's


resident director informing her that her services would be
terminated. Mrs. Querimit filed in the NLRC a complaint for
illegal dismissal, underpayment, overtime pay and maternity
benefits.
- The labor arbiter dismissed the complaint for lack of merit. On
appeal, the NLRC opined that borrowing money is not a ground
for termination of employment under the Labor Code and that
the loan is a "personal transaction" between Andrea Aliarte and
Mrs. Querimit "the respondent not being a privy to (the)
transaction and hence, had no cause to dismiss the complainant
from her job more so that the loan had earlier been paid and
settled." The petitioner filed an MFR. After it was denied, the
petitioner filed the instant petition.
ISSUE
WON private respondent was illegally dismissed
HELD
1. NO
Ratio Borrowing money is neither dishonest, nor immoral, nor
illegal, much less criminal. However, said act becomes a serious
misconduct that may justly be asserted as a ground for
dismissal when reprehensible behavior such as the use of a
trust relationship as a leverage for borrowing money is
involved.
Reasoning
- The fact that Aliarte has retracted her complaint is of no
moment. She loaned money to the respondent, not once but
twice and there can be no other assumption where the money
came from except from the trust funds intended for the ward.
The NLRC should have considered that a higher degree of
prudence is required of the foundation's employees especially
when it comes to financial matters affecting the petitioner's
wards. The petitioner solicits or "begs" for money from abroad
to support its wards. It cannot be a third person where that
money is involved.
Disposition The petition is GRANTED. The decision of the
NLRC is REVERSED and SET ASIDE. The decision of the Labor
Arbiter is REINSTATED.

2. COURTESY RESIGNATION
BATONGBACAL V ASSOCIATED BANK
168 SCRA 600
FERNAN; December 21, 1988
NATURE
Review of the decision of the NLRC
FACTS
- Bienvenido Batongbacal, a lawyer, worked for Citizens Bank
and Trust Company from 1961. On 1975, Citizens Bank and
Trust Company merged with the Associated Banking
Corporation. The merged corporate entity later became known
as Associated Bank. In the new bank, petitioner resumed his
position as assistant vice-president.
- On March 1982, he learned that his salary was very much
below compared to the other Asst. VPs of the bank. He wrote to
the Board of Directors asking that he be paid the proper
amount. Apparently, said letter fell on deaf ears.
- On March 15, 1982, the board approved the following
resolution:
BE IT RESOLVED that the new management be given the
necessary flexibility in streamlining the operations of the
Bank and for the purpose it is hereby resolved that the Bank
officers at the Head Office and the Branches with corporate
rank of Manager and higher be required, as they hereby are
required to submit IMMEDIATELY to the President their
courtesy resignations.
- Petitioner did not submit his courtesy resignation. On May 3,
1983, he received a letter from the Board saying that his

Labor Law 1
resignation has been accepted. Petitioner wrote to the
executive VP asking for reconsideration. He stated therein that
he thought the call for the submission of courtesy resignations
was only for erring "loathsome" officers and not those like him
who had served the bank honestly and sincerely for sixteen
years.
- Starting May 4, 1983, he was not paid. He filed for illegal
dismissal and damages with the NLRC. The NLRC ruled in favor
of the petitioner. On MFR, the NLRC reversed.
ISSUE
WON the bank may legally dismiss for refusal to tender the
courtesy resignation which the bank required in line with its
reorganization plan
HELD
NO
- While it may be said that the private respondent's call for
courtesy resignations was prompted by its determination to
survive, we cannot lend legality to the manner by which it
pursued its goalBy directing its employees to submit letters of
courtesy resignation, the bank in effect forced upon its
employees an act which they themselves should voluntarily do.
It should be emphasized that resignation per se means
voluntary relinquishment of a position or office. 11 Adding the
word "courtesy" did not change the essence of resignation. That
courtesy
resignations
were
utilized
in
government
reorganization did not give private respondent the right to use
it as well in its own reorganization and rehabilitation plan. There
is no guarantee that all employers will not use it to rid
themselves arbitrarily of employees they do not like, in the
guise of "streamlining" its organization. On the other hand,
employees would be unduly exposed to outright termination of
employment which is anathema to the constitutional mandate
of security of tenure
- The record fails to show any valid reasons for terminating the
employment of petitioner. There are no proofs of malfeasance
or misfeasance committed by petitioner which jeopardized
private respondent's interest.
- However, we agree with the Solicitor General and the NLRC
that petitioner is not entitled to an award of the difference
between his actual salary and that received by the assistant
vice-president who had been given the salary next higher to his.
There is a semblance of discrimination in this aspect of the
bank's organizational set-up but we are not prepared to
preempt the employer's prerogative to grant salary increases to
its employees. In this connection, we may point out that private
respondent's claim that it needed to trim down its employees as
a self-preservation measure is belied by the amount of salaries
it was giving its other assistant vice-presidents
Disposition Remanded to the NLRC to determine WON the
petitioner is a managerial employee

3. WORK ATTITUDE
ABSENCES
MANILA ELECTRIC CO V NLRC
[PAGE 186]
GSP MANUFACTURING CORP V CABANBAN
495 SCRA 123
CORONA; July 14, 2006
NATURE
Petition for review on certiorari from a decision and a resolution
of the Court of Appeals.
FACTS

A2010

- 252 -

Disini

- Cabanban worked with GSP Manufacturing Corporation (GSP)


as a sewer from February 7, 1985 until her alleged termination
on March 1, 1992.
- On June 16, 1992, respondent filed with the National Labor
Relations Commission (NLRC), National Capital Region
Arbitration Branch, a complaint against petitioners for illegal
dismissal, non-payment of holiday pay, service incentive leave
pay and 13th month pay. She claimed she was terminated by
petitioners because she failed to dissuade her daughter from
continuing her employment at the Sylvia Santos Company, a
business competitor of petitioners.In their defense, petitioners
argued that respondent abandoned her work on March 14, 1992
and that they reported this to the Department of Labor and
Employment on May 15, 1992.
- Labor arbiter found petitioners guilty of illegal dismissal.
Petitioners appealed to the NLRC, it was dismissed. Petition to
CA was also dismissed. They claim that these findings, based
solely on statements made by respondent in the affidavit
attached to her position paper, were arrived at arbitrarily.
ISSUE
WON respondent is guilty of abandonment
HELD
NO
- Abandonment as a just ground for dismissal requires the
deliberate, unjustified refusal of the employee to perform his
employment responsibilities. Mere absence or failure to
work, even after notice to return, is not tantamount to
abandonment. The records are bereft of proof that petitioners
even furnished respondent such notice.
- Furthermore, it is a settled doctrine that the filing of a
complaint for illegal dismissal is inconsistent with abandonment
of employment. An employee who takes steps to protest his
dismissal cannot logically be said to have abandoned his work.
The filing of such complaint is proof enough of his desire to
return to work, thus negating any suggestion of abandonment.
- Clearly, petitioners claim that respondents complaint was an
afterthought, having been filed a long time after the date of
the supposed abandonment, was utterly without merit. As the
Court of Appeals correctly pointed out, citing the case of Pare v.
NLRC, respondent had four years within which to institute her
action for illegal dismissal. Compared to the six months it took
the aggrieved employee in that case to file his complaint for
illegal dismissal, respondents 84 days was not unreasonably
long at all.
Disposition petition is hereby DENIED. The assailed decision
and resolution of the Court of Appeals in CA-G.R. SP No. 51161
are hereby AFFIRMED.

4. TERM EMPLOYMENT
BRENT SCHOOL V ZAMORA
[PAGE 94]

ROMARES V NLRC
294 SCRA 411
MARTINEZ; August 19, 1998
NATURE
Appeal from a decision of NLRC
FACTS
- Complainant-petitioner Romares has been hired and employed
by respondent PILMICO since Sept 1, 89 to Jan 15, 93, in a
broken tenure but all in all totalled to over a year's service.
Complainant's period of employment started on Sept 1, 89 up
to Jan 31, 90 or for a period of 5 months. Then on Jan 16 91, he
was hired again up to June 15, 91, or for a period covering

Labor Law 1
another 5 months. Then on Aug 16, 92, he was hired again up
to Jan 15, 93 or for a period of another 5 months. Thus, from
Sept 1, 1989 up to January 15, 1993, complainant has worked
for 15 months more or less and has been hired and terminated
3 times. In all his engagements by respondent, he was assigned
at
respondent's
Maintenance/Projects/Engineering
Dept
performing maintenance work, particularly the painting of
company buildings, maintenance chores, like cleaning and
sometimes operating company equipment and sometimes
assisting the regulars in the Maintenance/ Engineering Dept.
- Petitioners arguments: That having rendered a total
service of more than 1 year and by operation of law,
complainant has become a regular employee of respondent;
That complainant has performed tasks and functions which
were necessary and desirable in the operation of respondent's
business which include painting, maintenance, repair and other
related jobs; That complainant was never reprimanded nor
subjected to any disciplinary action during his engagement with
the respondent; That without any legal cause or justification
and in the absence of any time to know of the charge or notice
nor any opportunity to be heard, respondent terminated him;
That his termination is violative of the security of tenure clause
provided by law; That complainant be awarded damages and be
reinstated to his former position, be awarded backwages, moral
and exemplary damages and atty's fees.
- Respondents arguments: That complainant was a former
contractual employee of respondent and as such his
employment was covered by contracts; That complainant was
hired as mason in the Maintenance/Project Department and that
he was engaged only for a specific project under such
department; That when his last contract expired on Jan 15,
1993, it was no longer renewed and thereafter, complainant
filed this instant complaint; and; That since petitioner's
employment contracts were for fixed or temporary periods, as
an exception to the general rule, he was validly terminated due
to expiration of the contract of employment.
- LA ruled in favor of petitioner finding him to be a regular
employee and hence should be reinstated. NLRC reversed LA
decision ruling that petitioner was engaged in a fixed term
employment and as such, his termination was valid due to
expiration of employment contract. Hence, this appeal.
ISSUE
WON dismissal of complainant (under the just cause that such
employment was of term employment) was justified
HELD
NO
[a] Petitioner was deemed a regular employee. Petitioners work
with PILMICO as a mason was definitely necessary and desirable
to its business. PILMICO cannot claim that petitioner's work as a
mason was entirely irrelevant to its line of business in the
production of flour yeast feeds and other flour products. During
each rehiring, the summation of which exceeded 1 year,
petitioner
was
assigned
to
PILMICO's
Maintenance/Projects/Engineering Dept performing the same
kind of maintenance work such as painting of company
buildings cleaning and operating company equipment, and
assisting the other regular employees in their maintenance
works. Such a continuing need for the services of petitioner is
sufficient evidence of the necessity and indispensability of his
services to PILMICO's business or trade.
[b] Even assuming arguendo that petitioner was temporary EE,
he was converted to regular employee ff this rule: If the
employee has been performing the job for at least one year,
even if the performance is not continuous or merely
intermittent, the law deems the repeated and continuing need
for its performance as sufficient evidence of the necessity is not
indispensability of that activity to the business. Hence, the
employment is also considered regular but only with respect to
such activity and while such activity exists.
[c] In rehiring petitioner, employment contracts ranging from 2
to 3 months with an express statement that his temporary
job/service as mason shall be terminated at the end of the said

A2010

- 253 -

Disini

period or upon completion of the project was obtrusively a


convenient subterfuge utilized to prevent his regularization. It
was a clear circumvention of the employee's right to security of
tenure and to other benefits. It likewise evidenced bad faith on
the part of PILMICO.
[d] NLRC erred in finding that the contract of employment of
petitioner was for a fixed or specified period. From Brent v
Zamora: The decisive determinant in "term employment"
should not be the activities that the employee is called upon to
perform but the day certain agreed upon by the parties for the
commencement and termination of their employment
relationship. But, if from the circumstances it is apparent that
the periods have been imposed to preclude acquisition of
tenurial security by the employee, they should be struck down
or disregarded as contrary to public policy and morals.
Note however that, "term employment" cannot be said to be in
circumvention of the law on security of tenure if: (1) The fixed
period or employment was knowingly and voluntarily agreed
upon by the parties without any force, duress, or improper
pressure being brought to bear upon the employee and absent
any other circumstances vitiating his consent; or (2) It
satisfactorily appears that the employer and the employee dealt
with each other on more or less equal terms with no moral
dominance exercised by the former or the latter None of these
requisites were complied with.
Disposition Petition GRANTED. NLRC decision SET ASIDE. LA
decision REINSTATED

MEDENILLA V PHIL VETERANS BANK


PURISIMA; March 13, 2000
FACTS
- Petitioners were employees of the Philippine Veterans Bank
(PVB). On June 15, 1985, their services were terminated as a
result of the liquidation of PVB. On the same day of their
termination, petitioners were rehired through PVB's Bank
Liquidator.
- All of them were required to sign employment contracts which
provided that:
(1) The employment shall be strictly on a temporary basis and
only for the duration of the particular undertaking for which a
particular employee is hired; (3) The Liquidator reserves the
right to terminate the services of the employee at any time
during the period of such employment if the employee is found
not qualified, competent or, efficient in the performance of his
job, or have violated any rules and regulations, or such
circumstances and conditions recognized by law.
- January 18, 1991 their employment was terminated. The
reasons for which were "(a) To reduce costs and expenses in
the liquidation of closed banks in order to protect the interests
of the depositors, creditors and stockholders of Bank. (b) The
employment were on strictly temporary basis."
- Petitioners filed for illegal dismissal. Labor Arbiter found for
employees. NLRC however reversed decision
ISSUES
1. WON NLRC gravely abused its discretion in holding that the
employment contract entered into by the complainants and the
Liquidator of PVB was for a fixed-period
2. WON NLRC act with grave abuse of discretion in finding that
there was no illegal dismissal
HELD
1. NO
- Employment contract between parties states that:
(1) The employment shall be on a strictly temporary basis and
only for the duration of the particular undertaking for which you
are hired and only for the particular days during which actual
work is available as determined by the Liquidator or his
representatives since the work requirements of the liquidation
process merely demand intermittent and temporary rendition of
services."

Labor Law 1
- The Court has repeatedly upheld the validity of fixed-term
employment.
Philippine
National
Oil
Company-Energy
Development Corporation vs. NLRC gave two guidelines by
which fixed contracts of employment can be said NOT to
circumvent security of tenure:
1. The fixed period of employment was knowingly and
voluntarily agreed upon by the parties, without any force,
duress or improper pressure being brought to bear upon the
employee and absent any other circumstances vitiating his
consent;
or:
2. It satisfactorily appears that the employer and employee
dealt with each other on more or less equal terms with no
moral dominance whatever being exercised by the former on
the latter."
- The employment contract entered into by the parties herein
appears to have observed the said guidelines. Furthermore, it is
evident from the records that the subsequent re-hiring of
petitioners which was to continue during the period of
liquidation and the process of liquidation ended prior to the
enactment of RA 7169 entitled, "An Act to Rehabilitate
Philippine Veterans Bank
2. YES
- The reason given by the Liquidator for the termination of
petitioners' employment was "in line with the need of the
objective of the Supervision and Examination Sector,
Department V, Central Bank of the Philippines, to reduce costs
and expenses in the liquidation of closed banks in order to
protect the interest of the depositors, creditors and
stockholders
- In cases of illegal dismissal, the burden is on the employer to
prove that there was a valid ground for dismissal. Mere
allegation of reduction of costs without any proof to
substantiate the same cannot be given credence by the Court.
As the respondents failed to rebut petitioners' evidence, the
irresistible conclusion is that the dismissal in question was
illegal.
- the failure of respondent bank to dispute complainants'
evidence pertinent to the various unnecessary and highly
questionable expenses incurred renders the termination
process as a mere subterfuge, as the same was not on the basis
as it purports to see, for reason that immediately after the
termination from their respective positions, the same were
given to other employees who appear not qualified. What
respondent's counsel did was merely to dispute by pleadings
the jurisdiction of this Office and the claims for damages, which
evidentiary matters respondent is required to prove to sustain
the validity of such dismissals."
- As held by this Court, if the contract is for a fixed term and the
employee is dismissed without just cause, he is entitled to the
payment of his salaries corresponding to the unexpired portion
of the employment contract

MAGSALIN V NATIONAL ORGANIZATION OF


WORKING MEN
[PAGE 77]
LABAYOG V MY SAN BISCUITS INC
[PAGE 89]

5. PAST INFRACTIONS
PAST OFFENSES
STELLAR INDUSTRIAL SERVICE INC V NLRC
(PEPITO)
252 SCRA 323
REGALADO; January 24, 1996
NATURE

A2010

- 254 -

Disini

Special Civil Action for Certiorari


FACTS
- Stellar Industrial Services, Inc., an independent contractor
engaged in the business of providing manpower services,
employed private respondent Roberto H. Pepito as a janitor on
January 27, 1975 and assigned the latter to work as such at the
Maintenance Base Complex of the Philippine Airlines in Pasay
City.
- Pepito worked for 15 years.
- According to petitioner, private respondent committed
infractions of company rules ranging from tardiness to
gambling, but he was nevertheless retained as a janitor out of
humanitarian consideration and to afford him an opportunity to
reform.
- Stellar finally terminated private respondent's services on
January 22, 1991 because of Absent Without Official
Leave/Virtual Abandonment of Work Absent from November 2 December 10, 1990.
- Private respondent had insisted that during the period in
question he was unable to report for work due to severe
stomach pain and that, as he could hardly walk by reason
thereof, he failed to file the corresponding official leave of
absence. Attached was a medical certificate.
- Petitioner filed a complaint for illegal dismissal, illegal
deduction and underpayment of wages with prayer for moral
and exemplary damages and attorney's fees.
- LA was of the view that Pepito was not entitled to differential
pay, or to moral and exemplary damages for lack of bad faith
on the part of the company, he opined that private respondent
had duly proved that his 39-day absence was justified on
account of illness and that he was illegally dismissed without
just cause. He ordered the respondent to immediately reinstate
complainant to his former position as Utilityman, without loss of
seniority rights and with full backwages and other rights and
privileges appurtenant to his position until he is actually
reinstated.
- The respondent is further ordered to pay the complainant
reasonable attorney's fees equivalent to 10% of the amount
recoverable by the complainant.
- LAs decision was affirmed by NLRC
ISSUES
1. WON serious misconduct for nonobservance of company
rules and regulations may be attributed to Pepito
2. WON the extreme penalty of dismissal meted to him by
Stellar may be justified under the circumstances

HELD
1. NO
- Stellar's company rules and regulations on the matter could
not be any clearer, to wit:
"Absence Without Leave"
Any employee who fails to report for work without any
prior approval from his superior(s) shall be considered
absent without leave.
In the case of an illness or emergency for an absence
of not more than one (1) day, a telephone call or
written note to the head office, during working hours,
on the day of his absence, shall be sufficient to avoid
being penalized.
In the case of an Illness or an emergency for an
absence of two (2) days or more, a telephone call to
the head office, during regular working hours, on the
first day of his absence, or a written note to the head
office, (ex. telegram) within the first three (3) days of
his absence, and the submission of the proper
documents (ex. medical certificate) On the first day he
reports after his absence shall be sufficient to avoid
being penalized.
1st offense- three (3) days suspension
2nd offense- seven (7) days suspension

Labor Law 1
3rdoffense- fifteen (15) days suspension
4th offense- dismissal with a period of one (1) year
- There was substantial compliance with said company rule by
private respondent. He immediately informed his supervisor of
the fact that he could not report for work by reason of illness. At
the hearing, it was also established without contradiction that
Pepito was able to talk by telephone to one Tirso Pamplona,
foreman, and he informed the latter that he would be out for
two weeks as he was not feeling well. Added to this is his letter
to the chief of personnel which states that, on November 2,
1990, he relayed to his supervisor his reason for not reporting
for work and that, thereafter, he made follow-up calls to their
office when he still could not render services. As earlier noted,
these facts were never questioned nor rebutted by petitioner.
- While there is no record to show that approval was obtained
by Pepito with regard to his absences, the fact remains that he
complied with the company rule that in case of illness
necessitating absence of two days or more, the office should be
informed beforehand about the same that is, on the first day of
absence. Since the cause of his absence could not have been
anticipated, to require prior approval would be unreasonable.
On this score, then, no serious misconduct may be imputed to
Pepito. Necessarily, his dismissal from work, tainted as it is by
lack of just cause, was clearly illegal.
2. NO
- Petitioner's reliance on Pepito's past infractions as sufficient
grounds for his eventual dismissal, in addition to his prolonged
absences, is unavailing. The correct rule is that previous
infractions may be used as justification for an employee's
dismissal from work in connection with a subsequent similar
offense.
- In the present case, private respondent's absences, as already
discussed, were incurred with due notice and compliance with
company rules and fie had not thereby committed a "similar
offense" as those lie had committed in the past. Furthermore,
as correctly observed by the labor arbiter, those past infractions
had either been "satisfactorily explained, not proven,
sufficiently penalized or condoned by the respondent." In fact,
the termination notice furnished Pepito only indicated that he
was being dismissed due to his absences from November 2.
1990 to December 10, 1990 supposedly without any acceptable
excuse therefor. There was no allusion therein that his dismissal
was due to his supposed unexplained absences on top of his
past infractions of company rules. To refer to those earlier
violations as added grounds for dismissing him is doubly unfair
to private respondent. Significantly enough, no document or
any other piece of evidence was adduced by petitioner showing
previous absences of Pepito, whether with or without official
leave.
Disposition Petition dismissed

LA CARLOTA PLANTERS ASSN V NLRC


(COMPACION)
298 SCRA 252
VITUG; October 27, 1998
NATURE
Petition for certiorari which seeks to set aside and nullify the
decision of the NLRC promulgated on 25 September 1995
setting aside the LAs decision and directing the respondent to
pay complainant backwages and separation pay in lieu of
reinstatement, computed at one (1) month per year of service.
FACTS
- Compacion alleges that he was a regular employee of
petitioner since 1988 hired as truck driver; that on December
14, 1992, at the instance of the petitioner, he drove the truck
overloaded of sugarcane bound for La Carlota Sugar Central;
that while driving through Sitio Bacus, Ma-ao, Bago City, the
road was very slippery causing the truck to be outbalance (sic)
resulting to the truck turning right side down; that he was not
drunk when he drove the truck on December 14, 1992; that the

A2010

- 255 -

Disini

Security Guards of Central La Carlota issued a clearance to the


effect that he is cleared from whatever issues against him; that
Rene Baylon reported the incident only on March 1993 when
the incident happened on December 14, 1992 as shown by the
Police Blotter; that because of his illegal dismissal, he sought
the help of a legal counsel who helped him in filing this case for
which he claims for payment of attorney's fees.
- On the other hand, petitioner alleges that Compacion is a
truck driver of Nature's Beauty Trucking Services; as such, he
was assigned to Ma-ao Transloading Station, a loading station
of sugarcanes bound for Central La Carlota located at Brgy. Maao, Bago City, Negros Occidental; that on December 14, 1992,
Compacion who was very drunk and with a knife entered the
Ma-ao Transloading Station and harassed the office personnel
to the extent of stabbing the person of Gerry Flores who
fortunately was able to escape the said assault; that despite the
repeated warning made by the Shifting In-charge Rene Baylon
not to drive the truck, he drove the ten wheeler truck loaded
with 18 tons of sugarcane bound for La Carlota Central in a
reckless manner causing the truck to turn right side down
resulting in a damage to property paid by the owner to Mr.
Eulalio Pagunsan, owner of the bananas and pig pen hit and
destroyed by the truck; that the said Mr. Eulalio Pagunsan
observed that the driver Felix Compacion, at the time of the
accident, was very drunk; that because of this accident which
happened because of reckless driving, the truck underwent
major repair; that after the accident, driver Felix Compacion
was nowhere to be found, never reporting the accident to the
police authorities or to the owner; that despite repeated calls,
he refused to meet the owner nor did he report to the office
thus prompting the latter to write him a letter dated January 4,
1993 suspending him for 30 days; further requiring him to
report to the office and explain why he should not be
terminated.
- Petitioner further averred that during his employment,
Compacion was paid wages and other benefits in accordance
with law; that at the time of the accident, there was no rain and
the road was not slippery; that at the time he stopped
reporting, he has an outstanding account with respondent in the
amount of P3,650.00; that prior to this accident on December
14, 1992, specifically on November 27, 1992, Felix Compacion
was caught stealing diesel fuel from the drums owned by La
Carlota Planters Association for which he was admonished and
warned not to repeat the same.
ISSUE
WON there was valid, legal and just cause for the dismissal of
private respondent by petitioners
HELD
NO
Ratio The correct rule has always been that such previous
offenses may be so used as valid justification for dismissal from
work only if the infractions are related to the subsequent
offense upon which basis the termination of employment is
decreed. The previous infraction, in other words, may be used if
it has a bearing to the proximate offense warranting dismissal.
Reasoning
- Petitioners contend that sufficient factual and legal bases exist
to justify the dismissal of private respondent for misconduct. It
cites various infractions allegedly committed in the past by
private respondent; to wit:
a. Private respondent was caught twice stealing diesel fuel from
the drum of the petitioner's association;
b. He entered the transloading office on December 14, 1992
drunk, armed with bayonet knife, and harassed the personnel
therein, even unsuccessfully stabbing one Gerry Flores for two
(2) times; and
c. Private respondent failed to report for work since December
14, 1992 which is an obvious sign of guilt.
- The reliance by petitioners on the past offenses of private
respondent supposedly dictating his eventual dismissal is
unavailing. The complainant may have been at fault when he
figured in a vehicular accident causing damage to the company

Labor Law 1
truck; that fault, nevertheless, cannot be considered a just
cause for dismissal. Indeed, it has once been held that the
penalty of dismissal would be grossly disproportionate to the
offense of driving through reckless imprudence resulting in
damage to property. The claim of drunkenness on the part of
private respondent has not been substantiated; the allegation is
based solely on the uncorroborated statement made by one
Rene Baylon in his affidavit executed on 24 April 1993, months
after the accident had occurred in December of 1992.
Disposition the Court is constrained to dismiss, as it hereby so
DISMISSES, the instant petition for certiorari.

6. PROFESSIONAL TRAINING
RESIDENCY TRAINING
FELIX V BUENASEDA
[PAGE 55]

7. LOVE AND MORALS


IMMORALITY
SANTOS V NLRC (HAGONOY INSTITUTE ET AL)
287 SCRA 117
ROMERO; March 6, 1998
NATURE
Petition for certiorari
FACTS
- Mrs. Martin and Petitioner Santos were both teachers at the
Hagonoy Institute. Both were married to different people.
During the course of their employment, they fell in love, and
rumors about their relationship spread.
- Private respondent advised Mrs. Martin to take a leave of
absence, which she ignored. A week later, she was barred from
reporting for work and was not allowed to enter Hagonoys
premises, effectively dismissing her from employment
- Mrs. Martins case for illegal dismissal was successful because
the private respondent failed to accord her the necessary due
process in her dismissal.
- Meanwhile, HI set up a committee to investigate the veracity
of the rumors. After 2 weeks, the committee confirmed the illicit
relationship
- in view of this finding, petitioner was charged administratively
for immorality and was required to present his side of the
controversy. 5 months later, he was informed of his dismissal.
He thus filed a complaint for illegal dismissal.
- After a full blown trial was conducted, the Labor Arbiter
dismissed his complaint, but awarded him money as financial
assistance.
- petitioner filed an appeal with the NLRC which was dismissed
for lack of merit
ISSUE
WON the illicit relationship between the petitioner and Mrs.
Martin could be considered immoral as to constitute just cause
to terminate an employee under Article 282 of the Labor Code
HELD
YES
Reasoning

A2010

- 256 -

Disini

- Section 94 10 of the Manual of Regulations for Private Schools:


Causes of Terminating Employment. In addition to the just
cases enumerated in the Labor Code, the employment of school
personnels, including faculty, may be terminated for any of the
following causes:xxx xxx xxx E. Disgraceful or immoral conduct.
- To constitute immorality, the circumstances of each particular
case must be holistically considered and evaluated in light of
the prevailing norms of conduct and applicable laws. America
jurisprudence has defined immorality as a course of conduct
which offends the morals of the community and is a bad
example to the youth whose ideals a teacher is supposed to
foster and to elevate, the same including sexual misconduct.
Thus, in petitioner's case, the gravity and seriousness of the
charges against him stem from his being a married man and at
the same time a teacher.
- Having an extra-marital affair is an affront to the sanctity of
marriage, which is a basic institution of society. Even our Family
Code provides that husband and wife must live together,
observe mutual love, respect and fidelity. This is rooted in the
fact that both our Constitution and our laws cherish the validity
of marriage and unity of the family. Our laws, in implementing
this constitutional edict on marriage and the family underscore
their permanence, inviolability and solidarity.
- As a teacher, petitioner serves as an example to his pupils,
especially during their formative years and stands in loco
parentis to them. To stress their importance in our society,
teachers are given substitute and special parental authority
under our laws.
- Teachers must adhere to the exacting standards of morality
and decency. He must freely and willingly accept restrictions on
his conduct that might be viewed irksome by ordinary citizens.
The personal behavior of teachers, in and outside the
classroom, must be beyond reproach.
- Accordingly, teachers must abide by a standard of personal
conduct which not only proscribes the commission of immoral
acts, but also prohibits behavior creating a suspicion of
immorality because of the harmful impression it might have on
the students. - Likewise, they must observe a high standard of
integrity and honesty.
- From the foregoing, it seems obvious that when a teacher
engages in extra-marital relationship, especially when the
parties are both married, such behavior amounts to immorality,
justifying his termination from employment.
Disposition Petition DISMISSED

LOVE
CHUA-QUA V CLAVE
189 SCRA 117
REGALADO; August 30, 1990
NATURE
Petition for certiorari.
FACTS
- This would have been just another illegal dismissal case were
it not for the controversial and unique situation that the
marriage of herein petitioner, then a classroom teacher, to her
student who was fourteen (14) years her junior, was considered
by the school authorities as sufficient basis for terminating her
services.
- Private respondent Tay Tung High School, Inc. is an
educational institution in Bacolod City. Petitioner had been
employed therein as a teacher since 1963 and, in 1976 when
this dispute arose, was the class adviser in the sixth grade
where one Bobby Qua was enrolled. Since it was the policy of
the school to extend remedial instructions to its students,
Bobby Qua was imparted such instructions in school by
petitioner. In the course thereof, the couple fell in love and on
December 24, 1975, they got married in a civil ceremony
solemnized in lloilo City by Hon. Cornelio G. Lazaro, City Judge
of Iloilo.

Labor Law 1
- Petitioner was then thirty (30) years of age but Bobby Qua,
being sixteen (16) years old, consent and advice to the
marriage was given by his mother, Mrs. Concepcion Ong. Their
marriage was ratified in accordance with the rites of their
religion in a church wedding solemnized by Fr. Nick Melicor at
Bacolod City on January 10, 1976.
- On February 4, 1976, private respondent filed with the
subregional office of the Department of Labor at Bacolod City
an application for clearance to terminate the employment of
petitioner on the following ground: "For abusive and unethical
conduct unbecoming of a dignified school teacher and that her
continued employment is inimical to the best interest, and
would downgrade the high moral values, of the school."
- Petitioner was placed under suspension without pay on March
12, 1976.
- Executive Labor Arbiter rendered an "Award" in favor of
private respondent granting the clearance to terminate the
employment of petitioner.
- NLRC unanimously reversed the Labor Arbiter's decision and
ordered petitioner's reinstatement with backwages.
- Minister of Labor reversed the decision of theNLRC.
- Petitioner appealed the said decision to the Office of the
President of the Philippines. Presidential Executive
Assistant Jacobo C. Clave, rendered its decision reversing the
appealed decision.
- However, in a resolution dated December 6, 1978, public
respondent, acting on a motion for reconsideration of herein
private
respondent
and
despite
opposition
thereto,
reconsidered and modified the aforesaid decision, this time
giving due course to the application of Tay Tung High School,
Inc. to terminate the services of petitioner.
ISSUE
WON there is substantial evidence to prove that the antecedent
facts which culminated in the marriage between petitioner and
her student constitute immorality and or grave misconduct
HELD
NO
Ratio To constitute immorality, the circumstances of each
particular case must be holistically considered and evaluated in
the light of prevailing norms of conduct and the applicable law.
Reasoning
- Contrary to what petitioner had insisted on from the very start,
what is before us is a factual question, the resolution of which is
better left to the trier of facts.
- Considering that there was no formal hearing conducted, we
are constrained to review the factual conclusions arrived at by
public respondent, and to nullify his decision through the
extraordinary writ of certiorari if the same is tainted by absence
or excess of jurisdiction or grave abuse of discretion. The
findings of fact must be supported by substantial evidence;
otherwise, this Court is not bound thereby.
- We rule that public respondent acted with grave abuse of
discretion.
- As earlier stated, from the outset even the labor arbiter
conceded that there was no direct evidence to show that
immoral acts were committed.
- Nonetheless, indulging in a patently unfair conjecture, he
concluded that "it is however enough for a sane and credible
mind to imagine and conclude what transpired during those
times." In reversing his decision, the National Labor Relations
Commission observed that the assertions of immoral acts or
conducts are gratuitous and that there is no direct evidence to
support such claim, a finding which herein public respondent
himself shared.
- What is revealing, however, is that the reversal of his original
decision is inexplicably based on unsubstantiated surmises and
non sequiturs which he incorporated in his assailed resolution in
this wise:
". . . While admittedly, no one directly saw Evelyn Chua and
Bobby Qua doing immoral acts inside the classroom, it seems
obvious and this Office is convinced that such a happening
indeed transpired within the solitude of the classroom after

A2010

- 257 -

Disini

regular class hours. The marriage between Evelyn Chua and


Bobby Qua is the best proof which confirms the suspicion that
the two indulged in amorous relations in that place during
those times of the day..."
- With the finding that there is no substantial evidence of the
imputed immoral acts, it follows that the alleged violation of the
Code of Ethics governing school teachers would have no basis.
Private respondent utterly failed to show that petitioner took
advantage of her position to court her student. If the two
eventually fell in love, despite the disparity in their ages and
academic levels, this only lends substance to the truism that
the heart has reasons of its own which reason does not know.
But, definitely, yielding to this gentle and universal emotion is
not to be so casually equated with immorality. The deviation of
the circumstances of their marriage from the usual societal
pattern cannot be considered as a defiance of contemporary
social mores.
- It would seem quite obvious that the avowed policy of the
school in rearing and educating children is being unnecessarily
bannered to justify the dismissal of petitioner. This policy,
however, is not at odds with and should not be capitalized on to
defeat the security of tenure granted by the Constitution to
labor. In termination cases, the burden of proving just and valid
cause for dismissing an employee rests on the employer and his
failure to do so would result in a finding that the dismissal is
unjustified.
Disposition
Petition for certiorari granted. Decision of
respondent annulled and set aside.

DUNCAN ASSOCIATION V GLAXO-WELLCOME


[PAGE 43]

8. VIOLATION COMPANY RULES


APARENTE SR V NLRC (COCA-COLA BOTTLERS
PHIL)
331 SCRA 82
DE LEON JR; April 27, 2000
FACTS
- Rolando Aparante, Sr. was first employed by private
respondent Coca-Cola Bottlers Phils., Inc. (CCBPI), General
Santos City Plant as assistant mechanic in April 1970. He rose
through the ranks to eventually hold the position of advertising
foreman until his termination on May 12, 1988 for alleged
violation of company rules and regulations. His monthly salary
at the time of his termination was P5,600.
- On November 9, 1987, Aparante drove CCBPI's advertising
truck to install a panel sign. He sideswiped Marilyn Tejero, a
ten-year old girl. He brought Tejero to Heramil Clinic for first aid
treatment. As the girl suffered a 2 cm fracture on her skull
which was attributed to the protruding bolt on the truck's door,
she was subsequently transferred to the General Santos City
Doctor's Hospital where she underwent surgical operation. She
stayed in the hospital for about a month.
- Five days after the accident, he reported the incident to
CCBPI. At about the same time, he submitted himself to the
police authorities at Polomolok, South Cotabato for investigation
where it was discovered that he had no driver's license at the
time of the accident. In view thereof, FGU Insurance
Corporation, an insurer of CCBPI's vehicles, did not reimburse
the latter for the expenses it incurred in connection with
Tejero's hospitalization a total amount of P19,534.45.
- CCBPI conducted an investigation of the incident where
Aparente was given the opportunity to explain his side and to
defend himself.
On May 12, 1988, Aparente was dismissed for having violated
the company rules and regulations particularly Sec. 12 of Rule
005-858 for blatant disregard of established control procedures
resulting in company damages.

Labor Law 1
- The Labor Arbiter ordered his reinstatement without back
wages. The NLRC affirmed but reversed its ruling upon motion
of CCBPI. It declared the dismissal as one for just cause and
effected after observance of due process.
ISSUES
1. WON the NLRC erred in holding that CCBPI afforded petitioner
due process
2. WON the NLRC erred in upholding the dismissal despite its
initial finding that the CCBPI had implicitly tolerated petitioners
driving without a license
3. WON the infraction committed by petitioner warrants the
penalty of dismissal despite the fact that it was his first offense
during his 18 long years of satisfactory and unblemished
service
HELD
1. NO
Ratio The essence of due process does not necessarily mean or
require a hearing but simply a reasonable opportunity or a right
to be heard or as applied to administrative proceedings, an
opportunity to explain one's side. In labor cases, the filing of
position papers and supporting documents fulfill the
requirements of due process.
Reasoning
- Aparente was fully aware that he was being investigated for
his involvement in the vehicular accident that took place on
November 9, 1987. It was also known to him that as a result of
the accident, the victim suffered a 2 cm fracture on her skull
which led to the latter's surgical operation and confinement in
the hospital for which CCBPI incurred expenses amounting to
P19,534.45 which FGU Insurance Corporation refused to
reimburse upon finding that he was driving without a valid
driver's license. Thus, being aware of all these circumstances
and the imposable sanctions under CCBPI's Code of Disciplinary
Rules and Regulations, he should have taken it upon himself to
present evidence to lessen his culpability.
2. NO
Reasoning
- According to Aparente, he informed the company that he had
lost
his
license
five
months
before
the
accident.
Notwithstanding such fact, the company allowed him to
continue driving the vehicle assigned to him. Thus, he shifts the
blame to the company, claiming that it should have simply
ordered him to desist from driving the vehicle once it was
informed of the loss of his license. His contention is belied by
his very own admission in his position papers filed before the
Labor Arbiter and the NLRC that the company had in fact
prohibited him from driving immediately after he lost his
license, and had requested him to secure a new license.
However, through misrepresentations, he led CCBPI to believe
that he had procured another driver's license. Thus, he was
permitted to drive again.
3. YES
Ratio The law warrants the dismissal of an employee without
making any distinction between a first offender and a habitual
delinquent where the totality of the evidence was sufficient to
warrant his dismissal. In protecting the rights of the laborer, the
law authorizes neither oppression nor self-destruction of the
employer.
Reasoning
- Company policies and regulations, unless shown to be grossly
oppressive or contrary to law, are generally valid and binding
on the parties and must be complied with until finally revised or
amended, unilaterally or preferably through negotiation, by
competent authority. The Court has upheld a company's
management prerogatives so long as they are exercised in
good faith for the advancement of the employer's interest and
not for the purpose of defeating or circumventing the rights of
the employees under special laws or under valid agreements.
- First, Aparente's dismissal is justified by Company rules and
regulations. It is true that his violation of company rules is his
first offense. Nonetheless, the damage caused to private
respondent amounted to more than P5,000, thus, the penalty of

A2010

- 258 -

Disini

discharge is properly imposable as provided by CCBPI's Code of


Disciplinary Rules and Regulations.
- Second, Article 282, in order that an employer may dismiss an
employee on the ground of willful disobedience, there must be
concurrence of at least two requisites: The employee's assailed
conduct must have been willful or intentional, the willfulness
being characterized by a wrongful and perverse attitude; and
the order violated must have been reasonable, lawful, made
known to the employee and must pertain to the duties which he
had been engaged to discharge. We have found these
requisites to be present in the case at bar. The evidence clearly
reveals the willful act of Aparente in driving without a valid
driver's license, a fact that he even tried to conceal during the
investigation conducted by CCBPI. Such misconduct should not
be rewarded with re-employment and back wages, for to do so
would wreak havoc on the disciplinary rules that employees are
required to observe.
- In the instant case, we find the award to petitioner of
separation pay by way of financial assistance equivalent to 1/2
month's pay for every year of service equitable. Although
meriting termination of employment, petitioner's infraction is
not as reprehensible or unscrupulous as to warrant complete
disregard for the fact that this is his first offense in an
employment that has spanned 18 long years.
Disposition Decision of the NLRC is AFFIRMED.

9. CRIMINAL CASE
EFFECT OF ACQUITTAL
RAMOS V NLRC
298 SCRA 225
PUNO; October 21, 1998
NATURE
Petition for certiorari to annul NLRC decision
FACTS
- In 1978, Elizabeth Ramos was employed by United States
Embassy Filipino Employees Credit Cooperative (USECO)
- In 1993, the USECO Board created an Audit and Inventory
Committee to determine whether USECO has a sound financial
management and control mechanism.
- The committee found anomalies in USECOs lending
transactions. Petitioner and her co-employees, Luz Coronel and
Nanette Legaspi, were called to shed light on some items in the
Audit Committee Report, such as unrecorded loans, fabricated
ledger, falsification of documents, accommodations of payroll
checks, encashment of check/CPAs, resigned members,
unrecorded loan of resigned members and withdrawal of more
than the deposits.
- During the meeting, Beth admitted her serious offense in
regard to falsification of documents. When asked by the Board
to explain how recently resigned members and other resigned
employees in the past were able to secure loans, Beth replied
that she just wanted to help members without regard to
existing policies.
- In her written explanation, Beth said that the loans are
approved based on prerogatives of individuals in authority. She
said that, it is unfortunate that the USECU Staff had to resort
to creating dummy records. But since the loans are duly
acknowledged by the borrowers in other legitimate documents,
it is readily apparent that the records were made simply to
accommodate those borrowers beyond the authorized limits,
but never, never to defraud USECU.
- Ramos was preventively suspended for 30 days. Later,

Labor Law 1
petitioner was placed on forced leave with pay, pending the
completion of the investigation.
- USECO commissioned an external auditing firm to examine the
irregularities discovered in its lending practices. The auditor
confirmed the irregularities and also discovered shortages in
bank deposits.
- USECO dismissed the petitioner for loss of trust and
confidence. Petitioner countered with a complaint for illegal
dismissal, illegal suspension, underpayment of salary, moral
damages and attorneys fees.
- Labor Arbiter sustained the suspension and dismissal of
petitioner but ordered the payment of her unpaid salary.
ISSUES
1. WON there is just cause for petitioners suspension and
dismissal
2. WON the NLRC committed grave abuse of discretion in
granting
private
respondents
second
motion
for
reconsideration
HELD
1. YES
- Position of petitioner as Management Assistant requires a high
degree of trust and confidence.
- Loss of confidence is a valid ground for dismissal of an
employee. In the case at bar, USECO proved that its loss of
confidence on petitioner has a rational basis. The findings of the
labor arbiter on this factual issue are supported by the
evidence.
- Petitioner's explanation that the "loan practices" were made
for the benefit of the borrowing members and not to defraud
USECO cannot exonerate her.
Her unsound practices
endangered the financial condition of USECO because of the
possibility that the loans could not be collected at all.
- Petitioner was not denied due process before she was
suspended and later dismissed.
The records show that
petitioner was called by the USECO Board of Directors and
confronted with the findings of the Audit, and Inventory
Committee showing the irregularities she committed. She was
asked to explain in writing these irregularities.
Petitioner
submitted her written explanation. Thus, petitioner cannot
complain that she did not understand the charges against her.
She is educated and she immediately explained her side. Due
process simply demands an opportunity to be heard and this
opportunity was not denied her.
2. NO
- Section 14 of the Rules of the NLRC provides:
Section. 14. Motions
for Reconsideration.--Motions for
reconsideration of any order, resolution or decision of the
Commission shall not be entertained except when based on
palpable or patent errors, provided that the motion is under
oath and filed within ten (10) calendar days from receipt of
the order, resolution or decision, with proof of service that a
copy of the same has been furnished, within the
reglementary period, the adverse party, and provided further
that only one such motion from the same party shall be
entertained.
- The NLRC initially reversed the ruling of the labor arbiter on
the grounds that: (1) petitioner was denied procedural due
process and (2) the criminal case for estafa filed against her has
been dismissed by the Manila Prosecutor's Office for
insufficiency of evidence, particularly, for lack of proof that the
USECO was damaged by the acts attributed to petitioner.
- As discussed above, petitioner was not denied due process.
- Similarly, it is a well established rule that the dismissal of
the criminal case against an employee shall not
necessarily be a bar to his dismissal from employment
on the ground of loss of trust and confidence. The NLRC
corrected these patent errors when it granted private
respondent's second motion for reconsideration.
Disposition Petition dismissed for lack of merit.

CONVICTION

A2010

- 259 -

Disini

SAMPAGUITA GARMENTS CORP V NLRC (SANTOS)


233 SCRA 260
CRUZ; June 17, 1994
NATURE
Petition for review of a resolution of the NLRC
FACTS
- Theft was claimed to have been done by Santos, employee of
Sampaguita. It was alleged she attempted to bring out a piece
of cloth w/o permission.
- She was dismissed on this ground. She filed complaint for
illegal dismissal. Labor Arbiter ruled in favor of Sampaguita.
NLRC reversed and ordered reinstatement.
- Sampaguita also filed criminal action against Santos. She was
found guilty.
- Santos moved for execution of NLRC decision. Sampaguita
opposed and invoked her conviction in the criminal case.
ISSUE
WON subsequent conviction in criminal prosecution for an
offense will affect a previous administrative decision which
absolved the employee of the same offense
HELD
YES
- Once judgment has become final and executory, it can no
longer be disturbed except only for correction of clerical errors
or where supervening events render its execution impossible or
unjust.
- Here, the decision of NLRC ordering reinstatement had
become final and executory. Even so, we find that NLRC wasnt
correct in sustaining implementation
- The affirmance by RTC and CA of private respondent's
conviction is justification enough for NLRC to exercise this
authority and suspend execution of its decision.
Such
conviction, also upheld by this Court is a supervening cause
that rendered unjust and inequitable the decision mandating
the private respondent's reinstatement.
- Separation pay shall be allowed as a measure of social justice
only in those instances where the employee is validly dismissed
for causes other than serious misconduct or those reflecting on
his moral character. A contrary rule would, as the petitioner
correctly argues, have the effect of rewarding rather than
punishing the erring employee for his offense.
- The only award to which private respondent may be entitled is
for the amount as a penalty for effecting her dismissal without
complying with the procedural requirements.

DISMISSAL-CRIMINAL CASE
LACORTE V INCIONG (ESTRELLA, ASEAN
FABRICATORS INC)
166 SCRA 1
FERNAN; September 27, 1988
NATURE
Certiorari and Mandamus
FACTS
- Salvador Lacorte was hired as a warehouseman whose duties
were among others, to receive and store the raw and junk
materials used by respondent in its business.
- January 19, 1977: Lacorte offered to purchase some obsolete,
defective and non-usable junk materials from AFI, who agreed
and issued a cash invoice for the purchase of the scrap items.
- When Lacorte tried to bring out these items he was accosted
by AFI' s security guard and in the course of the investigation, it
was discovered that the items sought to be brought out by
complainant weighed more than what he actually purchased.

Labor Law 1
- Furthermore, it was found out that the items were not junk
since some parts were brand new and usable.
- As a consequence the respondent filed a case for qualified
theft against complainant before the Provincial Fiscal of
Bulacan.
- The criminal complaint was however, dismissed for
insufficiency of evidence.
- The application of AFI to terminate LACORTE was granted as
the latter was found by Labor Regional Director Estrella, to have
committed certain acts in breach of the trust and confidence of
his employer.
- On appeal, Deputy Minister of Labor Amado Gat Inciong
affirmed the aforementioned order. Hence, this present
recourse.
ISSUE
WON public respondents acted arbitrarily and/or with grave
abuse of discretion (considering that the criminal complaint was
dismissed) connection with the grant of the application for
clearance to terminate the employment of petitioner filed by AFI
HELD
YES
- The purpose of the proceedings before the fiscal is to
determine if there is sufficient evidence to warrant the
prosecution and conviction of the accused. In assessing the
evidence before him, the fiscal considers the basic rule that to
successfully convict the accused the evidence must be beyond
reasonable doubt and not merely substantial.
- On the other hand, to support findings and conclusion of
administrative bodies only substantial evidence is required.
- The evidence presented before the two bodies may not be
necessarily Identical.
- The appreciation of the facts and evidence presented is an
exercise of discretion on the part of administrative officials over
which one cannot impose his conclusion on the other.
- Sea-Land Service, Inc. v. NLRC: The conviction of an
employee in a criminal case is not indispensable to warrant his
dismissal, and the fact that a criminal complaint against the
employee has been dropped by the fiscal is not binding and
conclusive upon a labor tribunal.
- Also, the Court did not believe Lacortes claim that the real
reason behind his termination was his union activities.
- As regards Lacortes claim that there was no actual weighing
and examination of the boxes containing the scrap materials he
allegedly stole, the Court ruled that it was too late in the day for
Lacorte to raise these matters of facts in this petition and that
his evidence does not substantiate his claim.
- The Court considered the records of this case as a whole, and
was convinced that there is substantial basis for the Orders
issued by respondent labor officials.
Disposition Petition is dismissed for lack of merit.

GUILT OR INNOCENCE
CHUA V NLRC
218 SCRA 545
FELICIANO; February 8, 1993
NATURE
Petition for certiorari
FACTS
- The Union of Filipro Employees, of which petitioner Chua was a
member, declared a strike against the private respondent
company, Nestle Philippines, Inc. During the strike, several of
the striking employees threw stones at the trucks entering and
leaving the company premises. One truck. whose driver was
rendered unconscious by a stone hitting him on the head,
rammed a private vehicle and crashed into a beauty parlor
resulting in the death of three persons and extensive damage
to private property. Consequently, a criminal complaint for

A2010

Disini

- 260 -

multiple murder and frustrated murder was filed against


petitioner and several other employees who were believed to
be responsible for the stoning incident which resulted in the
deaths and property damage. The criminal complaint was
dismissed for insufficiency of evidence. The strike itself was,
however, declared illegal in two decisions of the National Labor
Relations Commission (NLRC) which were affirmed by the
Supreme Court.
- Subsequently, the union and its striking members offered to
return to work and were readmitted by the company except 69
union officers and 33 union members, including petitioner. The
union's counsel wrote to the private respondent requesting the
reinstatement of five employees, including petitioner. The
request, however, was denied. Petitioner received a notice of
dismissal from private respondent for having participated in the
illegal strike.
- Two days later, petitioner initiated a complaint for illegal
dismissal against private respondent company. The Labor
Arbiter rendered a decision finding that petitioner had been
validly dismissed. It was held that the evidence introduced by
private respondent, in the form of the testimony of Maniego,
Personnel Supervisor of its Cabuyao Plant, that he positively
saw and identified petitioner as one of the union members who
actively participated and manned the barricades during the
strike is "a concrete manifestation of an illegal act that is
frowned upon by law." Wishing to be reinstated also, petitioner
appealed the Labor Arbiter's decision to the NLRC which,
however, affirmed in toto the decision of the Labor Arbiter.
Hence, this petition.
ISSUE
WON the NLRC committed grave abuse of discretion in affirming
the decision of the Labor Arbiter
HELD
NO
- We find this contention to be without merit, Petitioner's
participation in the illegal strike and his commission of illegal
acts while the strike was in progress, i.e., he participated in the
barricade which barred people from entering and/or leaving the
employer's premises, had been sufficiently established by
substantial evidence, including the testimony of Mr. Maniego,
Personnel Supervisor at the Cabuyao Plant. Mr. Maniego
testified, among other things, that he was not able to report to
work because of the presence of the barricade. The law
prohibits any person engaged in picketing from obstructing free
ingress to or egress from the employer's premises for lawful
purposes.
- While the criminal complaint where petitioner was included as
one of the accused was dismissed for insufficiency of evidence,
the Court considers that the dismissal of the criminal complaint
did not preclude a finding by the competent administrative
authorities, that petitioner had indeed committed acts inimical
to the interest of his employer.
- Private respondent's guilt or innocence in the criminal case is
not determinative of the existence of a just or authorized cause
for his dismissal. This doctrine follows from the principle that
the quantum and weight of evidence necessary to sustain
conviction in criminal cases are quite different from the
quantum of evidence necessary for affirmance of a decision of
the Labor Arbiter and of the NLRC.
- Since petitioner's participation in the unlawful and violent
strike was amply shown by substantial evidence, the NLRC was
correct in holding that the dismissal of petitioner was valid
being based on lawful or authorized cause.
Disposition Petition dismissed.

10. MOONLIGHTING
AGABON V NLRC
[PAGE 35]

Labor Law 1
11. SUSPICION
EASTERN TELECOMMUNICATIONS PHILS INC V
DIAMSE
491 SCRA 239
YNAREZ-SANTIAGO; June 16, 2006
FACTS
- Maria Charina Damse is the Head of Building Services of ETPI.
She requested a cash advance of P150k for the renewal of
ETPIs business permits. The companys policy is cash advances
should be liquidated 15 days from the completion of the project
or activity, or else it will be deducted from the employees
salary, benefits or receivables.
- She was able to use a total of P97,151. The last payment was
made on Feb 26, 2001. She wasnt able to liquidate the cash
advance within 15 days.
- On July 13, 2001, ETPIs Finance Dept advised her to liquidate
the amount. She submitted a liquidation report on August 13,
2001. This report was refused by the Fin Dept for being late.
She was told that the entire amount would just be deducted
from her monthly salary starting Sept 2001. By Dec 2001, a
total of P23k had been deducted from her salary. She then
requested for reimbursement for P97,151. This was reviewed by
her supervisor and approved by HR and Fin Dept, and the amt
was credited to her ATM payroll acct.
- The Internal Audit Dept (IAD) apparently didnt know what was
going on. In Jan 2002, IAD found that her payroll acct had P86k.
They required her to withdraw P52,533 for the unliquidated amt
minus the deductions. She complied. The next day, they asked
her again for P74,462.82, which is the difference bet the P97k+
+ credited to her acct minus the P23k deductions. She complied
again. (I dont understand how the computations were made.)
- The next day, ETPI required Diamse to explain why she should
not be disciplined for unauthorized diversion or application of
company funds, and for acts of dishonesty, fraud, deceit and
willful breach of trust. She explained what that the liquidation
report wasnt accepted by the Fin Dept and she was instead
advised to do as she did. A month later, she was dismissed.
- LA ruled in her favor. NLRC reversed. CA reversed NLRC and
ordered separation pay etc instead of reinstatement because of
the strained relations bet the parties.
ISSUE
WON Diamse was illegally terminated
HELD
NO
- Employer wasnt able to prove that the employee was
terminated for valid and just cause.
LOSS OF TRUST AND CONFIDENCE v. SUSPICION
- To be a valid cause for dismissal, the loss of trust and
confidence must be based on a willful breach and founded on
clearly established facts. A breach is willful if it is done
intentionally, knowingly and purposely, without justifiable
excuse, as distinguished from an act done carelessly,
thoughtlessly, heedlessly or inadvertently. Loss of trust and
confidence must rest on substantial grounds and not on the
employer's arbitrariness, whims, caprices or suspicion,
otherwise, the employee would eternally remain at the mercy of
the employer.
- The SC held that the mere delay in the liquidation of the cash
advance cannot sustain a finding of loss of trust and confidence.
It was based on mere suspicion, without evidence to show that
Diamse misappropriated funds. In fact, all documents submitted
were found to be authentic. The evidence on record shows that
Diamse was able to liquidate the cash advance and that the
ensuing delay in its liquidation was attributable to ETPI.
- It cannot be presumed that Diamse misappropriated the funds
because to do so would do violence to her right to security of
tenure and the well-settled rule that the burden of proof is on
the employer to establish the ground for dismissal. Suspicion

A2010

- 261 -

Disini

has never been a valid ground for dismissal and the


employee's fate cannot, in justice, be hinged upon conjectures
and surmises.
- More suspicion with regard to the P86k in her ATM acct: The
company suspected that it came from the P97k erroneously
credited to her acct. They didnt bother to prove it. They
werent able to show any bank statements to that effect.
Disposition Petition denied. CA decision affirmed and modified
in that this case be REMANDED to the Labor Arbiter for the sole
purpose of computing Diamse's full backwages, etc.

14.06 TRANSFERS DISCHARGE


AND SUSPENSION
LANZADERAS V AMETHYST SECURITY AND
GENERAL SERVICES INC
404 SCRA 505
QUISUMBING; June 20, 2003
FACTS
- Amethyst has been supplying guard for Resin Industrial
Chemical Corp (RICC) and its sister company Phil. Iron
Construction and Marine Works (PICMW) since 1968. One
condition was that Amethyst must supply guards between 25
45 years old.
- In 1998, RICC/PICMW reminded Amethyst of this condition.
Amethyst in turn required of the guards assigned to
RICC/PICMW to submit copies of their birth certificates. Those
beyond the limit were told to report to the office for
reassignment.
- Amethyst was able to renegotiate with RICC/PICMW to the
effect that those beyond the age limit could be assigned as
firewatch guards in the same company. (SO they had a choice
of being assigned as firewatch guards in the same company or
be transferred to Cagayan de Oro.)
- The petitioners chose neither option and didnt report for
work. They filed illegal dismissal with the LA.
- LA held Amethyst and RICC/PICMW solidarily liable for P1.25M
to the petitioners. On appeal, NLRC reversed and set aside the
LAs decision on the ground that the relief of petitioners from
their posts was a legitimate exercise of prerogative on
RICC/PICMWs part.
- CA denied petitioners appeal on procedural grounds.
ISSUE
WON petitioners were illegally dismissed

HELD
NO
- In the first place, the petitioners knew of the age limit and
acted in bad faith when they werent honest about their ages.
- The condition imposed by respondent RICC/PICMW, as a
principal or client of the contractor Amethyst, regarding the age
requirement of the security guards to be designated in its
compound, is a valid contractual stipulation. It is an inherent
right of RICC/PICMW, as the principal or client, to specify the
qualifications of the guards who shall render service pursuant to
a service contract. It stands to reason that in a service contract,
the client may require from the service contractor that the
personnel assigned to the client should meet certain standards
and possess certain qualifications, conformably to the client's
needs.
- Security of tenure, although provided in the Constitution, does
not give an employee an absolute vested right in a position as
would deprive the company of its prerogative to change their
assignment or transfer them where they will be most useful.
When a transfer is not unreasonable, nor inconvenient, nor
prejudicial to an employee; and it does not involve a demotion

Labor Law 1
in rank or diminution of his pay, benefits, and other privileges,
the employee may not complain that it amounts to a
constructive dismissal.
- Case law recognizes the employer's right to transfer or assign
employees from one area of operation to another, or one office
to another or in pursuit of its legitimate business interest,
provided there is no demotion in rank or diminution of salary,
benefits and other privileges and not motivated by
discrimination or made in bad faith, or effected as a form of
punishment or demotion without sufficient cause. This matter is
a prerogative inherent in the employer's right to effectively
control and manage the enterprise.
- The petitioners were given an option to stay at RICC/PICMW as
firewatch guards or to be transferred to CDO as security guards.
The petitioners didnt report to the office to receive new
deployment instructions. They have no excuse not to heed
managements exercise of management prerogative.
Disposition Petition denied. CA affirmed.
Note The SC also denied on procedural grounds but went into
the issues to settle the matter completely.

WESTIN PHIL PLAZA HOTEL V NLRC (RODRIGUEZ)


306 SCRA 631
QUISUMBING; May 3, 1999
NATURE
Petition to review a decision of the NLRC
FACTS
- Private respondent Len Rodriguez was continuously employed
by petitioner in various capacities (pest controller, room
attendant, bellman, and doorman) from July 1, 1977 until his
dismissal on February 16, 1993.
- On December 28, 1992, private respondent received a
memorandum from the management transferring him from
doorman to linen room attendant. The position of doorman is
categorized as guest-contact position while linen room
attendant is a non-guest contact position.
- The transfer was allegedly taken because of the negative
feedback on the manner of providing service to hotel guests by
private respondent.
- Instead of accepting his new assignment, private respondent
went on vacation leave
- The President of the National Union of Workers in Hotels,
Restaurants and Allied Industries (NUWHRAIN) appealed to
management concerning private respondent's transfer, but the
director for human resources development, clarified that private
respondent's transfer is merely a lateral movement. She
explained that management believed that private respondent
was no longer suited to be in a guest-contact position, but there
was no demotion in rank or pay.
- When private respondent reported back to work, he still did
not assume his post at the linen room.
- On February 11, 1993, private respondent was served with a
memorandum asking him to explain in writing why no
disciplinary action should be taken against him for
insubordination. The memorandum noted that while private
respondent regularly came to the hotel everyday, he just stayed
at the union office.
- In his reply private respondent merely questioned the validity
of his transfer without giving the required explanation.
- On February 16, 1993, petitioner terminated private
respondent's employment on the ground of insubordination.
- Private respondent filed with the Department of Labor and
Employment a complaint for illegal dismissal against petitioner.
- The labor arbiter declared that the dismissal was legal.
Accordingly, the complaint was dismissed for lack of merit.
- On appeal, public respondent reversed the judgment of the
labor arbiter. It held that there was no just cause in dismissing
private respondent.
- Its motion for reconsideration having been denied, petitioner
filed this instant petition.
ISSUES

A2010

- 262 -

Disini

1. WON private respondent was guilty of insubordination, thus


giving petitioner just and valid cause for dismissal
2. WON the order of transfer was legal
HELD
1. YES
- Under Article 282 (a) of the Labor Code, as amended, an
employer may terminate an employment for serious misconduct
or willful disobedience by the employee of the lawful orders of
his employer or representative in connection with his work. But
disobedience to be a just cause for dismissal envisages the
concurrence of at least two (2) requisites: (a) the employee's
assailed conduct must have been willful or intentional, the
willfulness being characterized by a wrongful and perverse
attitude; and, (b) the order violated must have been
reasonable, lawful, made known to the employee and must
pertain to the duties which he has been engaged to discharge.
- In the present case, the willfulness of private respondent's
insubordination was shown by his continued refusal to report to
his new work assignment: 1. Upon receipt of the order of
transfer, private respondent simply took an extended vacation
leave; 2. When he reported back to work, he did not discharge
his duties as linen room attendantwhile he came to the hotel
everyday, he just went to the union office; 3. when he was
asked to explain why no disciplinary action should be taken
against him, private respondent merely questioned the transfer
order without submitting the required explanation.
2. YES
- It must be emphasized that this Court has recognized and
upheld the prerogative of management to transfer an employee
from one office to another within the business establishment,
provided that there is no demotion in rank or a diminution of his
salary, benefits and other privileges.
- This is a privilege inherent in the employer's right to control
and manage its enterprise effectively.
- Besides, it is the employer's prerogative, based on its
assessment and perception of its employee's qualifications,
aptitudes and competence, to move him around in the various
areas of its business operations in order to ascertain where the
employee will function with utmost efficiency and maximum
productivity or benefit to the company.
- An employee's right to security of tenure does not give him
such a vested right in his position as would deprive the
company of its prerogative to change his assignment or transfer
him where he will be most useful.
- Petitioner is justified in reassigning private respondent to the
linen room. Petitioner's right to transfer is expressly recognized
in the collective bargaining agreement between the hotel
management and the employees union as well as in the hotel
employees handbook. The transfer order was issued in the
exercise of petitioner's management prerogative in view of the
several negative reports vis-a-vis the performance of private
respondent as doorman. It was a lateral movement as the
positions of doorman and linen room attendant are equivalent
in rank and compensation. It was a reasonable relocation from a
guest contact area to a non-guest contact area.
Disposition Petition granted. NLRC decision reversed.

CASTILLO V NLRC (PCIB)


308 SCRA 326
GONZAGA-REYES; June 1999
NATURE
Petition for certiorari seeking to annul the NLRC Decision
FACTS
- Castillo was an employee of Philippine Commercial &
International Bank (PCIB) as Foreign Remittance Clerk. A
Jordanian national, went to PCIBs Ermita branch to claim a
foreign remittance in the amount of US$2T. He paid P450 as
commission charges as computed by petitioner. Upon recomputation, the correct amount of the charges amounted to
only P248.75.

Labor Law 1
- Because of this incident, Castillo received a Memorandum
regarding her REASSIGNMENT. In line with the Banks policy on
flexibility employee development and internal control, effective
immediately, you are hereby reassigned temporarily as
Remittance Clerk-Inquiry.
- She then filed with the NCR Arbitration Branch a complaintaffidavit for illegal dismissal asking for her reinstatement as
Foreign Remittance Clerk plus moral and exemplary damages.
- She received another memorandum: Relative to your
reassignment as Remittance Clerk-Inquiry, for internal control
purposes, you are hereby instructed that your specific duties
and responsibilities will be confined to handling of inquiring by
phone, by walk-in clients over the counter and to assist the FX
Supervisor-Inquiry & Investigation in verifying inquiries of
correspondent banks, agencies, other banks and branches.
- Castillo claimed that there was no legal basis for her transfer
and demotion order. Also, PCIB immediately appointed another
employee in her place and refused to allow petitioner to
perform her usual functions as she became a mere fixture in
the office premises to her gross humiliation. She was allegedly
barred from the office premises and was thereby constructively
dismissed without any legal ground and without due process.
- Labor Arbiter ruled that Castillo was constructively
dismissed, thus she was entitled to reinstatement with full
backwages without loss of seniority rights, privileges and other
rights granted by law.
- NLRC reversed LA: there was no demotion because the
position to which she was being reassigned belongs to the same
job level as her former position and both positions have the
same rate of compensation.
ISSUE
WON Castillo was constructively and illegally dismissed
HELD
NO
Ratio The Court, as a rule, will not interfere with an employers
prerogative to regulate all aspects of employment which
includes among others, work assignment, working methods,
and place and manner of work. It is the prerogative of the
employer to transfer and reassign employees for valid reasons
and according to the requirement of its business, provided that
the transfer is not unreasonable, inconvenient, or prejudicial to
the employee, and that there is no demotion in rank or a
diminution of his salary, benefits and other privileges. An
employees right to security of tenure does not give him such a
vested right in his position as would deprive the company of its
prerogative to change his assignment or transfer him where he
will be most useful.
- Constructive dismissal: The employer has the burden of
proving that the transfer and demotion of an employee are for
valid and legitimate grounds. Where the employer fails to
overcome this burden of proof, the employees demotion shall
no doubt be tantamount to unlawful constructive dismissal.
Reasoning
- PCIB was acting within its management prerogative to protect
its interest and that of its clients. NLRC upheld PCIBs
contention that the remittance clerk payment order/collection
item is given the same weight in terms of duties and
responsibilities as that of a remittance clerk inquiry. These
positions are of co-equal footing, co-important and of the same
level of authority and that the transfer did not entail any
reduction of wages and other benefits. This is because both
positions are in fact Remittance Clerks, which, in PCIBs
classification system, are both slotted at level S-III.
- It is not true that Castillo has become a mere fixture in the
office premises without any function and was given no
responsibilities. As a matter of act, had she accepted her new
position, she would have assumed a bigger responsibility, a big
departure from her former position where she merely did
routine processing work.
Disposition Petition dismissed.

A2010

- 263 -

Disini

OSS SECURITY & ALLIED SERVICES INC V NLRC


(LEGASPI)
325 SCRA 157
DE LEON JR; February 9, 2000
NATURE
Petition for certiorarii
FACTS
- Private respondent Eden Legaspi worked as a security guard
of OSS Security Agency from June 16, 1986. On January 17,
1986 petitioner Miguel and Victoria Vasquez acquired the
assets and properties of OSS and absorbed some of its
personnel, including Legaspi, who was assigned to render
security services to the different clients of petitioner. She was
last assigned at the Vicente Madrigal Condominium II located in
Ayala Avenue, Makati. In a memorandum, to petitioner, the
Building Administrator of VM Condominium II, complained of the
laxity of the guards in enforcing security measures and
requested that petitioner reorganize the men and women
assigned to the building to instill more discipline and proper
decorum by changing, if need be, some of the personnel,
replacing, if possible, on a temporary basis, the women
complement, to find out if it would improve the service.
- In compliance therewith, petitioner issued Duty Detail Order,
relieving Legaspi and another lady security guard of their
assignment at VM, for reassignment to other units or
detachments where vacancy exists. Thereafter, petitioner
detailed Legaspi to the Minami International Corporation in
Taytay, Rizal for 1 month to replace lady security guard who
was on leave. However, Legaspi did not report for duty at her
new assignment.
- Legaspi filed her complaint for under payment and
constructive dismissal. The Labor Arbiter upheld Legaspis
position and ordered OSS to reinstate complainant to her
former position and pay the latter backwages for 18 months.
Upon appeal, the NLRC affirmed said decision. Hence, this
petition.
ISSUE
WON the transfer of Legaspi was illegal and tantamount to
unjust dismissal
HELD
NO
- Service-oriented enterprises, such as petitioner's business of
providing security services, generally adhere to the business
adage that "the customer or client is always right". To satisfy
the interests, conform to the needs, and cater to the whims and
wishes of its clients, along with its zeal to gain substantial
returns on its investments, employers adopt means designed
towards these ends. These are called management prerogatives
in which the free will of management to conduct its own affairs
to achieve its purpose, takes from. Accordingly, an employer
can regulate, generally without restraint, according to its own
discretion and judgment, every aspect of business.
- In the employment of personnel, the employer can prescribe
the hiring, work assignments, working methods, time, place and
manner of work, tools to be used, processes to be followed,
supervision of workers, working regulations, transfer of
employees, work supervision, lay-off of workers and the
discipline, dismissal and recall of work, subject only to
limitations imposed by laws.
- Thus, the transfer of an employee ordinarily lies within the
ambit of management prerogatives. However, a transfer
amounts to constructive dismissal when the transfer is
unreasonable, inconvenient, or prejudicial to the employee, and
it involves a demotion in rank or diminution of salaries, benefits
and other privileges. In the case at bench, nowhere in the
record does it show that that the transfer of Legaspi was
anything but done in good faith, without grave abuse of
discretion, and in the best interest of the business enterprise.

Labor Law 1
- No malice should be imputed from the fact that Legaspi was
relieved of her assignment and, a day later, assigned a new
post. We must bear in mind that, unlike other contracts of
service, the availability of assignment for security guards is
primarily at heart subservient to the contracts entered into by
the security agency with its client-third parties. As such, being
sidelined temporarily is a standard stipulation in employment
contracts. When a security guard is placed "off detail" or on
"floating" status, in security agency parlance, it means "waiting
to be posted." Legaspi has not even been "off detail" for a week
when she filed her complaint.
- Evidence is wanting to support the Labor Arbiter's conclusion
that petitioner discriminated against private respondent when it
ordered her relief and transfer of assignment. Petitioner proved
that such transfer was effected in good faith to comply with the
reasonable request of its client, Madrigal Condominium
Corporation Incorporated (MCCI), for a more disciplined service
of the security guards on detail. The renewal of the contract of
petitioner with MCCI hinged on the action taken by the former
on the latter's request. Most contracts for security services
stipulate that the client may request the replacement of the
guards assigned to it. Besides, a relief and transfer order in
itself does not sever employment relationship between a
security guard and her agency.29 Neither was the transfer for
any ulterior design, such as to rid itself of an undesirable worker
or to penalize an employee for his union activities and thereby
defeat his right to self-organization.
- It appears that Legaspi declined the post assigned to her
inasmuch as she considered it "a booby trap of crippling and
dislocating her from her employment". She lived in V. Mapa,
Sta. Mesa, Manila, and her new assigned post is in Taytay, Rizal,
as against her previous post at VM Condominium II in Makati.
Her new assigned post would entail changes in her routine,
something that she was not agreeable with. But the mere fact
that it would be inconvenient for her, as she has been assigned
to VM Condominium II for a number of years, does not by itself
make her transfer illegal. Even Legaspi admitted that she was
assigned to render security service to the different clients of
petitioner. An employee has a right to security of tenure, but
this does not give her such a vested right in her position as
would deprive petitioner of its prerogative to change her
assignment or transfer her where her service, as security guard,
will be most beneficial to the client. Thus, there was no basis to
order reinstatement and back wages inasmuch as she was not
constructively dismissed. Neither is private respondent entitled
to the award of money claims for underpayment, absent
evidence to substantiate the same.

MENDOZA V RURAL BANK OF LUCBAN


433 SCRA 756
PANGANIBAN; July 7, 2004
NATURE
Petition for Review under Rule 45, ROC
FACTS
- April 1999, the Board of Directors of respondent bank (BANK)
issued a Board Resolution announcing the reshuffling of
assignments, without changes in compensation and other
benefits, of several officers and employees of the bank (in line
with the banks policy to familiarize bank employees with the
various phases of bank operations and to further strengthen the
existing internal control system). One of the employees
assigned to a new position is petitioner Elmer Mendoza, who
was transferred from his post as an appraiser to a Clerk for
Meralco collection.
- In May, Mendoza expressed his resentment on the reshuffling,
saying that it was a demotion and that he heard intrigues that
his demotion to a Clerk-Meralco collection was due to the
malicious machination of a certain public official who is the
friend of the Board chairman and with whom the relatives of
Mendoza had filed a falsification case against. He also said that
he had been working for 6 years in good standing in the bank

A2010

- 264 -

Disini

and that the reshuffling is a blatant harassment on the part of


the Board, an act which implicitly forces him to resign, and
which constitutes an unfair labor practice. He requested to have
his position as an appraiser retained. However, Mr. Daya (Board
Chairman) explained in a reply that the reshuffling is not a
demotion since his compensation as an appraiser is retained
and no reductions were made. Also, Mr. Daya explained the
objectives of the reshuffling, particularly the maintenance of an
effective internal control system recommended by Bangko
Sentral ng Pilipinas, and that it was the banks management
prerogative to do so. Also, Mendoza could retain position upon a
formal request to the board.
- In June, petitioner submitted 2 applications for LOA, and during
his 2nd LOA he filed a complaint before the Arbitration Branch of
NLRC for illegal dismissal, underpayment, separation pay and
damages.
- LA: in favor of Mendoza (entitled to reinstatement + full
backwages/ separation pay if reinstatement not possiblemoral
+ exemplary + attys fees)
- NLRC: in favor of Bank: no bad faith or malice on banks part;
petitioner only feel inconvenienced; petitioner not only
employee reshuffled; no clear, competent, convincing evidence
that he holds a vested right to the position of Appraiser.
- CA: affirm (Mendozas claims self-serving, no diminution, could
retain title upon formal request, no bad faith/malice; no
constructive dismissal he was the one who separated himself
from the banks employ)
ISSUES
1. WON Mendoza was constructively dismissed
2. WON the transfer of employees/ reshuffling was a valid
exercise of the banks management prerogatives
(Secondary Issues)
3. WON Serrano v. NLRC is applicable
4. WON NLRC and CA proceedings null
HELD
1. NO
- Findings of NLRC and CA were supported by substantial
evidence
Ratio
Constructive dismissal is defined as an involuntary
resignation resorted to when continued employment is
rendered impossible, unreasonable or unlikely; when there is a
demotion in rank or a diminution of pay; or when a clear
discrimination, insensibility or disdain by an employer becomes
unbearable to the employee.
Reasoning
- Petitioner presented no evidence to support his claims. (More
on the second issue)
2. YES
Ratio
In the pursuit of its legitimate business interest,
management has the prerogative to transfer or assign
employees from one office or area of operation to another -provided there is no demotion in rank or diminution of salary,
benefits, and other privileges; and the action is not motivated
by discrimination, made in bad faith, or effected as a form of
punishment or demotion without sufficient cause. This privilege
is inherent in the right of employers to control and manage
their enterprise effectively. The right of employees to security
of tenure does not give them vested rights to their positions to
the extent of depriving management of its prerogative to
change their assignments or to transfer them.
-Managerial prerogatives, however, are subject to limitations
provided by law, collective bargaining agreements, and general
principles of fair play and justice.
-TEST of validity of transfer of employees (Blue Dairy
Corporation v. NLRC): "The managerial prerogative to
transfer personnel must be exercised without grave abuse of
discretion, bearing in mind the basic elements of justice and fair
play. Having the right should not be confused with the manner
in which that right is exercised. Thus, it cannot be used as a
subterfuge by the employer to rid himself of an undesirable
worker. In particular, the employer must be able to show that
the transfer is not unreasonable, inconvenient or prejudicial to

Labor Law 1
the employee; nor does it involve a demotion in rank or a
diminution of his salaries, privileges and other benefits. Should
the employer fail to overcome this burden of proof, the
employees transfer shall be tantamount to constructive
dismissal, which has been defined as a quitting because
continued employment is rendered impossible, unreasonable or
unlikely; as an offer involving a demotion in rank and diminution
in pay. Likewise, constructive dismissal exists when an act of
clear discrimination, insensibility or disdain by an employer has
become so unbearable to the employee leaving him with no
option but to forego with his continued employment."
- Employees may be transferred based on their qualifications,
aptitudes and competencies to positions in which they can
function with maximum benefit to the company.
Reasoning
- Mendozas transfer complied with the test. Transfer made in
pursuit of valid objectives (see above, 1st paragraph inside
parenthesis); Mendoza was not singled out; no diminution of
salary, privileges, and other benefits.
3. NO
- No constructive dismissal, not entitled to monetary benefits as
awarded in the Serrano case.
4. NO
- Petitioners arguments regarding the Banks appeal before the
NLRC filed beyond the reglementary period was not raised in
CA, thus cannot be entertained if raised for the 1st time.
Disposition Petition is DENIED, and the June 14, 2002 Decision
and the September 25, 2002 Resolution of the Court of Appeals
are AFFIRMED.

12. RESIGNATION AND EFFECTIVITY


EMCO PLYWOOD CORP V ABELGAS
[PAGE 14]
SHIE JIE CORP/SEASTER EX-IM CORP V NATIONAL
FEDERATION OF LABOR
463 SCRA 569
SANDOVAL-GUTIERREZ; July 15, 2005
FACTS
- Respondents were employed by petitioner as fish processors.
Respondents staged a walk-out and abandoned their work,
bringing operations to a standstill. They were suspended for a
week. Petitioner claims that instead of coming to work, some of
the respondents submitted resignation letters and quitclaims.
Petitioner then sent the rest a notice terminating their services
for abandonment of work.
- the Labor Arbiter found petitioners guilty of unfair labor
practice for illegally dismissing respondents and awarding the
latter claims. On appeal, the NLRC reversed the decision. The
CA later reversed the decision again based on Article 277 which
requires that the employer prove that the termination was for a
valid or just cause. Hence this petition.
ISSUE
WON respondents made valid resignations and were thus not
illegally dismissed
HELD
NO
- Voluntary resignation is defined as the act of an employee,
who finds himself in a situation in which he believes that
personal reasons cannot be sacrificed in favor of the exigency
of the service; thus, he has no other choice but to disassociate
himself from his employment. Acceptance of a resignation
tendered by an employee is necessary to make the resignation
effective, which was not shown in the instant case.
- To constitute a resignation, it must be unconditional and with
the intent to operate as such. There must be an intention to

A2010

- 265 -

Disini

relinquish a portion of the term of office accompanied by an act


of relinquishment.
- It is illogical that respondents would file complaints of illegal
dismissal 17 days after filing their resignation letters. Such acts
negate any intention on their part to relinquish their jobs. It was
held in Molave Tours Corp. vs NLRC, By vigorously pursuing the
litigation of his action against petitioner, private respondent
clearly manifested that he has no intention of relinquishing his
employment, which act is wholly incompatible to petitioners
assertion that he voluntarily resigned.
Disposition WHEREFORE the petition is DENIED

13. ABOLITION OF POSITION


BENGUET ELECTRIC COOPERATIVE V FIANZA
425 SCRA 41
YNARES-SANTIAGO; March 9, 2004
NATURE
Review on certiorari
FACTS
- Josephine Fianza had been employed with petitioner Benguet
Electric Cooperative (BENECO) as Property Custodian under the
Office of the General Manager.
- BENECOs General Manager, Versoza, issued Office Order No.
42 addressed to Fianza communicating that she is temporarily
detailed to the Finance Department to assume the duties of a
Bill Distributor without any change in salary rate. This is line
with their efforts to reduce the cost of operation.
- Fianza acknowledged receipt of the letter under protest. She
avers that it amounts to a demotion because there are
significant differences in the educational qualifications, work
experience, skills and job description and the working
conditions of a Bill Distributor are totally different and more
strenuous and expose her to unfavourable and dangerous
circumstances, and therefore not similarly situated as that of
Property Custodian.
- In response, Versoza issued a Memorandum informing her
that the position of Property Custodian may eventually phased
out upon approval of the already proposed Table of
Organization as part of business decision.
- Still, Fianza refused to heed the order of the General Manager
and continued to work as Property Custodian despite successive
issuance of Memorandum until the management no longer
authorized her to perform the duties and functions of a Property
Custodian.
ISSUES
1. WON Fianzas transfer from Property Custodian to Bill
Distributor is valid
2. WON the position of Property Custodian is abolished and
WON the abolition is valid
HELD
1. YES
Ratio
The management has a wide latitude to regulate,
according to his own discretion and judgment, all aspects of
employment, including the freedom to transfer and reassign
employees according to the requirements of its business.
However, the transfer of an employee may constitute
constructive dismissal when it amounts to an involuntary
resignation resorted to when continued employment is
rendered impossible, unreasonable or unlikely; when there is a
demotion in rank and/or a diminution in pay, or when a clear
discrimination, insensibility or disdain by an employer becomes
unbearable to the employee.
2. YES
Ratio The abolition of a position deemed no longer necessary
is a management prerogative, and this Court, absent any
findings of malice and arbitrariness on the part of management,

Labor Law 1
will not efface such privilege if only to protect the person
holding that office.
Reasoning
- There was no showing that the position of Property Custodian
was abolished in order to single out Fianza, or that malice and
ill-will attended the phasing out of the position.

14. DISHONESTY
NAGUIT V NLRC (MANILA ELECTRIC)
408 SCRA 617
CARPIO-MORALES; August 12, 2003
NATURE
Petition for certiorari seeking to annul and set aside the
decision and resolution of the NLRC.
FACTS
- Petitioner Aniceto W. Naguit, Jr., an employee of respondent
Manila Electric Company (MERALCO) was dismissed after 32
years of service. At the time of his dismissal, he was
Administrative Officer of MERALCO.
- On June 5, 1987, petitioner informed his Supervisor-Branch
head Sofronio Ortega, Jr. that he would render overtime work on
June 6, 1987, a Saturday, and that after concluding his field
work on that day, he would proceed to Pagbilao, Quezon to
accompany his wife who was a principal sponsor to a kins
wedding.
- On June 6 Naguit proceeded to his field assignment to conduct
supervisory survey on re-sequence of customers account
numbers, and to supervise MERALCOs Operation FC
(apprehension of customers with illegally connected service). At
12:00 noon, he, along with his co-employee Accounts
Representative Fidel Cabuhat who drove his (petitioners) jeep,
proceeded to Pagbilao, Quezon.
- On June 8, the timekeeper prepared an Overtime Notice and
the corresponding Timesheet[9] wherein it was reflected that
petitioner worked from 8:00 a.m. to 5:00 p.m. on June 6 and 7.
Petitioner corrected the documents by erasing the entries made
for June 7. The documents were approved by petitioners
supervisor Ortega. Petitioner was thereafter paid for overtime
work on June 6.
- Documents including petty cash voucher covering Cabuhats
alleged overtime work on June 6 were also prepared on account
of which petitioner, as custodian of petty cash, released to
Cabuhat the amount of P192.00 representing meal allowance
and rental for a jeep.
- More than two years later, petitioner received from the Legal
and Investigation Staffs Head of MERALCO a letter stating that
the Special Presidential Committee (SPC) is in receipt of
information that he caused reimbursement of transportation
expenses for the work of Cabuhat not actually rendered. It
requested that he report to the Ortigas office Feb 27, 1990 for
the administrative proceedings.
- During the administrative proceedings, Naguit wiaved his right
to counsel and gave sworn statements denying the charges.
- Evidence against petitioner consisted primarily of the sworn
statements of Cabuhat who was charged along with petitioner
with falsification of time card; Olivia Borda, billings clerk; and
five customers of MERALCO. The statements tried to establish
that, petitioner induced Cabuhat to prepare a petty cash
voucher covering expenses for meal and rental of a jeep for the
June 6 alleged conduct by the latter of field verification of Bill
Omissions; that on petitioners invitation, Cabuhat also
repaired to Pagbilao, Quezon on June 6; and that petitioner
gave the petty cash payable to Cabuhat making it appear that
some collections for bill omissions were received from
customers on June 6 when in fact no such collections were ever
received from the customers in whose name official receipts
were issued.
- SPC found Naguit and Cabuhat guilty of falsification of time
cards under Sec. 7, par. 7 of the Company Code on Employee

A2010

- 266 -

Disini

Discipline. Additionally, petitioner was found guilty under Sec.


6, par. 24 of the Code for encouraging Cabuhat to commit an
act constituting a violation of the Code.
- MERALCO thus informed petitioner that he was, for falsification
of time card and encouraging and inducing another employee
to perform an act constituting a violation of the Company Code
on Employee Discipline, dismissed from the service with
forfeiture of all rights and privileges.
- Naguit filed a complaint with the NLRC against MERALCO for
illegal dismissal, he praying for reinstatement, backwages,
damages, attorneys fees and other awards he is entitled to.
Labor Arbiter found for Naguit.
- Meralco appealed which reversed the Labor Arbiters decision.
Petitioners claims
> the factual findings of the Labor Arbiter clearly show that he,
as an Administrative Officer, is covered by respondent
MERALCOs policy pertaining to field personnel, particularly
when he is designated to perform field assignments. As such,
he did not bother to correct the Overtime Notice which
indicated that he worked from 8 a.m. to 5 p.m., albeit he
actually worked until 12 noon, the company policy being that
even if an employee who had a field assignment did not
actually render 8 hours of work, he is deemed to have worked
for such duration provided he had completed the assigned task
as he claims he did.
ISSUES
1. WON Naguit is guilty of falsification
2. WON NLRC committed grave abuse of discretion when they
gave full credence to Cabuhats affidavits that he was induced
to claim overtime pat despite Cabuhats failure to affirm such in
the arbitral proceedings
3. WON there was valid ground for dismissal
HELD
1. NO
- The petitioner was in good faith when he did not correct the
entry in the Notice of Overtime and Timesheet reflecting that he
worked up to 5pm on June 6.
- Petitioner advised his superior Ortega about his rendering
overtime work the following day, June 6, 1987, after which he
would head for Pagbilao after concluding his work. If petitioner
had intended to do overtime work up to 5:00 p.m., there would
have been no need for him to advise Ortega that he would
thereafter go to Pagbilao. Since Ortega never refuted
petitioners claim about his advising him of his proceeding to
Pagbilao and in fact the grant and release of petitioners
overtime pay was approved by Ortega, who had the discretion
to judge the number of hours that can be foregone in light of
his (Ortegas) explanation that office personnel on field
assignment forego the convenience of the office, they [being]
exposed to the heat of the sun and the like, this Court would
not, as the Labor Arbiter did not, attribute malice to petitioner.
Thus, the Labor Arbiter held that Ortega opined that half day
would not be allowed. But, the fact remains that such discretion
is exercised, the limit of which was not shown to have been
disseminated to the employees, the qualifying factor being
whether the job was satisfactory or not. If on the contrary, there
was indeed no such practice or, that complainant, being an
office personnel, is removed from coverage thereof and
governed strictly by the time-rule such that he would have been
off at the actual completion of the assigned task, he would not
have bothered to inform his branch head - in effect a request
for permission of his planned trip to Pagbilao, Quezon
thereafter. That would have been meaningless gesture on the
part of the complainant.
- With the incentive scheme or tolerance of Naguit, there is no
resulting prejudice to Meralco so to speak of nor intention on
the part of complainant to cause it. What was done was
consistent with management policy on covering the overtime
work in the branch.
2. YES
- In labor cases, where the adverse party is deprived of the
opportunity to cross-examine affiants, affidavits are considered

Labor Law 1
hearsay unless the affiants are placed on the witness stand to
testify thereon. Cabuhats affidavits are inadmissible as
evidence.
3. YES
- Naguit, despite his knowledge that Cabuhat did not hire any
jeep nor conduct field verification on June 6, released the petty
cash representing Cabuhats meal allowance and rental fee for
a jeep. As custodian of the petty cash fund, he had the duty to
ascertain that the circumstances which brought about any claim
therefrom were in order.
He cannot now shirk from this
responsibility by indirectly pinning the blame on the approving
officer and asserting that the transgression was the result of
mere inadvertence, given his admission that he very well knew
that Cabuhat did not conduct any field work on June 6, 1987, he
(Cabuhat) having merely driven for him to Pagbilao.
- Petitioner thus committed dishonesty and breached
MERALCOs trust, which dishonesty calls for reprimand to
dismissal under MERALCOs rules.
- Dismissal is, however, too severe as a penalty in petitioners
case, given his 32 years of service during which he had no
derogatory record.
At the time petitioner was dismissed, he was still below the
retirement age of employees of MERALCO at 60. However, he is
now about 65. Imposing a penalty less harsh than dismissal and
ordering his reinstatement are thus functus oficio, the Labor
Arbiters order for his reinstatement not having been executed.
Disposition Decision and Resolution of the NLRC are hereby
SET ASIDE. Respondent MERALCO is, in light of the foregoing
discussions, hereby ORDERED to pay petitioner Aniceto W.
Naguit, Jr. his retirement benefits to be computed from the
inception of his service up to the time he reached 60 years of
age, in accordance with its retirement plan.

CONSTRUCTIVE DISCHARGE
DEFINED
PHIL JAPAN ACTIVE CARBON CORP V NLRC
(QUINANOLA)
171 SCRA 164
GRINO-AQUINO; March 8, 1989
NATURE
A petition for review
FACTS
- Quinanola had been employed in Phil. Japan since January 19,
1982, as Assistant Secretary/Export Coordinator.
He was
promoted to the position of Executive Sec. to the Executive Vice
President and General Manager. On May 31, 1986, for no
apparent reason at all and without prior notice to her, she was
transferred to the Production Department as Production
Secretary, swapping positions with Ester Tamayo. Although the
transfer did not amount to a demotion because her salary and
workload remained the same, she believed otherwise so she
rejected the assignment and filed a complaint for illegal
dismissal.
LA found that the transfer would amount to
constructive dismissal and her refusal to obey the order was
justified. Upon appeal to the NLRC, the Commission approved
the Labor Arbiter's decision but reduced to P10,000 the award
of moral damages and the attorney's fees to 10% of the
judgment.
ISSUE
WON Quinanola was constructively and illegally dismissed as a
result of her transfer or assignment to the Office of the
Production Manager even if she would have received the same
salary rank, rights and privileges
HELD

A2010

- 267 -

Disini

NO
- A constructive discharge is defined as: "A quitting because
continued employment is rendered impossible, unreasonable or
unlikely; as, an offer involving a demotion in rank and a
diminution in pay." In this case, Quinanolas assignment as
Production Secretary of the Production Department was not
unreasonable as it did not involve a demotion in rank (her rank
was still that of a department secretary) nor a change in her
place of work (the office is in the same building), nor a
diminution in pay, benefits, and privileges. It did not constitute
a constructive dismissal.
- It is the employer's prerogative, based on its assessment and
perception of its employees' qualifications, aptitudes, and
competence, to move them around in the various areas of its
business operations in order to "ascertain where they will
function with maximum benefit to the company. An employee's
right to security of tenure does not give him such a vested right
in his position as would deprive the company of its prerogative
to change his assignment or transfer him where he will be most
useful. When his transfer is not unreasonable, nor inconvenient,
nor prejudicial to him, and it does not involve a demotion in
rank or a diminution of his salaries, benefits, and other
privileges, the employee may not complain that it amounts to a
constructive dismissal.
- On the other hand, we reject the petitioner's contention that
the private respondent's absence from work on June 2 to June 3,
1986 constituted an abandonment of her job in the company
resulting in the forfeiture of the benefits due her. While she was
guilty of insubordination for having refused to move out of her
position as Executive Secretary to the Executive Vice-President
and General Manager of the company, dismissal from the
service would be a draconian punishment for it, as her
complaint for illegal dismissal was filed in good faith.
Disposition the decision of the NLRC insofar as it orders the
petitioner to reinstate the private respondent is affirmed, but
she shall be reinstated to her position as Production Secretary
of the Production Department of petitioner's corporation without
loss of seniority rights and other privileges. The awards of
backwages, moral damages and attorney's fees to the private
respondent are hereby set aside. No pronouncement as to
costs.

DUSIT HOTEL NIKKO V NUWHRAIN


466 SCRA 374
CALLEJO; August 9, 2005
NATURE
Petition for review on certiorari of the Decision of the Court of
Appeals
FACTS
The Case for Rowena Agoncillo
- Agoncillo was employed by the Hotel. After some time, she
was promoted as Supervisor of Outlet Cashiers and later
promoted as Senior Front Office Cashier.
- The Hotel decided to trim down the number of its employees
from the original count of 820 to 750.
- The Hotel offered a Special Early Retirement Program (SERP)
to all its employees. It was stated therein that the program was
intended to provide employees financial benefits prior to
prolonged renovation period and, at the same time, to enable
management to streamline the organization by eliminating
redundant positions and having a more efficient and productive
manpower complement.
- Union president Rasing, sought a commitment from the
management that the employees terminated due to
redundancy will not be replaced by new employees; nor will
their positions be given to subcontractors, agencies or casual
employees.
- A total of 243 employees, including Agoncillo, 161 of whom
were Union officers and members, were separated from the
Hotels employment. As a result, the membership of the Union
was substantially reduced.

Labor Law 1
- The Hotel wrote DOLE saying that the Hotel terminated the
employment of 243 employees due to redundancy. On the
same day, Agoncillo was summoned by Hotel Comptroller
Reynaldo Casacop, who gave her a letter of even date informing
the latter of her separation from service due to redundancy
effective close of office hours of April 30, 1996.
- Casacop advised Agoncillo to just avail of the Hotel's SERP, as
embodied in the inter-office memorandum of Masuda. He
informed her that she had the option to avail of the program
and that, in the meantime, he will defer the processing of her
termination papers to give her time to decide. On April 3, 1996,
Agoncillo finally told Casacop that she would not avail of the
SERP benefits. By then, she had decided to file a complaint for
illegal dismissal against the Hotel.
- Meanwhile, the Hotel temporarily closed operations because of
the renovation thereof.
- When news spread among the hotel employees that Agoncillo
would contest her termination before the NLRC, she was
summoned by Personnel Manager Leticia Delarmente to a
conference. Delarmente and Dizon repeatedly asked Agoncillo
to give back the original copy of the April 1, 1996 termination
letter. Agoncillo told them that the letter was already in the
possession of her counsel. Agoncillo was relieved when she
was given another letter of even date stating that, by reason of
her non-availment of the SERP, she was still considered an
employee but on temporary lay-off due to the ongoing
renovation of the Hotel and that she will just be advised
accordingly of her work schedule when the Hotel reopens.
- Delarmente and Dizon offered to reinstate Agoncillo but not to
her former position as Senior Front Office Cashier. Agoncillo
objected but informed them that she could accept the position
of Reservation Clerk. However, no response was received.
- She was told by Dizon that the Hotel was willing to reinstate
her but as an Outlet Cashier. Dizon explained that the Hotel
had already hired new employees for the positions of
Reservation Clerks. Agoncillo, however, pointed out that she
was already an Outlet Cashier Supervisor before her promotion
as Senior Front Office Cashier and that if she accepted the
position, it would be an unjustified demotion on her part. After
Agoncillos meeting with Dizon, the latter kept on promising to
find a suitable position for her. In those meetings, Dizon always
offered reinstatement to positions that do not require guest
exposure like Linen Dispatcher at the hotel basement or
Secretary of Roomskeeping. When Agoncillo refused, Dizon just
instructed her to return. Agoncillo had no specific position or
assigned task to perform.
- When the Hotel resumed operations, the Union filed a Notice
of Strike for unfair labor practice with the DOLE.
The Case for the Hotel
- Pursuant to the reorganization program, a reclassification of
positions ensued upon resumption of the Hotels operation.
Consequently, the position of Agoncillo as Senior Front Office
Cashier was abolished and a new position of Guest Services
Agent absorbing its functions was created. Considering that the
new position requires skills in both reception and cashiering
operations, respondent Hotel deemed it necessary to transfer
Agoncillo to another position as Outlet Cashier, which does not
require other skills aside from cashiering.
- The transfer of Agoncillo from Senior Front Office Cashier to
Outlet Cashier does not entail any diminution of salary or rank.
Despite which, she vehemently refused the transfer and
insisted that she be reinstated to her former position. Since
Agoncillo was not amenable to the said transfer, she did not
assume her new position and since then had stopped reporting
for work despite the Hotels patient reminder to act on the
contrary. Instead, she filed a complaint to question the
prerogative of the management to validly transfer her to
another position as she considers the transfer an act of
constructive dismissal amounting to illegal termination and
unfair labor practice in the form of union busting.
ISSUE

A2010

- 268 -

Disini

WON Agoncillo was illegally dismissed


HELD
YES
- We agree with the contention of the petitioners that it is the
prerogative of management to transfer an employee from one
office to another within the business establishment based on its
assessment and perception of the employees qualification,
aptitude and competence, and in order to ascertain where he
can function with the maximum benefit to the company. But,
like other rights, there are limits thereto. The managerial
prerogative to transfer personnel must be exercised without
grave abuse of discretion, bearing in mind the basic elements of
justice and fair play. Having the right should not be confused
with the manner in which that right is exercised.
- There is constructive dismissal when there is a demotion in
rank and/or diminution in pay; or when a clear discrimination,
insensibility or disdain by an employer becomes unbearable to
the employee.
- In the present case, the Hotel recalled the termination of
respondent Agoncillo when they learned that she was going to
file a complaint against them with the NLRC for illegal dismissal.
However, instead of reinstating her to her former position, she
was offered the position of Linen Dispatcher in the hotel
basement or Secretary of the Roomskeeping Section, positions
much lower than that of a Supervisor of Outlet Cashiers which
the respondent held before she was promoted as Senior Front
Office Cashier. With the said positions, the respondent would
not certainly be receiving the same salary and other benefits as
Senior Front Office Cashier.
- The offers by the petitioners to transfer Agoncillo to other
positions were made in bad faith, a ploy to stave off a suit for
illegal dismissal. In fact, Agoncillo had not been transferred to
another position at all.
- Even assuming, for the sake of argument, that the hotel had a
valid ground for dismissing [the] complainant and that it had
merely spared her such fate, the hotel is still guilty of illegal
dismissal. Had the hotel made the transfer of complainant in
good faith and in the normal course of its operation, it would
have been justified. In this case, however, the supposed transfer
was made only after complainant had been earlier terminated.
Complainants statement in her affidavit that she was
summoned by the hotel after news of her plan to contest her
dismissal circulated remains unrefuted. Furthermore, the hotel
has not explained why there was no official memorandum
issued to complainant formally informing her of her transfer.
All these lead to only one conclusion that the alleged transfer
was not made in good faith as a valid exercise of management
prerogative but was intended as a settlement offer to
complainant to prevent her from filing a case.
Disposition Petition is DENIED for lack of merit. Costs against
the petitioners.

MOBILE PROTECTIVE AND DETECTIVE AGENCY V


OMPAD
458 SCRA 308
PUNO; May 9, 2005
NATURE
Petition for review on certiorari of the decision of the CA
FACTS
- Private respondent, Alberto Ompad, was employed by the
petitioner as a security guard in 1990. He was assigned to the
various clients of Mobile. In June 1997, respondent was
assigned as a security guard at Manila Southwoods when he
inquired from the project manager of Southwoods if they have
already paid their backwages to the security agency. Ompad
claims that when the Agency found out about his query, he was
relieved from his post and never given another assignment.
- The petitioner on the other hand claims that Ompad was
assigned to another client, Valle Verde Country Club from

Labor Law 1
August 29 to October 31, 1997 after he was relieved from his
post at the Manila Southwoods. Petitioner further claims that
one of the guards at Valle Verde attested that Ompad had told
her that he would earn better if he just drives his tricycle full
time. On October 15, 1997, Ompad reported for work but he
was limping due to an accident he suffered while driving his
tricycle. Petitioner claims that he stopped reporting for work
after that date. On September 23, 1998, Domingo Alonzo,
operations manager of Mobile saw respondent and inquired as
to whether he was still interested in reporting for work. The
petitioner allegedly answered in the negative and it was at that
time that Alonzo advised him to resign. Ompad, he claims,
submitted his hand written resignation which also was a quit
claim.
- Petitioner contended that the letter of resignation was forced
on him in return for monies owed him. As he needed the
money, he had no choice but to comply. He however was only
being given Pesos 5,000 which he rejected. He filed this case
the following day.
- Ompad alleged that he was illegal terminated and claimed
underpayment or non-payment of wages, overtime pay,
premium pay for holiday and rest day, separation pay, etc.
- Labor Arbiter dismissed the complaint for lack of merit. The
NLRC reversed the decision. The CA also dismissed the action
for reconsideration, noting that there was no voluntariness in
the acts of Ompad in submitting the resignation letters. Hence
this action.
ISSUE
WON Ompad was illegally dismissed
HELD
YES
- The resignation letters of Ompad are dubious as they were
written in a language obviously not his and lopsidedly worded
to free the Agency from liabilities. The affidavits issued by the
witnesses of Mobile are suspect considering that these
witnesses were/are in fact employed by the petitioner.
Reasoning
- All the documentary evidence proves that respondent was
assigned to Valle Verde from September 29 to October 31, 1997
and that he stopped reporting for work on October 16, 1997.
After this period, respondent did not seem to have be given any
further assignment.
- The SC ruled that while it is true that security guards may be
put on floating status the same should last for only six months.
In the case at bar, there was no showing that Mobile lacked
engagements to which they can post their guards. Absent any
dire exigency justifying their failure to give respondent further
assignment, the only logical conclusion is that respondent was
constructively dismissed.
- Even assuming that Mobile was justified in not immediately
giving Ompad any assignment after October, the length of time
that he was put on floating status is tantamount to constructive
dismissal.
- In an illegal dismissal case, the onus probandi is on the
employer to prove that the dismissal was in fact for valid cause.
It was in this case also the burden of Mobile to submit evidence
that the resignation was voluntary on the part of Ompad.
Disposition Petition dismissed.

DUNCAN ASSOCIATION V GLAXO WELLCOME


[PAGE 43]
R.P. DINGLASAN CONSTRUCTION INC V ATIENZA
433 SCRA 263
PUNO; June 29, 2004
NATURE
This is an appeal from the decision and resolution of the Court
of Appeals, dated January 17, 2001 and October 30, 2002,

A2010

- 269 -

Disini

respectively, upholding the finding of constructive dismissal


against petitioner.
FACTS
- Petitioner R.P. Dinglasan Construction, Inc. provided janitorial
services to Pilipinas Shell Refinery Corporation (Shell
Corporation) in Batangas City. Private respondents Mariano
Atienza and Santiago Asi served as petitioners janitors
assigned with Shell Corporation since 1962 and 1973,
respectively.
- July 7, 1994 - Dinglasan called for a meeting and informed
them and 3 other employees that their employment with Shell
Corporation would be terminated effective July 15, 1994. They
were told that Dinglasan lost the bidding for janitorial services
with Shell. Dinglasan notified them that they may reapply as
helpers and redeployed in other companies where DInglasan
had subsisting contracts but they would receive only a
minimum wage. Atienza refused as the offer would be a form of
demotion --- they would lose their seniority status and would
not be guaranteed to work at regular hours.
- December 1994 Atienza et al filed a complaint against
Dinglasan for non-payment of salary with the DOLE district
office in Batangas City.
- February 1995 - during the conciliation proceedings with the
DOLE, Dinglasan sent notices to Atienza et al informing them
that they would be reinstated with Shell Corporation as soon as
they submit their barangay clearance, medical certificate,
picture and information sheet as per the new identification
badge requirements of Shell Corporation. Barangay officials met
with Dinglasan to signify Atienza et als willingness to to be
reinstated bringing with them said requirements
- May 1995 Atienza et al demanded the payment of their
backwages starting from July 15, 1994.
- June 1, 1995 Dinglasan notified Atienza et al they have been
declared absent without leave (AWOL) as they allegedly failed
to signify their intention to return to work and submit the badge
requirements for their reinstatement.
- June 13, 1995 Atienza et al wrote Dinglasan and insisted that
they had complied with the badge requirements. Accompanied
by the barangay officials, Atienza et al attempted to meet with
the officers of Dinglasan but the latter refused to dialogue with
them. As proof of their compliance with the Shell requirements,
Atienza et al submitted to the DOLE their x-ray results, dated
May 17 and 19, 1995 and their barangay certification, dated
May 13, 1995.
- NLRC > Atienza et al charged DInglasan with illegal dismissal
and non-payment of 13th month pay, with a claim for payment
of attorneys fees and litigation expenses, and a prayer for
reinstatement with payment of full backwages from July 15,
1994.
> Dinglasan gave a different version of the incident.
It
allegedly informed Atienza et al and the other affected
employees that they would be deployed to petitioners other
principal companies but that their work would be different.
Except for Atienza et al, all the affected employees accepted its
offer of redeployment and reported back to work. Atienza et al
failed to submit a resignation letter to signify their intention not
to return to work.
- during the pendency of the labor case Dinglasan in 2
separate notices informed Atienza et al that they could be
reinstated at Shell Corporation with no diminution in their salary
provided that they submit the documents for the new
identification badge requirement of Shell Corporation. Atienza
et al, however, refused to return to work until they were paid
their backwages. Consequently, Dinglasan was constrained to
consider them as having abandoned their work and to
terminate their employment on September 19, 1995. Dinglasan,
thus, justified the dismissal of Atienza et al on the grounds of
gross and habitual neglect of duties and abandonment of work.
- LABOR ARBITER > September 3, 1998, LA Andres Zavalla
rendered a decision finding that private respondents were
illegally
dismissed
from
service
and
ordering
their
reinstatement.
- NLRC > the decision of the labor arbiter was affirmed

Labor Law 1
- CA > PROCEDURAL: petition could not prosper as petitioner
failed to move for a reconsideration of the NLRC decision;
SUBSTANTIVE: upheld the findings of the labor arbiter and the
NLRC that:
(1) private respondents were constructively
dismissed as petitioners offer of reassignment involved a
diminution in pay and demotion in rank that made their
continued employment unacceptable; and, (2) private
respondents could not be considered to have abandoned their
work; DInglasans motion for reconsideration was denied
ISSUES
1. WON there is valid dismissal on the ground that they failed
to report back to the office and this abandoned their work
2. WON there was constructive dismissal
HELD
1. Ratio In an illegal dismissal case, the onus probandi rests
on the employer to prove that its dismissal of an employee is
for a valid cause. In the case at bar, Dinglasan failed to
discharge its burden. It failed to establish that Atienza et al
deliberately and unjustifiably refused to resume their
employment without any intention of returning to work.
- To constitute abandonment of work, two (2) requisites must
concur: first, the employee must have failed to report for work
or must have been absent without justifiable reason; and
second, there must have been a clear intention on the part of
the employee to sever the employer-employee relationship as
manifested by overt acts. Abandonment as a just ground for
dismissal requires deliberate, unjustified refusal of the
employee to resume his employment. Mere absence or failure
to report for work, after notice to return, is not enough to
amount to abandonment.
Reasoning
- the evidence negates the theory that they abandoned their
work.
(1) Atienza et al reported back to Dinglasans office a number of
times expressing their desire to continue working without
demotion in rank or diminution of salary.
This fact was
established by the corroborating testimony of barangay
officials, accompanied Atienza et al to Dinglasans office at least
ten (10) times to negotiate their redeployment on more
acceptable terms.
(2) in seeking reinstatement, Atienza et al also sought the
intervention of the DOLE to arbitrate the labor issue between
the parties.
(3) Atienza et al submitted the barangay clearances and x-ray
results required from them by petitioner for their reinstatement
as witnessed by the barangay officials.
(4) the records would bear that Atienza et al lost no time and
sought their reinstatement by filing an illegal dismissal case
against Dinglasan, which act is clearly inconsistent with a desire
to sever employer-employee relations and abandon their work.
- All these overt acts on the part of Atienza et al negate
Dinglasans claim of abandonment of work and prove beyond
doubt their steadfast desire to continue their employment with
petitioner and be reinstated to their former position.
2. YES
Ratio
Constructive dismissal is defined as quitting when
continued employment is rendered impossible, unreasonable or
unlikely as the offer of employment involves a demotion in rank
and diminution of pay.
Reasoning
- Dinglasan committed constructive dismissal when it offered to
reassign Atienza et al to another company but with no
guaranteed working hours and payment of only the minimum
wage.
The terms of the redeployment thus became
unacceptable for private respondents and foreclosed any choice
but to reject petitioners offer, involving as it does a demotion in
status and diminution in pay. Thereafter, for six (6) months,
Atienza et al were in a floating status. Interestingly, it was only
after Atienza et al filed a complaint with the DOLE that
Dinglasan backtracked in its position and offered to reinstate
Atienza et al to their former job in Shell Corporation with no
diminution in salary. Eventually, however, Dinglasan unilaterally

A2010

- 270 -

Disini

withdrew its offer of reinstatement, refused to meet with the


Atienza et al and instead decided to dismiss them from service.
Disposition petition is DISMISSED and the impugned decision
and resolution of the Court of Appeals, dated January 17, 2001
and October 30, 2002, respectively, are AFFIRMED in toto.

GO V CA (MOLDEX PRODUCTS INC)


430 SCRA 358
YNARES-SANTIAGO; May 28, 2004
NATURE
Petition for review decision of CA (which set aside resolutions of
NLRC)
FACTS
- Fernando Go was hired by Moldex Products Inc. in 1986 as a
salesman, then, over the years, was promoted to a Senior Sales
Manager. As such officer, he was responsible for overseeing and
managing the sales force of the company such as dealing with
clients, getting orders, entering into an agreement with clients
(subject to approval of higher management).
- Sometime in 1998, the EVP o Moldex called the attention of Go
regarding the discovery of alleged anomalies purportedly
committed by the sales people under Gos control. Such
anomalies stemmed from the disbursement of funds by Moldex
to govt officials to secure big supplpy contracts from the govt.
- Because of the issue, a number of employees were dismissed,
including those under Gos supervision. Go himself was
terminated, allegedly on account of command responsibility.
Moldex claimed that Go, obviously feeling guilty for not
exercising effective supervision over his subordinates,
submitted a letter of resignation dated October 12, 1998 but
effective on November 16, 1998. Moldex added that Go went
on leave from Oct 12, 1998 to Nov 16, 1998. While on leave, he
worked for the release of his clearance and the payment of 13th
month pay and leave pay benefits. On the other hand, Go
contends that he was not investigated. The investigation only
involved other sales people. He filed a complaint for
constructive dismissal. LA ruled for Go (there was illegal
dismissal), NLRC affirmed, but CA set aside the decisions,
relying on evidence that Go was actively performing his normal
duties and functions during the months immediately prior to his
resignation, contrary to the finding of constructive dismissal.
ISSUE
WON there was constructive dismissal
HELD
NO
Ratio Constructive dismissal exists where there is a cessation
of work because continued employment is rendered impossible,
unreasonable or unlikely. It is present when an employees
functions, which were originally supervisory in nature, were
reduced, and such reduction is not grounded on valid grounds
such as genuine business necessity.
Reasoning
- Apparently, Go still fully exercised the prerogatives and the
responsibilities of his office as the Senior Sales Manager during
the time that the said functions were supposedly removed from
him. Therefore, there can be no constructive dismissal to speak
of.
- Go claims that his separation from employment with Moldex
was a case of constructive dismissal, an allegation which the
company refutes with its own set of evidence pointing to the
Gos voluntary resignation.
- It should be remembered that Go has submitted a letter of
resignation. It is thus incumbent upon him to substantiate his
claim that his resignation was not voluntary but in truth was
actually a constructive dismissal. This the petitioner failed to
do. His bare allegations, when uncorroborated by evidence,
cannot be given credence.

Labor Law 1

A2010

- on the other hand, Moldex presented confidential sales


evaluation forms that prove that Go was still performing his
duties and responsibilities one month prior to his resignation.
- While on leave, he worked for the release of his clearance and
the payment of his 13th month pay and leave pay benefits. In
doing so, he in fact performed all that an employee normally
does after he resigns. Resignation is the formal pronouncement
or relinquishment of an office. The voluntary nature of Gos acts
has manifested itself clearly belie his claim of constructive
dismissal.
The totality of the evidence indubitably shows that Go resigned
from employment without any coercion or compulsion from
respondent. His resignation was voluntary.
Disposition: Petition denied, and decision of CA AFFIRMED.

ACUNA V CA
[PAGE 12]
POSEIDON FISHING V NLRC
[PAGE 98]

CONSTRUCTIVE
DISCHARGE
ILLEGAL DISMISSAL

AND

MARK ROCHE V NLRC


313 SCRA 356
BELLOSILLO; August 31, 1999
FACTS
- On different dates, private respondents filed separate
complaints for underpayment of wages and non-payment of
overtime pay against petitioners Mark Roche International
(MRI), Eduardo Dayot and Susan Dayot. Private respondents
sought the assistance of a labor organization which helped
them organize the Mark Roche Workers Union (MRWU).
Apparently irked by the idea of a union within the company,
petitioners ordered private respondents to withdraw the petition
and further threatened them that should they insist in the
organization of a union they would be dismissed. Unfazed,
private respondents refused. As expected, private respondents
were discharged from work. Petitioners disclaimed knowledge
of any deficiency owing to private respondents since all the
benefits due them as required by law were fully paid, except
overtime pay which they were not entitled to on account of
their being piece-rate workers. The Labor Arbiter rendered his
decision declaring as illegal the constructive dismissal of private
respondents and ordered their reinstatement, payment of
backwages, salary differentials and proportionate 13th month
pay and service incentive leave pay. On appeal, the National
Labor Relations Commission (NLRC) affirmed the decision of the
Labor Arbiter, but set aside the award of service incentive leave
on the ground that private respondents were not entitled
thereto as they were piece-rate workers. Petitioners moved for
reconsideration, but it was denied. Hence, the present petition.
ISSUE
WON the dismissal of private respondents was a constructive
dismissal or an illegal dismissal
HELD
- Constructive dismissal or a constructive discharge has been
defined as a quitting because continued employment is
rendered impossible, unreasonable or unlikely, as an offer
involving a demotion in rank and a diminution in pay. In the
instant case, private respondents were not demoted in rank nor
their pay diminished considerably. They were simply told
without prior warning or notice that there was no more work for
them. After receiving the notice of hearing of the petition for
certification election on 27 October 1992, petitioners
immediately told private respondents that they were no longer

- 271 -

Disini

employed.
Evidently it was the filing of the petition for
certification election and organization of a union within the
company which led petitioners to dismiss private respondents
and not petitioners' allegations of absence or abandonment by
private respondents. The formation of a labor union has never
been a ground for valid termination, and where there is an
absence of clear, valid and legal cause, the law considers the
termination illegal.

GLOBE TELECOM INC V FLORENDO


390 SCRA 201
September 27, 2002
NATURE
Petition for review on certiorari of a decision of CA.
FACTS
- FLORENDO, a Senior Account Manager of Globe, filed a
complaint for constructive dismissal against Globe with some
key officials [GLOBE et al., for brevity] and FLORENDOs
immediate superior Cacholo Santos [SANTOS, for brevity].
FLORENDO complained that SANTOS never submitted her
performance evaluation report thereby depriving her of salary
increases and incentives which other employees of the same
rank had been receiving; reduced her to a house-to-house
selling agent (i.e. a direct sales agent) of company products
("handyphone") despite her rank as supervisor of company
dealers and agents; never supported her in the sales programs
she presented; and, withheld all her other benefits.
- GLOBE et al., on the other hand, claimed that FLORENDO
abandoned her work; that her complaint rested on a purely
private disagreement with her immediate superior, and that she
filed the complaint without consulting the companys grievance
process.
ISSUE
WON FLORENDO can be constructively dismissed from service
HELD
YES
Ratio Constructive dismissal exists where there is cessation of
work because "continued employment is rendered impossible,
unreasonable or unlikely, as an offer involving a demotion in
rank and a diminution in pay." All these are discernible in
FLORENDOs situation. She was singularly edged out of
employment by the undesirable treatment she received from
her superior, who discriminated against her without reason.
(See above for SANTOS acts against FLORENDO.) And although
FLORENDO continued to have the rank of a supervisor, her
functions were reduced to a mere direct sales agent. This was
tantamount to a demotion. She might not have suffered any
diminution in her basic salary but GLOBE et al. did not dispute
her allegation that she was deprived of all benefits due to
another of her rank and position, benefits which she apparently
used to receive.
- Far from blaming SANTOS alone, FLORENDO also attributes
her degraded state to GLOBE et al. She cited GLOBE et al.'s
indifference to her plight as she was twice left out in a salary
increase, without GLOBE et al. giving her any reason. It eludes
belief that GLOBE et al. were entirely in the dark as the salary
increases were granted across-the-board to all employees
except FLORENDO. It is highly improbable that the exclusion of
FLORENDO had escaped GLOBE et al.'s notice. The absence of
an evaluation report from SANTOS should have been looked into
by GLOBE et al. for proper action. If a salary increase was
unwarranted, then it should have been sufficiently explained by
GLOBE et al. to FLORENDO. And despite GLOBE et al.s claim
that FLORENDO did not brought her problem against SANTOS to
the company's grievance machinery, it remains uncontroverted
that FLORENDO had inquired from GLOBE et al. why her other
benefits had been withheld and sought clarification for her
undeserved treatment but GLOBE and SANTOS remained mum.

Labor Law 1
- Thus, the dispute was not a mere private spat between
FLORENDO and her superior; the case overflowed into the realm
of FLORENDO's employment. And at the very least, GLOBE et al.
were negligent in supervising all of their employees.
- In constructive dismissal, the employer has the burden of
proving that the transfer and demotion of an employee are for
just and valid grounds such as genuine business necessity. The
transfer must not involve a demotion in rank or a diminution of
salary and other benefits. If the employer cannot overcome this
burden of proof, the employee's demotion shall be tantamount
to unlawful constructive dismissal. The award of back wages in
the instant case is justified upon the finding of illegal dismissal.
Disposition CA decision that FLORENDO abandoned her work,
SET ASIDE. GLOBE et al. to pay FLORENDO full back wages from
the time she was constructively dismissed until her
reinstatement, and to cause immediate reinstatement of
FLORENDO to her former position, without loss of seniority
rights and other benefits.

PREVENTIVE SUSPENSION
GLOBE-MACKAY CABLE AND RADIO CORP V NLRC
(SALAZAR)
206 SCRA 702
ROMERO; March 3, 1992
NATURE
Appeal from a decision of NLRC
FACTS
- Imelda L. Salazar was employed by Globe-Mackay Cable and
Radio Corporation (GMCR) as general systems analyst. Also
employed by petitioner as manager for technical operations'
support was Delfin Saldivar with whom private respondent was
allegedly very close.
- GMCR, prompted by reports that company equipment and
spare parts worth thousands of dollars under the custody of
Saldivar were missing, caused the investigation of the latter's
activities.
- The report prepared by the company's internal auditor
indicated that Saldivar had entered into a partnership styled
Concave Commercial and Industrial Company with Richard A.
Yambao, owner and manager of Elecon Engineering Services
(Elecon), a supplier of petitioner often recommended by
Saldivar; that Saldivar had taken petitioner's missing Fedders
airconditioning unit for his own personal use without
authorization and also connived with Yambao to defraud
petitioner of its property; that Imelda Salazar violated company
regulations by involving herself in transactions conflicting with
the company's interests. Evidence showed that she signed as a
witness to the articles of partnership between Yambao and
Saldivar. It also appeared that she had full knowledge of the
loss and whereabouts of the Fedders airconditioner but failed to
inform her employer.
- GMCR placed Salazar under preventive suspension for 1
month, thus giving her 30 days within which to, explain her
side. But instead of submitting an explanation, private
respondent filed a complaint against petitioner for illegal
suspension, which she subsequently amended to include illegal
dismissal, vacation and sick leave benefits, 13th month pay and
damages, after petitioner notified her in writing that she was

A2010

- 272 -

Disini

considered dismissed in view of her inability to refute and


disprove the findings.
- Labor Arbiter ordered petitioner company to reinstate private
respondent to her former or equivalent position and to pay her
full backwages and other benefits
- NLRC affirmed the aforesaid decision
ISSUES
1. WON the suspension was illegal
2. WON Art.2794 of the Labor Code should apply
HELD
1. NO
Ratio By itself, preventive suspension does, not signify that the
company has adjudged the employee guilty of the charges she
was asked to answer and explain. Such disciplinary measure is
resorted to for the protection of the company's property
pending investigation any alleged malfeasance or misfeasance
committed by the employee.
Reasoning
- The investigative findings of GMCR which pointed to Saldivar's
acts in conflict with his position as technical operations
manager, necessitated immediate and decisive action on any
employee closely, associated with Saldivar. The suspension of
Salazar was further impelled by the discovery of the missing
Fedders airconditioning unit inside the apartment private
respondent shared with Saldivar. Under such circumstances,
preventive suspension was the proper remedial recourse
available to the company pending Salazar's investigation.
- If at all, the fault, lay with Salazar when she ignored
petitioner's memo giving her ample opportunity to present her
side. Instead, she filed her complaint for illegal suspension
without giving her employer a chance to evaluate her side of
the controversy.
2. YES
Ratio Where a case of unlawful or unauthorized dismissal has
been proved by the aggrieved employee, or on the other hand,
the employer whose duty it is to prove the lawfulness or
justness of his act of dismissal has failed to do so, then the
remedies provided in Article 279 should find, application.
Reasoning
- It must be recalled that the present Constitution has gone
further than the 1973 Charter in guaranteeing vital social and
economic rights to marginalized groups of society, including
labor. To be sure, both Charters recognize "security of tenure"
as one of the rights of labor which the State is mandated to
protect. But there is no gainsaying the fact that the intent of the
framers of the present Constitution was to give primacy to the
rights of labor and afford the sector "full protection," at least
greater protection than heretofore accorded them, regardless of
the geographical location of the workers and whether they are
organized or not.
- that the right of an employee not to be dismissed from his job
except for a just or authorized cause provided by law has
assumed greater importance under the 1987 Constitution with
the singular prominence labor enjoys under the article on Social
Justice. And this transcendent policy has been translated into
law in the Labor Code
- The intendment of the law in prescribing the twin remedies of
reinstatement and payment of backwages is, in the former, to
restore the dismissed employee to her status before she lost
her job, for the dictionary meaning of the word "reinstate" is "to
restore to a state from which one had been removed" and in the
latter, to give her back the income lost during the period of
unemployment.
4

The following provision on security of tenure is embodied in Article 279 reproduced


herein but with the amendments inserted by RA 6715:
In cases of regular employment, the employer shall not terminate the services of-an
employee except for a just cause or when authorized by this Title. An employee who
is unjustly dismissed from work shall be entitled to reinstatement without loss of
seniority rights AND OTHER PRIVILEGES and to his FULL backwages, inclusive of
allowances, and to his other benefits or their monetary equivalent computed from the
time his compensation was withheld from him up to the time of his ACTUAL
reinstatement.

Labor Law 1
- Over time, the following reasons have been advanced by the
Court for denying reinstatement under the facts of the case and
the law applicable thereto; that reinstatement can no longer be
effected in view of the long passage of time or because of the
realities of the situation; or that it would be "inimical to the
employer's interest; " or that reinstatement may no longer be
feasible; or that it will not serve the best interests of the parties
involved; or that the company would be prejudiced by the
workers' continued employment; or that it will not serve any
prudent purpose as when supervening facts have transpired
which make execution on that score unjust or inequitable or, to
an increasing extent, due to the resultant atmosphere of
"antipathy and antagonism" or "strained relations" or
"irretrievable estrangement" between the employer and the
employee. In lieu of reinstatement, the Court has variously
ordered the payment of backwages and separation pay or
solely separation pay.
- If in the wisdom of the Court, there may be a ground or
grounds for non-application of the Art.279, this should be by
way of exception, such as when the reinstatement may be
inadmissible due to ensuing strained relations between the
employer and the employee.
- Here, it has not been proved that the position of private
respondent as systems analyst is one that may be
characterized as a position of trust and confidence such that if
reinstated, it may well lead to strained relations between
employer and employee. Hence, this does not constitute an
exception to the general rule mandating reinstatement for an
employee who has been unlawfully dismissed.
- To rely on the Maramara report as a basis for Salazar's
dismissal would be most inequitous because the bulk of the
findings centered principally oh her friend's alleged thievery
and anomalous transactions as technical operations' support
manager. Said report merely insinuated that in view of Salazar's
special relationship with Saldivar, Salazar might have had direct
knowledge of Saldivar's questionable activities. Direct evidence
implicating private respondent is wanting from the records.
Disposition
The assailed resolution of NLRC is AFFIRMED.
Petitioner GMCR is ordered to REINSTATE Salazar and to pay her
backwages equivalent to her salary for a period of 2 years only.
The decision is immediately executory.

SEPARATE OPINIION
MELENCIO-HERRERA [dissent]
- I believe there is just cause for dismissal per investigative
findings.

PHIL AIRLINES INC V NLRC (CASTRO)


292 SCRA 40
ROMERO; July 8, 1998
NATURE
Appeal from a decision of the NLRC affirming the decision of the
LA
FACTS
- Private Respondent Edilberto Castro, an employee
(manifesting clerk) of PAL was apprehended by govt. authorities
while about to board a flight to H.K. Castro and co-employee
Arnaldo Olfindo were found to be in possession of P39,850 and
P6,000 respectively, in violation of Central Bank (CB) Circular
265, as amended by CB Circular 383, 1 in relation to Section 34
of R.A. 265, as amended.
- Upon knowledge of this incident, PAL required respondent to
explain within 24 hrs why he should not be charged
administratively. He failed to comply and was placed on
preventive suspension effective March 27, 1984 for grave
misconduct. An investigation was later conducted wherein
respondent admitted ownership of the confiscated money but
denied any knowledge of CB Circular 265. Respondent, through

A2010

- 273 -

Disini

the PAL Employees Association (PALEA) then sought not only


the dismissal of his case but also prayed for his reinstatement.
- 3 years and 6 months after his suspension, PAL issued a
resolution finding respondent guilty of the offense charged but
nonetheless reinstated the latter explaining that the period
within which he was out of work shall serve as penalty for
suspension. Upon reinstatement, respondent filed a claim
against PAL for backwages and salary increases granted under
the CBA covering the period of his suspension which the latter,
however, denied on account that under the existing CBA, an
employee under suspension is not entitled to CBA salary
increases granted during the period covered by his penalty.
- Labor Arbiter De Vera rendered a decision in favor of Castro;
limiting his suspension to 1 month; ordering PAL to pay his
salaries, benefits, and other privileges from April 26, 1984 up to
Sept. 18, 1987 and to pay his salary increases accruing during
the period aforesaid. Moral damages and exemplary damages
were likewise awarded. On appeal, the NLRC affirmed the LA
decision but deleted the award of moral and exemplary
damages, hence, this petition.
ISSUE
WON an employee who has been preventively suspended
beyond the maximum 30-day period is entitled to backwages
and salary increases granted under the CBA during his period of
suspension
HELD
YES
- The rules are rather clear under Secs. 3 and 4, Rule XIV of the
Omnibus Rules Implementing the Labor Code:
Sec.3. Preventive suspension. The employer can place the
worker concerned under preventive suspension if his
continued employment poses a serious and imminent threat
to the life or property of the employer or of his co-workers
Sec.4. Period of suspension. No preventive suspension shall
last longer than 30 days. The employer shall thereafter
reinstate the worker in his former or in a substantially
equivalent position or the employer may extend the period of
suspension provided that during the period of extension, he
pays the wages and other benefits due to the workers. In
such case, the worker, shall not be bound to reimburse the
amount paid to him during the extension if the employer
decides, after completion of the hearing, to dismiss the
worker.
Reasoning
- It is undisputed that the period of suspension of respondent
lasted for 3 years and 6 months. PAL, therefore, committed a
serious transgression when it manifestly delayed the
determination of respondents culpability in the offense
charged. The provisions of the rules are explicit and direct;
hence, there is no reason to further elaborate on the same.
- PAL faults the LA and the NLRC for allegedly equating
preventive suspension as remedial measure with suspension as
penalty for administrative offenses. This argument is
inaccurate. As held in Beja Sr. v CA: Imposed during the
pendency of an administrative investigation, preventive
suspension is not a penalty in itself. It is merely a measure of
precaution so that the employee who is charged may be
separated, for obvious reasons, from the scene of his alleged
misfeasance while the same is being investigated. While the
former may be imposed on a respondent during the
investigation of the charges against him, the latter is the
penalty which may only be meted upon him at the termination
of the investigation or the final disposition of the case. A
cursory reading of the records reveals no reason to ascribe
grave abuse of discretion against the NLRC; its decision was
grounded upon petitioners manifest indifference to the plight of
its suspended employee and its consequent violation of the
Implementing Rules of the Labor Code.
- As the NLRC correctly ruled: The long period of preventive
suspension could even be considered constructive dismissal
because were it not for his letters demanding his reinstatement,
PAL by its inaction appeared to have no plan to employ

Labor Law 1
respondent back to work. The manifest inaction of PAL over
the pendency of the administrative charge is indeed violative of
Castros security of tenure because without any justifiable
cause and due process, his employment would have gone into
oblivion.
- PAL contends that when respondent consented to the
resolution that the entire period of suspension shall constitute
his penalty for the offense charged, the latter is thereby
estopped to question the validity of said suspension. We concur
with the labor arbiter when he ruled that the ensuing conformity
by respondent does not cure petitioner's blatant violation of the
law, and the same is therefore null and void- We do not
question the right of the petitioner to discipline its erring
employees and to impose reasonable penalties pursuant to law
and company rules and regulations. Having this right,
however, should not be confused with the manner in which that
right must be exercised. Thus, the exercise by an employer of
its rights to regulate all aspects of employment must be in
keeping with good faith and not be used as a pretext for
defeating the rights of employees under the laws and applicable
contracts. Petitioner utterly failed in this respect.
Disposition Petition is DISMISSED for lack of merit. Assailed
decision is AFFIRMED.

VALIAO V CA
[PAGE 11]
CADIZ V CA
[PAGE 224]
MARICALUM MINING CORP V DECORION
487 SCRA 182
TINGA; April 12, 2006
NATURE
Appeal from decision of CA
FACTS
- Decorion was a regular employee of Maricalum Mining who
started out as a Mill Mechanic and was later promoted to
Foreman I.
- On April 11, 1996, the Concentrator Maintenance Supervisor
called a meeting which Decorion failed to attend as he was then
supervising the workers under him. Because of his alleged
insubordination for failure to attend the meeting, he was placed
under preventive suspension on the same day. He was also not
allowed to report for work the following day.
- May 12, 1996, Decorion was served a Notice of Infraction and
Proposed Dismissal to enable him to present his side. On May
15, he submitted to the Personnel Department his written reply.
A grievance meeting was held upon Decorion's request on June
5, during which he manifested that he failed to attend the
meeting on April 11 because he was then still assigning work to
his men. He maintained that he has not committed any offense
and that his service record would show his efficiency.
- July 23, 1996, Decorion filed before the Labor Arbiter a
complaint for illegal dismissal and payment of moral and
exemplary damages and attorney's fees.
- In the meantime, the matter of Decorion's suspension and
proposed dismissal was referred to Atty. Roman G. Pacia, Jr.,
Maricalum Mining's Chief and Head of Legal and Industrial
Relations, and recommended that Decorion's indefinite
suspension be made definite with a warning that a repetition of
the same conduct would be punished with dismissal. Maricalum
Mining's Resident Manager issued a memorandum on August 2
placing Decorion under definite disciplinary suspension of 6
months which would include the period of his preventive
suspension which was made to take effect retroactively.

A2010

- 274 -

Disini

- On September 4, 1996, Decorion was served a


memorandum informing him of his temporary lay-off due to
Maricalum Mining's temporary suspension of operations and
shut down of its mining operations for 6 months, with the
assurance that in the event of resumption of operations, he
would be reinstated to his former position without loss of
seniority rights.
- Decorion, through counsel, wrote to Maricalum Mining on
October 8, 1996, requesting that he be reinstated to his former
position. The request was denied with the explanation that
priority for retention and inclusion in the skeleton force was
given to employees who are efficient and whose services are
necessary during the shutdown.
- Labor Arbiter found Decorion's dismissal illegal and ordered
his reinstatement with payment of backwages and attorney's
fees. According to the labor arbiter, Decorion's failure to attend
the meeting called by his supervisor did not justify his
preventive suspension. Further, no preventive suspension
should last longer than 30 days.
- The NLRC, however, reversed the labor arbiter's decision and
dismissed Decorion's complaint. The reversal was premised on
the finding that the case was litigated solely on Decorion's
allegation that he was dismissed on April 11, 1996. However,
during the grievance meeting held on June 5, 1996, Decorion
left it up to management to decide his fate, indicating that as of
that time, there was no decision to terminate his services yet.
According to the NLRC, to consider the events that transpired
after April 11, 1996 and make the same the basis for the finding
of illegal dismissal would violate Maricalum Mining's right to due
process.
- CA reinstated decision of labor arbiter. It held that Decorion
was placed under preventive suspension immediately after he
failed to attend the meeting called by his supervisor on April 11,
1996. At the time he filed the complaint for illegal dismissal on
July 23, 1996, he had already been under preventive
suspension for more than 100 days in violation of Sec. 9, Rule
XXIII, Book V of the Omnibus Rules Implementing the Labor
Code (Implementing Rules) which provides that no preventive
suspension shall last longer than 30 days.
-

Maricalum Minings MFR was denied.

ISSUE
WON Decorion was dismissed or merely under preventive
suspension
HELD
- Decorions preventive suspension has already ripened into a
constructive dismissal.
- Sections 8 and 9 of Rule XXIII, Book V of the Implementing
Rules provide:
Section 8. Preventive suspension. The employer may
place the worker concerned under preventive
suspension if his continued employment poses a serious
and imminent threat to the life or property of the
employer or his co-workers.
Section 9. Period of Suspension No preventive
suspension shall last longer than thirty (30) days. The
employer shall thereafter reinstate the worker in his
former or in a substantially equivalent position or the
employer may extend the period of suspension provided
that during the period of extension, he pays the wages
and other benefits due to the worker. In such case, the
worker shall not be bound to reimburse the amount paid
to him during the extension if the employer decides,
after completion of the hearing, to dismiss the worker.
- Preventive suspension is justified where the employee's
continued employment poses a serious and imminent
threat to the life or property of the employer or of the
employee's co-workers. Without this kind of threat,
preventive suspension is not proper.
- Decorion was suspended only because he failed to attend
a meeting called by his supervisor. There is no evidence to
indicate that his failure to attend the meeting prejudiced

Labor Law 1
his employer or that his presence in the company's
premises posed a serious threat to his employer and coworkers. The preventive suspension was unjustified.
- Decorion's suspension persisted beyond the 30-day
period allowed by the Implementing Rules. In Premiere
Development Bank v. NLRC the Court ruled that
preventive suspension which lasts beyond the maximum
period allowed by the Implementing Rules amounts to
constructive dismissal.
- At the time Decorion filed a complaint for illegal
dismissal, he had already been suspended for 103 days.
Decorion's preventive suspension had already ripened into
constructive dismissal at. While actual dismissal and
constructive dismissal do take place in different fashion,
the legal consequences they generate are identical. His
employment may not have been actually terminated in the
sense that he was not served walking papers but there is
no doubt that he was constructively dismissed as he was
forced to quit because continued employment was
rendered impossible, unreasonable or unlikely by
Maricalum Mining's act of preventing him from reporting
for work.
- Article 286 of the Labor Code, which provides that the
bona fide suspension of the operation of a business or
undertaking for a period not exceeding six (6) months
shall not terminate employment, may not be applied in
this case. The instant case involves the preventive
suspension of an employee not by reason of the
suspension of the business operations of the employer but
because of the employee's failure to attend a meeting.
The allowable period of suspension in such a case is only
30 days as provided by the Implementing Rules.
Disposition Petition denied. CAs decision affirmed.

RATIONALE
KWIKWAY ENGG WORKS V NLRC (VARGAS)
195 SCRA 526
MEDIALDEA; March 22, 1991
FACTS
- Respondent Vargas was formerly employed by Kwikway as
bookkeeper and secretary. As bookkeeper, it was her duty to fill
up the check vouchers and indicate therein the name of the
customer agent and the amount payable to each before they
are presented to the agents for signing.
- The new branch manager (BM) discovered that several blank
vouchers already contained the signatures of the mechanic
agents. BM confronted the branch cashier in charge of the
vouchers, Marina Corpus, concerning the irregularity. Corpus
explained that Vargas was aware of this practice. When asked
for an explanation, Vargas stated that the procedure has been
the practice in that office since the time of the former branch
manager who had knowledge thereof.
- BM informed the head office with his discovery. Kwikways VP
conducted an investigation. On the following day, Vargas and
Corpus were placed under preventive suspension for an
indefinite period of time on the ground of loss of trust and
confidence.
- Vargas was informed of the result of the investigation.
Kwikway offered her a chance to resign with separation pay,
which she accepted.
- The Labor Arbiter rendered a decision directing the
reinstatement of respondent Vargas to her former position with
backwages. NLRC affirmed the decision of the labor arbiter.
- Petitioner: the nature of the position of Vargas involves trust
and confidence. That private respondent's acts of dishonesty as
well as her active participation in violating and infringing
company accounting procedure which allowed the cashier to
personally misappropriate sums of money provide sufficient
basis for dismissing respondent. That Vargas was aware that
her cashier Corpus was committing acts of dishonesty and

A2010

- 275 -

Disini

misappropriation of company funds but she did not report the


matter to her superiors in the company. That the actuations of
Vargas were in violation of the company's code of conduct,
which is punishable by dismissal.
ISSUES
1. WON the dismissal of respondent Vargas was for a just and
valid cause
2. WON respondent Vargas was deprived of her constitutional
right to due process
HELD
1. YES
- The rule is settled that if there is sufficient evidence to show
that the employee has been guilty of breach of trust or that his
employer has ample reason to distrust him, the labor tribunal
cannot justly deny to the employer the authority to dismiss
such employee.
- Jurisprudence abounds with cases recognizing the right of the
employer to dismiss the employee on loss of confidence. More
so in the case of supervisors or personnel occupying positions
of responsibility, loss of trust justifies termination
- The mere existence of basis for believing that the employee
has breached the trust of employer is sufficient and does not
require proof beyond reasonable doubt.
- Clearly, respondent Vargas's position involves a high degree of
responsibility requiring trust and confidence. Her position
carries with it the duty to observe proper company procedures
in the fulfillment of her job as it relates closely to the financial
interests of the company.
2. YES
Notice and Hearing
- The twin requirements of notice and hearing constitute
essential elements of due process in cases of employee
dismissal: the requirement of notice is intended to inform the
employee concerned of the employer's intent to dismiss and the
reason for the proposed dismissal; upon the other hand, the
requirement of hearing affords the employee an opportunity to
answer his employer's charges against him accordingly to
defend himself therefrom before dismissal is effected. Neither
of these two requirements can be dispensed with without
running afoul of the due process requirement of the 1987
Constitution.
- In the instant case, the records are bereft of any indication
that a formal notice of the charge was given to the respondent
prior to the suspension or that the said investigation gave
adequate opportunity to the respondent to defend herself. It is
important to stress that an employee whose services are sought
to be terminated, has the right to be informed beforehand of his
proposed dismissal or suspension as well as of the reasons
therefor and to be afforded an adequate opportunity to defend
himself from the charges leveled against him. We give respect
to the following conclusions of the labor arbiter and respondent
Commission:
It is patent from the respondent's submission that written
notice specifying the causes for termination was never
furnished to complainant. Neither does it appear that she was
given enough opportunity to explain her side and defend
herself with the assistance of a representative of her choice is
she so desires.
Preventive Suspension
- Further, the preventive suspension of respondent Vargas for
an indefinite period amounted to a dismissal and is violative of
Section 4, Rule XIV of the Implementing Rules of the Labor Code
which limits the preventive suspension to thirty (30) days. The
said rule also provides that "the employer shall thereafter
reinstate the worker in his former or in a substantially
equivalent position or the employer may extend the period of
suspension provided that during the period of extension, he
pays the wages and other benefits due to the worker." (Pacific
Cement Company Inc. v. NLRC
Disposition the petition GRANTED. The questioned decision of
the respondent NLRC insofar as it ordered the reinstatement of

Labor Law 1
respondent Rosalinda Vargas with payment of three (3) years
backwages is REVERSED and SET ASIDE.
- Petitioner company is ordered to pay an indemnity of
P1,000.00 to respondent Vargas (For failure of the employer to
comply with the requirements of due process in terminating the
employees service, it shall be liable to indemnify the employee
in the sum of P1,000.00 as damages)

GATBONTON V NLRC (MIT, CALDERON)


479 SCRA 416
AUSTRIA-MARTINEZ; January 23, 2006
NATURE
Petition for review on certiorari
FACTS
- Gatbonton is a professor at the Mapua Institute of Technology,
a member of the Faculty of Civil Engineering. In November
1998, a student filed a complaint of sexual harassment against
Gatbonton. He was then placed under preventive suspension
for 30 days pending investigation.
- Gatbonton filed a complaint against MIT for illegal suspension.
- Petitioner questioned the validity of the administrative
proceedings with the Manila RTC but the case was terminated
on May 21, 1999 when the parties entered into a compromise
agreement wherein MIT agreed to publish in the school organ
the rules and regulations implementing the Anti-Sexual
Harassment Act.
The Labor Arbiter later declared that the
preventive suspension imposed was illegal.
- Both respondents and petitioner filed their appeal from the
Labor Arbiters Decision, with Gatbonton questioning the
dismissal of his claim for damages. The NLRC favorable granted
the appeal of respondent MIT. Gatbonton.
ISSUE
WON the NLRC erred in dismissing Gatbontons claim for
damages stemming from an alleged illegal suspension
HELD
YES
Ratio Preventive suspension is a disciplinary measure for the
protection of the companys property pending investigation of
any alleged malfeasance or misfeasance committed by the
employee. The employer may place the worker concerned
under preventive suspension if his continued employment
poses a serious and imminent threat to the life or property of
the employer or of his co-workers. However, when it is
determined that there is no sufficient basis to justify an
employees preventive suspension, the latter is entitled to the
payment of salaries during the time of preventive suspension.
Reasoning
- R.A. No. 7877 imposed the duty on educational or training
institutions to promulgate rules and regulations in consultation
with and jointly approved by the employees or students or
trainees, through their duly designated representatives,
prescribing the procedures for the investigation of sexual
harassment cases and the administrative sanctions therefor.
- Petitioners preventive suspension was based on respondent
MITs Rules and Regulations for the Implemention of the AntiSexual Harassment Act which provides that any member of the
educational community may be placed immediately under
preventive suspension during the pendency of the hearing of
the charges of grave sexual harassment against him if the
evidence of his guilt is strong and the school head is morally
convinced that the continued stay of the accused during the
period of investigation constitutes a distraction to the normal
operations of the institution or poses a risk or danger to the life
or property of the other members of the educational
community.
- But the said rules and regulations were published only on
February 23, 1999.
- The Mapua Rules is one of those issuances that should be
published for its effectivity, since its purpose is to enforce and

A2010

Disini

- 276 -

implement R.A. No. 7877, which is a law of general application.


Thus, at the time of the imposition of petitioners preventive
suspension on January 11, 1999, the Mapua Rules were not yet
legally effective, and therefore the suspension had no legal
basis.
- The Court also finds that there is insufficient legal basis to
justify the preventive suspension because of the absence of the
required requisites. Under the Mapua Rules, an accused may
be placed under preventive suspension during pendency of the
hearing under any of the following circumstances:
(a) if the evidence of his guilt is strong and the school head is
morally convinced that the continued stay of the accused
during the period of investigation constitutes a distraction to
the normal operations of the institution; or
(b) the accused poses a risk or danger to the life or property of
the other members of the educational community.
Disposition petition is PARTIALLY GRANTED. The decision of
Labor Arbiter is reinstated while the decisions of the CA and the
NLRC are set aside.

NUMBER OFFENSES
APARENTE V NLRC
[PAGE 240]

Das könnte Ihnen auch gefallen